Вы находитесь на странице: 1из 641

PRESSURE TRANSIENT TESTING:

DESIGN AND ANALYSIS

Vol. II

• By

James T. Smith

0

COPYRIGHT

By

James T. Smith
P.O. Box 1990
Cody, Wyoming 82414
^ Tele p hone: (307) 527-6494
E-Mail: jtsmith@180com.net

ALL RIGHTS RESERVED

This book, or any part thereof, may not be reproduced


in any form without permission of the author.

0 0603
. TABLE OF CONTENTS

Page

VI. PRESSURE TRANSIENT BEHAVIOR IN


FRACTURED RESERVOIRS

Types of Fractured Systems . . . . . . . . . . . . . . . . 6-2


Infinite Conductivity, Single plane, Vertical
Fractures . . . . . . . . . . . . . . . . . . . . . . . . . . . 6-3
Flow Regimes Associated with Fractures .... 6-4
Conventional Analysis of Pressure Data ..... 6-13
Permeability and Fracture Length ..... 6-13
Beginning of Pseudoradial Straight
Line . . . . . . . . . . . . . . . . . . . . . . . 6-17
Short Producing Times . . . . . . . . . . . . 6-19
Formation Damage . . . . . . . . . . . . . . 6-19
Wellbore Storage . . . . . . . . . . . . . . . 6-22
Average Reservoir Pressure . . . . . . . . . 6-24
^ Type C urve Ana l ys i s . . . . . . . . . . . . . . . . 6 - 32
Permeability and Fracture Length .... . 6-32
Limitations of Type Curve Analysis .... 6-34
Type Curve Versus Conventional
Analysis . . . . . . . . . . . . . . . . . . . . . 6-35
Finite Conductivity, Single Plane, Vertical
Fractures .. . . . .......... .... . . . . . . . 6-39
Uniform Flux Fractures . . . . . . . . . . . . . . . 6-40
Finite Conductivity Fractures . . . . . . . . . . 6-40
Naturally Fractured Reservoirs . . . . . . . . . . . . . . 6-44
Summary . . . . . . . . . . . . . . . . . . . . . . . . . . . 6-46
References . . . . . . . . . . . . . . . . . . . . . . . . . . 6-49
Nomenclature . . . . . . . . . . . . . . . . . . . . . . . . 6-52
Summary of Major Equations . . . . . . . . . . . . . . . 6-54
Problems .......................... . 6-55


iii
Page
^
LBF. BILINEAR FLOW - LIQUID RESERVOIRS

Flow Periods For A Vertically Fractured Well ... . LBF-1


Fracture Linear Flow . . . . . . . . . . . . . . . . LBF-1
Bilinear Flow . . . . . . . . . . . . . . . . . . . . . LBF-1
Formation Linear Flow . . . . . . . . . . . . . . . LBF-3
Pseudoradial Flow . . . . . . . . . . . . . . . . . . LBF-3
Bilinear Flow Equations . . . . . . . . . . . . . . . . . . LBF-3
Constant Formation Face Rate . . . . . . . . . . . LBF-3
Constant Formation Face Pressure . . . . . . . . LBF-4
Bilinear Flow Graphs . . . . . . . . . . . . . . . . . . . . LBF-5
Constant Formation Face Rate . . . . . . . . . . . LBF-5
Constant Formation Face Pressure . . . . . . . . LBF-7
End of Bilinear Flow . . . . . . . . . . . . . . . . . . . . LBF-9
Constant Formation Face Rate . . . . . . . . . . . LBF-9
Constant Formation Face Pressure . . . . . . . . LBF-9
Analysis of Bilinear Flow Data . . . . . . . . . . . . . . LBF-11
Liquid - Constant Rate . . . . . . . . . . . . . . . LBF-11
Liquid - Constant Pressure . . . . . . . . . . . . . LBF-13
Effect of Flow Restrictions . . . . . . . . . . . . . . . . LBF-14
Effect of Wellbore Storage . . . . . . . . . . . . . . . . LBF-16
References . . . . . . . . . . . . . . . . . . . . . . . . . . LBF- 18

NF. INTRODUCTION TO PRESSURE


TRANSIENT TESTING IN NATURALLY
FRACTURED RESERVOIRS

Warren-Root Model . . . . . . . . . . . . . . . . . . . . . NF-1


Idealized Model of Naturally Fractured
Reservoir . . . . . . . . . . . . . . . . . . . . . . . . NF-1
Assumptions in Warren-Root Model . . . . . . . NF-1
Primary Parameters . . . . . . . . . . . . . . . . . NF-2
Ideal Semilog Behavior . . . . . . . . . . . . . . . NF-3
Conventional Semilog Analysis . . . . . . . . . . NF-4
Type Curve Analysis . . . . . . . . . . . . . . . . . . . . NF-4
Double Porosity Type Curves . . . . . . . . . . . NF-4

iv
Page

Type Curve Matching . . . . . . . . . . . . . . . . NF-7


Type Curve Analysis . . . . . . . . . . . . . . . . NF-9
Type Curve Analysis When Total System
Behavior is Not Observed . . . . . . . . . . . . . NF-10
Analysis With Pressure Derivatives . . . . . . . . NF-11
Fissured Versus Multilayered Reservoirs .... NF-13
Example Problem . . . . . . . . . . . . . . . . . . . . . . NF-14
References .... ..... ........... . .. .. . NF-24

VII. GAS WELL TESTING

Introduction . . . . . . . . . . . . . . . . . . . . . . . . . . 7-1
Pressure Drawdown Test . . . . . . . . . . . . . . . . . 7-3
p2-Method . . . . . . . . . . . . . . . . . . . . . . . 7-3
Limitations . . . . . . . . . . . . . . . . . .. 7-4
Permeability Determination . . . . . . . .. 7-5
Skin Factor . . . . . . . . . . . . . . . . . .. 7-6
^ Evaluation of Turbulence . . . . . . . . .. 7-6
Flow Efficiency . . . . . . . . . . . . . . .. 7-8
Pseudosteady State . . . . . . . . . . . . .. 7-9
p-Method . . . . . . . . . . . . . . . . . . . . . . .. 7-13
Limitations . . . . . . . . . . . . . . . . . .. 7-13
Permeability Determination . . . . . . . .. 7-13
Skin Factor . . . . . . . . . . . . . . . . . .. 7-14
Flow Efficiency . . . . . . . . . . . . . . .. 7-14
Real Gas Pseudo-Pressure, m(p) . . . . . . . .. 7-14
Evaluation of m(p) . . . . . . . . . . . . .. 7-16
Pressure Drawdown Equation . . . . . . .. 7-22
Permeability Determination . . . . . . . .. 7-22
Skin Factor . . . . . . . . . . . . . . . . . . . 7-23
Flow Efficiency . . . . . . . . . . . . . . .. 7-24
Pseudosteady State Flow . . . . . . . . . .. 7-24
Advantages of m(p) Method . . . . . . .. 7-24
Pressure Buildup Test . . . . . . . . . . . . . . . . . .. 7-29
^ p2-Method . . . . . . . 7-29
Horner Analysis . . . . . . . . . . . . . . .. 7-30

v
Page

Permeability . . . . . . . . . . . . . . . 7-30
Skin Factor . . . . . . . . . . . . . . . 7-30
Average Pressure . . . . . . . . . . . 7-31
Flow Efficiency . . . . . . . . . . . . 7-32
Miller-Dyes-Hutchinson Plot . . . . . . . . 7-38
Agarwal Method . . . . . . . . . . . . . . . 7-39
p-Method . . . . . . . . . . . . . . . . . . . . . . . . 7-39
Horner Analysis . . . . . . . . . . . . . . . . 7-39
Permeabi lity . . . . . . . . . . . . . . . 7-40
Skin Factor . . . . . . . . . . . . . . . 7-40
Average Pressure . . . . . . . . . . . 7-40
Flow Efficiency . . . . . . . . . . . . 7-40
Other Methods of Analysis . . . . . . . . . 7-40
Real Gas Pseudopressure, m(p), Method ... . 7-41
Horner Analysis . . . . . . . . . . . . . . . . 7-41
Permeability . . . . . . . . . . . . . . . 7-41
Skin Factor . . . . . . . . . . . . . . . 7-41
Flow Efficiency . . . . . . . . . . . . 7-42
Average Pressure . . . . . . . . . . . 7-42
Other Methods of Analysis . . . . . . . . . 7-42
Problems of Interpretation . . . . . . . . . . . . . . . . . 7-50
Wellbore Storage . . . . . . . . . . . . . . . . . . . 7-50
Bounday Effects .................. . 7-53
Short Producing Time . . . . . . . . . . . . . . . . 7-53
Changing Gas Properties . . . . . . . . . . . . . . 7-53
Type Curve Analysis . . . . . . . . . . . . . . . . . . . . 7-55
Fractured Gas Wells . . . . . . . . . . . . . . . . . . . . 7-56
Conventional Method of Analysis . . . . . . . . . 7-57
Type Curve Analysis . . . . . . . . . . . . . . . . 7-59
Gas Well Deliverability Testing . . . . . . . . . . . . . 7-67
Flow-After-Flow Tests .............. . 7-68
Empirical Method of Analysis . . . . . . . 7-70
Theoretical Method of Analysis ..... . 7-85
Summary . . . . . . . . . . . . . . . . . . . . 7-89
Isochronal Tests . . . . . . . . . . . . . . . . . . . 7-89
Empirical Method of Analysis . . . . . . . 7 - 93

vi
Page

Theoretical Method of Analysis .. .... 7-93


Modified Isochronal Test . . . . . . . . . . . . . . 7-95
Analysis of Data When Pseudosteady
State Flow is Not Attained . . . . . . . . . . . . . 7-99
Use of Real Gas Pseudopressure, m(p),
in Gas Well Deliverability Analysis . ...... 7-101
Summary . . . . . . . . . . . . . . . . . . . . . . . . . . . 7-102
References . . . . . . . . . . . . . . . . . . . . . . . . . . 7-103
Nomenclature . . . . . . . . . . . . . . . . . . . . . . . . 7-106
Summary of Major Equations . . . . . . . . . . . . . . . 7-109
Problems . . . . . . . . . . . . . . . . . . . . . . . . . . . 7-114

GBF. BILINEAR FLOW - GAS RESERVOIRS

Bilinear Flow Equations . . . . . . . . . . . . . . . . .. GBF-1


Constant Formation Face Rate . . . . . . . . . .. GBF-1
Constant Formation Face Pressure . . . . . . .. GBF-2
i Bilinear Flow Graphs . . . . . . . . . . . . . . . . . . .. GBF - 3
Constant Formation Face Rate . . . . . . . . . .. GBF-3
Constant Formation Face Pressure . . . . . . .. GBF-4
End of Bilinear Flow . . . . . . . . . . . . . . . . . . . . GBF-6
Constant Formation Face Rate . . . . . . . . . .. GBF-6
Constant Formation Face Pressure . . . . . . .. GBF-8
Analysis of--Bilinear Flow Data . . . . . . . . . . . . .. GBF-10
Gas - Constant Rate . . . . . . . . . . . . . . . .. GBF-10
Gas - Constant Pressure . . . . . . . . . . . . . .. GBF-11
References . . . . . . . . . . . . . . . . . . . . . . . . .. GBF-13

VIII. INJECTION WELL TESTING

Pressure Falloff Test in Unit-Mobility Ratio


Reservoirs . . . . . . . . . . . . . . . . . . . . . . . . . . . 8-2
Permeability . . . . . . . . . . . . . . . . . . . . . . 8-6
Skin Factor . . . . . . . . . . . . . . . . . . . . . . 8-6
^ Flow Efficiency . . . . . . . . . . . : : : : : : : : : 8-7
Miller-Dyes-Hutchinson Method 8-7

vii
Page

Average Reservoir Pressure . . . . . . . . . . . . 8-8


Wellbore Storage . . . . . . . . . . . . . . . . . . . 8-14
Pressure Falloff Analysis in Non-Unit Mobility
Ratio Systems . . . . . . . . . . . . . . . . . . . . . . . . 8-14
Two-Bank System . . . . . . . . . . . . . . . . . . 8-15
Three-Bank System . . . . . . . . . . . . . . . . . 8-17
Pressure Falloff Analysis in Fractured
Reservoirs . . . . . . . . . . . . . . . . . . . . . . . . . . . 8-24
Injectivity Test . . . . . . . . . . . . . . . . . . . . . . . . 8-33
Step Rate Testing . . . . . . . . . . . . . . . . . . . . . . 8-36
Introduction . . . . . . . . . . . . . . . . . . . . . . 8-36
Test procedure and Analysis . . . . . . . . . . . . 8-37
Pressure versus Rate Plot . . . . . . . . . . 8-37
Multirate Analysis . . . . . . . . . . . . . . 8-45
Factors Affecting SRT Analysis . . . . . . . . . . 8-68
Time Step Size . . . . . . . . . . . . . . . . 8-68
Rate Increment . . . . . . . . . . . . . . . . 8-74
Wellbore Storage . . . . . . . . . . . . . . . 8-75
Changing Wellbore Storage . . . . . . . . . 8-80
Skin Effects . . . . . . . . . . . . . . . . . . 8-84
Selection of p; for SRT Analysis ...... 8-85
Step Rate Test on a Fractured Well ....... . 8-92
Two-Step Rate Test . . . . . . . . . . . . . . . . . 8-93
Analysis Methods . . . . . . . . . . . . . . . 8-94
Design Considerations . . . . . . . . . . . . 8-101
References . . . . . . . . . . . . . . . . . . . . . . . . . . 8-104
Nomenclature . . . . . . . . . . . . . . . . . . . . . . . . 8-106
Summary of Major Equations . . . . . . . . . . . . . . . 8-108
Problems ......................... . 8-110

IX INTERFERENCE AND PULSE TESTING

Introduction . . . . . . . . . . . . . . . . . . . ....... 9-1


Interference Test . . . . . . . . . . . . . . . . ....... 9-4
Homogeneous Isotropic Reservoir . . ....... 9-7
Single Active Well . . . . . . . ....... 9-7

viii
Page

Multiple Active Wells . . . . . . . . . . . . 9-16


Homogeneous Anisotropic Reservoir . . . . . . . 9-24
Heterogeneous Anisotropic Reservoirs . . . . . . 9-36
Pulse Testing . . . . . . . . . . . . . . . . . . . . . . . . . 9-36
Introduction . . . . . . . . . . . . . . . . . . . . . . 9-36
Theoretical Background . . . . . . . . . . . . . . . 9-38
Homogeneous Isotropic Reservoirs . . . . . . . . 9-41
Test Design - Graphical Method . . . . . . 9-54
Test Design - Analytical Method . . . . . . 9-57
Test Analysis - Graphical Method .... . 9-60
Test Analysis - Analytical Method .... . 9-62
Homogeneous Anisotropic Reservoirs . . . . . . 9-64
Heterogeneous Anisotropic Reservoirs . . . . . . 9-65
Vertical Pulse Testing . . . . . . . . . . . . . . . . 9-65
Summary . . . . . . . . . . . . . . . . . . . . . . . . 9-66
References . . . . . . . . . . . . . . . . . . . . . . . . . . 9-67
Nomenclature . . . . . . . . . . . . . . . . . . . . . . . . 9-70
^ Summary of Major Equat i ons . . . . . . . . . . . . . . . 9 - 72

X. DRILLSTEM TESTING

Introduction . . . . . . . . . . . . . . . . . . . . . . . ... 10-1


Equipment and Procedures . . . . . . . . . . . . . . ... 10-3
Analysis of Pressure Data . . . . . . . . . . . . . . ... 10-11
Pressure Buildup Data . . . . . . . . . . . . . ... 10-11
Flow-Period Data . . . . . . . . . . . . . . . ... 10-14
Wireline Formation Tests . . . . . . . . . . . . . . ... 10-15
Interpretation of DST Pressure Charts . . . . . . . ... 10-16
References ... . . .... .... ... ... . . . . ... 10-20
Nomenclature ... .... ........... .. . . .. 10-22
Summary of Major Equations . . . . . . . . . . . . ... 10-23

APP. APPENDIX A: FLUID AND ROCK PROPERTY


CORRELATIONS


ix
• Chapter 6

PRESSURE TRANSIENT BEHAVIOR IN FRACTURED RESERVOIRS

A significant percentage of the wells tested each year


have fractures within their drainage area. Most of these
fractures are the result of using hydraulic fracturing as
a method of well stimulation. Since its inception in 1947,
hydraulic fracturing has become commonplace throughout the
world, particularly in regions noted for low permeability
and accompanying low productivity. Other fractures are less
expected, having resulted from drilling operations, or from
injection operations associated with waterflooding or other
enhanced recovery projects. Further, many reservoirs are
naturally fractured as a result of tectonic stresses within
the earth.
The methods of analyzing pressure data presented in
previous chapters were based upon an ideal radial flow model
which assumed,--among other things, a homogeneous formation.
We have already seen that any reservoir heterogeneity which
represents a significant departure from these assumed
conditions will cause measured formation face pressures to
deviate from the behavior predicted by this ideal model. A
fracture is a heterogeneity; accordingly, it is necessary to
modify previously used methods of pressure analysis when
pressure data are from a fractured well.
The objective of this chapter is to show how fractures
affect pressure data, and to present methods which can be
used to calculate permeability, skin factor, average reservoir
pressure and fracture length. Calculations involving both
conventional methods and type curves will be presented.
Further, the effects of wellbore storage and skin will be
discussed.
The equations and procedures presented in this chapter
apply specifically to the pressure buiZdup test. However,
other tests are handled in an analogous manner, and are well
documented in the literature. Also, this chapter will only
consider Ziquid systems; gas reservoirs will be discussed in
Chapter 7.

I. TYPES OF FRACTURED SYSTEMS

In practice, one may encounter several types of


fractured systems. For purposes of discussion, these may
be classified' as:

1) hydraulically fractured wells with fractures


o infinite conductivity;
2) hydrau %ly fractured wells with fractures of
finite conductivity;
3) wells intersecting natural fracturs;
4) wells producing from n^turally frac >>^Pd or
jointed, systems, but not directly intersecting
the fractures.

Fractures may be further categorized as vertical, inclined


or horizontal.
Hydraulically fractured wells tend to have vertical
fractures2. Horizontal fractures occur only at shallow depths
and, in practice, are seldom encountered. Accordingly, this
discussion of hydraulic fractures will be Zimited to vertical
fractures of types (1) and (2). Further, it will be assumed
that hydraulic fractures are fully penetrating, i.e., they
extend from top to bottom of the subject formation. Theory
and procedures which pertain to the analysis of horizontal
fractures are available3'`
0

6-2
• II. INFINITE CONDUCTIVITY, SINGLE PLANE, VERTICAL FRACTURES

Most hydraulic fractures have only one plane, tend to


be vertical, and fully penetrate the formation. Further,
most fractures have high conductivity. We will consider in
this section fractures which have infinite conductivity.
Infinite conductivity fractures are those in which pressure
is constant along the fracture at any given time, i.e., there
is no pressure loss in the fracture. Although pressure may
change with time in an infinite conductivity fracture, it will
not change with position along the fracture at a particular
time. Figure 6.1 presents a schematic of a vertical fracture
within a square drainage area. dP^.. ►^a^or+

kf _W

WELLBORE kf^,1= ^ar^uvtlca-Vduc,h'v'h^

• i
I ^
1
h
a5^n-vu ^ ac^Wu^

I
I Xf _ ^ vt^+ fu,^^G t
^--- ( -----
FRACTURE
2Xe ^/
^
I
^
j/
^ .

i
i
i
DRAINAGE
BOUNDARIES

Fig. 6.1: Schematic of a well with a single plane,


vertical fracture, and the associated
drainage area.


6-3
1. Flow regimes associated with fractures
[When a reservoir is fractured, the resulting pressure
behavior can no longer be described by conventional radial
flow theory.I Instead, it has been shown5-' that pressures
exhibit Linear flow behavior at early test times. Later,
pressure transients move away from the fracture with a shape
that is semi-elliptical in geometry; we will refer to this
as transitional fl--. As pressure waves continue to propa-
gate away from the fracture, they begin to approach a radial
geometry; pressures measured at the wellbore during this
time behave somewhat like radial flow and, consequently, this
period is referred to as pseudoradial flow. If a fracture is
relatively short compared to the distance to the nearest
boundary, true radial flow may be achieved. The flow geometry
during these three periods is illustrated by Fig. 6.2.

EARLY •
111 ^ 11 1
LATER MUCH LATER

. - .

Fig. 6.2: Flow geometries at various times in the


drainage area of a fractured well. •
6-4
A. Linear flow
At early test times, the linear flow behavior associated
with a fractured system can be described by the linear
diffusivity equation which was previously presented as Eq.
2.43 in Chapter 2:

912P = ^]Jct 2R (6.1)


0.0002637k at
ax2

Subject to appropriate initial and boundary conditions, this


equation can be solved to determine the pressure behavior of
a fractured well. During the transient flow period, a well
producing at constant rate will have a flowing formation
face pressure, according to Eq. 6. 1, equal to SO.hda+y or►,dtiions :
l. Constav.f 3• P=^^ Q
^ 2. ^ansiek4 -F'lou7 ^F^
_ 16.3qB ut ^ - 141.2qB^
(s-sf) (6.2)
pwf = pi Af [ckj kh

where Af represents the area of the fracture face. In


particular, the flowing pressure at the time of shut-in
(i.e., t=tp and At=0), assuming that flow is transient, is
-'2
pwf(At=0) = pi - 16A3qB u^ _
f t tk L i - 141•2 Bu (s-s)
kh f
(6.3)

If a well is shut in for a buildup test after producing


for a time, tp, the shut-in pressure, pws, can be predicted
using Eq. 6.2 and the principle of superposition expressed
by Eq. 5.2:

t +At t +At
pi pws (pi pwf)ql p +(pi pwf)-ql p
0 t
p

^
_ 16 3qB u(tp+At) 141.2qBu
pi pws Af ^ctk + kh (s-sf)

6-5
nc.W - old

//"' co ^)
16.3(- )B uAt ^ + 141.2(- ) BU (s-sf)
+ ^ &\ [ ckj
II •
16.3qB z
[ (tp+At) Z - (At) 2] (6.4)
pws pi Af [ctkj

Equations 6.3 and 6.4 can be combined to yield

Pws = pwf(At=0) - 16A B ^ [(tp+At) ^ - (at)


f t

i
+ 16.3qB tP + 141khqBU (s-sf).
(6.5)
Af t

^ Fracture face area, Af, can be written in terms of the


fracture half-length, xf, and fracture height, h. A
fracture has two faces of total length 2xf; thus the area
of the fracture faces is 4x fh. When this area is
substituted for Af in Eq. 6.5,

P ws = pw f(^t=0) - 4.06 ^ -^ 2[(t +Ot) z - ( At) Z]
f ^ t p

rv1c Y
+ 4.064qB utP + 141khqBp ( s-sf). (6.6)
O t
^^ f

b
Equation 6.6 is a very important result because it
indicates that a plot of pws versus the tandem square root
function, [(tp+At) 2-(At) 2] , will yield a straight line of
negative slope during linear flow. This is a diagnostic
feature of linear flow and is very useful in detecting
fractures. A schematic of the tandem square root plot is
illustrated by Fig. 6.3. It is noted that when linear
flow ends, data begin to deviate from the straight line.


6-6
STRAIGHT LINE
INDICATING

.... \ LINEAR FLOW

111'r
END OF
LINEAR FLOW

Li t -

0 S 10 15 20.

t +^t V77, hrs^`

Fig. 6.3: Schematic of tandem square root


plot illustrating the effect of
q linear flow.

^ t,c'b _ 0 .3J ^e.. C.Q,p,t^

L When producing time is significantly longer than shut-in


time, i.e., tp >> A t, Eq. 6.6 reduces to

pws "= pw f(At=0) + 4.06hcIB


f ¢Lc t
k Z(^t) z

+ 141.2aBU s-s
^^G (6 .7
kh ( f)

pWs = pw f(At=0) + mLF (^t) z+ 141khqBU (s s f) (6.8)

where: ^
4.064r{B u z
mLF = Xfh (6.9)
T .


6-7
Equation 6.9 indicates that when producing time is sufficiently
long, which is generally the case, a plot of pws versus At? •
will yield a straight line of positive slope during linear
flow. The square root plot is depicted by Fig. 6.4. Again,
the straight line feature of this plot is very important in
,the detection of fractures; further, the slope of this line
will be required in the calculation of permeability and
fracture length.

SLOPE = mLF, psi/hr2

• ® o •
M • •
V) •

V)

END OF
LINEAR FLOW

^pw f(A t0

Alt, h r s 12

Fig. 6.4: Schematic of square root plot


illustrating the effect of
linear flow.


6-8
jj-YU-Qn fLLrAr ^^ &

2. ^tK -4 Sh^'cY.t; 1([n^/- Gv^ cQ.tf('Jt^<aLv^ PtO


^

^ Another important feature of linear flow is observed


if we neglect the skin term in Eq. 6.7 and rearrange the
equation as

4.064c1B -2
pws - pwf(At=0) = Xfh (^t) z (6.10)
[4ctkj

Take the logarithm of^- Eq. 6.10 to obtain

_ 4,064 B 2
log Cpws pwf(At=0)] = log X--^-

+ 2 log At. (6.11)

The form of Eq. 6.11 suggests that a log-log plot of


LPwspwf(At=o)] versus At will trace a straight line
during linear flow. Moreover, it is significant that the
rsZope of this Zine will be one-haZf.l This relationship
is illustrated by Fig. 6.5.

10'

.^
HALF-SLOPF LINE

• • •


1() •
^

V)
END OF
LItiEAR FLOtiu`

1
10-1 1 10 1i)^

.^t , h T' s

Log-log plot showing the effect


of linear flow.

6-9
The straight line relationships illustrated by Figs.
6.4 and 6.5 provide distinctive and easily recognizable
evidence of a fracture. When properly applied, these plots
are the best diagnostic tool available for the purpose of
detecting a fracture.
When the duration of the linear flow period is short,
as it often is, care must be taken not to misinterpret the
data. It is common in this situation for skin effects or
wellbore storage effects to alter pressures to the extent
that the linear flow straight line does not occur, or is
very difficult to recognize. Interpretation in such cases
is difficult. This problem will be discussed in greater
detail in a subsequent section.

B. Transitional Flow
As pressure waves move away from the fracture, flow
ceases to be linear. Fluid always flows in a direction
perpendicular to isopotential lines; these lines of constant
pressure become semi-elliptical in shape soon after a test
begins and the flow lines are altered accordingly. A
schematic of flow lines is illustrated in Fig. 6.2 for this
flow regime. Because of the changing flow geometry,
pressure behavior during transitional flow is difficult to
describe mathematically. We will not attempt to analyze
pressures measured during this period using conventional
methods; however, the data can be analyzed using type curves.

C. Pseudoradial Flow
Following the period of linear pressure behavior, and
a transition period, pressures begin to exhibit character-
istics similar to a radial flow system; hence, this period
is referred to as pseudoradial flow. The pressure behavior
during this time is very dependent upon fracture penetration,
i.e., flow is almost radial for short fracture lengths, but
becomes linear as the fracture length increases to the
drainage radius of the well. This dependence upon fracture

6-10
length is illustrated by the theoretical Horner buildup
curves of Russell and Truitte presented in Fig. 6.6. Note
that fracture penetra tion is expressed as a ratio of
fracture half-length, xf, to the drainage radius, xe, of the
subject well.

3.9

4.1

XE 1 , 0 (LINEAR FLOW)
4.3 -

4.5 - Xe
} N
'„ ^

V) 4.7 -
3 03

4.9


^

5.1 ^
/

Q+-'1
5.3

IOS 104 103 10 1

t +At
p
^^n at
C,U.^P^x " et 1 U^8'^ $^vGLI ^nt u. f.rLtG, AGtGQd. OL^

Fig. 6.6: Effect of fracture penetration on


Horner buildup (After Ref. 8).

Some very important observations can be made from Fig.


6.6. First, the effect of fracture penetration, xf/xe, on
the Horner buildup curve is obvious; [the buildup behavior*
varies from radial to linear as fracture length increases
from zero to xe.l Second, there is no true straight line on
any of the curvesJ with a non-zero fracture penetration.
The only true straight line is the line which represents
• radial flow, i.e., xf/xe = 0; the slope of this line is

6-11
inversely proportional to reservoir permeability. It is ^
noted, however, that each fracture curve has a maximum slope
at its inflection point, and exhibits an apparent straight
line at this point. Third, the buildup curves are very
similar in shape to what one would expect in a homogeneous
reservoir. For example, the slow increase in pressure at
early buildup times is characteristic of wells in unfractured
reservoirs which have a wellbore storage problem. Thus,
fractures do not cause a distinct shape on the Horner plot,
and cannot always be detected from this p lot; the square
root plot and log-log plot must be used for this purpose. A
typical Horner plot for a fractured well is depicted by Fig.
6.7.

= mPP, PSI/CYCLE

^
^

10
^
102 103 10"

t +At
p
At

Fig. 6.7: Schematic of Horner plot for


a fractured well.


6-12

\ NORNER PLOT FOR ALL


7 PRODUCING TIMES

3 \ \ ^ ^ \ \ \ ^^ ,

0.05

V `^ \ ` \ `^ `^1•^^2

10-1
9 ` ^^`OpJ
8 ^
M/LLER - DYES -NUT-CN/NSON PO-'
PLOT

0 0.2 0.4 0.6 0.8

Xf/Xe

0
^ Finally, if the buildup for a fractured reservoir is
analyzed using conventional radial flow theory, and the
maximum slope on the Horner plot is used, the slope will
always be too small, the error increasing as fracture
penetration increases. This will result in a calculated
flow.capacity which is too large, an erroneous average
pressure, and a skin factor which is too small.
Obviously, the correct interpretation of pressure
data from a fractured reservoir is not possible unless the
presence of the fracture is first detected. Further,
conventional radial flow theory must be modified before
interpreting these systems.

2. Conventional analysis of pressure data


Pressure data from fractured systems can be analyzed
using either conventional plots, discussed here, or type
curves, which will be discussed in a subsequent section.

A. Permeability and fracture length


It was shown previously in Fig. 6.4 that a plot of
pwf(At=0)] versus At 1-2 will yield a straight line
[pws
during linear flow. The slope of this line, defined by
Eq. 6.9, can be rearranged to compute the fracture half-
length as

_ 4.064 c. B z
[^p ](6 . 1?_)
xf mLFh ctk

Equation 6.12 relates fracture length and permeability;


accordingly, if either is known from another source, Eq. 6.12
can be used to compute the unknown quantity. In most cases,
however, the objective of the test is to estimate both k and
xf. Since we only have one equation which contains two
unknowns, a second independent equation relating k and xf
is needed to determine these values. This relationship can

6-13
be determined from a theoretical computer model.
Figure 6.6 presented the theoretical buildup curves
of Russell and Truitta. It is observed from Fig. 6.6 that
all of the curves build up to a maximum slope, mPR, which
represents pseudoradial flow. This slope, which is a function
of fracture penetration, xf/x e , is not the true radial flow
slope and, consequently, will not yield the true permeability.
The radial flow straight line on Fig. 6.6 is the only line
which will give the true permeability. The maximum slope
can be used, however, to calculate an apparent permeability.
Notice that the pseudoradial slope for a fractured well is
always Less than the true radial flow slope; it follows,
since permeability is inversely proportional to the slope of
the semilog straight line, that apparent permeability
computed from the pseudoradial maximum slope will always be
Larger than true permeability. Since true permeability is
known from the computer model, and since an apparent perine-
ability can be computed from the maximum slope of each
pseudoradial curve, it is possible to determine a permeability
correction factor, F which is a function of fracture
cor l
penetration:

k (6.13)
Fcor ka

This factor was originally presented as a function of xf/xe


by Russell and Truitt8. However, a more recent model by
Raghavan9 is believed to be more accurate, and is presented
in Fig. 6.8.
Figure 6.8 gives permeability corrections for both
Horner and MDH plots. The solid line applies to the Horner
plot for all producing times. The remaining lines apply to
the MDH plot; notice that the MDH correction is a function
of producing time up to tDA = 0.12. Because the Horner
correction is independent of producing time, and since the
correction is smaller than required by the MDH plot, the

6-14
• /ER PLOT FOR ALL
PRODUCING 77MES

\ \ \ \ ^\
^\ \ \ \\\\
\\\ \ \ \ \
cd
\\ \ \ \ \ \\\^^
\ \ \ \ \^ \ \\ ^^
^\ \; \ \ \ ^^ ^^^ O./P
\\ \ \ ^ ^.^
II ^^^+J
^ \ \ \\ \ `\

0 ^ \\ \\ \\ ^\ 0. 02
U
\\ \\\ \\\ \^^^0`/1
0o/
\ \^ "o
10-^ \ \ ^^^s
\ ^
^,C?90

M/LLER -OY£S -HUTCHlNSON


PLOT

0 0.2 0.4 O.6 0.8 I

• Fig. 6.8:
xf/xe

Correction factor for


permeability estimated from
pressure buildup tests in
vertically fractured wells,
assuming sufficient shut-in
time to reach maximum slope
(After Ref. 9).

Horner method is recommended10 for analyzing fractured wells.


We are now in a position to estimate permeability and
fracture length. Substitute Eq. 6.13 into Eq. 6.12 to obtain

x4.064qB
_ u '2 (6.14)
f mLFh [ctFcorka]

The apparent permeability, ka, is computed from the pseudo-


radial maximum slope of the Horner plot as


6-15
k = -162.6 BP h (6.15)
a m
PR 0

Combining Eqs. 6.14 and 6.15,

4.064 B mPRh
xf = mL 162.6qB^ctFcor

0.319 mPRaB Z (6.16)


or 9
xf ° mLF ^cthFcor

Equations 6.16, 6.13, and the relationship between Fcor


and xf/xe depicted by Fig. 6.8, can be solved simultaneously
to determine k and xf. The required procedure is:

1) Determine from the half-slope line on


the log-log plot the data which represent
linear flow.
2) Identify the linear flow data on the square
root plot and construct a best-fit straight
line through these data. Determine the slope,
m, of this line. NOTE: If it is determined
that the linear flow straight line is masked
by storage effects, the conventional method
of analysis cannot be used; in this situation,
use type curves to analyze the data.
3) Construct a Horner plot of the buildup data
and determine if the pseudoradial straight
line is present. If present, determine the
slope, m R, of this line; if the maximum
slope has P not been reached, it is recommended
that type curves be used for the analysis.
Guidelines will be presented in the next
section to determine where the pseudoradial
straight line begins.
4) Compute the apparent permeability, ka, using
Eq. 6.15.
5) Estimate the distance, xe, to the drainage
boundary of the test well.
6) Assume a value of the fracture half-length, xf.
7) Based on the assumed value of xf, determine Fcor
from Fig. 6.8.
8) Compute xf using Eq. 6.16.

6-16
09) Comp are the value of xf computed in Step 8 to
the assumed value. If these values are equal,
the assumed xf is the correct fracture half-
length, and F is the correction factor for
permeability. corIf assumed and computed values
of X f are not equal, repeat steps 6-9.
10) Compute the true permeability as

k = kaFcor' (6.17)

B. Beginning of pseudoradial straight line


It should be observed that the correction factors in
Fig. 6.8 assume the test well was shut in sufficiently
long to reach the maximum slope. If a well does not
achieve the maximum slope, it is obvious from Fig. 6.6
that the slope will be too small, and the calculated apparent
permeability too large. Even though the apparent permeability
is corrected in this case, the corrected permeability will
still be too large.
^ Wattenbarger and Ramey" have shown that an approximate
relationship exists between the pressure change at the end
of linear flow, Apelf, and the beginning of the pseudoradial
straight line, Opbsl. In particular,

Apbsl - 2Apelf (6.18)

This rule of thumb, commonly referred to as the double-Ap


ruZe, can be applied based on information from the log-log
plot: Determine the value of [pws pwf(At=0)] at which
the half-slope line ends on the log-log plot, double this
value of Ap and determine from the log-log plot the shut-
in time, otbsll at which this pressure difference oGcurs.
The double-Ap rule is illustrated by Fig. 6.9. It is also
noted that

Atbsl ^__ l0Atelf (6.19)


6-17
10z

HALF-SLOPE LINE •
2Ape 1 f • •
-^••
10 , •
Apelf
----------

At elf Athsl

1 ^
10-1 1 10 102

At , hrs

Fig. 6.9: Use of the log-log plot to approximate the


beginning of pseudoradial flow.

which is equivalent to Eq. 6.18, and is commonly referred to


as the 10At rule. This means that the correct pseudoradial
flow line should begin approximately one log-cycle beyond the
end of linear flow.
Again, it is emphasized that the relationships expressed
by Eqs. 6.18 and 6.19 are approximations based on theoretical
studies. However, if application of these rules to a buildup
test indicate that shut-in time is too short for the maximum
slope to develop, it is recommended that type curves be used
to analyze the data. Type curves will be discussed in a
subsequent section.


6-18
C. Short producing times
When the producing time is short, it is recommended that
the tandem square root plot be used instead of the regular
square root plot. Further, it is recommended that all
calculations be made using the Agarwa112 equivalent time
which was previously defined by Eq. 5.79.

D. Formation Damage
If the slope of the Horner plot is corrected for the
fracture, i.e., m = mpR/Fcorl the total skin factor can be
computed using the radial flow equation previously presented
as Eq. 5.17:

p wf P lhr. - log
S = 1.151 k + 3.23 . ( 6.20)
m ^uctrw

Recall, however, that the total skin is a composite of


several effects as defined by Eq. 3.31:

(6.21)
s= sd + sr + sp + st + sf + ssw'

The dominant skin term in a fractured well is generally sf.


Due to the stimulation effect of the fracture, sf will be a
large negative-number; consequently, even if damage is
present, s will generally be a negative number.
An accurate evaluation of formation damage requires
that we be able to determine sd. From Eq. 6.21,

sd = s - sr - sp - st - sf - ssw. (6.22)
^

In order to compute sd using Eq. 6.22, sf must be available


from another source. Unfortunately, using current technology,
we have no reliable method to determine sf. Therefore, the
^ skin due to damage cannot be evaluated using Eqs. 6.20 and
6.22.

6-19
The best method"' 14 to evaluate damage in a fractured
well is to use the square root plot. In an ideal well with- •
out damage, the square root straight line will extrapolate
to pwf(At=0) at At=0; this was illustrated previously by
-Eq. 6.10 and Fig. 6.4. However, when a well is damaged, the
,intercept pressure will be greater than pwf(At=0); this is
illustrated by Eq. 6.7 and Fig. 6.10.

mLF •
^ • •

Pint

END OF
LINEAR FLOW



pwf(At=0) •
V/At, h r s il

Fig. 6.10: Effect of skin on the square


root plot.

The total skin factor exclusive of sf, i.e., s-sf, can


be determined from the square root plot. According to Eq.
6.7, extrapolation of the linear flow straight line to At=0
will result in an intercept pressure, Pint' equal to

_ 141.2qBu
(6.23)
pint pwf(At=0) + kh (s-sf)
0

6-20
With Pint obtained from the square root plot, we can
calculate

pwf(It=0)]kh
[pint (6.24)
s-sf = - 141.2qTiu

The skin effect due to restricted entry, sr, and


slanted wells, ssw, are not applicable to fractured
wells. Therefore,

sd = (s-sf) - sp - st. (6.25)

If the effects of turbulence and perforations are


negligible, which is true in most liquid systems,

sd = s-sf. (6.26)

^ Combining this equation with Eq. 6.24, the pressure loss


due to damage is shown to be

= 141.2cfBU (s-s f.)


(APs ) d - (AP )
s s-sf kh

pwf("t=0), (6.27)
(Aps)d Pint

Thus, it is not necessary to compute s or sf to determine


the pressure loss due to damage.
The log-log plot can also be valuable as a means of
recognizing skin i n a well. In the absence of skin and
storage effects, an infinite conductivity fracture causes
early-time data to form a half-slope line on the log-log
plot. However, if skin effects are present, the early-
time data will form a slope less than one-half.-1S. This is
illustrated by Fig. 6.11. Note on Fig. 6.11 that skin
causes the early-time data to approach the half-slope line
from above. Unfortunately, if wc.llbure storage effects are
4111

6-21
also present, this observation does not apply; wellbore
storage causes the log-log plot to have a slope greater •
than one-half at early times, thereby masking the presence
of skin in the well.

100

.,I HALF SLOPE LINE

^ ^• •
/ •
SKIN EFFECT •

+-)
a 10
• • ^
• • • ^ APPROXIMATE END
OF LINEAR FLOW
U1

1
1 10

,^t , hrs
100 1000

Fig. 6.11: Effect of skin on the log-log
plot of pressure buildup data.

Finally, Eq. 6.8 indicates that damage will not affect the
straight line on the square root plot; although the half-slope
line may not appear on the log-log plot, the square root
straight line will be present. Again, however, if wellbore
storage effects are present, this will dominate pressure
behavior and neither the half-slope line on the log-log plot,
or the straight line on the square root plot will be present.

E. Wellbore storage
Wellbore storage affects buildup behavior in fractured
systems much like it does in non-fractured reservoirs.
Figure 6.12 is a log-log graph depicting buildup data from •
6-22
a fractured well with wellbore storage. At early times
when wellbore storage controls the data, a unit-slope line
is formed. As time increases, data deviate from the-unit-
slope line and approach the line of half-slope. If the
linear period is short, or if wellbore storage effects are
severe, the half-slope line may be completely masked.

100
HALF SLOPE LINE

.^
^ • • ®
UNIT SLOPE LINE •

+J 10 4 ^
APPROXIMATE END
• OF LINEAR FLOW

APPROXIMATE END

• 1
OF WELLBORE
STORAGE CONTROL

1 10 100 1000

At, hrs

Fig. 6.12: Effect of storage on the log-log


plot for a vertically fractured
well.

It is not uncommon for a_non-fractured well with wellbore


storage to be mistaken for a fractured well. Figure 5.19
shows the log-log plot for a non-fractured well with storage;
notice that in the transition from the unit slope line to
radial flow, there will be a time interval in which the data
form a half-slope line. Consequently, the log-log plot for
ideal radial flow with storage can have the same appearance
as a fractured well. Fortunately, it is possible to distinguish
• between these two situations using the double-Ap rule and the
following procedure".

6-23
1) Determine from the log-log plot the pressure
drop, Apelf, which corresponds to the end of
the half-sIopc line.
2) Double the value of Ap obtained from Step 1,
i.e., 2APelf' and enter the log-log plot with
this value.
3) If the well is fractured, a horizontal line
representing 2Ap f will intersect the buildup
data at the appr^^tlmate shut-in time where
pseudoradial flow begins (see Fig. 6.9).
However, if the well is not fractured and the
half slope line is simply the transition from
storage to radial flow, the line representing 2APelf
will not intersect the test data; instead, it
will lie above the test data.

F. Average reservoir pressure


The volumetric average reservoir pressure within the
drainage volume of the test well can be determined using the
methods presented in Chapter 5. Because of minimum data
requirements, the author prefers to use the Muskat method;
however, this method can only be applied when data are
available at long shut-in times. The reader is referred to
Chapter 5 for a review of the procedures and limitations of
the methods commonly used to estimate average pressure.

Example 6.1: Conventional analysis of buildup data from


a hydraulically fractured well

Problem. The following data is from an oil well which was


fractured at the time of initial completion. Based upon
well spacing, the drainage radius of the well is estimated
to be 1,065 feet. At the time this well was shut in for a
buildup test, it had produced for 67 days; the producing
rate at shut-in was 17S STB/D. Using the buildup data
presented, estimate permeability, fracture length, and the
pressure loss due to formation damage.

0
6-24
^ p`V f(At=0) = 2,567 psia
qo = 175 STB/D
aw = 0
qg = 113.7 Mscf/D (reservoir is above the bubble point)
B 0 = 1.28 RB/STB
Po = 1.42 cp
h = 31 ft
= 12%
ct = 20 x 10-6 psi-1
xe = 1,065 feet
tp = 67 days = 1,608 hrs

At, hrs Ap, psi pws, psia t4-At/At At-'2, hrsZ

0.1 17 2584 16081 0.316


0.2 23 2590 8041 0.447
^ 0.3 27 2594 5361 0.S48
0.4 33 2600 4021 0.632
0.5 36 2603 3217 0. 70 7
0.6 39 2606 2681 0.775
0.7 41 2608 2298 U.837
0.8 43 2610 2011 0.894
0.9 45 2612 1788 0.949
1.0 48 2615 1609 1.00
1.5 58 2625 1073 1.22
2.0 66 2633 805 1.41
3.0 78 2645 537 1.73
4.0 88 2655 403 2.00
6.0 103 2670 269 2.45
8.0 115 2682 202 2.83
10.0 128 2695 162 3.16
15.0 150 2717 108 3.87
20.0 167 2734 81.4 4.47
^ 30.0 181 2748 54.6 5.47
40 . 0 212 2779 41.2 6.32

6-25
60.0 241 2808 27.8 7.75
80.0 262 2829 21.1 8.94
100.0 283 2850 17.1 10.00
150.0 320 2887 11.7 12.25
200.0 339 2906 9.04 14.14
300 .0 379 2946 6.36 17. 32
400.0 396 2963 5.02 20.00
456.0 408 2975 4.53 21.35

Solution. Figures 6.13 - 6.15 present log-log, square root,


and Horner plots of the buildup data, respectively. It is
noted from Fig. 6.13 that a half-slope line, indicating
linear flow, exists on the log-log plot for approximately
0.7 hours. A straight line through these same points is
shown on the square root plot in Fig. 6.14. The slope,
mLF, of the square root straight line is

mLF = 50 psi/hr.

According to the 10At rule, the pseudoradial straight


line should begin on the Horner plot at Atbsl - 7 hours;
this corresponds to a Horner time ratio of 231. It is noted
on Fig. 6.15 that the straight line begins slightly later
than this but, considering the approximate nature of this
rule of thumb, is in reasonable agreement with the predicted
time. The slope, mpR, of the pseudoradial straight line is

mPR = - 209 psi/cycle.

The apparent permeability of the reservoir is computed


using Eq. 6.15:

k a = - 162.6 mpR


6-26
• • •
103

HALF-SLOPE LINE

.^
^

r-
CD
102

OF LINEAR
FLOW

10

10-1 1 10 102

At, hrs

Fig. 6.13: Log-log plot, Ex. 6.1.


2800

= 50 PSI/HR 2

2 750

2700

Ln

V)
2650
N
00

2600

Pint = 2,568 PSIA


-I 1 11111

2550 t-
0 1
-

2
^

3 4 .
*
-

6
- -

'
VA t hrs z

Fig. 6.14: Square root plot, Ex. 6.1.

• • •
Ll • •
3000

2900
m PR = - 209 PSI/CYCLE

2800
^

a, 2700

2600

2500

1 10 102 103 10`'

t {'At
p
At

Fig. 6.15: Horner plot, Ex. 6.1.


ka
(162.6) (175) (1. 28) (1. 42)
(-209)(31 •
ka = 8.0 md

The correct permeability and fracture length can be


obtained by solving Eq. 6.16 simultaneously with the cor-
rection factor in Fig. 6.8:

Assume xf = 100 feet:

From Eq. 6.16,


2
_ 0.319 mPRaB
xf mLF ^cthFcor

_ 0.319 _ (-209)(175)(1.28) z
xf 50 (0.12) (20x10-6) (31) Fcor

x = 160
f %
F Z
cor

The value of xf/xe is:

100 = 0.094.
1065
xf/xe

From Fig. 6.8,

Fcor = 0.92.

Therefore,


Xf= 160
167 feet.
(0.92)

6-30
• Since assumed and calculated values are not equal, this
procedure must be repeated.

Assume xr = 17

1
xf/xe 1065 = 0.163

Fcor = 0.85

160
x - = 174.
f (0.85) 2

Since assumed and calculated values of fracture length are


equal, it is concluded that the fracture length is 174 feet
and the permeability correction factor is 0.85. The correct
permeability is, from Eq. 6.17,

0 k = kaFcor

k = (8.0) (0.85)

k = 6.8 md.

The pressure loss due to formation damage can be


estimated from the square root plot. It is observed from
Fig. 6.14 that the intercept pressure is:

Pint = 2,568 psia.

The pressure loss due to damage can be estimated using Eq. 6.27:

(Aps)d pint pwf(At=0)

= 2568 - 2567

(Aps)d = 1 psi.

6-31
Thus, it appears that damage is negligible in this well.
0

3. Type curve analysis


An alternate approach to fracture well analysis is to
use type curves. This method eliminates the trial-and-error
procedure required by conventional analysis. In those cases
where wellbore storage completely masks the linear flow data,
or when shut-in time is not long enough for pseudoradial flow
to begin, type curves represent the only available method of
analysis.
Figure 6.16 presents the type curve16 for a well
producing from an ideal reservoir with a single-plane, vertical
fracture of infinite conductivity. Like the Ramey type curve
discussed in Chapters 3 and 5, this is a theoretical plot of
dimensionless pressure versus dimensionless time. Dimension-
less pressure has the same definition as for radial flow, i.e.,

0.00708kh[pws-pwf(At=0)] (6.28)
PD = qBU '

Dimensionless time is defined as

tDxf _ 0.0002637kAt (6.29)


- 2
^uctxf

Figure 6.16 does not include the effects of skin or wellbore


storage and, consequently, cannot be used to determine the
skin factor.

A. Permeability and fracture length


The use of Fig. 6.16 to obtain k and xf requires a
curve matching procedure similar to that used with the
Ramey type curve, i.e., a log-log plot of [Pws pwf(At=0)]
versus At is prepared on tracing paper and is matched with

6-32
• • •
102

10

1^
U4
W

10-
1 0-2 10-1 10 102 l0g

'W" ^S
tDxf
• , ^^S
6.16: Type curve for single plane, infinite conductivity, vertical fracture
Fig (After Ref. 16) .
6- ^WiLt S^^Vy--
the type curve. A match point is selected, and values of
[pws pwf(At=0)]M and pDM are read from the ordinate of the
data curve and type curve, respectively. Substitute these
values into Eq. 6.23 and compute k as

k = 14qBp PDM (6.30)


[Pws-Pwf(At=0)]M

Similarly, determine the values of AtM and (tDxf)M which


define the match point. Then,

1-
0.0002637k A tM Z (6.31)
X
f ^Uct ^ tD
Xf) M
J

The skin factor and average reservoir pressure must be


obtained using conventional methods previously described.

^
B. Limitations of type curve analysis
The type curve presented in Fig. 6.16 is based on
pressure drawdown theory. Consequently, it should not be
applied in the conventional manner to buildup data unless
t p >> At. When the producing time is short, the Agarwal
method12, presented in Chapter 5, should be used. The
Agarwal method requires that equivalent time, Ot e , be
substituted for At in all plots and calculations.
Figure 6.16 applies only to fractures of infinite
conductivity. If pressure loss in the fracture is signifi-
cant because of low conductivity, or because the fracture
is very long, Fig. 6.16 will give incorrect results. The
most obvious error in this situation will be a calculated
fracture length which is much less than the true fracture
length. Solutions for fractures with finite conductivity
will be presented in the next section.
When damage is present on the face of the fracture,
this can cause the shape of the log-log plot to be

6-34
distorted; this was discussed in a previous section. Since
the type curve in Fig. 6.16 does not include the effect of
damage, this distortion can result in an incorrect match
if the data affected by damage are not recognized and the
distortion ignored in the process of curve matching. Skin
does not affect the shape or slope of the square root plot;
consequently, the conventional method of analysis will not
be affected by the presence of skin.

C. Type curves versus conventional methods


Assuming that the correct type curve is being used,
i.e., that we are working with fractures of very high
conductivity, the type curve offers the same accuracy as
the conventional method, and is easier to use. Further,
the type curve eliminates the problem of defining the linear
flow data, and deciding if the test was run long enough to
achieve pseudoradial flow.
Disadvantages of the type curve include the requirement
that pwf(At=0) be accurately measured. If this pressure is
incorrect, the shape of the log-log plot will be distorted
resulting in possible misinterpretation of the data. Further,
if a well is severely damaged, this will also cause dis-
tortion of the log-log plot. It'is recommended in these
situations, if they are recognized, that the conventional
method be used.
With the exception of the problem areas which were
noted, the author prefers to use type curves in the analysis
of fractured wells. They are easy to use, give acceptable
accuracy, and they eliminate most problems associated with
defining flow regions required by the conventional method.
This is in contrast to the analysis of non-fractured systems
where it was recommended that type curves never be used if
a conventional analysis is possible.


6-35

Example 6.2: Type curve analysis of buildup data from a
hydraulically fractured well

Problem. Analyze the buildup data presented in Ex. 6.1


using the type curves for infinite conductivity, vertical
fractures.

Solution. A log-log plot of the buildup data was prepared


on tracing paper as presented in Fig. 6.17. A match of this
data with the type curve for a vertical, infinite conductivity
fracture is presented in Fig. 6.18. From the match point,

(AP) M = 100

(PD) hl = 0•. 56

(A t) NI = 100 ^

(tDxf)M - 2.7.

Permeability is computed using Eq. 6.30:

k = 141.2qBu (pD)M
h
EPws-pwf(at=0)]M

k = (141.2) (175) (1.28) (1.42) 0.56


31 100

k = 8.1 md.

The fracture half-length is, according to Eq. 6.31

Xf
=
0.0002637k
^uct
(At) M
tD TM_
z

6-36
• • •

103

• ••

• •
.^{
^ • •

• •

0

4--) 102 •

rn L4-4 •

•®®
V)
• •


10

10-1 1 10 102 103


D tG
At-, hrs

Fig. 6.17 Log-log plot, Ex. 6.2.

0
00

10 102 10 3

At, hrs

Fig. 6.18: Type curve match of data in Ex. 6.2.

• • •
= (0.0002637) (8.1) 100 2
X 2. 7
f [(0 .12) (1.42) (20x10-6)

xf = 152 ft.

These values compare favorably to k = 6.8 md and xf = 174


feet computed using the conventional method.

III. FINITE CONDUCTIVITY, SINGLE PLANE, VERTICAL FRACTURES

It is commonly observed, especially when working with


massive hydraulic fractures, that fracture lengths computed
using the infinite conductivity model are much shorter than
design lengths. This difference often represents several
orders of magnitude. For example, a fracture designed to
be 1000 feet long may be determined from pressure buildup
testing to be 10 feet long. These discrepancies have led
engineers to conclude that many fractures have a smaller
flow capacity than can be adequately described by infinite
capacity solutions. This is easily understood when one
considers that a fracture with finite conductivity must be
significantly longer than an infinite conductivity fracture
to produce an equivalent effect on well test data.
Recognizing the inadequacy of infinite conductivity
solutions for certain applications, researchers i''18 have
developed type,curves for finite capacity fractures. We
will consider first type curves for uniform flux fractures,
which are a special type of finite conductivity fracture.
Finally we will consider type curves which present solutions
as a function of fracture capacity.


6-39
rI,',^ ^ _ ^ dP

1. Uniform flux fractures


The assumption of infinite conductivity implies that
there is no pressure loss in a fracture, i.e., that the
pressure is constant along the fracture. A uniform flux
fracture is one in which pressure does change along the
fracture; however, the pressure change is small, and the
uniform flux condition gives the appearance of a high, but
not infinite, fracture conductivity. The type curve16 for
a uniform flux fracture is presented in Fig. 6.19. This
curve, a plot of PD versus tDxf, does not include the effect
of skin or wellbore storage. Dimensionless pressure, pD,
and dimensionless time, tDxfl were previously defined by
Eqs. 6.23 and 6.24, respectively. It should also be noted
that this type curve represents a solution to pressure
drawdown equations; accordingly, previously described
modifications should be made when the curve is used to
analyze pressure buildup data where the producing time is
short compared to the maximum shut-in time.
Direct comparison of Figs. 6.16 and 6.19 shows that
the pressure behavior of a uniform flux fracture is very
similar to that of an infinite conductivity fracture.
However, data from a high percentage of fractured wells
seem to best match the uniform flux solution. It is not
obvious before analyzing data from a well test which curve
should be used; accordingly,lit is recommended that test
data be compared to both type curves, and that the curve
which provides the best match be used for the analysis _j

2. Finite conductivity fractures


Type curves which describe pressure behavior in
fractures of finite conductivity have been developed by
Agarwal, et alla and Cinco, et all' Figure 6.20 presents
the Agarwal type curve for a vertical, single plane fracture
producing at constant formation face rate. Solutions for
wells producing at constant formation face pressure are also

6-40
• • •

10'2 10'1 1 10 102 109

tDxf

Fig. 6.19: Type curve for a vertical, single plane, uniform flux fracture
(After Ref. 16).
Q

U)
U)

IT
a 1(
N
N
^
a)
r-i

0
.H
U)
9
U)

10-) 10 4 10-j 10-1 10-1 1

Fig . 0 ,?O: Type curve for finite con Of ivity vertical fracture ( After Ref. is 1141

0
available1e. Like the infinite conductivity type curve,
Fig. 6.20 is a plot of dimensionless pressure, pD, versus
dimensionless time, tDxf' however, this solution is
complicated by an additional variable, fracture capacity,
which is defined as

k w
(6.32)
FcD kx
f

where: w = fracture width, ft


kf = fracture permeability, md.

It is observed in Fig. 6.20 that FcD ranges from 0.1 to


500; values greater than 500 approximately represent an
infinite capacity fracture. Significantly, Eq. 6.32 shows
that fracture capacity, kfw, is not the only parameter that
causes FcD to be large; high values of FcD may also be due
^ to low formation permeability or short fracture length.
The procedure required to use Fig. 6.20 is the same as
described for infinite conductivity fractures. However,
problems exist which make use of this type curve difficult.
Practically speaking, it is difficult to obtain a
unique solution for xf and k using Fig. 6.20. First, it is
noted that the curves representing different fracture capaci-
ties do not have distinct shapes; consequently, it is
difficult to obtain a unique match. Second, it is observed
that the curves all have slopes less than one-half; this is
important because, as noted in our previous discussion,
infinite conductivity fractures with skin may exhibit a
similar shape (Fig. 6.11). Accordingly, it is difficult to
differentiate between finite capacity fractures and damaged
infinite capacity fractures; depending on the decision made,
calculated results will differ significantly. Finally, if
weilbore storage effects are present, the analysis is
further complicated.

6-43
The most widespread application of finite capacity
type curves has been in the analysis of data from tight
gas reservoirs which have experienced massive hydraulic
fracturing. Massive hydraulic fractures consistently
'exhibit finite capacity behavior. Type curves, such as
the one presented in Fig. 6.20, are the best tool available
to properly analyze pressure transient data from these
systems. Finite capacity fractures require very specialized
analysis techniques that are beyond the scope of this text.

IV. NATURALLY FRACTURED RESERVOIRS

Natural fractures are caused in brittle rocks by


tectonic stresses within the earth. Accordingly, an almost
infinite variety of naturally fractured systems is possible.
Many theoretical studies 19 32 of naturally fractured systems
have been made but, because of the diversity of these
reservoirs, no single method can be recommended. Unfortu-
nately, very little pressure transient data from naturally
fractured systems have been published.
Pollard20 developed a method to estimate average pressure,
formation damage , fracture volume , and matrix volume for
fractured limestone reservoirs commonly found in Venezuela.
This method has also been used in fractured coal and siltstone
units of Alberta , Canada2° , and in the Asmari reservoirs
of Iran 30. However, in studies by Warren and Root19 and
KazemiZ' , it was concluded that this method can give false
indications of damage, incorrect fracture volume, and if the
method is applied to non-fractured reservoirs, it may
erroneously indicate the reservoir is fractured. Further,
it has been recommended by Earlougher10, that this method
not be used.
Warren and Root19 developed a model based on an
idealized fracture system where the matrix was assumed to
be composed of equal-sized blocks and the space between

6-44
the blocks represented the fractures. The results of this
model were later confirmed by Kazemi27. The pressure
behavior predicted by this model is illustrated by the Horner
plot in Fig. 6.21. Two straight lines of equal slope are
shown to occur. The permeability can be computed from the
slope of either of these lines in the normal manner. Further,
the skin factor and average pressure can be evaluated using
data from the second line. The vertical separation between
the lines is related to the volume of the fracture system.
Quite often the first line is obscured by wellbore storage;
further, the test does not last long enough in many cases to
develop the second line. Accordingly, it is very easy to
misinterpret the pressure response in these situations for
another type of heterogeneity.





• •

SLOPE = m

10 102 103 104 105

t +At
-p
At

Fig. 6.21: Pressure buildup behavior

• predicted by the Warren and Root"


model.

6-45
It was concluded by Odeh22 in a study of reservoirs
with "homogeneous fracturing" that it was not possible to
distinguish between fractured and homogeneous reservoirs.
Further he concluded that a Horner plot could be analyzed
in the same way as for homogeneous reservoirs to obtain
reservoir properties.
Type curves can also be useful in the analysis of
naturally fractured reservoirs. Data from many naturally
fractured systems seem to match the uniform flux type curve
presented in Fig. 6.19, to obtain permeability and fracture
length; the fracture length, however, has only qualitative
meaning since these curves rigorously apply only to vertical,
single plane fractures. Recently developed type curves 12-13
which treat the reservoir as a dual porosity system
potentially offer the most general solution available for
the analysis of data from naturally fractured reservoirs.
In summary, no single method can be applied to naturally
fractured reservoirs. We can generally determine from curve
shape if the data is behaving in a manner predicted by one
of the theoretical models which are available; if so, that
model might be satisfactory for data analysis. Otherwise,
it might be necessary to develop an empirical, numerical or
theoretical model for the subject reservoir.

V. SUMMARY

Fractured reservoirs can be analyzed using pressure


transient tests to determine formation permeability, average
formation pressure, formation damage and fracture length.
The pressure behavior of a fractured well is significantly
different from the radial flow behavior of homogeneous
reservoirs. Consequently, conventional methods of pressure
analysis, based on radial flow theory, cannot be directly
applied to these systems. The pressure behavior of a

6-46
fractured well is dependent upon both the length and
capacity of the fracture involved. An accurate analysis of
a fractured well is only possible if, first, it is recognized
that the formation is fractured and, second, that some
knowledge of fracture capacity is available.
When fracture capacity is large enough to be assumed
infinite, early-time buildup pressures behave linearly and,
in the absence of severe wellbore storage or skin effects,
will plot as a straight line on a square root plot, and as a
half-slope line on a log-log plot. These plots can be
combined with a semilog graph to calculate permeability, skin
and fracture length. This conventional method of fractured
well analysis requires an iterative procedure. Pressure
data can also be analyzed using type curves; this'method has
the advantage of not requiring a trial-and-error calculation,
but it does not yield any quantitative information about
formation damage. The choice of conventional versus type
curve analysis is a matter of personal preference since
both methods give valid results.
Finite capacity fractures behave differently from
infinite capacity fractures. Further, the pressure response
of a well which intersects a finite capacity fracture is
dependent upon the magnitude of fracture capacity. Pressure
tests from wells with finite capacity fractures are best
analyzed using appropriate type curves.
A major difficulty in fractured well analysis is
determining whether to use infinite capacity or finite
capacity solutions to analyze the data. This is particularly
true if the test data are altered by wellbore storage or skin
effects. The behavior of the test data sometimes gives a
clue as to the nature of the fracture; more often, however,
the decision is not clear. Since computed results vary
significantly depending upon the solution method used, it is
concluded that we do not have the ability, with existing
^ technology, to determine unique values of permeability and
fracture length in many reservoirs.

6-47
Naturally fractured reservoirs can be very difficult
to analyze because of the variety of possible fracture
systems and, correspondingly, the variety of pressure
responses possible from these systems. A number of theoretical
models are available to analyze naturally fractured reservoirs;
however, these models are generally only applicable to
reservoirs with characteristics similar to that assumed in
the theoretical development.

6-48
REFERENCES

1. Raghavan, R.: "Modern Well Test Analysis--The Log-


Log Type Curve Approach," Continuing Education Course
No. 9, The Mid-Continent Section of SPE, 1975.

2. Howard, G. C., and Fast, C. R.: Hydraulic Fracturin ,


Monograph Series, Society of Petroleum Engineers op,
Dallas (1970) 2.

3. Gringarten, A. C. and Ramey, H. J., Jr.: "Unsteady-


State Pressure Distributions Created by a Well With
a Single Horizontal Fracture, Partial Penetration, or
Restricted Entry," Trans., AIME (1974) 257, 413.

4. Gringarten, A. C., Ramey, H. J., Jr., and Raghavan,


R.: "Applied Pressure Analysis for Fractured Wells,"
Trans., AIME (1975) 259, 887.

5. Dyes, A. B., Kemp, C. E., and Caudle, B. H.: "Effect


of Fractures on Sweep-out Patterns," Trans., AIME
(1958) 213, 245.

6. Prats, M., Hazebrock, P., and Stickler, W. R." Effect


^ of Vertical Fractures on Reservoir Behavior - Compressible
Fluid Case," Soc. Pet. Eng. J. (June, 1962) 87.

7. Scott, J. 0.: "The Effect of Vertical Fractures on


Transient Pressure Behavior of Wells," J. Pet. Tech.
(Dec., 1963) 1365.

8. Russell, D. G. and Truitt, N. E.: "Transient Pressure


Behavior in Vertically Fractured Reservoirs," Trans.,
AIME (1964) 231, 1159.

9. Raghavan, R., Cady, G. V. and Ramey, H. J., Jr.: "Well


Test Analysis for Vertically Fractured Wells," Trans.,
AIME (1972) 253, 1014.

10. Earlougher, R. C., Jr.: Advances in Well Test Analysis,


Society of Petroleum Engineers of TIME, Dallas ( 1977 ) S.

11. Wattenbarger, R. A. and Ramey, H. J., Jr.: "Well Yest


Interpretation of Vertically Fractured Gas Wells," Trans.,
AIME (1969) 246, 625.

12. Agarwal, R. G.: "A New Method to Account for Producing


Time Effects When Drawdown Type Curves Are Used to
Analyze Pressure Buildup and Other Test Data," paper SPE
9289 presented at the SPE 55th Annual Fall Technical
Conference and Exhibition in Dallas, Sept. 21-24, 1980.

6-49
13. Clark, K. K.: "Transient Pressure Testing of Water
Injection Wells," J. Pet. Tech. (June, 1968) 639.

14. Smith, J. T. and Cobb, W. M.: "Application of


Transient Pressure Analysis To Wells With Hydraulically
Induced Vertical Fractures" Proc., Southwestern
Petroleum Short Course, Lubbo^Texas (April, 1979).

15. Raghavan, R.: "Some Practical Considerations in the


Analysis of Pressure Data," Trans., AIME (1976) 262,
1256.

16. Gringarten, A. C., Ramey, H. J., Jr., and Raghavan, R.:


"Unsteady-State Pressure Distributions Created by a
Well With a Single Infinite-Conductivity Vertical
Fracture," Trans., AIME (1974) 257, 347.

17. Cinco, H. L., Samaniego, F. V. and Dominguez, N. A.:


"Transient Pressure Behavior for a Well With a
Finite-Conductivity Vertical Fracture," Soc. Pet.
Eng. Jour. (Aug., 1978) 253.

18. Agarwal, R. G., Carter, R. D., and Pollock, C. B.:


"Evaluation and Performance Prediction of Low-
Permeability Gas Wells Stimulated by Massive Hydraulic
Fracturing" J. Pet. Tech. (March, 1979) 362; Trans.,
AIME, 265.

19. Warren, J. E. and Root, P. J.: "The Behavior of


Naturally Fractured Reservoirs," Soc. Pet. Eng. Jour.
(Sept. 1963) 245-255; Trans., AIME,22^-

20. Pollard, P.: Evaluation of Acid Treatments From


Pressure Build-Up Analysis," Trans., AIME (1959) 216,
38-43.

21. Pirson, Richard S. and Pirson, Sylvain J.: "An


Extension of the Pollard Analysis Method of Well
Pressure Build-Up and Drawdown Tests," paper SPE 101
presented at the SPE-AIME 36th Annual Fall Meeting,
Dallas, Oct. 8-11, 1961.

22. Odeh, A. S.: "Unsteady-State Behavior of Naturally


Fractured Reservoirs," Soc. Pet. Eng. Jour. (March
1965) 60-64; Trans., AIME, 234.

23. Warren, J. E. and Root, P. J.: "Discussion of Unsteady-


State Behavior of Naturally Fractured Reservoirs,"
Soc. Pet. Eng. Jour. (March 1965) 64-65; Trans., AIME,
234.

24. Morris, Earl E. and Tracy, G. W.: "Determination of


Pore Volume in a Naturally Fractured Reservoir,"
paper SPE 118S presented at the SPE-AIME 40th Annual
Fall Meeting, Denver, Oct. 3-6, 1965.

6-50
25. Huskey, William L. and Crawford, Paul B.: "Performance
of Petroleum Reservoirs Containing Vertical Fractures
in the Matrix," Soc. Pet. Eng. Jour. (June 1967) 221-228;
Trans., AIME, 240.

26. Adams, A. R., Ramey, H. J., Jr., and Burgess, R. J.:


"Gas Well Testing in a Fractured Carbonate Reservoir,"
J. Pet. Tech. (Oct. 1968) 1187-1194; Trans., AIME, 243.

27. Kazemi, H.: "Pressure Transient Analysis of Naturally


Fractured Reservoirs With Uniform Fracture Distribution,
Soc. Pet. Eng. Jour. (Dec. 1969) 451-462; Trans., AIME,
246.

28. Kazemi H., Seth, M. S., and Thomas, G. W.: "The


Interpretation of Interference Tests in Naturally
Fractured Reservoirs With Uniform Fracture Distribution,"
Soc. Pet. Eng. Jour. (Dec. 1969) 463-472; Trans., AIME,

29. Schwartz, F. W.: "Response Testing of Piezometers in


Fractured Porous Media," Can. Geotech. J. (1975) 12, 408.

30. Andresen, K. H., Baker, R. I., and Raoofi, J.: "Develop-


ment of Methods for Analysis of Iranian Asmari Reservoirs",
paper 14, Proc., 6th World Petroleum Congress, 1963, II.

31. Bourdet, N. and Gringarten, A. C.: "Determination of


Fissured Volume and Block Size in Fractured Reservoirs
by Type Curve Analysis", paper SPE 9293 presented at
the SPE 55th Annual Technical Conference and Exhibition,
Dallas, Texas, Sept. 21-24, 1980.

32. Gringarten, A. C., Burgess, T. M., Viturat, D.,


Pelissler, J. and Aubry, M.: "Evaluating Fissured
Formation Geometry from Well Test Data: A Field Example"
paper SPE 10182 presented at the SPE 56th Annual Fall
Technical Conference and Exhibition, San Antonio, Texas,
Oct. 5-7, 1981.

33. Gringarten, A. C.: "Interpretation of Tests in Fissured


Reservoirs and Multilayered Reservoirs with Double Porosity
Behavior: Theory and Practice", paper SPE 10044 presented
at the SPE International Petroleum Exhibition and
Technical Symposium, Bejing, China, March 26-28, 1982.

6-51
• NOMENCLATURE - CHAPTER 6

Af = fracture face area, ft2


B = formation volume factor, RB/STB
ct = total formation compressibility, psi-1
FcD = fracture capacity, dimensionless
correction factor for permeability in fractured
F cor = reservoirs = k/ka, dimensionless

h = net formation thickness, ft


k = volumetric average effective permeability of formation
within the drainage area of the test well, md
ka = apparent permeability from pseudoradial semilog
straight line, md
kf = permeability of fracture, md
m = slope of radial flow semilog straight line, psi/log-cycle
m LF = slope of linear flow straight line on square root
12
plot, psi/hr-
m = slope of pseudoradial semilog straight line, psi/log-cycle
PR
p = pressure, psia
= dimensionless pressure
PD
pDM = dimensionless pressure corresponding to match point
on type curve
p = initial pressure, or stabilized pressure at the beginning
1 of a drawdown test, psia
= intercept pressure on square root plot, psia
pint
= flowing formation face pressure, psia
pwf
pwf(At=0) = flowing formation face pressure at time of shut-in
for a buildup test, psia
= static formation face pressure during buildup test, psia
pws
q = flow rate, STB/D,
s = total skin factor, dimensionless
= skin due to permeability alteration, dimensionless
Sd
sf = skin due to fracture, dimensionless
sp = skin due to perforations, dimensionless
Sr = skin due to restricted entry, dimensionless
is s sw = skin due to slanted well, dimensionless

6-52
st = skin due to turbulence
t = time, hrs
dimensionless time based on fracture length
tDxf =
dimensionless time corresponding to match point
(tD x f)M =
on type curve
tp = producing time prior to buildup test, hrs
w = fracture width, ft
Xe = distance from well to drainage boundary, ft
xf = fracture half-length, ft
pressure change at beginning of pseudoradial straight
Apbsl-
line, psi
pressure change at end of linear flow, psi
Apelf=
Aps = pressure loss due to skin, psi
pressure loss associated with damage skin factor,
(Aps)d =
sd, psi

(Aps)s-sf = pressure loss associated with (s-sf), psi


(Qp)M = pressure change corresponding to match point on
type curve, psi
At = shut-in time, hrs
shut-in time at which pseudoradial straight line
Atbs1=
begins, hrs
shut-in time at which linear flow ends, hrs
Atelf=
(At)M= shut-in time corresponding to match point on type
curve, hrs
u = viscosity, cp
porosity, fraction


b-53
i• SUNWARY OF lNtAJOR EQUATIONS - CHAPTER 6

Equation Number Equation


in Text

u 12
6.12 x f = 4 06 qB
m $ctk
LF h

6.13 F = k
co r k
a

162'^c^
6.15 ka = -
PR

-2'
0.319 mPRaB
6 . 16 X = mLF
f
^cthFcor

I • 6.17 k = k F
a cor

6.27 (AP s)d = pint pwf("t=0)

0 .00708k h [p^^s-pwt (At=0)]


6 . 28
PD = (I B^.

f = 0.0002 637kAt
6.29
tD x
^uct x f2

6.30 k = 141.2qBu pDti9


h [pws pw f (At=0) ] ht

?Z
6.31 x = [0.00026 37k 4t\t
f ¢uct tD
• 6.32
kfw
FcD kx
f

6-54
PROBLEMS

PRESSURE TRANSIENT BEHAVIOR IN FRACTURED RESERVOIRS

1. The discovery well in an oil reservoir was hydrau-


lically fractured during the initial completion. After
flowing for 325 days at an average production rate of
419 STB/D, the well was shut in for a buildup test. The
pressure buildup data`` are presented along with other
reservoir data. You are requested to analyze this data
using conventional methods-to determine the following
information:

(a) Effective permeability, k

(b) Half-fracture length, xf

(c) Total skin factor, s

(d) Skin pressure loss based on total skin


factor

(e) Flow efficiency based on total skin factor

(f) Pressure loss due to formation damage

Initial reservoir pressure, pi, psia ....... 3,770


Drainage area, t1, acres . ................... 1,600
(Not fully developed, the well
is in the center of the
reservoir as is best known)
Wellbore radius, r w, ft . ................... 0.28
Porosity, 1, fraction ... ................... 0.12
Thickness, h, ft ........ ................... 82
Total compressibility, c t, psi-1........... 21x10-6
Viscosity, u, cp ........ ................... 0.65
F orma ti on vo l ume f ac t or, B, RB/STB ......... 1 . 26

6-55
Flow rate, q, STB/D ........................ 419
Producing time, t P , hours .................. 7,800
Flowing pressure at shut-in, pwf(At=0),
psia .................................... 3,420

t + At
At, hrs P At
Pws pwf (At=0) , Psi Pws l psia

0 0 3,420 -
0.0833 11 3,431 9.36 x 104
0.167 15 3,435 4.67 x 104
0.25 18 3,438 3.12 x 10"
0.5 24 3,445 1.56 x 104
0.75 29 3,449 1.04 x 10"
1 32 3,452 7.80 x 10 3
2 43 3,463 3.80 x 103
3 51 3,471 2.60 x 103
4 57 3,477 1.95 x 103
5 62 3,482 1.56 x 103
6 66 3,486 1.30 x 103
7 70 3,490 1.12 x 103
8 75 3,495 9.76 x 102
9 78 3,498 8.68 x 102
10 80 3,500 7.81 x 102
12 86 3,506 6.51 x 102
24 108 3,528 3.26 x 102
36 124 3,544 2.18 x 102
48 135 3,555 1.64 x 102
60 143 3,563 1.31 x 102
72 150 3,570 1.09 x 102
96 162 3,582 8.23 x 10
120 170 3,590 6.60 x 10
144 180 3,600 5.52 x 10
192 190 3,610 4.16 x 10
240 200 3,620 3.35 x 10

6-56
I0
d
Ip+ I

s0T ,,0I E0T 01 0T


.,,.. .r. .!:, : ... r . . . ... . . _ . .. . . ^, ., ,; OOb4.
f i
,
. .
I
, .
71 ,
li. . . . I .. I
.:.

,. , . ^ li I - I ^{,
1.''li Ifr ^ I ! •11 ,.
I 1 I Ill I

I w!
11

1 1
r 1 ^r.ll 11

li OSb2
r l ^1 ^ . ^ .. . . . I 1^ l
..I ^I ^ {•.I . . I ^ . . . . . { i 1 .1 ! 1 r r ` I i
" ! ' ! ' , , i 11 , r
I } I . ^ I

! 'll i ^^.^.^ If . . I I ' . ,} 11.^ I :,1 l i.r I l^ .ll^ lr^ ,l I^ ^Ir


f I 1! ,.

^" . i I . l 11+,

1, 11 ,; ^ ;, 1 1, ,^
1. • ^ 1 .1 . . f•'I »^ ^ ^ . ... .... .. r .. . . . I' .I . 11 ii .I !,II . I.r', { II I. ^I i r
., I• . ^' ,
I{I1 ^^!
f ' l .^I i . ^ . . ..I . . l. . It. ^ I
^

.;
Il i,
I
fit I
.
,
1•! , id !
^: ,^,^
- _-- ::
..,. , . ,.r
:;:
,, .
^, ^; i
r ,;,. .,; ..; ,i ', .
OOS^
,t , ! 1 ; 11 • {^1 1 ,'I1 1 ., , I..i I { , { i;' ^ ^ , .
' . . ...

r-_L-n
I .
^ I I ^ 1 {
. .II ! I ^ !^ 1 1

^
{
^
OSS^
1 1• - I ,^:1 ; { ' ,;.' .
i. , : .. ;1 ., { r r ...... : . rl ^. lr { ^
^ F^•
I ^ 1 r 1 ^
' { I
r 1 ^ w
. 1 ^I I ^ . ^, '. ^1 1 ,^ ^ _ . ^ •I . . , ^ ^^ LI^I^ ^I , . . .
1{ I^ IV , + +; I 1 ^ 'i
:. 1 I 1l ' :; .1
I,.
^ j. II1., ,.r I! ^^L^; ^ii• •^ f! 1 i -'!r ;II i f ! j^, , i,. ".. . ^,, j.,, I ,^! : ^ ^ ^ ^ i Ir. ;.;{,.1 „r ,;, j ^ { r
, I I ; • , ^ ^^ iil ^ 1.
009E
: Ii l ^:{• ^ r!: ,1 ,^^:',i ,11 i I!1 ^ 1 , I I^ 1 I: 1 ^lt ^( ^ 4j^ .^ i

1
! f "f ^ !' ^t t1 f ^ ^f f l ^ ' tI :

l ', I 0S9^
^ ' ll; i; li l 1 ,I i1 I; ^i E. :; ;;r 1'• , .
i 1 I ^ : i, •I If it
^ f 1.
,'r I T ih'IK0 2fd ,, , I''; ^, ( I 1a ^
rrl I

1 u^'
I.
'I II^
i• ^l I
1 i • 1 1 ^
i^I I Lb .I , ^III I, ^I. ^{ l I
'_j , I I ^ + ^,I '1^I.I IIII I. .f ,
1
i ^''

I . II I 11 i
r.

^'t
I It ,I I, ^'jl

if I
I .

I ^ •1 . 1 II . ,! I I ^'i I I
l
, . .. . .. ^ , ^ I1 i,
I" i
OO L£

• • ^
8S-9

r n
m
Ul D y
x A

n3^
n
m
in


D r
0 L

• ->.:,^
9
NO. 340-20 DIETZGEN GRAPH PAPER
20 X 20 PER INCH
DIETZGEN CORPDRATION
raAOe iH U.S.A.

2.. Use type curves to compute the permeability and fracture
length associated with the well described in Prob. 1.
Compare your results to those determined in Prob. 1 using
conventional methods.

3. The following pressure drawdown data were obtained from


a well with a finite conductivity fracture. Using the
Agarwal finite conductivity type curve, compute the
fracture length, xf, and the fracture conductivity, kfw.

= 30% h = 30 ft
ct = 20 x 10-6 psi u = 0.85 cp
B o = 1.65 RB/STB qo = 250 STB/D
rw = 0.25 ft k = 7.8 md (prefrac test)

t, hrs Psi
pi pwf,

0.25 S7
0.50 68
1.0 79
2.5 106
5.0 134
10.0 168
20.0 210
30.0 238
40.0 261
50.0 280
60.0 298
70.0 311
80.0 321
90.0 334
100.0 343
150.0 384


6-60
^{ - - leZ.6 l3

mPR

B ^IZ
V'"pIL
Ck y+ FCC,,


0 PROBLEM PTA6-1: HYDRAULICALLY FRACTURED OIL
WELL, HIGH CONDUCTIVITY

Pressure buildup data from a hydraulically fractured oil well are


presented in file PTA6-1.PAN. Analyze this test for permeability and
fracture length using both type curve and conventional methods. Verify
your results with Quick Match.

0
s • •

PTA6-1: Lo g -Lo g Plot Q uick Match

ick Match Results


ertfcal fracture - Infinite oor4uctridy
In acting
Constant compressiblliy
Ca = 0.0246 bbUpsi
K = 6.9764 md
100 5f = 0.037
= 154.203 It
pi = 3764.6091 psia

r-.

[1.

ed

10
'Oe

loll,

0.1 1 • 10 100
Equivalent Time (hours) - Tp=7800.0
3650 PTA6-1: Semi-Lo Plot Qu ick Match

3600

,---, 3550
Quick Match Results
Vertical fracture - infinite condu ctivity
Infinitely acting
Constant compressibility
Cs = 0.0246 bbUpsi
3500 K = 6 . 9764 md
Sf = 0.037
Xf = 154.203 ft
IN = 3764.6091 psia

3450

3400 1
10 10A 1000 10000 3650
Homer Time Function - Tp=7800.0

0 0 0
• • •

3650 PTA6-1: S q uare Root Plot Quick Match

3600

-3550
y
C^. uick Match Results
Vertical fracture - infinite oonduaf '
Infinitely acting
Constant compressibility
Cs = 0.0246 bbupsi
3500 K = 6_9764 md
Sf = 0_037
f = 154.203 ft
Pi = 3764.6091 Psia

3450

3aog0
75 80 85 9
Tandem Square-root Function - T=7800.0
PROBLEM PTA6-3: PRESSURE DRAWDOWN TEST,
0 HYDRAULICALLY FRACTURED WELL,
FINITE CONDUCTIVITY

File PTA6-3.PAN contains the pressure drawdown data from a post-frac


test. Analysis of a pre-frac test yielded a permeability of 7.8 md. Using the
pre-frac permeability as an aid in your analysis, determine the length and
conductivity of the fracture.

0
0 • ^1

PTA6-3: Cinco-Ley Type Curve


Match Results
ertial fracture - finite condu&jvity
'
nfinkeN acting
K z 7.8 md
a 166.5329 ft
1000 Fcd -5
IdW 6494.7842 md.R
pr ^ 5.5511

'5100

10

0.1 1 10 100 1000


Elapsed Time hours
• • • I

PTA6-3: Lo -Lo P lot Quick Matc h

y 100
L1.

Quick Match Resifts


Mfcal fracttre - finite candudi
InWdy actinp
CwStwt OwPvSS"wy
s =0 6bUpsf
K -7.a md
5f -0
-106.533 ft
Fcd =5
Pi = 3644 psFa

10 100
Elapsed Time (hours)
_ • • •

3600 PTA6-3: Semi-l og Plot Quick Match

3525

^-,3450
4
.^
.g
^
H
^
q
N
t-.
3375

Quick Match Results


Vertical fnKture - We conducOvity
InfiMtely acting
3300 C ar"Aart comPressibft
Cs -0 bbUps!
K -7.8 md
Sr w0
Xf a 166.533 ft
Fcd c5
Pi - 3644 psia
^ 0. 1
35 1 10 1 00 1 00
Elap sed Time hours
• • • .

PTA6-3: Wong-Harrington Type Curve


1000
Match Resufts
eRical fracture - We corAxtylty
Infinitely acting
K -7.1634 md
f 6173.8747 ft
Fcd = 6.2632
IdW = 7d25.9741 md.ft
-5.6464

g100

10

0.1 1 10 100 1000


Elapsed Time hours
BILINEAR FLOW - LIQUID RESERVOIRS


I. FLOW PERIODS FOR A VERTICALLY FRACTURED WELL

Figure 1 depicts the various flow periods which are


associated with finite conductivity vertical fractures.

A. Fracture Linear Flow


This is the first flow period which occurs in a fractured
system. Most of the fluid which enters the welibore during
this period of time is a result of expansion within the
fracture, i.e., there is negligible fluid coming from the
formation. Flow within the fracture during this time period
is linear.
Equations which can be used to predict the flowing
formation face pressure, pwf, during fracture linear flow
are presented by Cinco, et all, for the constant rate case.
^ This reference also presents an equation which predicts the
time when this flow period ends. Unfortunately, fracture
linear flow occurs at a time which is to early to be of
practical use in well test analysis.

B. Bilinear Flow
This flow period is called bilinear flow because two
types of linear flow simultaneously occur. One flow is
linear incompressible flow within the fracture and the other
is linear compressible flow in the formation. Most of the
fluid which enters the wellbore during this flow period comes
from the formation. Fracture tip effects do not affect well
behavior during bilinear flow; accordingly, unless a well
test is run sufficiently long for bilinear flow to end, it
will not be possible to determine fracture length from the
data.


LBF- 1
Well

I I i We I I^ ^.IJt
Fracture

(a) (b)
FRACTURE LINEAR FLOW BILINEAR FLOW

\ ( /
Fracture
1 Fracture

Well

/ I \
(C)
(d)
FORMATION LINEAR F L 0 W
PSEUDO-RADIAL FLOW

Fig. 1: Flow periods for a vertically fractured well (Ref. 1)

• • ^
Bilinear flow was first recognized by Cinco, et all
Since its introduction into the literature, the use of
bilinear flow analysis to characterize both formation and
fracture properties has been documcnted2 6 The details of
analyzing bilinear flow data will be detailed in subsequent
discussions.

C. Formation Linear Flow


The analysis of formation linear flow is covered in
the course manual.

D. Pseudoradial Flow
The analysis of pseudoradial flow is covered in the
course manual.

II. BILINEAR FLOW EQUATIONS

0 A. Constant Formation Face Rate

Dimensionless Pressure:

kh(pi pwf)
PD 141.2qBu (1)

Dimensionless Time:

_ 0.0002637kt
tDxf ( 2 J

^uctxf
r

Dimensionless Fracture Conductivity:

kfw
FCD kxf (3)


LBF- 3
Bilinear Flow Equation:

_ 2.45 ', (4)



PD F2 tDx f
CD

_ _ 44.1aBU (5)
t4
pi pwf h(k fw) Z(^Uctk) °

Bilinear Slope (graph of pi pwf versus t°):

44.1qBu (6)
mbf 1i (kFw) ^ (^uctk) 4

B. Constant Formation Face Pressure

Dimensionless Rate:

_ 141.2 BU (7)
•i
aD
kh pi pwf

Bilinear Flow Equation:

1 _ 2.72 4 (8)
aD F CD 11 tDxf

1 _ 48.9BU
t4 (9)
q
(pi Pwf)h(kfw)!z(^uctk)'

Bilinear Slope (graph of q versus t

48.9Bu
,Z (10)
m bf = -
(pi pwf) h (kfw) (wtk)
E
LI3F -4
NOTE: The equations presented in this section are
^ written specifically for pressure drawdown
-rests. These equations can be modified for
pressure buildup tests by replacing the
pressure differential Ap = pi-pwf, and the
producing time, t, with appropriate values
as shown in the following table:

Test Differential Time

Drawdown Ap = pi-p«f t

Buildup At or At e
Qp - 1'ws l) wf

III. BILINEAR FLOW GRAPHS

A. Constant Formation Face Rate


When the rate of a well is maintained constant, the
pressure change at the Formation face is described by Eq. S.
This equation indicates that a plot of pi pwf (pws-pwf for
^ ,
buildup tests) versus t' (At i4 for buildup tests) will yield
a straight line with slope, mbf, predicted by Eq. 6. This
plot is illustrated by Fig. 2. When bilinear flow ends,
the straight line will end and the plot will exhibit curva-
ture which is concave upward or downward depending upon the
value of the dimensionless fracture conductivity, 1' CD'
When FCD < 1.6, the curve will be concave downward; a value
of FCD > 1.6 will cause the curve to be concave upward.
When FCD > 1.6, bilinear flow ends because the fracture
tip begins to affect wellbore behavior. If a pressure
transient test is not run sufficiently long for bilinear
flow to end when FCD > 1.6, it is not possible to determine
the length of the fracture. When FCU < 1.6, bilinear flow

L13F-S
^
ICD > 1.66
^
^
/

HV)
I/
i F('D < 1.6
SLOPE = mbf

END OF
BILINEAR FLOW

1.
1-
t ° , lirs `'

Fig. 2: Bilinear flow graph for a constant


rate well.

^j
in the reservoir changes from predominatly one-dimensional
(linear) to a two-dimensional flow regime. In this case, it
is not possible to uniquely dctcrmine fracture length even
if bilinear flow does end during the test.
A more diagnostic plot to recognize the occurrence
of bilinear flow is the log-log plot. From lid. 5,

44 .
log (Pi Pwr) = log ^ lqB+ 1 4 log t (11^
h_( kfw) (^uctk)
[

Equation 11 indicates that a log-log plot of pi-pw£ versus


t will yield a straight line with a one-fourth slope; this
is illustrated by Fig. 3.

LBF -6
• SLOPE

H
V)

t, hrs

Fig. 3. Log-log plot showing effect of ideal


bilinear flow for the constant rate
case.

1]
B. Constant Formation Face Pressure
When formation face pressure remains constant, the
formation face rate will change with time as described by
Eq. 9. According to Eq. 9, a plot'of 1/q versus t4 should
yield a straight line with slope, mbf, defined by Eq. 10;
this graph is depicted by Fig. 4. Following the end of
the bilinear flow period, the curve for F CD < 2.8 will be
concave downward and the curve for F CD > 2.8 will be concave
upward. The straight line caused by bilinear flow ends for
the same reasons as described for the constant rate case.
Equation 9 also indicates that a log-log plot of 1/q
versus t should yield a straight line with a slope of one-
fourth:

log (1) = log 48 .9BU/


^+ 1 log t (1 2)
4

• [TP iPwfhkfwctk ^
q )
I ]

L13P- 7
^Cll > 2. 5

/
/
^
SLOPE = F CD < Z. 8
mb f

--I Ic+

END OF
BILINEAR FLOW

.
t 4

Fig. 4: Bilinear flow graph for a constant


pressure well.

This plot, illustrated by Fig. 5, is the primary diagnostic



tool by which bilinear flow can be recognized.

SLOPE = 1

Fig. 5; Log-log plot illustrating the effect of


ideal bilinear flow for the constant

pressure case.

LBF-8
IV. END OF BILINEAR FLOW

A. Constant Formation Face Rate


The relationship between (tDxf)ebf and FCD is depicted
graphically by Fig. 61 This relationship can be approximated
as:

0.1
F CD ' 3' (t Dxf) ebf 2 (13)
FCD

0.0205 (FCD - 1.5)-1-53 (14)


1.6 < FCD < 3: (t Dxf)ebf -

FCD < 1.6: (tDxf)ebf - (FCDS - 2.5)-4 (15)

For the case where FCD > 3, the dimensionless pressure


at the end of bilinear flow is

1. 3
^ (pD)ebf F CD$ (16)

Therefore,

1.38
FCD = ^ (17)
pD

and,

194.9 Bu
(15^
F CD
kh pi-pwf ebf
.

B. Constant Formation Face Pressure


The relationship between (tDxf)ebf and FCD is
presented graphically by Fig. 74 This relationship can be
approximated by the following equations:

• LBF-9
11 A
10-1
I .i
_z
10
^
a^
r-,
w
X
Q
^ 103

_ 4
10

10 1 1
10- 1 101 102

FCD
•i
Fig. 6: Dimensionless time for the end
of the bilinear flow period
versus dimensionless fracture
conductivity, constant rate
case (Ref. 1).

x 0'Z
FCD > S: ( tDxf)ebf _ 6.94 2 10-' (19)
F CD

2 < FCD < 5: See Fig. 7

0.5 < FCD < 2: (tDxf)ebf = 1•58 x 10'3 FCD1-6 (20)

For the case where FCD > 5,


LI3F- 10
1 _ 1.40 (21)
r I^c,r)
^ a ^Ch
^

Therefore,

FCD = 1 . 40 (ClD)ebf (22)

and,

197.7clebfB^'
(23)
rCD kh
p i-pwf

V. ANALYSIS OF BILINEAR FLOW DATA

The conventional analysis of bilinear flow data requires


two plots - a log-log plot of the appropriate rate or pressure
function versus t, and a cartesian plot of the appropriate
^ rate or pressure function versus t°.

A. Liquid-Constant Rate
The following procedure can be used to analyze bilinear
flow data for fracture conductivity and fracture length.
When rate is constant:

1) Make a log-log plot of (pi-pwf) versus t.


2) Determine if any data fall on a straight
line of quarter slope.
3) If there are data with a q uarter slope in
Step 2, plot pi-p`^f versus t4 on cartesian
paper and ide^itify those d ata wh ich form
the bilinear flow straight line.
4) Determine the slope, mbf, of the bilinear
flow straight line.
5) Using the slope, m, from Step 4, compute
th e fracture con du^fivit y, k f w , usi ng E q. 6:

L1iP-11

CONSTANT PRESSURE
PRODUCTION

F,Z=5
^
14-4
10

F RACTURE
TIP ^ •

^Q4 t RESERVOIR
WELL
Xf

6 51 1h 1 I 11111 1 11 1 11,111 1 1 1 1 1 1111

lo'' I z.s 10 1oz


FCD

Fig. 7: Dimensionless time to the end of


bilinear flow for constant pressure
production (Ref. 4).


LBr-12
2
k [_44.1ciB '4 )
t mbCh(Ouctk) °

It should be noted that this calculation


can only be made if k is known from a
prefrac test.
6) If the bilinear flow straight line ends
and the data rise above the straight line,
determine the value of Ap, i.e., Apebf, at
which the line ends. Then, from Eq. 18,
FCD can be computed as

_ 194.9 Bu (18)
1^h
FCD pl pwf)ebf

With FCD known, the fracture length can be


computed using Eq. 3:

kfw

• Xf kFCD
(25)

It should be noted that Eq. 18 assumes


FCD > 3. If enough data are available
beyond bilinear flow, a type curve
match should be attempted to verify
that this is true.

B. Liquid-Constant Pressure
When formation face pressure remains constant during a
test, the following procedure can be used to analyze the
bilinear flow data for fracture conductivity and fracture
length:

1) Make a log-log plot of 1/q versus t.


2) Determine if any data fall on a
straight line of quarter slope.
3) If any data form a quarter slope in
Step 2, plot 1/cl versus t4 on
cartesian paper and determine the
11 slope, mbf, of the bilinear flow

LBF-13
straight line.
4) Using the slope, In from Step 37,
compute the fracture conductivity,
kfw, using Eq. 10:

48.9BU . (26)
k w =
f Lmbfjwf)ht¼

5) If the bilinear flow line ends and


the data rise above the straight
line, determine the value of q where
the line ends, i.e., qebf' Then,
from Eq. 23, F CD can be computed as

197.7qebfBu
r-_ _ (23)
CD kh pi pwf

With FCD known, the fracture length


can be computed using Eq. 25:

k w
Xf = ^ . (25)
CD

Equation 23 assumes FCD > 5; accordingly,


if enough data are available beyond
bilinear flow, a type curve match should
be attempted to verify that this is true.

VI. EFFECT OF FLOW RESTRICTIONS

When a flow restriction exists in the formation adjacent


to the fracture, or when a r ^striction occurs in the fracture
near the wellbore, the ideal bilinear flow behavior discussed
previously will be altered. Ideal bilinear flow results in a
straight line on a cartesian plot of 4p (constant rate) or

LBF-14
^ 1/q (constant pressure) versus t; further, this line passes
through the origin. Bilinear flow still exists when a flow
restriction is present; however, the restriction causes
an extra pressure drop, aps, in the system. This additional
pressure loss does not alter the slope, mbf, of the bilinear
flow straight line; instead, rather than passing through the
origin, the line will have an intercept equal to Aps for the
constant rate case. This behavior is depicted by Fig. 8.

DAMAGE OR
^I
Ul
CHOKED FRACTURE


IDEAL
AP
s

0
0
t°, hrs°

Fig. 8: Effect of a flow restriction on


bilinear flow, constant rate case.

A log-log plot of Ap (constant rate) or l/q (constant


pressure) versus t will exhibit a straight line with quarter
slope for ideal bilinear flow. The slope of this line will
be altered, however, when a flow restriction is present.
This behavior is depicted by Fig. 9 for the constant rate
case.

LBF-15

DAMAGE OR
.. ^, CHOKED FRACTURE

f r

SLOPE = 1
4

t, hrs

Fig. 9: Effect of a flow restriction on the


log-log plot for the constant rate
case.

VII. EFFECT OF WELLBORE STORAGE



6Vellbore storage will alter or completely mask the
bilinear flow straight lines ideally expected on the
cartesian and log-log plots of Ap or 1/q versus time.
Figure 10 depicts the effect of storage on a plot of Ap
versus t° for the constant rate case. The corresponding
effect of storage on the log-log plot is shown in Fig. 11.
It has been reported by Cinco, et all, that the end of
wellbore storage effects occurs approximately three log
cycles after the end of the unit slope line.


LBF-1G

.H
IDEAL BILINEAR
FI.06V

^
^ EFFECT OF
/ WELLBORE STORAGE
/

,. 1.

tQ, hrs

Fig. 10: Effect of wellbore storage on a plot of


op versus 01 for the constant rate case.

SLOPE = 1
^! 4

UNIT SLOPE ^
.^,

3 LOG CYCLES

t, hrs

• Fig. 11: Effect of wellbore storage on the


log-log plot for the constant rate
case.

LBF-17
REFERENCES

clIlcU , It , and ;D11C:L1]11ego , F . : '" Ff ^1I1S1CIlt 1 ' 1'CSSUI ' C:

Analysis for Fractured Wells," J. Pet. Tech. (Scpt.,


1981) 1749-1766.

Cinco, H., and Samaniego, F.: "Transient Pressure


Analysis: Finite Conductivity Fracture Case Versus
Damaged Fracture Case," SPE 10179, presented at the
56th Annual Fall Technical Conference and Exhibition
of the Society of Petroleum Engineers, held in San
Antonio, Texas, October S-7, 1981.

Cinco, H.: "Evaluation of Hydraulic Fracturing by


Transient Pressure Analysis Methods," SPE 10043,
presented at the International Petroleum Exhibition
and Technical Symposium of the Society of Petroleum
Engineers, held in Bejing, China, March 18-26, 1982.

Bennett, C. 0., Reynolds, A. C., and Raghavan, R.:


"Performance of Finite Conductivity Vertically
Fractured Wells in Single-Layer Reservoirs," SPE
11029, presented at the 57th Annual Fall Technical
Conference and Exhibition of Society of Petroleum
Engineers, held in New Orleans, Louisiana, Sept.
26-29, 1982.

Guppy, K. H., Cinco, H., and Ramey, H. J., Jr.:


"Pressure Buildup Analysis of Fractured Wells
Producing at High Flow Rates," J. Pet. Tech. (Nov.,
1982) 2656-2666. ^

l:odriguez, F., Horne, R. N. and Cinco, H.: "Partially


Penetrating Vertical Fractures: Pressure Transient
Behavior of a Finite Conductivity Fracture," SPE 130S7,
presented at the 59th Annual Technical Conference and
Exhibition held in Houston, Texas, Sept. 16-19, 1984.


LBF-18
Transient Pressure Analysis
for Fractured Wells
Heber Cinco-Ley,' SPE, Stanford U.
Fernando Samaniego-V.,' SPE, Inst. Mexicano del Petr6leo

Summary
A new technique is presented for analyzing pressure the dimensionless fracture conductivity
transient data for wells intercepted by a finite- ( k fb f/kx f) >_ 300; all other cases, such as those
conductivity vertical fracture. This method is based represented by long or poorly conductive fractures,
on the bilinear flow theory, which considers transient must be analyzed by considering a finite-conductivity
linear flow in both fracture and formation. It is fracture model.
demonstrated that a graph of p,t, f vs. t produces a Exploitation of low-permeability gas reservoirs has
straight line whose slope is inversely proportional to required stimulation of wells by massive hydraulic
h f( kfb f) ,A. New type curves are presented that fracturing (MHF) techniques. Vertical fractures of
^ overcome the uniqueness problem exhibited by other large horizontal extension are created as a result of
type curves. this operation; consequently, pressure drop along the
fracture cannot be neglected.
Introduction Several papers25-3^ have been published on the
A large amount of information concerning well test behavior of finite-conductivity vertical fractures.
analysis has appeared in the literature over the last Type-curve matching has been proposed as an
three decades. As a result of developments in this analysis method under these conditions; however,
area, three monographs 1,2,3 and one book4 have some regions of the curves present a uniqueness
been published covering different aspects of pressure problem in the analysis. Barker and Ramey31 in-
transient analysis. Ramey5 also has presented a dicated that the use of published type curves becomes
review on the state of the art. practical when a large span of pressure data is
The analysis of pressure data for fractured wells available.
has deserved special attention because of the number The purpose of this work is to present a new in-
of wells that have been stimulated by hydraulic terpretation technique for early-time pressure data
fracturing techniques. A summary of the work done for a well intercepted by a finite-conductivity vertical
on flow toward fractured wells' was presented by fracture, including the criteria to determine the end
Raghavan6 in 1977. of wellbore storage effects. In addition, new type
It was recognized early that intercepting fractures curves are discussed to overcome the uniqueness
can strongly affect the transient flow behavior of a problem exhibited by previous curves at intermediate
we117-9 and that, consequently, the application of and large time values.
classical methodslo-1z to the analysis of transient
pressure data in this situation may produce erroneous Transient Pressure Behavior
results. Several methods 13-24 were proposed to solve for Fractured Wells
this problem. Consider a vertically fractured well producing at a
These analysis techniques consider a well in- constant flow rate, q, in an infinite, isotropic,
tersected by either an infinite-conductivity vertical homogeneous, horizontal reservoir that contains a
fracture or a uniform-flux vertical fracture. Cinco- slightly compressible fluid of constant com-
Ley et al. 25 demonstrated that the assumption of pressibility c, and viscosity A. The porous medium
nfin.ite fracture conductivity is valid whenever has a permeability k, porosity g5, thickness h, and
0-
' Now with Petr6leos Mexicanos and U. Nacional de Mexico. initial pressure pi.
0149•2138/8110009-7490S00.25 Let us assume that the well is intercepted by an
Copyright 1981 Society of Petroleum Engineers of AIME undeformable, fully penetrating vertical fracture of
SEPTEMBER 1981 1749
TABLE 1 - SI PREFERRED UNITS, CUSTOMARY UNITS, half-length x f, width bf, permeability k f, porosity
AND UNIT CONVERSION CONSTANTS
USED IN THESE SYSTEMS
Of, and total compressibility cfr (Fig. 1). The


properties of both the reservoir and the fracture are
Parameter independent of pressure and the flow in the entire
or Variable SI Preferred Units Customary Units system obeys Darcy's law. In addition, it is con-
k µm2 md venient to assume that pressure gradients are small,
h m ft gravity effects are negligible, and flow entering the
q0 m31d STB/D
m31d Mscf/D
wellbore comes only through the fracture.
q9
µ Pa•s cp With these assumptions, the unsteady-state flow in
B m3 /m3 RBiSTB the system can be described by the diffusivity
0 fraction fraction equation.' For details on the boundary conditions
cr Pa-1 psi-'
kPa psi
and a semianalytical solution for this flow problem,
m(p) kPa2/Pa•s psi2/cp see Ref. 25.
t hours hours The general solution for the wellbore flowing
CYQ 1,842 141.2 pressure pwf for oil is given by25
ag 1,293 1,424
3.6x10-9 2.637x10-4
bb^slo 34.97 44.1 kh(pi-pw.j)
bbr9 24.57 444.75 =pD (1D ,r)fD,C^f) . . . . . . (1)
0.3918 8.128 aoqBµ I
ailo
61f 0.275 81.97 and, for gas,
K R
C m3/Pa cu ft/psi
kh[m(p;)-m(pf)] -
a q7. pwD (tDf ,?1lD,C1Dl),
8
....................(2)

where
Qkt
1Dxf = 0µctx2f (3 )

rifD = k k

and
cl
. .....................
. . . . (4)
0
bO 1 . ................ . . . . . . (5)
CjDl ^rx

pwD represents the dimensionless pressure drop; it is


a function of dimensionless time, tDxf , dimen-
sionless fracture hydraulic diffusivity, ,t fD, and di-
mensionless fracture storage capacity, C fDf ao, ag
and a are unit conversion constants (see Table 1).
Cinco-Ley et a1.25 showed that for practical values
of dimensionless time the pressure behavior depends
on two parameters only: the dimensionless time tDx ,
Fig. 1 - Finite-conductivity vertical fracture in an infinite- and the dimensionless fracture conductivity
slab reservoir.
(kb) D. The former was defined in Eq. 3 and the
latter may be obtained from Eqs. 4 and 5. By
10Z
definition, the dimensionless fracture conductivity is

D E
1 C (kfbf)D= ^ .....................(6)
1 n^- a
CASE I g ^ I
P and it appears to be related to CfDf and rlfD as
ND m\kihiJ()
, follows.
16Z a Z 500
A G 1H
CASE II (kfb f) D = 7rCfDf'qjp . . . . . . . . . . . . . . . . . . . (7)
F
164 L
Fig. 2 shows the general behavior of a well with a
1010 108 106 ,0 idZ .n2
finite-conductivity vertical fracture. There, we show
tDxf a log-log graph of dimensionless wellbore pressure,
pwD, vs. dimensionless time, tDx . For the sake of
Fig. 2- Low-log graph of typical cases for fractured wells. simplicity only two cases are 4resented. Case 1
represents the behavior of a low-conductivity
1750 JOURNAL OF PETROLEUM TECHNOLOGY
Well

Well if I t t
Fracture

`°) (b)
FRACTURE LINEAR FLrJW BILINEAR FLOW

Fracture
Fracture

Well

.f ^

(= ) /^
(d)
I
FORMATION LINEAR FLOW
PSEUDO-RADIAL FLOW

• Fig. 3- Flow periods for a vertically fractured well.

fracture (k fb f) p=0.1; Case 2 considers a highly Fracture Linear Flow Period


conductive fracture, (kb) D = 500. These cases This behavior occurs at very small values of dimen-
were selected because they exhibit all the features of sionless time, and.it is exhibited by all cases. During
the transient pressure behavior for a fractured well. this flow period, most of the fluid entering the well-
An analysis of Fig. 2 shows that the transient bore comes from the expansion of the system within
behavior of a well with a finite-conductivity vertical the fracture and the flow is essentially linear, as
fracture includes several flow periods. Initially, there shown in Fig. 3a.
is a fracture linear flow characterized by a half-slope The dimensionless pressure response at the
straight line; after a transition flow period, the wellbore is given by*
system may or may not exhibit a bilinear flow period,
indicated by a one-fourth-slope straight line. As time PwD (kfbf)D `17r?7fDtpXf .
increases, a formation linear flow period might
............ (8)
develop. Eventually, in all cases, the system reaches a Hence, for oil,
pseudoradial flow period (see Fig. 3).
Points A, F, and L in Fig. 2 represent the end of _ B^oqB At
pW pt . . . . . . . . . . . . '(9)
the fracture linear flow period (half-slope straight f bfh kfOjcrr
line). The bilinear flow period (one-quarter-slope and for gas,
straight line) is defined by Segments B-C and G-H;
this behavior is not present when the fracture has a S^ ^T r
m(pwf) =m(pi)
high storage capacity and a high conductivity (lower
I ^Jlofpcr
curve in Case 2). where So and SVg are unit conversion constants.
The formation linear flow period is shown by the
These equations indicate that a log-log graph of
half-slope straight line between Points I and J, and it
pressure vs. time yields a straight line whose slope is
is only exhibited by fractures of high conductivity,
equal to one half. A graph of pressure or pseudo-
(k fb f) D >_ 300. Points D and K show the beginning pressure vs. the square root of time also gives a
^ of the pseudoradial flow period.
straight line whose slope depends on the fracture
We present a detailed description of both the characteristics excluding the fracture half-length x f.
fractured linear flow and the bilinear flow. The The fracture linear flow ends when
formation linear flow and the pseudoradial flow have
been discussed in the literature. 14,15,17,19,22,25 'See Appendix A for derivation.

SEPTEMBER 1981 1751


10-1
0
10-2

t Debf
3
10 3

(kf bi) p = o.in


2 o.zn
End of Straight
pw D Line _0,5 rr" l 10 4

i - ' 5¶Soo

L
10-5
0
0 n.7 na nr (z& i 10-1 101 102
4
Xi (kI bi) D
Fig. 4-pWo vs. tpY for a well with a finite-conductivity Fig. 5-Dimensionless time for the end of the bilinear flow
vertical fraciure. period vs. dimensionless fracture conductivity.


t 0.01(kfbf)D. lower curve corresponding to a high value of CjDf
^^D (11) does not exhibit the bilinear flow behavior because
Dx f `
fracture tip effects are felt before this flow regime is
Unfortunately, this flow period occurs at a time too established (Segment L-I).
early to be of practical use. The dimensionless welibore pressure for the
bilinear flow period is given by'
Bilinear Flow Period
ir
To the best of our knowledge, this is a new type of pwD t ^Dj
x ^ . . . . . . . . . (12)
flow behavior that has not been considered in the I'(1.25) 2(kfbj)D
literature. It is called bilinear flow because two linear or
flows occur simultaneously. One flow is a linear 2.45
incompressible flow within the fracture and the other pwlv= ( k^t^Dxf . . . . . . . . . . . . . . . . ( 13 )
is a linear compressible flow in the formation, as
shown in Fig. 3b. A bilinear flow exists, as shown in This equation indicates that a graph of PwD vs.
Appendix A, whenever most of the fluid entering the D
° t produces a straight line whose slope is
wellbore comes from the formation and fracture tip 2.45/ (k , intercepting the origin. Fig. 4
effects have not yet affected the well behavior. presents that type of graph for different values of
Let us now examine this behavior in a log-log (kfbJ)D•
graph of pw vs. tDx (Fig. 2). In Case I An important feature of this graph is that after the
[(k fb f} D = O. lg, the biliiear flow exists between bilinear flow period (straight-line portion), the curves
Points B and C after a transition flow period for (krbf) D!51.6 are concave downward and the
represented by Segment A-B. The pressure behavior curves for (k bf ) D > 1 6 are concave upward.
for the bilinear flow exhibits a straight line whose The end ofthe straight-line portion of the curves
slope is equal to one fourth. The duration of this depends on the fracture conductivity and may be
period depends on both ( k fb f) D and C fD f. Case 2
[( k fb f) D = 500] may or may not exhibit the bilinear
flow period, as shown by the upper and lower curves.
The upper curve for Case 2 corresponds to a low
expressed by
t'bf=
0.1
(kfbf) D
for ( kfbf)D>_3, . . . . . . (14a) •
value for CfDf and does exhibit bilinear flow over a
short period of time (Segment G-H); however, the 'See Appendix A for derival ion.

1752 JOURNAL OF PETROLEUM TECHNOLOGY



Q
a

CL
a
rn
0
J

Slope= 14

\ _t Log t

Fig. 6-Graph for analysis of pressure data of bilinear Fig. 7- Log-log graph of pressure data for bilinear flow.
flow.

and, for gas,


tDebJ-0.0205[(kfbf)D -1.5] -1.53
• for 1.6 <- ( k fb f ) D :5 3, . . . . . . . . . . . (14b) Am (P)
h(kf by
abjgqT
(O^.ctk)
,, ^ . . . . . . . . (16)
and
where Ap is the pressure (pseudopressure) change for
4.55 -a a given test. Both Sbfo and bb g are unit conversion
tD,bf=[ -2.5 ] constants and are given in Table 1.
Eqs. 15 and 16 indicate that the pressure change is
both inversely proportional to the square root of the
for (kfbf) D :5 1.6 . . . . . . . . . . . . . . . (14c) fracture conductivity and directly proportional to the
fourth root of time.
According to Eqs. 15 and 16, a graph of ap vs. VT
Fig. 5 shows a graphical representation of these
produces a straight line passing through the origin,
equations.
whose slope, m bf , for oil, is given by

Bilinear Flow Analysis _ SbloqBl^ . . . . . . . . . . . . (17)


Bilinear flow is exhibited by finite-conductivity mbf h(kfbf) T(OIc74 ,
fractures with a small dimensionless storage capacity,
and, for gas,
CfD f. Any attempt to analyze pressure data observed
over this flow period using conventional methods (p
_ qT
vs. NFI-or p vs. log t) will produce erroneous results. . . . . .. . .. . . . (18)
We present the appropriate analysis method based on mbf h(kJbJ) ^g(Olictk)"" '
the bilinear flow theory. Hence, the product h (k fbJ) V' can be estimated by
using the following equations. For oil,
Basic Equations and Graphs
8b.ro qBµ ............(19)
From Eq. 13, the pressure drop for oil may be ex-
pressed as h(kfbf) mbf(O^rk)`'
and, for gas,
LI bblo4Btc
_ ^ .......... (15) --- . . . (20)
AP h(kfbJ)' (Oucrk)'/+ h(kfbf) V2 mbf(^k4 k)'• .
SEPTEMBER 1981 1753

AP

mbf AP

tevt'S

A PS
ZBE^WIOR 1iEUBORE

4^
^ t_ ^

Fig. 8-Bilinear-flow graph for a fracture with a flow Fig. 9- Wellbore storage effect on the bilinear flow graph.
restriction near the wellbore.

These equations indicate that values of reservoir entire thickness of the formation. This is possible
properties must be available to estimate the group simply by using the fracture height, hf, instead of the
h(kfb f) 2 . Note that this analysis technique tends to
decrease the effect of the error introduced when poor
information on reservoir properties (i.e., k, 0, and
cr) is used.
formation thickness, h.
In cases where a flow restriction (low conductivity,
turbulent flow) exists within the fracture in the
vicinity of the wellbore; bilinear flow still occurs and

All comments on the concavity of the curve in Fig. the pressure data analysis discussed can be applied
4 are valid for the curve in Fig. 6. (see Fig. 8). An extra pressure drop, Ops, is created in
From Eqs. 13 and 14, if ( k fb f) p z 3, the this case and the straight-line portion does not in-
dimensionless pressure drop at the end of the bilinear tercept the origin. These situations distort the
flow period is given by straight-line portion in the log-log graph.
If wellbore storage affects the system, the bilinear
1.38 flow pressure behavior may be masked, as shown in
(pwD)e$f = (kjbf)D . . . .. . . . . . . . . . .. . . (21) Fig. 9, and data analysis becomes difficult (if not
impossible) with current interpretation methods.
Hence, the dimensionless fracture conductivity can Flow Regime Identification
be estimated using the following equation.
and New Type Curves
_
The pressure behavior of a fractured well may exhibit
1.38
k b . ................ ( 22 ) several flow periods for practical values of time:
( f f) D (pwD)ebf bilinear flow, formation linear flow, and
(pwD ) ebf can be calculated using Eq. 1 or Eq. 2 and pseudoradial flow.
(Ap) ebf or Am( P) ebr obtained from the bilinear Pressure data for each flow period should be
flow graph. analyzed using a specific interpretation method (i.e.,
From Eqs. 15 and 16, a graph of log ap vs. log t pw vs. V- 4 pw vs. ft, and pw vs. log t for bilinear,
yields a quarter-slope straight line (Fig. 7) that can be linear, and pseudoradial flows, respectively).
used as a diagnostic tool for bilinear flow detection. The log-log graph has been used commonly as a
diagnostic tool to detect different flow regimes in a
Extensions and Limitations transient pressure test. The use of type curves in the
The region disturbed during bilinear flow includes
only the fracture and its vicinities because it occurs at
early time, even in partially penetrating fracture
systems. Thus, the equations and graphs discussed in
analysis of pressure data for fractured wells
represents a major step in that area. The first type
curves for fractured wells were resented by
Gringarten et. a!19 Cinco-Ley et al. ZF showed that

the previous section for bilinear flow can be extended the infinite-conductivity vertical fracture solution of
to cases where the fracture does not penetrate the Gringarten and Ramey17 behaves like the solution
1754 JOURNAL OF PETROLEUM TECHNOLOGY
for a finite-conductivity vertical fracture of
( k fb f) D >_ 300; furthermore, they showed that the (k r
unlform-flux vertical fracture behaves as a variable- 0 10°1

• conductivity vertical fracture. The type-curve


analysis offers an advantage over the specific analysis
methods already mentioned because it can be applied
.O
Y t02

Q
slope' 2
^

to interpret at once pressure test data that correspond slope= 4 ti


to different flow periods. In addition, the type-curve
InPACiludiS ST44T
analysis could indicate when the different graphical 9r SEMI-lQ6 STMAIGMT
tIMt
techniques apply.
Experience has shown that, in some cases, ap-
plication of the type curves available for finite-
tpX (kfbi)p
conductivity vertical fractures does not yield unique f
results. This is because the shape of pressure
Fig. 10 -Type curve for vertically fractured wells.
behavior curves is similar for different values of
dimensionless fracture conductivity over some
regions of the type curves. A close inspection of Fig. or
2 indicates that the uniqueness problem can exist if
pressure data to be analyzed occur at either the tel .
(kfbf) D = 1.25 x 10 -2 ..........(26)
bilinear flow period (one-fourth slope) or the tb/f
pseudoradial flow period.
These equations apply when (k fb f) D > 100.
A convenient presentation of the type curve
Fig. 10 can be used as a type curve to analyze
published by Cinco-Ley et al., 25 Ramey et al. , 31 and
pressure data for a fractured well. Pressure data on a
Agarwal et a1.29 is given in Fig. 10. We show a graph
graph of log [1p vs. log t is matched on a type curve to
of log [pwD(kfbf)DI vs. log [tDx (kfbf) D].` The
determine (Ap)M, [PwD(kfbf)D]M, ( t)M, [tDx t
main feature of this graph is that^ for all values of
(k fb ) D the behavior of both bilinear flow (quarter ' (kfbf) ^]M, [(kfbf)D]M, ( tebf)M, ( tblf)M, anCl
slopej and the formation linear flow (half slope) is ( t bss/ ) M. From this information, we can estimate the
following.
given by a single curve. Note that there is a transition
period between the bilinear and linear flows. The
Dimensionless Fracture Conductivity.
dashed line in this figure indicates the approximate

• start of the pseudoradial flow period (semilog


straight line). Also shown are the end of the bilinear
flow and the start of the formation linear flow; the
[(kfbf)D] M
Formation Permeability. For oil,
time for the end of the bilinear flow from Fig. 10
agrees with the results presented in Fig. 5. The groups
of variables used in Fig. 10 were derived in Appendix __ ao9BlA
[PwD' (kfbf)D]M . . . . .
B, where it is shown that, for some values of fracture . . . (27)
k° h(Ap)M [(kfbf)D]M
conductivity, bilinear flow ends when fracture tip
and, for gas,
effects are felt at the wellbore.
Although many curves are presented for
(k fb f) D > 207r, the shape of these lines is essentially _ «XqT ^wD' (kfbf)D]M .. .. . (28)
the same. The only difference is the duration of the kg h[Am(P)]M [(kfbf)D]M
formation linear flow (half slope); that is, the higher
the fracture conductivity, the longer the linear flow
Fracture Half-Length.
period.
The beginning of the formation linear flow occurs
at tDxf (kfb f) D_ 102; that is,
ak(t)M [(krbf)D]Nt ^
(29)
100 Xf __ Oµct 1t^f ' (k ff f DM
tDb1f- (kfbf) D . (2
(23)
3)

The end of this flow period is given by19 Fracture Conductivity.

kfbf=kxf[ (kfbf) D]M . . . . . . . . . . . . . . . . . ( 30)


t Delf = 0.016 . . . . . . . . . . . . . . . . . . . . . . . . . . (24) End of Bilinear Flow.
Hence, the fracture conductivity may be estimated as
follows: (tebf)M

• (kfbf)D
10
t^ . ................... (25)
Beginning of Formation Linear Flow.

'A large-scale copy of this graph may be obtained from the authors. (tbtf)M

SEPTEMBER 1981 1755


102 Beginning of Semilog Straight Line.
End of wellbcre
^ 5toraqe Effects
(tbss!)M
ry (' 101 1 1/d
These results can be obtained if a large span of
pressure data is available. It should be kept in mind
^;
that specific analysis graphs must be used for dif-
ferent flow regimes to obtain a better estimate of
2
4 1
both fracture and reservoir parameters.
Now we discuss cases where all pressure data fall
10-1
on a very small portion of the type curve and a
10-1 1 101 102 103 1od
complete set of.information may not be obtained.
(kfbf )D '/'
F:(tDxf1 Y1tOxf
(C )
Df
Case 1. Pressure data exhibit one-fourth slope on a
Fig. 11 -Type curve for wellbore storage under bilinear log-log graph (Fig. 7). According to the discussion
flow conditions.
in the previous section, these data correspond to the
bilinear flow period and a unique match with Fig. 10
cannot be attained. The bilinear flow type of analysis
is the only method available for this case to obtain
information regarding the fracture characteristics
(kt bflp ^c>>
10d (kfbf).
lDn.
A minimum value for fracture half-length, xf, can
`!? 10. be estimated from Eqs. 14a through 14c for the end
102
Cl of the bilinear flow; that is, for ( k fby ) D z 3, we
,D have
0 a 10(3(kfb f) 2teb^
a3 1 X
---- Approximate stazt . (31)
^^b Log
9
ofineemilog
L
S Straight fy Ogcfk
1o-z Generally, wellbore storage affects a test at early
10-4 10-2 10Z lOd 106 108 time; thus, it is expected to have pressure data
z distorted by this effect, causing deviation from the
t pXt ( kf bt)p
one-fourth slope characteristic of this flow period.
Fig. 12-Type-curve matching for data in bilinear and Fig. 11 may be applied to analyze pressure data for
transitional flow. this case even if the duration of the test is not enough
to reach the one-fourth slope portion. It is important
to note that pressure behavior in Fig. 11 for both
wellbore-storage-dominated and bilinear-flow
portions is given by a single curve that completely
eliminates the uniqueness matching problem. The
correlating parameters Fi (pwD ) and F2 ( tDX f) used
in this figure are derived in Appendix C and defined
in Fig. 11.
The end of wellbore storage effects in Fig. 11
a occurs when F2 ( tDX f) -2x 10 ; that yields
0
J
17.25 3 ^
t ews 32
(kjbJ) 2h4^clk. . . . . . . . . . .
From observation of the results presented in Fig. 11,
we can see that the end of wellbore storage effects
occurs three log cycles after the end of the unit slope.
Log t
This criterion is useful to determine whether the
Fig. 13-Pressure data for a half-slope straight line in a proper straight-line portion for bilinear flow analysis
log-log graph. is chosen (see Fig. 9). If Fig. 11 is used as a type
curve, the following information may be obtained:
[F1 (pwD)]M, [F2(tDxf )IM, (Ap)1y, and (t)M.
Hence, we can estimate the following.

Wellbore Storage Constant. For oil,

C_ 27rao#4Bju(t)M [F1 (pwD)]M


. . . . (33)
(dp)M [F2(tDzf)I M

1756
JOURNAL OF PETROLEUM TECHNOLOGY
r'
q w
Xf

10-1

10 2
102 103
10- 1 1 l01

(kf bf ) D
Fig. 14- Effective welitaore radius vs. dimensionless fracture conductivity for a vertical fracture.

and, for gas, and, for gas,

• C_ 27ragQ9T(t)M [F'1 (PwD)IM ^


[6in (P)] M JF2(tDxf)IM
. . . . . . . (34) ( ^',L)
agqT
xf = h[Am(P)lM
[PwD(kf$f)D,M, .(38)

Fracture half-length and fracture conductivity. For


Fracture Conductivity. For oil, oil or gas,
_ 0.4 C ^ao9'Bµ[Fi (PwDAt 3
kfbf Oct k (aP)M kf bf
O(t)M I MI . . (39)
xf xf 4^t4ctk[tDxf(kfbf) D1M'
................ ........ (35)
and, for gas,
and

_ 0.4 C agqTlFi ( pwD)JM^3


kfbf 2 ^1 Ocrk [ AM(P)IM kfbf= ( xf) ( k} . . . . . . . . . . . . . . . . . . . . (40)
...................... (36)
Since the formation permeability generally is
Case 2. Pressure data partially match the curve for known from prefracture tests, the dimensionless
the transition period between bilinear and linear fracture conductivity can be estimated by using
flows (Fig. 12). In this case, the remaining portion results from Eq. 37 or Eq. 38. Some of these results
of the data may correspond to bilinear or linear flow also can be obtained from a specific analysis method
and the type-curve match is unique because the corresponding to the flow period exhibited by data
transition period has a characteristic shape. This other than the transition flow region (i.e., bilinear
comment is valid for dimensionless fracture con- flow or linear formation flow, as discussed in the
ductivities, (k fb f) p >_ 57r. bilinear flow analysis section and in Ref. 14).
From the type-curve match of pressure data for If all pressure data fall on the transition period of
this case in Fig. 10, we obtain [px,D(kfbf)DJM• the curve, type-curve matching (Fig. 10) is the only

• [tDa f(kfbf) o] M, (At) M, and (ap) M.


Hence, for oil,
kf bf
a° qBµ LpwD(kfbf)D,M^
h(AP)M .. .. ( 37 )
analysis method available.

Case 3. Pressure data exhibit a half-slope line on a


log-log graph. (Fig. 13). For this case there is no
C xf ^ unique match with Fig. 10; however, the linear flow

SEPTEMBER 1981 1757


lal different values of fracture conductivity are similarly
Q Olbf)p .__ tnd [ euinur rio..
shaped for the pseudoradial flow period. However,


WD .I. the transient pressure behavior shown in Fig. 10 can
0.

Bpannlnq / Srtloq
be correlated to analyze these cases better.
Sttslghc Lim
For the pseudoradial flow period, a fractured well
End ot Linear fta+
behaves like an unfractured well with an effective
10-1 wellbore radius being a function of dimensionless
5+

11:
fracture conductivity, (k fbI ) D. Fig. 14 presents a
100.
graph of dimensionless effective wellbore radius,
10-2 rw/xf, vs. dimensionless fracture conductivity,
^.. 10'3 10-2 10•1 I 101 102 103
(kfbf)D. Notice that for large values of
IDr'
W (kfbf)D(>300), the dimensionless effective
we1[bore radius is 0.5, as mentioned by Prats et al. 8
Fig. 15-Type curve for a finite-conductivity vertical
fracture. If the dimensionless time is defined by using ru,
instead of xf, a graph of pwD vs. ID,' provides a
single curve for the pseudoradial flow period for all
analysis presented by Clark14 can be applied to values of dimensionless fracture conductivity (see
obtain fracture half-length if formation permeability Fig. 15). This curve provides an excellent tool for
is known. In addition, a minimum value for the type-curve analysis of pressure data partially falling
dimensionless fracture conductivity, (kJb f) D, can in the pseudoradial flow period because the
be estimated using Eq. 26. Note that tbtf in Fig. 13 remaining data must follow one of the curves for
represents a maximum value for the time of the different fracture conductivities. Fig. 14 must be
beginning of the linear flow period and tef represents used as an auxiliary curve to determine (kfbf)D
a minimum value for the time of the endfof the half
when using Fig. 15.
slope.
Application of Fig. 15 to match pressure data
If wellbore storage effects are present at early provides ( pwD)M, (1D, ) M, (^)M. (t) M, and
times in a test for this case, the analysis can be made [(kfbf)D],y. Hence, the following equations are
using the t lype curve presented by Ramey and given.
Gringarten.2
Reservoir Permeability. For oil,
Case 4. Pressure data partially match the curve for
the pseudoradial flow period. If a large span of
pressure data is not available, a unique match would
not be obtained by using Fig. 10 because curves for k= h(AP)1y (pwD)M' . . .
. . . . . . . . . . . . . (41) •
l02
Match Point I

0 -
[et]r ^ 1 hnur (kfbf)p > S. {itfb f l D
[-,mlp1jH=]oPsl2 1
P
]C^ 0.35 hour
tsbi

[Py,U(kibipyI kfbtlp I° 3 .69 x 10-^ ZO>


Q ^ Itd. 2.5 hour °j
r M tebt
es ]0.
Q 10
U

f'!` --- --- ^- ---^-- _ r ^ - - ^ - - 2.

"
Q
I

[ ai,'nest,F
E
< 1
1a8 -- _ __ =x---^ --_I-_ ---t-- -- t
Beginning of 5ec+ilog Straight Line ^ t
0
^ 1 t t
0 t ^ I t
( t
[p 10-1 t


io7 i- - 11 3 --1--102---i 10-1 -^ -- ^ - 1Q1--
l0-2 10-1 tpx (kt bt)p 101 l02 At Ihoursl

Fig. 16-Type-curve matching for Well A.

1758 JOURNAL OF PETROLEUM TECHNOLOGY


and, for gas, TABLE 2-TEST INFORMATION AND ANALYSIS RESULTS
FOR WELL A

_ agqT Reservoir Data


(42) Production rate q, MscflD 7,350
k h[^(p)]Nr (pD)M. Producing time tp, hours 2,640
Formation thickness h, ft 118
Porosity o, fraction 0.1
Effective Wellbore Radius. For oil or gas, Permeability k, md 0.025
Formation temperature T, °R 690
Average gas viscosity µ, cp 0.0252
r' = ak ( t ) `u (43 ) Total compressibility cr, psi -^ 0.129 x 10-3
Flowing weilbore pressure p,rt, psia 1,320
w 01Ct(tDr 'W )M
Analysis Results Type-Curve Bilinear
By using [(kfbf) D]M in Fig. 14, we obtain Analysis Flow Analysis
( rw /x f) Fig. 14; hence, kfbr, md•ft 148 154
xr, ft 373 368
.
^ ktbt
r" ......... . (44) 1, md-ft/ft 0.3978 0.4185
xf [rw/xf]Fig.14 . ........ xt J
(klbt)D 15.9 -16.71
The pressure data falling in the pseudoradial flow
period also must be analyzed using the semilog r„„ ft 171.6 -169.3
methods to estimate k and rw .
The discussion in this section clearly indicates (as
mentioned by Agarwal et a1.29) that caution and
diligence should be exercised when applying the type-
curve matching technique. To perform the type of From the time-match information and Eq. 39, we can
analysis presented in the four cases, it is necessary to calculate
have prefracture information.
It should be kept in mind that application of the xf=0.3978
type-curve analysis method to pressure buildup tests
is appropriate when the producing time is large.
2.637x10- XI

• Examples of Application
Three examples illustrate the application of several of
the methods and theory previously discussed.
0.1 x0.025x 1.29x 10-4 x0.025x3.69x 10-2

=373 ft.
Well A Now, application of Eq. 40 yields
A buildup test was conducted in this fractured well
producing in a low-permeability reservoir. Table 2
presents the information and results of the analysis kfb f =0.3978 x 373 = 148 md-ft.
for this test. Fig. 16 shows a log-log graph of pressure
data matching the type curve given in Fig. 10. Notice We also can estimate
that the first data points are influenced by wellbore
storage and the rest of the data fall in both the (kb ) ^ ^
bilinear and the transition flow periods. The match- ffD- Xf k
point results also are presented in Fig. 16. A
minimum value for ( k fb f) D can be estimated from
_ 0.3978
the position of the last data point with respect to the =15.9.
type curves; for this case, ( k fb f) D min - 5v. 0.025
The end of wellbore storage occurs at ap-
proximately 0.35 hour and the end of bilinear flow is From Fig. 14, r,,/x1= 0.46.
at 2.5 hours. We also see that the formation linear
flow period was not reached in this test. r;,, = 373 x 0.46 =171.6 ft.
Since the test was not long enough to match a
curve for a specific value of (kfbf)D, this example
corresponds to Case 2 in the type-curve-analysis Fi^ 17 shows the bilinear flow graph [An (p) vs.
section. At ` ] for this example. Based on the information
Using the pressure data match and Eq. 38, we provided by Fig. 16, the correct straight line is
obtain drawn. The slope of this line is 1.62 x 108 psi2/cp-

• ^ 1,424x7,350x690x6.5
Xf 118X 10
hr'^' , and at the end of bilinear flow
Am (P) ebf'= 2•05 x 10g psi2/cp-hrl/' . Notice that the
pressure curve after the end of the bilinear flow
period is concave upward, indicating that
= 0.3978 md-ft/ft. (kfbf)D>1.6.

SEPTEMBER 1981 1759


Io

u B
0
C' lYrv° eoneaw Yp+a[Aa - IktE,10 .1.0 °° 109
0
6 °
n
U
°
°0°

?
E 4 °°0P
4 ;ylso,.r p°o .°^ . 61
_v 10a
E
a 5 iope =
a ^ ^^ cna o( ylln.ar rlw

`^N.llbo[a 6toraq•

0 1 2 4 10"-
10-2 lo-I 1 101 102

/41h°Ur114) Al Ihoursl

Fig. 17 - Bilinear flow graph for Well A. Fig. 18 - Log-log graph of pressure data for Well B.

TABLE 3-TEST INFORMATION AND ANALYSIS RESULTS Hence,


FOR WELL B

Reservoir Data
Production rate q, Mscf/D 1,675 xf k(klbf)D
Producing time tp, hours 1,800
Formation thickness h, ft 85 154
Porosity 0, fractIon 0.11 368 ft.
Permeability k, md 0.0045 0.025 x 16.71
Formation temperature T, °R
Average gas viscosity µ, cp
Total compressibility ct, psi -^
Flowing wellbore pressurep,f, psla
675
0.025
0.152 x 10-3
1,250
From Fig. 14, rti,, /xf =0.46.

r;,, = 368 x 0.46 = 169.3 ft.



Analysis Results Type-Curve Bilinear
Analysis Flow Analysis We see that the results obtained by using both the
krbr, md-ft 95.3 bilinear flow and the type-curve methods are ap-
xf, ft - z692 proximately the same; this fact increases confidence
in the analysis performed.
( kXbl ), md-ftlft - 50.1377
Well B
(kf br) p - 30.6 A buildup test was run after fracturing this gas well.
Information about the test and results of analysis are
rw, ft - s325
presented in Table 3. A log-log graph of pressure
data (Fig. 18) indicates that the test was completely
dominated by bilinear flow (quarter slope),
corresponding this example to Case I in the type-
From Eq. 20, curve-analysis section.
The bilinear flow graph (Fig. 19) yields a straight
(k1bJ)^' ' =(444.75x7,350x690)/t 1.62x 108 x 118 line whose slope, mbf, can be used to calculate
( k fb f) using Eq. 20:

(0.1 x 0.0252 x 1.29 x 10 - 4 x 0.025)) 4


(k fbf)'^ = (444.75 x 1,675 x 675)/[0.92 x 108
.'. (k fb f) =154 md-ft.
x85x(0.11 x0.025x 1.52x 10-4
Using Eqs. 22 and 2,
x0.0045)I41,

(k f )
1.38
f f D _ ^ /0.025x118x2.05x108\
l 1,424 x 7,350 x 690 J
or

kfbf = 95.3 md-ft.



=16.71. If we assume that the last data point corresponds to
1760 JOURNAL OF PETROLEUM TECHNOLOGY
0 n
2

101 0eglnning ot Scn::cg


Vl , '
[1 Straight Line
103^ --^ ^ ---1
0 1— - -
Ikfbt)0 n3cc
I I 1 (al. • 100 psi

a FD1M . 0.76
pw D
E I Idt7M ^ 1 hr

I Iepi,1M . 0.19
10'1
LIIILI
i! ! --I
10"1 1 10 1 0T 103
L t IhOursl
10-Z
2 10'2
0 I 3
1^4 1/d ID r'
^t^ ,
(6t) (hour I w

Fig. 19 - Bilinear flow graph for Well B. Fig. 20-Type-curve matching for Well C.

the end of the bilinear flow period and (k fb f) p? 3, TABLE 4-TEST INFORMATION AND ANALYSIS RESULTS
then from Eq. 31, FOR WELL C

Reservoir Data
4 lO x 2.637 x]0-4 x(95.3)2 X18 Production rate q, STBID 220
xf 2: 0.11 x0.025 x 1,52x 10-4 x0.0045 Producing time tP, hours 1,890
Formation thickness h, ft 49
= 692 ft, Porosity k, fraction 0.15
Viscosity µ, cp 0.8

• and

y<95.3
Total compressibility ci, psi -^
Formation volume factor Bo, bbi1STB
Wellbore radius r,„ ft
Flowing wellbore pressure p„,i , psia
17.6 x 10 -6
1.2
0.25
1,704
= 0.137? md-ft/ft.
xf 692 . Analysis Results Type-Curve Semilog
Analysis Analysis
From Eq. 6, k, md 2.07 2.28
k< 95.3 (kt)bt)0 27r -
(kfbf)Dkxf ` xf, ft 88.7 -
0.0045x692
ktbr, md-ft 1,156 -
rw, ft 36.89 30.37
.'.(kfbf)DS30.6. S -4.99 -4.8

From Fig. 14, rw/xf_<0.47; hence, rw :!:-:0.47x692,


rw :5325 ft.
Well C
After a flowing time of 1,890 hours, a buildup test Using information from the time match in Eq. 43,
was run on this fractured oil well. Information for
the test and analysis results are presented in Table 4. 2.637x10-4x2.07x1
Fig. 20 shows a log-log graph of the pressure data;
rw 0.15x0.8x17.6x10-6 x0.19
from this graph we can see that neither a one-fourth
slope nor a half slope is exhibited by the data. This
figure also shows that pressure data match the curve = 36.9 ft.
for (k fb^ ) p= 27r given in Fig. 15 and the last six
points fall on the semilog straight line. From Fig. 14, r;y/xf=0:415; hence,
From the pressure match obtained in Fig. 20 and 36.9
Eq. 41, we estimate xf 415 =88.9ft.

• k=
141.2x220x 1.2 x0.8 x0.34
49x 100
0.
The skin factor is estimated by
r 0.25
s=1nr" =1n36 9 = -4.99.
=2.07 md. w

SEPTEMBER 1981 1761


Conclusions
Based on the material presented in this work, the


6
following remarks are pertinent.
S 1. The transient flow behavior of a vertically
fractured well may exhibit four flow periods: (a)
a 4 fracture linear flow, (b) bilinear flow, (c) formation
/:307Ps./cYcIe
linear flow, and (d) pseudoradial flow. Bilinear flow
'- 3 is a new type of flow that has not been considered
a
before.
2 2. A new technique is presented to analyze data in

e ^
the bilinear flow period. It is shown that, during this
1 /si flow period, a graph of p x, f[or m (p,yf ) ] vs. V_Tyields
Ihr
/ a straight line whose slope is inversely proportional
0
io-l ,ni ,«Z ,r to h f(k fb f) "'
At 1hoursl 3. New type curves are now available for pressure
analysis of fractured wells. The uniqueness problem
Fig. 21 - Semilog graph for Well C. in the analysis is reduced considerably with the use of
these type curves.
4. Prefracture information about the reservoir is
necessary to estimate fracture parameters.
5. The type-curve analysis method must be used
simultaneously with the specific analysis methods
The fracture conductivity is (p,ti,f vs. ^rt_, pµ,f vs. V_t, and pH,f vs. log t) to produce
reliable results.
kfbf= (kfbf)Dkxr
Acknowledgments
= 2,7r x 2.07 x 88.9 = 1, 156.2 md-ft. Part of this work was develo,ped at Stanford U. and
the Inst. Mexicano del Petroleo. We are grateful to
Fig. 21 is a semilog graph for this example. The many people, especially H.J. Ramey Jr. for his
correct semilog straight line has a slope m = 307 encouragement and helpful comments.
psi/cycle and ( Ap ) 1 hr =- 47 psi. The formation
permeability can be calculated as
_ 162.6 qBµ
Nomenclature
by = fracture width
B = formation volume factor

k mh
c = compressibility
_ 162.6x220x 1.2x0.8 C = wellbore storage coefficient
= 2•28 md. FI , F2 = correlating parameters for wellbore
307 x 49
storage
The skin factor is
h = formation thickness
hJ = fracture height
s =1.151 [ (AP) 1h, _ log ( ^µc r 2 ) + 3.2275]
r W k = permeability
k fbJ = fracture conductivity
47 (kfbf)D = dimensionless fracture conductivity
=1.151 m= slope of semilog straight line, gas
307
pseudopressure
2.28 mbf = slope of straight line for bilinear flow
- log
0.15 x0.8 x 17.6x 10-6 x(0.25)2 p = pressure
q = well flow rate
+3.2275J = -4.8. r,,, = wellbore radius
rw = effective wellbore radius
s = skin factor or Laplace space variable
Finally, the effective wellbore radius is t = time
r;W =rwe-'s =0.25 e4.8 At = shut-in time
T = reservoir temperature
= 30.37 ft. x,y = space coordinates
xf = fracture half-length
The results provided by both the type-curve a,,6 = unit conversion constants r-^
analysis and semilog analysis methods are in good
r = gamma function
agreement. From these examples it is demonstrated
that type-curve analysis, when applied properly, 71 = hydraulic diffusivity
provides an excellent diagnostic tool and a technique ,0 = porosity
to estimate both reservoir and fracture parameters. µ = fluid viscosity

1762 JOURNAL OF PETROLEUM TECHNOLOGY


Subscripts 20. Wattenbarger, R.A. and Ramey, H.J. Jr.: "Well Test In-
terpretations of Vertically Fractured Gas Wells," J. Pet.
b = beginning Tech. (May 1969) 625-632; Trans., AIME, 246.
bf = bilinear flow 21. Ramey, H.J. Jr. and Gringarten, A.C.: "Effect of High-

• g = gas
D = dimensionless
e = end
Volume Vertical Fractures on Geothermal Steam Well
Behavior," Proc., Second United Nations Symposium on the
Use and Development of Geothermal Energy, San Francisco,
May 20-29, 1975.
!;^z
^^'

22. Cinco, H., Ramey, H.J. Jr., and Miller, F.: "Unsteady-State
f = fracture, flowing Pressure Distribution Created by a Well With an Inclined
i = initial Fracture," paper SPE 5591 presented at the SPE 50th Annual
If = linear flow Technical Conference and Exhibition, Dallas, Sept. 30-Oct. 3,
1975.
o = oil
23. Raghavan, R. and Hadinoto, N.: "Analysis of Pressure Data
I = total for Fractured Wells: The Constant-Pressure Outer Boun-
xf = based on xf dary," Soc. Pet. Eng. J. (April 1978) 139-150; Trans., AIME,
w = wellbore 265.
24. Raghavan, R., Uraiet, A., and Thomas, G.W.: "Vertical
Fracture Height: Effect on Transient Flow Behavior," Soc.
References Pet. Eng. J. (Aug. 1978) 265-277.
1. Matthews, C.S. and Russell, D.G.: Pressure Buildup and 25. Cinco, H., Samaniego, F., and Dominguez, N.: "Transient
Flow Tests in Wells, Monograph Series, SPE, Dallas (1967) 1. Pressure Behavior for a Well with a Finite-Conductivity
2. Ramey, H.J. Jr., Kumar, A., and Gulati, M.S.: Gas Well Test Vertical Fracture," Soc. Pet. Eng. J. (Aug. 1978) 253-264.
Analysis Under Water-Drive Conditions, AGA, Arlington, 26. Holditch, S.A. and Morse, R.A.: "The Effects of Non-Darcy
VA (1973). Flow on the Behavior of Hydraulically Fractured Wells," J.
3. Earlougher, R.C. Jr.: Advances in Well Test Analysis, Pet. Tech. (Oct. 1976) 1169-1178.
Monograph Series, SPE, Dallas (1977) 5. 27. Ramey, H.J. Jr., Barker, B., Arihara, N., Mao, M.L., and
4. Theory and Practice of Testing of Gas Wells, third edition, Marques, J.K.: "Pressure Transient Testing of Hydraulically
Energy Resources Conservation Board, Calgary, Alta. (1975). Fractured Wells," paper presented at American Society
5. Ramey, H.J. Jr.: "Practical Use of Modern Well Test Topical Meeting, Golden, CO, April 12-14, 1977.
Analysis," paper SPE 5878 presented at the SPE 46th (Annual 28. Cinco, H. and Samaniego, F.: "Effect of Wellbore Storage
California Regional Meeting, Long Beach, April 8-9, 1976. and Damage on the Transient Pressure Behavior of Vertically
6. Raghavan, R.: "Pressure Behavior of Wells Intercepting Fractured Wells," paper SPE 6752 presented at the SPE 52nd
Fractures," Proc., Invitational Well-Testing Symposium, Annual Technical Conference and Exhibition, Denver, Oct. 9-
Berkeley, CA, Oct, 19-21, 1977. 12, 1977.
7. Dyes, A.B., Kemp, C.E., and Caudle, B.H.: "Effect of 29. Agarwal, R.G., Carter, R.D., and Pollock, C,B.:
Fractures on Sweep-Out Patterns," Trans., AIME (1958) 213, "Evaluation and Prediction of Performance of Low-
245, Permeability Gas Wells Stimulated by Massive Hydraulic
8. Prats, M., Hazebroek, P., and Stickler, W.R.: "Effect of Fracturing," J. Pet. Tech. (March 1979) 362-372; Trans.,

q Vertical Fractures on Reservoir Behavior - Compressible-


Fluid Case," Soc. Pet. Eng. J. (June 1962) 87-94; Trans.,
AIME, 267.
30. Barker, B.: "Transient Flow to Finite-Conductivity Vertical
AIME, 225. Fractures," PhD dissertation, Stanford U., Palo Alto, CA
9. Scott, J.O.: "The Effect of Vertical Fractures on Transient 1977.
Pressure Behavior of Wells," J. Pet. Tech. (Dec. 1963) 1365- 31. Barker, B. J. and Ramey, H. J. Jr.: "Transient Flow to Finite-
1369; Trans., AIME, 228. Conductivity Vertical Fractures," paper SPE 7489 presented
10. Horner, D.R.: "Pressure Build-Up in Wells," Proc., Third at the SPE 53rd Annual Technical Conference and Exhibition,
World Pet. Cong., The Hague (1951) Sec. II, 503-523. Houston, Oct. 1-3, 1978.
11. Miller, C.C., Dyes, A.V., and Hutchinson, C.A. Jr.: 32. Scott, J.O.: "A New Method for Determining Flow
"Estimation of Permeability and Reservoir Pressure from Characteristics of Fractured Wells - Application to Gas Wells
Bottom-Hole Pressure Build-Up Characteristics," Trans., in Tight Formations," paper 78-T-2, AGA 1978 Transmission
Al ME (1950) 189, 91-104. " Conference, Montreal, T-279-186.
12. Matthews, C.S., Brons, F., and Hazebroek, P.: "A Method
for Determination of Average Pressure in a Bounded
Reservoir," Trans., AIME (1954) 201, 182-191. APPENDIX A
13. Russell, D.G. and Truitt, N.E.: "Transient Pressure Behavior Short-Time Transient Pressure
in Vertically Fractured Reservoirs," J. Pet. Tech. (Oct. 19(4)
1159-1170; Trans., Al ME, 231. Behavior for a Well With
14. Clark, K.K.: "Transient Pressure Testing of Fractured Water a Finite-Conductivity Fracture
Injection Wells," J. Pet. Tech. (June 1968) 639-643; Trans.,
AIME, 243. Derivation of Solution
15. Millheim, K.K. and Cichowicz, L.: "Testing and Analyzing
Low-Permeability Fractured Gas Wells," J. Pet. Tech. (Feb. Let us consider the system described in the text of this
1968) 193-198; Trans., AIME, 243. paper. A fractured well produces at a constant flow
16. Raghavan, R., Cady, G.V., and Ramey, H.J. Jr.: "Well-Test rate from an infinite reservoir. At small time values,
Analysis for Vertically Fractured Wells," J. Pet. Tech. (Aug. the pressure behavior of the system is not affected by
1972) 1014-1020; Trans., AIME, 253.
17. Gringarten, A.C. and Ramey, H.J. Jr.: "Unsteady-State
the tips of the fracture. In addition, the flow in the
Pressure Distributions Created by a Well With a Single In- formation is essentially linear and perpendicular to
finite-Conductivity Vertical Fracture, Soc. Pet. Eng. J. (Aug. the fracture plane. The reservoir and the fracture
1974) 347-360; Trans., AIME, 257. may be treated as two different homogeneous
18. Gringarten, A.C. and Ramey, H.J. Jr.: "Unsteady-State regions.25
Pressure Distributions Created by a Well With a Single

• Horizontal Fracture, Partial Penetration or Restricted En-


try," Soc. Pet. Eng. J. (Aug. 1974) 413-426; Trans., AIME,
257.
19. Gringarten, A.C., Ramey, H.J. Jr_, and Raghavan, R.:
If the flow within the fracture is assumed to be
linear, the pressure behavior can be described by
a2pfD 2 apD I
"Applied Pressure Analysis for Fractured Wells," J. Pet. axD2 + (kfbf)D aYD yD=0
Tech. (July 1975) 887-892; Trans., AIME, 259.

SEPTEMBER 1981 1763


1 aPJD The two partial differential equations are already
................ (A-1)
rtJD atDxl coupled by the boundary conditions. Application of
for 0<xD < oo, tDxJ >0. the Laplace transformation with respect to time to
Eqs. A-1 through A-8 and simplification yields
Initial Condition. a2,bfD 2 aPD
PfD = O, tDXf = O, 0 :5 XD < oo. . . . . . . . . . . . (A-2) axDa + (kfbJ)D ayD yd=o
Boundary Conditions.
ir S
aPfD I = -(kJbJ)D,
aXD XD=0 tDxf >O ,
_ pfD; 0 <xD < oo. . . . . . . . . . . (A-9)
1I
............. (A-3) Boundary Conditions.

and a,OfD
l r
- - , . . . (A-10)
axD xd =0 (kfbf)D . . . . . . .
lim p J p =0, > 0 . . . . . . . . . . . . .. .. .. .. (A-4)
XD- 00 lim pjp=0 . . . . . . . . . . . . .. . . . . .. . . (A-11)
XD-00
where, in oilfield units,
_ kh [p; -pfj and
PJD 141.2 qBµ a2p 2
ay D =spD; 0<yD <oo. . . . . . . . . . . . (A-12)
D
_ kh [P; -P) Boundary Conditions.
pD 141.2 qBp
PD I.vD=o =P.rD. . . . . . . . . . . . . . . . . . . . . (A-13)
_ 0.000264 kt lim pD = 0 . . . . . . . . . . . . . . . . . . . . . . . (A-14)
tDxJ tX 2
OAC J YD-

where
x
XD= - ,
xJ PjD (XD, S) 1 Dxf [PfD (XD.tDXf )
and
and
Y
YD= x .
J PD (YD, S) = °C f pxf [PD (YD, t Dxf ) ^ •

The variables not included above are defined in the Now let us define
text. p and pJ represent the formation pressure and
the fracture pressure, respectively, x is the flow PD (r.s) =-Cyo [PD (YD,s)1 •
direction in the fracture, and y is the formation flow
direction perpendicular to the fracture plane. Application of the Laplace transformation, with
The transient flow in the formation may be respect to yD, to Eq. A-12 yields
described by a
a2 rZpp -rpD IYo=o - yp I =spD. (A-15)
p^= apD ; yD=o
0 <yD < oo;
ayD atDx, By solving for pD and considering Eq. A-13, we
obtain
tpxf >0 . . . . . . . . . . . . . . . . . . . . . (A-5)
app l
Initial Condition.
__ rPfD r2
+ OY 0
PD -s D yD = . . . . . . . . . . . (A-16)
PD = 0; 0 <yD < oo; tpxj = 0 . . . . . . . . . . . . . (A-6)

Boundary Conditions. The inversion of this equation with respect to'yD


yields
PD I yv=o =PJD; tDxf>0 . . . . . . . . . . . . . . . (A-7)
PD =PJD cosh ( NFSyD ) + aPD
ayD yo = 0
and
0
lim PD =0; tDXf >0 . . . . . . . . . . . . . . . . . (A-8) sinh (NISyD )
YD- 00 Vs (A-17)

1764
JOURNAL OF PETROLEUM TECHNOLOGY
From Eqs. A-14 and A-17, we can write Total Formation Flow

aPD I _ -p . ............. . (A-18)


Eqs. A-18 and A-20 can be used to calculate the
fraction of the well flow rate that is being produced

• ayD YD=O
Next, substitution of Eq. A-18 into Eq. A-9 allows
us to write 'a partial differential equation with only
one dependent variable:
from the formation. The formula for this case ap-
pears to be
9'formation = 1
q - ex P( 72CrDJ
4t Dxr 2 1
82 s 2NFs
+ p ........ (A-19)
8xD Z r1rD (krbj) D ^
The solution of Eq. A-19 with boundary conditions •erfc( 2 )Dx" ). . . . . . . . . . (A-25)
given by Eqs. A-10 and A-11 can be expressed as TCrDJ
( f S 2^ l For small values of tDx , the amount of fluid
arexpzDlr^ + (krbJ)DJ ^ coming from the formation is negligible; however, at
PJD= (krbJ)D S1ns + (k /2
large values of tDx , most of the produced fluid
comes from the frormation. This illustrates the
f ID J bf D physical behavior of the system.
........ ............. ..(A-20)
APPENDIX B
The pressure at the wellbore pwo is calculated at Derivation of Correlation Parameters
XD = 0; thus, for Bilinear and Linear Flows
If we consider a fracture of length xr, Eq. A-4
z (A-21)
p",p = ( s 2t^ becomes
(krbr)D sL,l.r^ + (kr br )D ^ 'PfD
=o . . . . . . . . . . . . . . . . . . . . . . (B-1)
8xp xD=t
Finally, the Laplace inversion of Eq. A-21 with
respect to tDxl produces Using Eq. B-1 instead of Eq. A-4 and considering
incompressible flow within the fracture, the Laplace
IDxf transformation solution for the problem stated in

• PwD
= (krbr) D 0

?IJ°X
Appendix A is

PwD=
-?r+
00

1; 27rexp^-2n
erfc
[ (krbr)o (tDxt ] dX.
n=0

s '/z
^l 1 s(kj'bJ)D
L (k^bJ)D J
.......................(A-22)

Unfortunately, Eq. A-22 is too complex to analyze %z


both the short- and long-time behavior of the (kJ^] , (B-2)
21 s
solution. Eq. A-21 may be used for this purpose.
or
Short-Time Behavior
00
The short-time approximation of the solution can be pwD (krbr) p = -A+ F, 2aexp^-2n
obtained from Eq. A-21 by taking the limit as the n=0
variables approaches infinity.Thus,
s s
PwD = (krb r) DS 3/2 ' . . . . . . . (A-23) [ V (kJbJ)D2l ^
2
(kf
Inversion of this formula produces Eq. 8.

Long-Time Behavior s
[2 ^^ J . . . . . . . . . . . . . (B-3)
We can obtain the solution for large values of time by (krbr) D
taking the limit of Eq. A-21 as s approaches zero;
hence, Hence,
a

0
_
pwD (kJbf ) p = (kJ^ f [ (krbr) DZ ]. . . ( B-4)
PwD N12(krbJ)D s5i4 . . .. . . . . .. . . .. (A-24)
Inverting Eq. A-24 produces Eq. 12. This solution From Laplace inversion tables,
also may be obtained by considering incompressible
flow within the fracture. pwD (kJbJ) D =F[tDxf' (krbJ) D2 ] . . . . . . . (B-6)

SEPTEMBER 1981 1765


CD4/3 S 5/4
Hence, a graph of pwp (kfbf)D vs. tDxf (kfbf)2 4/3
(k b) D
yields a single curve. C^j3 p wD =^^^[ ( kj,b^, ) D2/3
For small values of dimensionless time, Eq. B-3
yields Eq. A-24, corresponding to bilinear flow.
The long-time approximation of Eq. B-3 yields +a
l
CDf /3S
(kfbf)D2/3
(C-4) •
717
/ . ........................ ( B-7)
pwD 2s" From properties of the Laplace transformation,
which is the Laplace transform of the dimensionless
pressure for the formation linear flow. L-lI(as) = QF[a] . . . . . . . . . . . . . . . . . . . (C-5)

APPENDIX C Hence, Eq. C-4 can be written as


Derivation of Correlation Parameters (kfbf)D2/3 _^ ( k b )D2/3 ^
for Bilinear Flow with (CDl) 1/3 pwD- (C.D
C f) 4/3 tDxf .
Wellbore Storage Effects
When wellbore storage effects are considered in the
flow problem defined in Appendix A, the boundary
....... ............ (C-6)
condition at xD = 0 becomes
This means that the bilinear flow solutions including
717 wellbore storage are represented by a single curve
apfl (1 CDfaPwD (C-1)
3XD Xo=o (kfbf)D BtbXfl when graphed in terms of
( k b ) D2/3
where ( k b ) D2/3
PwDVS.^fDx/.
_ C CD f 1/3 (CDf)
CDf 2^^lhxf2 . . . . . . . . . . . . . . . . . . . . (C-2)

and C is the wellbore storage coefficient. SI Metric Conversion Factors


If flow within the fracture is considered in- cp x 1.0* E-03 = Pa•s
compressible and Eq. C-1 is taken instead of Eq. A- ftx3.048 E-01 = m
3, the solution for the wellbore pressure in the psi x 6.894 757 E+00 = kPa


Laplace space is given by 'Conversion factor is exact. JPT
7r
PwD = 2(kfbf)D S514+7rCDfS2' . . , . . . (C-3)
Original manuscript received In Society of Petroleum Engineers office Aug.
10, 1978. Paper accepted for publication June 4, 1979. Revised manuscript
received July 6, 1981. Paper (SPE 7490) first presented at the SPE 53rd Annual
which can be written as Technical Conference and Exhibition, held In Houston, Oct. 1-3, 1978.


1766 JOURNAL OF PETROLEUM TECHNOLOGY
INTRODUCTION TO PRESSURE TRANSIENT TESTING

IN

NATURALLY FRACTURED RESERVOIRS

I. WARREN-ROOT MODEL

1. Idealized Model of Naturally Fractured Reservoir

VUGS 'vMATR/X S

Fig. 1: Schematic illustration of a naturally


fractured reservoir and its idealization.
After Warren and Root.

2. Assumptions in Warren-Root Model


• S ingle„p a se f l ow
• The material containing the primary porosity
is homQ.geneous and isotropic, and is contained
within a systematic array of identical
rectangular parrelopipeds.
• All of the secondary porosity is contained
within an orthogonal system of continuous,
uniform fractures which are oriented so that
each fracture is parallel to one of the princip':l
axes of permeability; the fractures normal to
each of the principal axes are uniformly spaced
and are of constant width; a different fracture
spacing or a different width may exist along
each of the axes to simulate the proper degree
of anisotropy.
• The complex of primary and secondary porosities
is homogeneous but may be anisotropic; flow can
occur between the primary and secondary po-
rosities, but flow between primary porosity
elements is not permitted.
• Flow to the wellbore is only through the
fractures. -
• Horizontal reservoir '}ranSPo<^
• Constant thickness reservoir
• Slightly compressible fluid
• Infinite reservoir
• Uniform initial pressure
• Constant rate production during pressure
drawdown tests, and prior to pressure
buildup tests.

3. Prima ry Param e ters


In ad dition to permeability and skin, the primary
parameters which control the behavior of double-porosity
reservoirs are storativity and the interporosity flow
coefficien t.

A. Storativity

(^Vct)f
• ^ - (1)
^Vct f+m

w = storativity
V = ratio of the total volume of one medium
to the bulk volume
= porosity ( ratio of pore volume in the
medium to the total medium volume)
ct = totil compressihility

NF-2
f = fissures (or fractures)

• f+m = total. system, i.e., fissures plus matrix

Interporosity Flow Coefficient


B.

k
m
a = ar2 (2)
w kf

A = interporosity flow coefficient


a= a parameter characteristic of system
geometry
km = matrix permeability

kf = fissure permeability

rw = wellbore radius

:l . Ideal S e m i 1 o K Be havior L.,

• y,6„{Y g Ic ^9 i nS ' fD r cSpb vJ

^Sl, dp SLOPE = m
^

• • - `
SLOPE = m -- -•--a-7 -1

At., 4t1

t + At
..^^.
At
Fig. 2: Ideal scmilog behavior predicted by
Warren-Root model.

NF-3
S. Conventional Semilog Analysis

^
kf = -162.6 mh
qBO (3)

a-^--
w = 10 ^mf (4)

Buildup Tests

($Uct)furW(tp+Otl)
Ykft P Atl or (S)

(^Uct)f+murW(tp+At2)
a = (6)
-ykftpAt2

where: y = exponential of Euler's constant = 1.78


At1, At, = transition times from semilog
plot.

Drawdown Tests

(^Vc)furw (^Uct)f+mur2w
(%)

ykftl ykft2

II. TYPE CURVE ANALYSIS

1. Double Porosity Type Curves


Type curves, developed by Flopetrol-Johnston, are
available which describe the pressure behavior in double
porosity reservoirs with either pseudosteady state or
transient interporosity flow. Figures 3 and 4 depict these
curves for pseudosteady state and transient interporosity
flow, respectively. These curves, which describe pressure
drawdown behavior, can also be used to analyze pressure

NF-4
ROPEfAOL JOHNSTON ATA
WELL WITH WELLBORE STORAGE AND SKIN 228. nx Erato.
f low 77530 vsuxae-P!N - FRANCE
INFINITE ACTING RESERVOIR WITH DOUBLE POROSITY BEHAVIOR - pseudo steady st
Postal aGdress : B.P. 557
The use of this type-curve is described in World Oil - October-1983 : INTERPRETING WELLTESTS IN FRACTUREL 77006 MeM, Cedex FRANCE

kh ROPETROL JOHNSTON NAM


A• q rw2 km 0.8936C C T- 2 p
cc FOR OIL -pp . kh AP FOR GAS-p0 µ(P)Z(P) dp 100 Maoro ea,l^d
kf 0 mcthr.2 141.2q9N S.03010<q T Pu SupartmN. TeMas 77478. U.SA
R0 Postal stltlress : P.O. Box 36389
_ tp Pp_ kh Tyc 2 P
dt.dP' dt.dp'
(^Vci ^.c1mVet m C- 0.000295 k C CDp0 141_2hq By CO 5.030104q T Px p(P)1(P)
Houston. Texas 770J6. U.S.A.
D µ
10 22
i T:

Type curve for double porosity


reservoir with pseudosteady
state interporosity flow.

Q .;
U ^ -- . ....._.._
O
=

10
w
>

W
tl7 ^
w
¢
^ . _, . . ..._
^

a . .... ...: .... .i


0
z
APPRO}
Q OF UN
LOG-LOG ;
w
¢
^
w
¢
Q.
^
^
w
Z
0
^
Z
w
0

ey• \^

10' :. : 10..._ .... . . .


101 . ., - ^ , 102
10•i

DIMENSIONLESS TIME GROUP t,/Cp

0
FLOPETAOLJOHNSTON

• .

FLOPETROLJOHNSTON ATA
228, rue Erman
77530 Yaw4eFNM • FRANCE
POROSITY'BEHAVIOR - transient IOrO$Ity flow
Postal ad&m ; B.P. ss7
The use of this type-curve is descri bed in World Oit - April 1984: NEW TYPE CURVES AID ANALYSIS OF FISSU RED ZONE WELL TESTS by D.BOURDET, A.ALAGOA • J.A. AYOUB, Y. M. PIRARD 77006 Mdn CeOex FRANCE
pp+.]P
kh p dp IOD OP^^NSTON NMI
k 1.8914 ( COe CD - 0.8936C FOR GAS- p INC 2
A . rrrw2 !^ ur submatrkbwcks FOR OIL- Pp - kh dp
t x mcthrw,2 141.2qB^ 0-5.030 104q T Psc Po P (p) -L(p1 g„qart",Texas n478.U5A
tD kA t0 kh Tsc 2 P Postal PO.Box36369
r.. , _ P'. e n^ dl.dp' - M AP' I^OUStpn. Texas 770.'!6. U5 A
- r D r= P 0-< non rne.. T - .. I ., 1 i 0 ;Z-1
(mVctll ♦
102 - -

Fig Type curve for double porosity


reservoir with transient
interporosity flow.
0
a
0
U

a . .....
.....
;.........
0
¢
10
W
> _-J-T'

^, o :. .. .
w


a
z r
¢ -
--- ---APPRC
CL° OF UI
w tAG-lOG
¢
D
U)
U)
W

t - -
N
^
J
Z
0 : . .__.__-_• . ...
rn
Z
Uj
^ .. . .. .. ..

t0 it0 _ .! :...___..._ : .__....:... . 1- '._...,_-._. ..._...


t . ^ ^ .. , .. .103
102

filMI .V.`i IOtiLES: 1 Lti1E. GHOUP t.^ICti

^
buildup data provided that equz=vaZent time is used in
preparing the required log-log plot.

2. Type Curve Matching


Type curve matching requires that a log-log plot of
pressure change versus test time be prepared; Figure 5 depicts
an ideal plot for a pressure buildup test. The early time
data represents radial flow from the fracture system into the
wellbore; these data, when matched on the type curve,
should follow one of the curves for a homogeneous reservoir
designated by the parameter CD e2s. From this early time
match, we determine [(Cne2s)r]hl. This match is illustrated in
Fig. 6 up to Point A on the curve for CDe2$ = W.

0
N • ^
^


^ o ^ ^ ^


^

^ •
FISSURE TRANSITIONAL TOTAL SYSTEM
FLOW FLOW FLOW

ate , hrs

Fig. 5: Log-log plot of pressure buildup data


depicting typical behavior of double
porosity reservoir.


NF- 7
As interporosity flow starts from the matrix into the
^ fractures, the data leave the CDe2$ curve and follow one of
the transition curves designated by the parameter ae-2s
From this match, we determine (ae-2$)M. This match is
illustrated on Fig. 6 between points A and B on the curve for
Xe-2s = 10-4.
Finally, when all production is from the matrix, the
2S
pressure leaves the transition curve and follows a new CDe
curve below the one matched by the early-time data. This
data represents the total system, i.e., fracture plus matrix,
and from this match we obtain the value of [(CDe2S)f+m]M'
This match is illustrated on Fig. 6 by data to the right of
Point B on the curve for CDe2S = 10.
From an arbitrary match point, and from the parameters
represented by the matched curves, the following information
is obtained:

[(C D e 15)f]M (pD)M

is (C De Zs ) f+m]^,^ (Ate)M

t
(^e 2s)M [ cD ] M .

(Ap) r,q

3. Type Curve Analysis


Using the information obtai.ned from the type curve
match, the following calculations can be made:

A. Fracture Conductivity

r^
kfh = 141.2qBu (pp l J
M


NF-9
B. Wellhore Storage Factor

C = 0.000295kh
tD D) ^1 ^ 9.)

C. Dimensionless Welibore Storage Factor

0.894C
(C D)f+m (10)
(^ct)f+mhrw

D. Total Skin Factor

2S
S = 1 In (CDe )f+m (11)
2 C
LDf^m

E. Storativity

zs
w = (C De ) f+r:' ^
(1 ? ;
(CDe2s)f

F. Interporosity Flow Coefficient

a = (ae-2s)Me2s (13)

4. Type Curve Analysis When Total System Behavior is


Not Observed
The foregoing analysis assumes that test conditions
permit the total reservoir behavior to be observed in the
test data. Because of wellbore storage or near wellbore
problems, it is not unusual for early or transitional data
to be masked. Further, if a test is not run sufficiently
long, the late time data may not be present. If any of the
flow periods are absent in a test, a complete test analysis
will not be possih7e.
0

NN-10
When early-time fissure behavior is not observed
^ because of wellbore storage, or other problems, type curve
analysis will yield the same information as when the total
system is observed except the value of (CDe2$)f will be
too small. Accordingly, only an upper limit for the value
of w will be obtained. When early-time and transitional
data are absent from the test, the system will behave like
a homogeneous reservoir; in this case, only values of kfh,
C, and a maximum value of s can be determined.

5. Analysis With Pressure Derivatives


It is often difficult to distinguish between double
porosity reservoirs and homogeneous reservoirs with certain
types of boundary effects or heterogeneities. This is
particularly true if total reservoir behavior is not observed
during a test. For example, suppose that the first straight
line on the semilog plot, caused by flow from the fissures,
is masked by the effects of wellbore storage. With only the
^ transitional and total system response present, it is obvious
from Fig. 2 that the Horner plot could be misinterpreted as
representing a well near a fault, a well off-center in a
bounded drainage area, a change in mobility away from the
well, a multilayered reservoir, or a variety of other boundary
effects or heterogeneities. In these situations, additional
information is needed to interpret the test.
A useful way to distinguish between homogeneous and
double porosity reservoirs is to examine a log-log plot of
the pressure derivative versus test time, i.e., d(dp)/d[In(t);
versus t for drawdown tests, or d(Ap)/d[ln(dte)] versus Ote
for buildup tests. Figure 7 illustrates the Horner plot and
pressure derivative plot that would typically be expected for •
a double porosity reservoir with early-time wellbore storage
effects. Notice in Region A that wellbore storage causes the
pressure derivative to hump in a manner identical to that observed
in homogeneous reservoirs; this behavior is independent of
reservoir type. Following the wellbore storage hump, the

NF-11
A 13 C I D ^,^`U^\

m
.,..,

• •
• • ^^^Pti
Si
V)

^ SLOPE = m

^S • SLOPE = m
^^55^ ••



A te, hrs

A - WELLBORE STORAGE
B - RADIAL FLOW FROM FISSURES

r-,
C
D
-
-
DOUBLE POROSITY BEHAVIOR
TOTAL SYSTEM BEHAVIOR Si
4J
a eQ
• ^
• A '^• B C D
'C7 • •

• • • • • • • • •
• •

I •
• •

• • •

ate , hrs

Fig. 7: Behavior of pressure and pressure derivative


for a double porosity reservoir.


NF-12
pressure derivative stabilizes; this characteristic, seen
^ in Region B, is caused by radial flow from the fissures, and
is identical to the behavior of radial flow in a homogeneous
reservoir. Following stabilization, the pressure derivative
humps beZozv the stabilized line; it is this characteristic,
shown in Region C, that is diagnostic of a double porosity
reservoir. Finally, the pressure derivative again stabilizes,
shown by Region D, representing the total system behavior.

6. Fissured Versus Multilayered Reservoirs


The double porosity models we have presented represent
both fissured and multilayered reservoirs with high permeability
contrast between the layers. These two systems exhibit
identical behavior; consequently, it is impossible to dis-
tinguish between the two types using only pressure-time data.


NF-13
EXAMPLE PROBLEM: Pressure buildup test in a naturally
fractured oil reservoir

Problem. The following data (Da Prat, et al., SPE 13054, 1984)
were obtained during a pressure buildup test of a naturally
fractured reservoir in the Machiques field located in Western
Venezuela. Prior to shut-in, the subject well was produced
at a rate of 3,224 STB/D for 56 hours.

q = 3,224 STB/D = 4.8%


h = 65 ft rw = 0.2917 ft
Bo = 1.82 RB/STB
T = 270°F
Po = 0.362 cp tp = 56 hrs

ct = 24.5x10-6 psi-1 pwf(At=o) = 9,670 psia

At Ot e p ws p ws p w f(At=o)
(hours) (hours) ( psia)

0.25 0.249 10,095 425


0.50 0.496 10,255 585
0.75 0.740 10,451 781
1.00 0.982 10,564 894
1.25 1.223 10,631 961
1.50 1.461 10,703 1,033
1.75 1.697 10,743 1,073
2.00 1.931 10,806 1,136
2.25 2.163 10,814 1,144
2.50 2.393 10,830 1,160
3.00 2.847 10,854 1,184
3.50 3.294 10,878 1,208
4.00 3.733 10,878 1,208
5.00 4.590 10,902 1,232
6.00 5.419 10,902 1,232
7.00 6.222 10,917 1,247


NF-14
8.00 7.000 10,925 1,255
9.00 7.754 10,933 1,263
10.00 8.485 10,941 1,271
12.00 9.882 10,941 1,271
14.00 11.20 10,950 1,280
16.00 12.44 10,959 1,289
18.00 13.62 10,958 1,288
20.00 14.74 10,973 1,303
24.00 16.80 10,982 1,312
28.00 18.67 10,989 1,319
32.00 20.36 10,989 1,319
36.00 21.44 10,989 1,319
40.00 23.33 11,045 1,375
45.00 24.95 11,109 1,439
50.00 26.42 11,109 1,439
55.00 27.75 11,109 1,439
70.00 31.11 11,141 1,471
72.00 31.50 11,149 1,479
75.00 32.06 11,149 1,479
79.00 32.77 11,156 1,486
83.00 33.44 11,165 1,495
87.00 34.07 11,165 1,495
92.00 34.81 11,181 1,511
97.00 35.50 11,181 1,511
102.00 36.15 11,189 1,519
107.00 36.76 11,197 1,527
112.00 37.33 11,200 1,530
117.00 37.87 11,200 1,530
122.00 38.38 11,200 1,530
127.00 38.86 11,200 1,530
138.00 39.84 11,229 1,539
140.00 40.00 11,229 1,539
142.00 40.16 11,252 1,582
145.00 40.40 11,252 1,582
148.00 40.63 11,268 1,598
153.00 41.00 11,284 1,614

NF-15
157.00 41.28 11,292 1,622
160.00
1.64.00
41.48
41.75
11,300
11,300
1,630
1,630 ^
169.00 42.06 11,308 1,638
174.00 42.37 11,316 1,646
178.00 42.60 11,316 1,646
183.00 42.88 11,316 1,646

a) Using the Flopetrol double porosity type curves, analyze


the pressure buildup data for fissure conductivity, kfh, well-
bore storage factor, C, total skin factor, s, storativity
ratio, w, and interporosity flow coefficient, ^.

b) Calculate kfh and w using a conventional semilog analysis


of the data.

Solution. Recognizing that producing time, tp, is small


compared to shut-in time, At, pressure will be graphed as a
function of equivalent time, Ate, which is defined as

t At
Ate = t p +At
p

This is not necessary for the semilog analysis; it is necessary


for the type curve analysis, however, since the type curves
being used rigorously only apply to pressure drawdown data
and cannot be used to analyze buildup data unless tp »At.

a) A log-log plot of [pws pwf(At=o)] versus Ote was prepared


and is depicted by Fig. 1. These dat a were matched with the
Flopetrol double porosity type curves for pseudosteady state
flow; this match is presented in Fig. 2. The early data,
corresponding to the pressure buildup response of the fissures,
match the theoretical curve for


NF-16
(Cpezs)f = 1.


10"

^ • ^ •• M 04 1 M• M

103

N

102
• 10-1 10° 10'

Ate, hrs

Fig. 1: Log-log plot of test data.

Following the fissure response, the transition data match the


curve for

-ZS -2
ae = 5x10
0

Finally, with the total system affecting the buildup, the


late-time data match the curve for

• (Clle2s)f+m = 10-1

NF-17
A . ,i rw,2 km Cp _ 0.8936C FOR OIL -Pp . kh kh TK 2
.1P FOR GAS - Pp - P dp
kt octhr,,,2 tat.2a8n 5030 10<v T p_ ^ (p)LIPI
^ (mVct)t .11
C . 0000295 ^ C
(mVct),(mVCt)m O CpPp. M7.2qBµ ALdP CpPp.5.030 70aQ T p- µIp^7(P)
t01

ti

0
n
0
U z
0

a
:D
O
¢
O to

z
a
w
n
!7
w
00 Cr

w
3
a
0
z
a

¢
V)
U)
¢
a ,
N
N
W
Z
O
vi
z

t01 ,p _ipi
. 10.
DIMENSIONLESS TIME GROUP ID/CD

Fig. 2: Match of test data on the double porosity type curve for pseudosteady state
interporosity flow.

0 • •
From the chosen match point, the pressure and time
match values are

(At) Im = 100 hours

(tD/CD)M = 430

(Op)m = 1,000 psi

(PD) M = 1.2.

The fracture conductivity, kfh, is computed according to


the relationship

(PD) M
kfh = 141.2qBU (Op)
M

kfh = (141.2)(3,224)(1.82)(0.362)(1^000)

0 k h = 359.9 md-ft.
£

Accordingly, the fissure permeability is indicated to be

,
k
_ fn _ 359.9
kf h 65

kf = S.S md.

The wellbore storage factor is computed from the time


match as

0.000295kh (At) M
C= u
( t D CD)M

^. _ (0.09 5) (5. 5) (65) 100


^ O.362 430

NF-19
C = ().065 bbl./psi.

The dimensionless wellbore storage factor, based on the total


system, is

0.894C
(CD)f+m _
Wt)f+mhrzw

_ _ (0.894)(0.068)
(CD)f+m (0.048)(24.5x10-6)(65)(0.2917)2

(CD)f+m = 9,347.

The total skin factor can be approximated from the match


value of (CDe2$)f+m as

[(CDe2s) f+m
s ln
2
TCD f+m .

1
S = 2 in (0
9.341 7)

s = -5.72.

The storativity ratio, w, was defined by Warren and


Root to be

( ^ VC } J 1
w - l

($Uct f + ^Vct III

This parameter can be computed from the type curve match


as


NF-20
( D
( e2s

(CI)e
2S)f+m
)E
1
0.1

0.1.

The interporosity flow coefficient, A, was defined by


Warren and root as

k
^ m
arw kf

This parameter is estimated from the transition data as

A = (ae2$)e2s

a = (5x10-2)(e "." )

^ a = 5.37x10-'

b) A semilog plot of the test data is presented in Fig. 3.


It is observed that the early and late data form parallel
straight lines as predicted theoretically by Warren and Root.
The fissure conductivity can be estimated using the slope of
either of these lines according to the following equation:

kfh = 162.6 ^

(162.6) (3,224) (1.82) (0.362)


kfh = - 860

kfh = 401.6.

Accordingly, the fissure permeability is

q
NF-21
11,40

11,20

11,00

10,80
ct
^

z ; 10,60

N
N

10,40

10,20

10,00
1O-' 10° 10' 102
t At
p
t +At
p

Fig. 3: Semilog plot of test data using equivalent time.

• • •
_ k f h _ 401.6
kf h 65

^
kf = 6.2 md.

It is noted that these values are in excellent agreement


with those from the type curve analysis.
The storativity ratio, w, was shown by Warren and
Root to be rclated to the differential pressure, ap,
separating the parallel lines on the semilog plot according
to the equation

10- (Sp/m)

From Fig. 3, it is noted that Sp = 660 psi. Therefore

W = 10-(660/860)

w = 0.17.
0


NF-23
REFERENCES

1. Warren, J. E. and Root, P. J.: "Behavior of Naturally


Fractured Reservoirs," Soc. Pet. Eng. J. (Sept. 1963)
245-55; Trans., AIME, 228.

2. Bourdet, D. and Gringarten, A. C.: "Determination of


Fissured Volume and Block Size in Fractured Reservoirs
by Type-Curve Analysis," paper SPE 9293 presented at
the 1980 SPE Annual Technical Conference and Exhibition,
Dallas, Sept. 21-24.

3. Aguilera, R.: Naturally Fractured Reservoirs, Penn Well


Publishing Company, Tulsa, Oklahoma 1980 .

4. Van Golf-Racht, T. D.: Fundamentals of Fractured


Reservoir Engi.neerin , Elsevier Scientific Publishing
Company, New York 1982).

5. Bourdet, D., et al.: "A New Set of Type Curves Simplifies


Well Test Analysis," World Oil (May 1983).

6. Bourdet, D., et al.: "Interpreting Well Tests in


Fractured Reservoirs," World Oil (Oct. 1983).

^ %. Gringarten, A. C.: "Interpretation of Tests in Fissured


and Multilayered Reservoirs With Double Porosity
Behavior: Theory and Practice," Jour. Pet. Tech.
(April 1984) 549-564.

8. Bourdet, D.: "New Type Curves Aid Analysis of Fissured


Zone Well Tests," World Oil (April 1984).

9. Da Prat, G., et al.: "Use of Pressure Transient Testing


to Evaluate Fractured Reservoirs in Western Venezuela,"
paper SPE 13054 presented at the 1984 SPE Annual
Technical Conference and Exhibition, Houston, Sept. 16-19.


NF-24
4p Distinguished Author Series

Interpretation of Tests in Fissured


and Multilayered Reservoirs With
Double-Porosity Behavior:
Theory and Practice
by Alain C. Gringarten, SPE

Alain C. Gringarten is vice president of Scientific Software•fntercomp in Denver. He


earlier was director of engineering with Johnston-Macco-Schiumberger in Houston and
during 1978-81 head of well test interpretation with Flopetrol-Schlumberger in France.
A graduate of Stanford U., Gringarten has published more than 30 papers on well test
analysis and related subjects. He currently serves on the SPE Editorial Review
Committee.

Summary


This paper summarizes current knowledge of reservoirs convincing explanation for some commonly observed
with double-porosity behavior. These include both flow peculiarities.
naturally fissured reservoirs and multilayered reservoirs There has been no unified approach to the problem;
with high permeability contrast between layers. The heterogeneous reservoir behavior in the literature is
first part presents available solutions to the direct still considered too complex and too diverse to be
problem (i.e., solutions to the diffusivity equation) that analyzed in a systematic and unique way. The main
have appeared in the oil and groundwater literature reason is the general belief that an interpretation model
over the past 20 years, The second part discusses must closely approximate the actual complexity of the
methods for solving the inverse problem-i.e., reservoir. The observation of a very large number of
identifying a double-porosity behavior and evaluating well tests in many different formations around the
all corresponding well and reservoir parameters. world, however, reveals that the number of possible
Several field examples demonstrate various aspects behaviors during a well test is limited; therefore, only
of double-porosity behavior and illustrate how a limited number of interpretation models is required
additional knowledge of the reservoir (e.g., fissured for well test analysis. This is because during a well
vs. multilayered, gas saturation, etc.) can be obtained test, the reservoir is acting only as a filter between an
from numerical values of the reservoir parameters. input signal, the change in flow rate, and an output
Practical considerations for planning tests in double- signal, the change in pressure, and only high contrasts
porosity reservoirs also are included. in physical properties within the reservoir can be
highlighted.
Introduction In practice, a test reveals only that the reservoir acts
The movement of underground fluids is of interest in as one single medium (homogeneous behavior) or as
many different engineering fields and, consequently, two interconnected media (heterogeneous behavior).
has been the subject of active research over the past The terms "homogeneous" and "heterogeneous" are
40 years. related to reservoir behavior, not to reservoir geology.
Interpretation procedures, however, are well "Homogeneous" means that the permeability
established only for porous fluid-bearing reservoirs measured in a test and that measured in a core are the
considered reasonably homogeneous. Fluid-flow same, although the resulting numbers may be
behavior in heterogeneous formations is still the different. "Heterogeneous" means that these

• subject of much debate. It is agreed only that


conventional methods primarily developed for
homogeneous reservoirs may be inadequate, and that
new specific approaches may be required to provide a
permeabilities are likely to be different.

The Double-Porosity Model


The particular case of heterogeneous behavior where
0149-2136184l0041-0044$00.25
only one of the two constitutive media can produce to
Copyright 1984 Society of Petroleum Engineers of AIME the well is called "double-porosity" behavior.

APRIL 1984 549


Although the corresponding double-porosity model has medium in the vicinity of that same point. p f is the
been the subject of many studies in the past 20 years, pressure measured at the bottom of the well during a
its use is still not fully understood by the practicing test.
engineer. This model is discussed in detail in the A very important characteristic of a double-porosity
remainder of this paper. system is the nature of the fluid exchange between the
The double-porosity model initially was introduced two constitutive media, or interporosity flow.
for the study of fissured reservoirs. t-s Later, several Interporosity flow was assumed b^ Barenblatt et at.
authors proposed the two-layered model as an and by subsequent authors z'3,5'7•t to occur under
alternative solution.9•10 In reality, both fissured and pseudosteady-state conditions:
multilayered reservoirs with high permeability contrast
between layers can be represented by the same double- kn,
porosity model (( and exhibit the same double-porosity Sq=a-(p^, -pf) . . . . . . . . . . . . . . . . . . . . . . . . (1)
behavior during a test. As a result, it is impossible to
distinguish between the two types from pressure-tcst
behavior alone. As discussed below, distinction can be where k,,, is the permeability in the least permeable
made only under certain conditions from the numerical medium. a, a parameter characteristic of the geometry
values of the well and reservoir parneters. of the system, has the dimension of a reciprocal area
A detailed review of published articIcs oil double
porosity as applied to fissured reservoirs was presented 4n(n+2)
in Ref. l 1. The lollowing summarizes only current « = P^ . ...........................(2) a
knowledge of the doublc-porosity model. Solutions to
the direct problem (i.e., predicting the pressure
behavior of a double-porosity reservoir) are presented where it is the number of normal sets of planes
first, followed by a detailed discussion of the inverse limiting the least permeable medium (n=1 for a
problem (i.e., identifying a double-porosity reservoir
from test data alone and evaluating double-porosity
reservoir parameters). The latter is illustrated with
multilayered reservoir) and f is a characteristic
dimension of such a block.
Other authors6's-1o,13-t5 have assumed transient

field examples. interporosity flow. Among these, de Swaan6 presented
a particularly interesting a^proach used in several
Direct Problem: Mathematical Models for subsequent publicationsg'1-•14'17 (see Appendix).
The question of the interporosity flow condition
Double-Porosity Behavior
likely to be found in practice has been the subject of
The double-porosity concept was introduced by much debate in the past few years. Pseudosteady state
Barenblatt et al. I As mentioned before, the double- has been shown to be a In-time approximation of
porosity model assumes the existence of two porous transient interporosity flow, and the pseudosteady-
regions of distinctly different porosities and state assumption has been justified because any
permeabilities within the formation, Only one of the transient effect was likely to be of short duration, as
porous media has a permeability high enough to could be inferred from published formulas describing
produce to the well. This would be the fissure system the initiation of pseudosteady-state flow. 16 Indeed, the
in a fissured reservoir or the most permeable layer in a majority of tests I have seen appear to exhibit a
two-layercd reservoir. For simplicity. I sometimes in pseudosteady-state interporosity flow behavior. Yet
the following call it "the fissures" and refer to it with some other tests seemed to indicate a transient
the subscript f. interporosity flow behavior, which is distinctly
The second porous medium does not produce different. 17
directly into the well but feeds fluid into the first A possible explanation of this apparent inconsistency
medium and acts as a source. This would correspond can be found in an unpublished paper by Cinco,* who
to the matrix blocks in a fissured reservoir and to the suggests that interporosity flow always occurs under
less permeable layer in a two-layered reservoir. Again, transient conditions but can exhibit a pseudosteady-
for simplicity, I refer to it as "the blocks" and use the state-like behavior if there is significant impediment to
subscript in. The subscript f+m stands for the total the flow of fluid from the least permeable medium to
system. the most permeable one, as in the case of calcite
A basic assumption in the model in Ref. 1 is that deposit in a fissured reservoir, for instance. In fact,
any infinitesimal volume contains a large proportion of
each of the two constitutive media. As a consequence,
each point in space is associated with two pressures,
natnely: (1) the average fluid pressure, pf, in the most
interporosity flow can exhibit any intermediate
behavior, depending upon the magnitude of the
interporosity skin.

As occurred in the case of reservoirs with
permeable medium in the vicinity of the point and (2)
the average fluid pressure, p,,,, in the least permeable 'Cinco-Loy, H.. Personal communication (Oct. 28, 1983).

550 JOURNAL OF PETROLEUM TECHNOLOGY



homogeneous behavior, the first solutions for double-
porosity behavior were limited to line-source wells.
Although Barenblatt et al. I introduced the double-
porosity concept, they only derived the pressure in the
blocks with the restriction of zero compressibility in
the most permeable medium.
The first complete line-source solution in terms of
pressure in the most permeable medium was done by
Warren and Root2 for pseudosteady interporosity flow.
These authors showed that two parameters, in addition
to permeability and skin, controlled double-porosity
behavior, These are: (1) the ratio w of the storativity
in the most permeable medium to that of the total
reservoir, tj t2
LOG OF PRODUCTION TIME

(0vCr)f
W= . ..........................•(3) Fig. 1-Drawdown test in a double-porosity reservoir
(0Vcr )j+m (Warren and Root2): two parallel semilog straight
lines.
where V is the ratio of the total volume of one
medium to the bulk volume, and 0 is the porosity of
that medium (ratio of pore volume in the medium to
the total medium volume); and (2) the interporosity Conversely, as Warren and Root had derived their
flow coefficient X, direct solution for fissured systems, they suggested
that this two parallel semilog straight-line behavior
q A=arW km (4)
was characteristic of fissured reservoirs. They noted,
however, that it was also characteristic of stratified
kf
formations (i.e., it belongs to double-porosity
behavior).
where k f is the permeability of the most permeable Warren and Root indicated that the reservoir
medium. permeability-thickness product, kh (in practice, the
Other line-source solutions subsequently published permeability-thickness product of the most permeable
are essentially identical to that of Warren and medium, kfh, as the matrix blocks do not flow to the
Root, 3,5-7 or they consider transient interporosity well), could be obtained from the slope m of the two
flow. 7,14,15
semilog straight lines; w, from their vertical
Wellbore storage and skin were added to the displacement Sp:
pseudosteady-state interporosity flow solution of
Warren and Root by Mavor and Cinco. 12 This w=10-bpi,R; ..............................(5)
solution then was extended by Bourdet and
Gringarten17 to account for transient interporosity flow and X, from the time of intersection of the horizontal
and to generate type curves useful for the analysis of line drawn through the middle of the transition curve,
double-porosity systems. A similar solution later was with either the first (t i) or the second (t2) semilog
published by Cinco and Samaniego. 13 straight line. This was shown by Bourdet and
Gringarten 17 to yield
Inverse Problem: Identification of
Double-Porosity Behavior From Well-Test Data
Conventional Semilog Analysis. The first (0Vcc);ur2
PI' _ (4Vcr)f+mµrW
identification method was proposed by Warren and (6)
Root. Z These authors evaluated approximate forms of ykft 1 7kft 2
their pseudosteady-state interporosity flow solution and
found that they yielded two parallel straight lines on a
semilog plot (Fig. 1). The first straight line represents in drawdown tests, and

• homogeneous semilog radial flow in the most


permeable medium acting alone, whereas the second
straight line corresponds to semilog radial flow in the
total reservoir. The two straight lines are separated by
1^= (0 Vc,)fµrw(tP+Art) _ Vcr)f+,^urw(tp+At2)
a transition period during which pressure tends to yk ftP At l yk fr p At Z
stabilize. ..........................(7)

APRIL 1984 551


^
in buildup tests. tP in Eq. 7 represents the duration of in Coe2S based on the storativity of the medium under
the drawdown preceding the buildup. consideration), and behavior during interporosity flow
As t) and t2 can only be approximated, the value of from the least permeable medium into the Inost
a obtained by this method (and by others published in permeable one: PD = ff(tnICD, Cpe2s, Xe-2s).
the literature 18) is not very accurate but usually Bourdet and Gringarten 17 thus designed two
remains within the order of magnitude of the correct drawdown type curves for a well with wellbore storage
value. A more accurate method by type-curve analysis in an infinite reservoir with double-porosity behavior,
is discussed later. corresponding to the two extreme interporosity flow
The existence of the two parallel semilog straight conditions identified by Cinco, ` namely, restricted and
lines, and therefore the possibility of obtaining w and unrestricted interporosity flow. The former corresponds
X from test data, was disputed by Odeh,3'19 who physically to a high skin between the most and the
found that some fissured reservoirs could behave like least permeable media and is mathematically
homogeneous systems. Odeh investigated the same equivalent to the pseudosteady-state interporosity flow
double-porosity model as Warren and Root but for solution. The latter corresponds to zero interporosity
different ranges of parameters. skin.
In the case of transient interporosity flow, a third The type curve for restricted interporosity flow
semilog straight line was shown to be present during presented in Fig. 2 is obtained as the superposition of:
transition, 17 with a slope equal to half that of the two (1) the drawdown type curve for a well with wellbore
parallel semilog straight lines. 13-15 storage and skin in a reservoir with homogeneous
The conditions of the existence of the various behavior, presented in Ref. 21 (the continuous curves
semilog straight lines have been the subject of much in Fig. 2), and (2) restricted interporosity flow curves,
discussion. It is generally believed that the first function of Ae -2S (the dashed lines in Fig. 2). 17
straight line, representing the most permeable medium, The type curve for unrestricted interporosity flow
can exist only at very early times and is likely to be
shadowed by welibore storage effects. Therefore, a
common perception is that, in practice, only
parameters characterizing the homogeneous behavior
presented in Fig. 3 is obtained as the superposition of:
(1) the drawdown type curve for a well with wellbore
storage and skin in a reservoir with homogeneous
behavior, Ref. 21, and (2) unrestricted interp orosity

of the total system, k fh, can be obtained, as contended flow curves, function of 6[(CDe2S)J+mAe- S], shown
by Odeh3-assuming, of course, that the as dashed lines in Fig. 3.17 These transition curves are
corresponding semilog straight line is present-and in fact the homogeneous curves of Ref. 21, shifted by
those specific to the fissures (w,X) are usually not a factor of 2. S is a function of the shape of the matrix
accessible. blocks and is given by: 6= 6/ryZ = 1.89 for
To minimize wellbore storage effects and thus avoid horizontal slab matrix blocks, and 6= 10/3y2 = 1.05
masking development of the first semilog straight line, for spherical matrix blocks, where -y is the exponential
several authors have advocated the use of a downhole of the Euler constant.
shut-in tool. 2,20 This, however, can help only in A typical behavior of the well pressure in a double-
multilayered reservoirs, not in fissured reservoirs. As porosity reservoir is sketched in Fig. 4. At early
discussed in another part of this paper, storage effects times, production comes only from the most permeable
in fissured reservoirs include the effect of fissures medium, and the pressure drop follows one of the
intersecting the well and usually are one or two orders homogeneous curves with CDe2S =(Coe2S) f. This
of magnitude greater than storage effects in the corresponds to the heavy line up to Point A on the
wellbore. curve called "fissures" on Fig. 4.
Type-Curve Analysis. An answer to the question of As interporosity flow starts from the least permeable
the existence of the various double-porosity semilog medium into the most permeable one, the pressure
straight lines, and, more generally, a solution to the leaves the CDeZS curve and follows one of the
inverse problem in double-porosity reservoirs in the transition curves (the heavy line between A and B
presence of wellbore storage and skin was presented on Fig. 4).
recently by Bourdet and Gringarten. 17 They showed Finally, when all production comes from the least
that double-porosity behavior is controlled by the permeable medium, the pressure leaves the transition
independent variables (defined in Appendix) PD, curve and follows a new C e2S curve below the first
to/Cp, CDe2S, w, and Xe-25, and that it is possible one, with CDe 2S =(CneZg)f+,n, corresponding to
to represent the behavior of a well with wellbore
storage and skin in an infinite reservoir with double-
porosity behavior, PD = f(to/CD, Cpe25 wXe-2s),
homogeneous behavior of the total reservoir. This
corresponds to the heavy line after Point B on the
curve labeled "blocks + fissures" on Fig. 4.

as a combination of the homogeneous behavior of each Because the type curve for wellbore storage and skin
constitutive porous medium, with wellbore storage and
skin at the well: PD = fy(tolCo, CL)e2S) (with Co 'Cinco-Ley, H.: Personal communication (Oct. 28, 1983).

552 JOURNAL OF PETROLEUM TECHNOLOGY


_pm
lo=
-Co. 2S
_ Coe Is
APPROXIMATE -qn
• • •• START OF SEMI LOG •t4 APPROXIMATE 1030
STRAIGHT LINE _,ps START OF SEMI-LOG
10 STRAIGHT LIN E -1030
-.•t.'25 q1
• p ^01111M10/MOIM CY/VeA --IDa
Y O s ••.... on 1nnA10M ewra ^ -- - -
103
sq^

.1#
0.5

° ID _Q5
10,3
1
....... .... J .... 10-D
q.. .. .....

/ ^.

10 102 103 104 t0-1


_ L D c 0.000295 ^! 'L
CD 11 C 10-I I 10 }p/co 102 103 104

Fig. 2-Bourdet and Gringarten's type curve" for a well with Fig. 3-Bourdet and Gringarten's type curve 17 for a well with
wellbore storage and skin in a double-porosity wellbore storage and skin in a double-porosity
reservoir (restricted interporosity flow). reservoir (unrestricted interporosity flow).

in a reservoir with homogeneous behavior also 10


includes the case of an infinite conductivity vertical

• fracture with wellbore storage (all continuous curves


below CDe2S = 1 in Figs. 2 and 3), the double-
porosity type curves in Figs. 2 and 3 yield information
on the quality of the well, depending on the CDe2S
102

10
_Cpe2S

---:le-2s

FlS R£

curve matching the homogeneous behavior of the most ^S+BysmoT^__ B -_7__ E


permeable medium: damaged if (CDe2S)^ is greater ........ ............
than !0 t; normal (nondamaged) if (CDe' ) f is
between 103 and 105; acidized if (CDe2S)f is between
5 and 0.5; and fractured if (CDeZS) f is less than 0.5. 10-1
These limits, of course, are only approximate.
The use of the double-porosity type curves is 1
10-21
discussed in the remainder of this paper, to illustrate 10-1 1 10 102 103 1o4 105 106
some characteristic features of double-porosity tpiCD
behavior. Most comments concern the type curve of
Fig. 2, for the sake of simplicity and because, in my Fig. 4-Schematic of double-porosity log-log behavior.
experience, restricted interporosity flow is the most
common behavior found in practice. These comments
and conclusions extend readily to unrestricted
interporosity flow. storage and skin homogeneous curve (corresponding to
Drawdown Analysis. When the total system the most permeable mediutn):
behavior is seen during a test, as in Fig. 4, tog-log (CDe2s) f+m
analysis of drawdown data with the type curves of
(CDe2s).f (8)
Figs. 2 and 3 yields all the parameters normally
obtained with the wellbore storage and skin type curve
for homogeneous reservoirs, 21 namely: k fh from the Occasionally, the first CDe2S curve coincides with
pressure match, C from the time match, and S from the transition curve so that„the well drawdown pressure
the match with the CDe2S curve for which OVcr is follows the transition curve from the very beginning
available (most permeable medium or total reservoir, before merging into a CDe2S curve corresponding to

• usually the latter), plus the fissuration parameters: X


from the match with the transition curves )`e-2S or
S[(CDe2s)f+m1Xe -251; w from the ratio of the CDe2S
value for the last wellbore storage and skin
the total reservoir. This situation occurs when (OVcr) f
is very small compared to (0Vcr)f+,n. The actual
(CDeZS) j curve may in fact be to the left of the Xe -2S
curve, and the actual (CDe2S) j value may be greater
homogeneous curve (corresponding to the total than that of the CDe2S curve coinciding with the early
reservoir) to the CDe2S value for the first wellbore part of the transition curve. Log-log analysis in such a
APRIL 19&4
553
L,
case still yields kfh, C, S, and X as before, but only w) but also and primarily on the well condition (S in
an upper limit of w: CDe2S and Xe -2S). This makes it difficult to predict
the existence of the two semilog straight lines and,
moreover, does not guarantee that the two semilog
(CDezS)f+m
..................(9) straight lines found in a test will be found again in
(Cpe2S)f,lim subsequent tests, and vice versa. Of course, if the two
semilog straight lines exist, they must be parallel,
because the permeability thickness of the total system
In practice, it is difficult to detect w values of less
is equal to that of the most permeable medium.
than 0.001 by type-curve analysis.
In a drawdown test in a double-porosity reservoir
Of course, if only a portion of the complete double- with unrestricted interporosity flow, the additional
porosity drawdown behavior shown in Fig. 4 is
half-slope semilog straight line exists for test data
obtained during the test, only limited information can
matching the transition curves in Fig. 3 after the
be extracted from the test data. For instance, it is not dotted curves labeled "approximate start of semilog
possible in practice to read Xe-"2S values much greater
radial flow on transition curves."
than unity. In such a case, the most permeable
In a buildup, conditions for the occurrence of the
medium behavior is not visible, and only the last
two parallel semilog straight lines on a Homer plot
CDe2S curve, corresponding to homogeneous behavior
are: (1) the preceding drawdown must be long enough
of the total reservoir, can be obtained in the test. This for total reservoir behavior to be reached-i.e., the last
may occur when the blocks in the least permeable CDe2S curve; (2) semilog radial flow must exist on
medium are very small and the well is hydraulically
the first CDe2S curve before transition occurs; and (3)
fractured; the double-porosity reservoir then would buildup time must be greater than the time required to
behave like a homogeneous one, with the
reach semilog radial flow on the final CDe2s


transmissivity of the most permeable medium, drawdown type curve. The existence of the two
and the total storativity.
parallel semilog straight lines on a Homer plot thus
Similarly, the double-porosity nature of the reservoir
requires a drawdown of adequate duration in addition
may remain unnoticed if Xe -2$ is small and the test
to the other conditions found for drawdown tests.
not long enough, so that only the CDe2S for the most
The duration of the drawdown is of primary
permeable medium is recorded during the test (up to
importance for the analysis of buildup tests in double-
Point A in Fig. 4). In that case, analysis can only
porosity reservoirs and controls the number of
provide the same parameters as with homogeneous
parameters that can be extracted from test data. Its
systems: k fh, C, and S. The value for S would in fact
impact on buildup test analysis is examined in detail
be a maximum if the total storativity, instead of that of
below.
the most permeable medium, is used in the skin
calculations. Buildup analysis. In practice, drawdown data are
difficult to analyze because they usually are perturbed
Finally, another alternative behavior is seen when
by variations of flow rate. As a result, analysis is
drawdown stops during transition. This case is
examined in detail in connection with buildup analysis. often made on buildup data only.
The type curves of Figs. 2 and 3 provide an Analysis of buildup data in double-porosity
explanation for the presence or the absence of the two reservoirs, however, is a lot more complicated than in
parallel semilog straight lines described by Warren and homogeneous formations. The main reason is that, for
Root 19 and of the semilog straight line during log-log analysis, log-log buildup type curves are
unrestricted interporosity transition flow. 13-15,17 required; drawdown type curves are usually inadequate
Because the pressure drop in double-porosity behavior because drawdown and buildup durations are often of
follows two homogeneous CDe2S curves in Fig. 2 and the same order, especially in exploration tests.
three homogeneous CDe2S curves in Fig. 3, Buildup type curves for a well with wellbore storage
respectively, two or three semilog straight lines may and skin in a double-porosity reservoir can be
be present if conditions for semilog radial flow are constructed as the drawdown type curves of Figs. 2
satisfied on each CDe 2S curve. and 3 by superposing buildup type curves for a well ,
In a drawdown test in a double-porosity reservoir with wellbore storage and skin in a homogeneous
with restricted interporosity flow, this requires reservoir with the transition curves. As a result,
transition during buildup occurs at the same Ap level


matching each of the two CDe2S drawdown curves in
Fig. 2 beyond the dotted line, which indicates the as in drawdown but at a later time, assuming, of
approximate start of semilog radial flow. It is obvious course, that duration of drawdown is long enough for
from Fig. 2 that the occurrence of the two semilog total reservoir behavior to be seen. If this is not the
straight lines requires a particular combination of case, the problem becomes even more complicated.
(CDe2S) f, Xe-25, and w. It depends not only on the Homer analysis is also more delicate than with
characteristics of the fluid and of the reservoir (X and homogeneous reservoirs and requires a lot of caution.

554 JOURNAL OF PETROLEUM TECHNOLOGY



Fig. 5 illustrates several possibilities found in practice.
Five buildup curves are shown on a Homer plot, each
computed for a different drawdown duration. The
corresponding log-log plots are presented in Figs. 6
through 10.
0
Fig. 6 presents an example of drawdown and
ri
buildup log-log plot when drawdown duration is long ;

enough for the total system to be seen. This is the a

most desirable situation. All well and reservoir -4


parameters can be extracted from drawdown data by
log-log analysis. In the same way, log-log analysis of
buildup data (Curve E in Fig. 6) with the
corresponding buildup type curve will yield all the -6
103 104 105 106 107
well and reservoir parameters (k fh, C, S, (,,, and X) if 102
1 tp At )/5t
the buildup test is long enough to reach the last CI)e2s
buildup curve.
Homer analysis, on the contrary, will yield all the Fig. 5-Typical double-porosity behaviors on a Horner plot.
parameters only if the two parallel semilog straight
lines exist (this is the case in the example selected for
preparing Fig. 5). If only the last semilog straight line
exists, only kfh, S, and p` can be obtained; i.e., the
fissuration parameters w and X are accessible only "ORAWDOWN TYPE-CURVES
through log-log analysis. In both cases, the intercept TRANSITION CURVE
^ BUILD-UP TYPE-CURVE
D
p` of the second semilog straight line represents the

• reservoir initial pressure.


Another case of interest is presented in Fig. 7.
Drawdown duration is such that only the most
permeable medium is produced: drawdown pressure
ff

^
data remain on the first CDe2S curve, the transition ^
c
curve is not reached. As discussed before, the double-
porosity nature of the reservoir cannot be diagnosed
from drawdown data, nor from buildup data, either on ro-I 1 10 102 103 104 ,o5 106

a log-log or on a Homer plot ( Curve A in Figs. 7 and DIMENSIONLESS TIME, IDICD

5, respectively).
Log-log analysis with the type curve for a well with Fig. 6-Double-porosity buildup log-log behavior when total
wellbore storage and skin in a homogeneous system is produced during drawdown.
reservoir21 can yield all homogeneous reservoir
parameters (k fh, C, and S) from either drawdown or
buildup data. S is only a maximum value if the total
system storativity is used in the skin calculations, ,0,
instead of that for the most permeable medium. If it is
known that the reservoir is a double-porosity system DRAWDOWN TYPE-CURVES
TRANSITION CURVE
- BUILD UP TYPE -CURVE
(e.g., from tests in other wells), a maximum value for 10 ----------
END Or DRAWDOWN -
X can be obtained by using the Xe -2S transition curve -------- ............
crossing the drawdown CDe2S curve for the most
permeable medium at a tolCo value corresponding to
the dimensionless production time. w cannot be
evaluated. ,0-1
.::

Homer analysis is possible only if semilog radial


flow exists on the first CDe2S curve for the most
permeable medium. If this is the case, as for Curve A 10-=
102 103 104 105 100
10-+ 1 10
in Fig. 5, it is possible to obtain kfh, S, and p".


DIMENSIONLESS TIME, tOICD
Again, S is only a maximum; p' represents the
reservoir initial pressure.
Fig. 7-Double-porosity buildup log-log behavior when only
The examples shown in Figs. 8 through 10 all the most permeable medium is produced during
correspond to drawdowns terminated during transition. drawdown.
In Fig. 8, the drawdown stops after transition has
APRIL 1984 555

p --- DRAYVOOWN TvPE.CURVES DRAWOOWN TYPE-CURVES
TRANSITION CURVE TRANSITION CURVE END OF DRAWDOWN
W ^ 9UIL0-UP TYPE-CURVE BUILDiJP TYPE-CURVE
10
0

uS ._. _......._.
ff
^ -
^
W
^ 10.I

101 1 10 102 103 pa ps Joe 10-1 1 10 102 103 104 10s 106
DIMENSIONLESS TIME, IDICD DIMENSIONLESS TIME, tDfCD

Fig. 8-Double-porosity buildup log-log behavior when Fig. 10-Double-porosity buildup log-log behavior when
drawdown stops in transition, drawdown stops in transition.

fact that the final semilog straight line, indicated by


w-
the label m=1.151 in Fig. 5, is not reached. There is
DRAWDOWN TYPE-CURVES
TRANSITION CURVE
another straight line, however, with a slope almost
W BUILD-UP TYPE CURVE
10 equal (at least in the example discussed) that could be
mistaken for the Homer semilog straight line. If used


fE for Homer analysis, this "wrong" straight line may
^ yield a kfh close to the actual one, but a wrong value
Q 00 . 000
of the skin and of p`. If p` is taken to represent the
Z
reservoir pressure, signs of depletion could be found
^ 104 000
s erroneously by comparison with other tests with
different drawdown durations.
1o-2 In Fig. 9, the drawdown stops in the middle of the
101 1 10 102 103 104 105 106
transition period on the ?,e -ZS curve. This case is very
DIMENSIONLESS TIME. V00
similar to that of Fig. 8 except that the constant
pressure portion during the buildup (Curve C)
Fig. 9-Double-porosity buildup log-log behavior when coincides with the transition curve on the log-log
drawdown stops in transition.
match.
As for the preceding case, log-log analysis yields
k fh, C, a maximum value for S (if total storativity is
used), and a maximum value for A. In addition, a
started but before the stabilized transition pressure, maximum value for co can be obtained from the
corresponding to the Xe -2S curve, has been reached. buildup CDe2S type curve passing through the last
The corresponding buildup (Curve B in Fig. 8) starts buildup point.
on the buildup curve for the most permeable medium Homer analysis (Curve C in Fig. 5) is similar to that
and then flattens out like a constant pressure boundary. with Curve B. In this case, however, the constant
The total reservoir CDe2S curve is not seen in practice pressure portion usually is well defined, thus giving a
whatever the buildup duration. minimum value of the reservoir average pressure, p. If
As the constant pressure portion on the log-log semilog radial flow is seen in the most permeable
occurs below the level of the Xe -2S curve, a medium (the "first" semilog straight line), a
maximum value of Xe -2S can be obtained by fitting a maximum value of w can be obtained from Eq. 5, with
Xe -ZS transition curve through these points. As with Sp=p-p f, p j being the intercept of the "first"
curve A discussed above, log-log analysis yields kfh, semilog straight line. In most practical cases, the
C, and maximum vaiues of S and A. buildup is not long enough to see the total system
On the Homer plot, the constant pressure portion
gives a minimum value for the reservoir average
pressure (Curve B in Fig. 5). If the buildup is too
short for this constant pressure effect to be seen, the
behavior. Even if it is long enough, the "second"
semilog straight line is not well defined.
Finally, in Fig. 10 the drawdown stops just before
reaching the total system curve. On the buildup type

buildup curve on the Homer plot is very similar to the curve (Curve D in Fig. 10), the pressure tends to
Curve A discussed before. The difference lies in the stabilize just above the Xe-2S curve, so a minimum
556 JOURNAL OF PETROLEUM TECHNOLOGY
q
value of Xe -2S can be found, in theory, by fitting a formations. I now think that the homogeneous model
transition curve through the constant pressure points. with a uniform-flux fracture, or any other boundary
In nondamaged or stimulated wells, it may even be condition, is not adequate for describing fissured
possible to find a unique combination of w and Xe -zs reservoirs. As a matter of fact, a number of tests
if distinct evidence of the total reservoir behavior can initially interpreted with the uniform-flux fracture have
be seen in the buildup data. As before, k fh, C, and a been reinterpreted with the double-porosity model.
maximum value of S are obtained from log-log Results were found to provide a much more realistic
analysis if the total storativity is used. description of the reservoir, as supported by other
From the Homer plot, the buildup appears like a knowledge, than that obtained with the homogeneous
constant pressure boundary effect in nearly all practical model with a uniform flux fracture.
cases (Curve D in Fig. 5). However, if the buildup is An efficient way to distinguish between
very long, the shape on the Homer plot is more homogeneous and heterogeneous behavior is to
characteristic of double-porosity behavior but the examine on a log-log plot the derivative of Ap with
"second" semilog straight line may not be well- respect to the natural log of At, in the case of a
defined. A minimum value of average reservoir drawdown, or with respect to the natural log of
pressure and a maximum value of w can be obtained Ot/(tP +At) in the case of a buildup, as a function of
as described before. At. Such a plot is characterized by a stabilization
during semilog radial flow. The shape of the derivative
for each behavior is drastically different, 24,25 with
Analysis With Pressure Derivatives. From the
double-porosity behavior exhibiting a characteristic
description of the various features of a double-porosity
hump below the semilog radial flow stabilization level
reservoir, it is evident many of the behaviors described
during transition, that allows unambiguous
in this paper can be analyzed by using a homogeneous
identification of the behavior, provided the quality of
model with appropriate boundary conditions. This is
pressure data is adequate (Fig. 11). The pressure
obvious for the cases illustrated in Figs. 5, 9, and 10

• with Curves C and D, when drawdown stops during


transition. Curves C and D could be analyzed in terms
of a homogeneous reservoir with a constant pressure
derivative also allows easy differentiation between an
infinite reservoir with double-porosity behavior and a
bounded reservoir with homogeneous behavior (whose
Ap vs. At traces are superposed in Fig. 12, although
boundary or in terms of a closed homogeneous
they correspond to very different kh values). In the
reservoir. In the same way, the last CDe 2S curve in
latter case, effects of boundaries appear above the
Fig. 4, corresponding to total reservoir behavior, could
semilog radial flow stabilization level, with a
be mistaken for a sealing fault in a homogeneous
stabilization at twice that level for a sealing fault.
reservoir.
Thus, there is often an alternative to the double-
Field Examples
porosity model that uses the homogeneous model and
attributes to boundary effects the features that The following presents several field examples to
characterize double-porosity behavior. Fortunately, the illustrate the various double-porosity behaviors
results of such an interpretation are often questionable, described in the first part of this paper. These
distances to boundaries are often ridiculous (usually examples have been selected from many tests with
less than 100 ft [30 m]), and reservoir sizes are double-porosity behavior that I have seen and are
incredibly small (often less than 40,000 sq ft [3716 fairly typical of what is found in practice.
m3]). Moreover, as discussed later, values for C and S Such examples are scarce in the literature. z,s, t t,zo,26
may suggest a fissured reservoir even if the analysis One main reason is that most authors were trying to
has been performed with the homogeneous model. illustrate the two parallel semilog straight line feature,
In some cases, results from interpretation with the which is the exception rather than the rule. The
homogeneous model appear reasonable. In such a following examples are used to introduce new
case, no choice can be made without additional information that cannot be derived from the theoretical
information. developments presented so far but have been
The homogeneous model has been used extensively discovered through experience.
and is still used for the analysis of fissured
reservoirs. 11 In fact, it was the only real tool available Fissured vs. Multilayered Reservoirs. It was stated
before knowledge of the double-porosity model in the beginning of this paper that the double-porosity
reached the state described in this paper. Among the model represents the behavior of both fissured and

• various possibilities, the homogeneous model with a


uniform-flux vertical fracture is certainly the most
popular. It had been foundzz,zs to describe reasonably
multilayered reservoirs with high permeability contrast
between the layers. As a result, it is not possible, from
the shape of the pressure vs. time curve alone, to
well the behavior of wells intersecting natural fractures distinguish between the two possibilities. All that can
and often has been used to analyze tests in fissured be diagnosed is a double porosity behavior.

APRIL 1984 557



- HOMOGENEOUS BEHAVIOR, INFINITE RESERVOIR
- - HOMOGENEOUS BEHAVIOR, SEALING FAULT
-- DOUBLE POROSITY BEHAVIOR, INFINITE RESERVOIR

PRESSURE
HOMOGENEOUS BEHAVIOR
u O
00^ ^^^OA ^
APPROXIMATE START APPROXIMATE START
OF INFINITE ACTING / OF INFINITE ACTING
0 HOMOGENEOUS BEHAVIOR HOMOGENEOUS BEHAVIOR
OOUBLE POROSITY
o° 10" BEHAVIOR (CO. 204- to$
c00. =sff.nr 1,510°
X.1% 510-10

l0's
1 10 to, 10, lo' l0°
fp/CD

Fig. 11-Derivatives for homogeneous and double-porosity


behavior.

LOG Q 1

Fortunately, experience shows that this distinction is Fig. 12-Derivatives for homogeneous behavior in a bounded
possible from the numerical values of the wellbore reservoir and double-porosity behavior in an infinite
storage constant C and of the skin S if the well is not reservoir.
damaged. This is illustrated in Figs. 13 and 14, and in
Table 1.
Figs. 13 and 14 present two examples of tests in
double-porosity reservoirs, performed before and after
an acid job. The details of the analyses are not shown,
with the transition curve in the test after acid. As a
result, w could not be found before acid, and only an

only the final log-log matches with the double-porosity upper limit was found from post-acid data.
type curve of Fig. 2. For each match, we have shown Let us now compare results before and after acid for
as a heavy line the double-porosity buildup (or each well. In Well I there is no variation in k fR, as
drawdoviwn, as appropriate) type curve fitted through should be expected, nor in w and A. C has increased
the measured pressure points; the initial and final from 0.016 bbl/psi to 0.025 bbl/psi; this 50% increase
CDe2S curves, corresponding to the most permeable could be attributed to the acid job. The skin has
and the least permeable medium, respectively, are decreased from +3.4 to -3.9, which indicates a
indicated as dashed lines, and the Xe -2S transition successful stimulation.
curves are shown as dotted lines. For clarity, the type In Well 2, on the other hand, k fh has decreased (but
curves of Fig. 8 are not shown, only those limiting the the pre-acid value is only approximate) while X
various zones (damaged, nondamaged, acidized, and remains the same. But C has increased by almost one
fractured wells). order of magnitude, from 0.017 bbl/psi to 0.13
Fig. 13 corresponds to unpublished buildup data bbl/psi. Skin has decreased from +3.4 to - 1.5.
from Well 1, whereas Fig. 14 presents drawdown data The increase in C after an acid job and the resulting
before acid and buildup data after acid from Test A in high value of the wellbore storage constant are
Well 2, whose analysis was presented in Ref. 26. characteristic of,fissured formations. Prior to the acid
From the plots in Figs. 13 and 14 there appears to job, when the well is damaged, most of the fissures
be no significant difference between the two series of intersecting the wellbore are plugged and the volume
tests from Wells 1 and 2, except that they match of the fluid communicating with the wellbore is just
different double-porosity type curves. However, the wellbore volume. The wellbore constant is thus
differences become apparent when one considers the equal to the one that could be computed from
numerical values of the parameters shown in Table 1. completion data if a value of fluid compressibility in
Because all the flow components could be identified the wellbore is available.
for Well 1 (initial and final CDe2S curve and transition After the acid job, on the other hand, fissures
Xe -2S curve), it was possible to extract all the well


become open to the wellbore and the volume of the
and reservoir parameters pertinent to the double fluid in direct communication with the well is equal to
porosity model from the test data (i.e., k fh, C, S, w, the wellbore volume plus the volume of the fissures
and X). On the other hand, the initial CDe2S curve, intersecting the well. The resulting wellbore storage
representing the most permeable medium, could not be constant may be one or two orders of magnitude
determined for Well 2 frorri the test before acid, due to higher than before acid.
lack of early-time data, and was found to coincide For this reason, downhole shut-in is not particularly
558 JOURNAL OF PETROLEUM TECHNOLOGY

IXJILV^/PTrM CURVES
- ORAM/OO.VN TYPE CURVE
I 1^1 DRAW00N11 DATA
- OwOb oworAT
- BUILD W TYPE CURVE
^ • ^ ^ BVILD UP DATA

p TRANSITION CURVE ...G.. l


1 wcco guufl ,.
W11L
sao^rro .rt^.

N ^ ............. ^ n:crw[o
,^ i ^ wtu
i i:
S^ 1

Yj
lW

>0;AAWD-C=
1" TYPE-CVRYE
oqe^ss

10.11- T0-1
10.1 10 107 103 104 10.1 1D 107 103 100
DIMENSIONLESS TIME,tDICO DIMENSIONLESS TIME. IDICD

Fig. 13-Well 1 type-curve match for test before and after acid Fig. 14-Well 2 type-curve match for Test A before and after
in a multilayered reservoir. acid in a fissured reservoir.

TABLE 1-COMPARISONS BETWEEN INTERPRETATION


RESULTS FROM WELL 1 AND WELL 2 (TEST A)

Well 1 Well 2, Test A


(Multilayered Reservoir) (Fissured Reservoir)
^ Before Acid After Acid Before Acid After Acid
krh, md-ft 565 565 416,600 347,000
C, bbl/psi 0,016 0.025 0.017 0.13
S + 3.4 -3.9 + 3.4 - 1.5
w 0.10 0.10 7 <-0.06
a 0.97x10- 1.0x10-5 3.6x10-5 3.6x10-5

useful in fissured formations, except maybe with is even at the limit between the regions for damaged
damaged wells. and nondamaged wells. Yet skins are negative: -3.9
On the other hand, there is no significant change in for Well 1 and -1.5 for Well 2.
the wellba•e storage constant following an acid job in In reality, double-porosity reservoirs exhibit
a multilayered reservoir. As a result, fissured pseudoskins, as created by hydraulic fractures. It is my
reservoirs can be distinguished from multilayered experience that a skin of around -3 is normal for
reservoirs with high permeability contrast between nondamaged wells in formations with double-porosity
layers by means of the numerical value of the wellbore behavior, Acidized wells may have skins as low as
storage constant, but only if the well is not damaged. -7, whereas a zero skin usually indicates a damaged
No distinction is possible from pressure and rate data well.
alone if the well is damaged. In the case of Well 2, the skin (-1.5 after acid)
In this case, it can be concluded that Well I is in a would indicate that the well is still damaged; as a
multilayered reservoir, whereas Well 2 is in a fissured result, some of the fissures communicating with the
formation. These conclusions are supported by well may still be plugged, and C could increase further
information from other sources. if a new acid job were performed.
Nondamaged or acidized wells in double-porosity
Skin Value for Nondamaged Wells. Another formations thus are characterized by a very negative
interesting property of double-porosity reservoirs skin. This is associated with a high wellbore-storage
(whether fissured or multilayered) is illustrated by the constant in fissured reservoirs. Conversely, a very high
examples in Figs. 13 and 14. Notice that for both wellbore-storage constant and a very negative skin
^-^ wells, the initial CDe2S curves, corresponding to the should suggest a fissured reservoir, even if the well
fissure system in Well 2 and to the most permeable exhibits a homogeneous behavior. In general, this
layer in Well 1, lie, after acid, in the nondamaged occurs when the test is too short, so that only the first
well region of the type curves, not in the acidized well CDe 2S curve corresponding to the fissures is seen in
region, as should be expected. For Well 2 (Fig. 14), it the test data. An example follows.
APRIL 1984 559

TABLE 2-COMPARISON BETWEEN INTERPRETATION
RESULTS FROM WELL 2 (TEST A), WELL 3, AND WELL 4
-- TRANSITION CURVE
FISSURE STST.CURVE Well 2
TOTAL 4VSTCURVE
• • • BUILD-UP DATA
Test A Well 3
After Acid After Acid Well 4
0 k,h, md-ft 347,000 2,260 90
^ C, bbl/psi 0.13 0.19 2 x 10 -3
S -1.5 -5.1 -4
_<0.06 ? ?
2 ^ 3.6x10-5 ? 2,5x10

102 103
(tp. At ) /-1t 15: the first CDe2S curve (A), the total system CDe2S
curve (B), and the transition curve. The "second"
Fig. 15-Well 2 dimensionless Horner plot for Test A after semilog straight line is reached by the buildup data.
acid. Fig. 16 presents drawdown and buildup data for
Well 3 of Ref. 26, corresponding to Curve A of Figs.
5 and 7. This is the case where drawdown stops on the
first CDeZS curve before transition is reached. As a
TD=
result, the data exhibit a homogeneous behavior and
- DRAWDOWN TTiE-CUNV[
there is no evidence of a heterogeneous system except
Q UL1flDRl1WDOWN DATA
from the value of the parameters listed in Table 2: C
n

10
- BWLD-UP TTPE-CURVE
• ^ ^ BUILD-UP DATA

_ _ _ .C D,IlD MLL

» i^ ciTUCD
(0.19 bbl/psi) is very large and the skin (-5.1) veiy
negative, thus suggesting a fissured reservoir. In fact,
some other wells in the same reservoir were found to
exhibit a double-porosity behavior. Consequently, S is

e
only a maximum value, since total storativity was used
W in the computations and a maximum value for X can
0 be computed (A < 3 x 10 -7 ).
/ DIIAWDDWN^^`RJE
The third example (Well 4 in Fig, 17) corresponds
to the case where drawdown was stopped during
10•1 1 ID 102 103 1wA
DIMENSIONLESS TIME, VCD
transition. As a result, buildup pressure in Fig. 17
becomes stabilized at long buildup times. Analysis was
performed as described earlier to yield kfR, C, a
Fig. 16-Well 3 type-curve match for drawdown and buildup maximum value for S, and a maximum value for X. A
data.
maximum value for w could not be evaluated because
of insufficient data at constant pressure (see the Homer
plot on Fig. 18). This case corresponds to Curve B in
Figs. 5 and 8, where drawdown stops after transition
Effect of Production Time on Buildup Behavior. has started but before the stabilized transition pressure,
This section illustrates the various buildup behaviors corresponding to the Xe-2S curve, has been reached.
described earlier in the text. Table 3 summarizes interpretation results for Well 2
The two tests in Fig. 13 for Well 1 and the test after (Fig. 14), Well 3 (Fig. 16), and Well 4 (Fig. 17). It
acid in Fig. 14 for Well 2 are examples of buildup shows clearly the dependency of results on the
tests where the total system is seen in the test data. duration of drawdown: all reservoir parameters can be
For these tests, the duration of the drawdown was obtained only if both drawdown and buildup are long
sufficient for the total system to be present in enough for total system behavior to be seen in the
drawdown data, but these were not adequate for test data.
analysis because of fluctuations in the flow rates.
As indicated in Table 1, it is possible to extract all
the parameters pertinent to the double-porosity model
from the test data.
Fig. 15 illustrates the Homer plot for the test after
acid in Well 2. This corresponds to Curve E in Fig. 5,
Variation of w and ). With Time. Discussion so far
has been based on the assumption that w and X were
constant. This is not always the case, especially when
reservoir pressure falls below bubble-point pressure.

except that there is no "initial" semilog straight line. The reason is that w and X both depend on fluid
All the various flow components are indicated in Fig. properties, not just on rock characteristics. w from Eq.
560 JOURNAL OF PETROLEUM TECHNOLOGY

BUILD-UP TYPE-CURVES
S l -^_--- _
O
Pnsaurebwn0
DOUBLE POROSITY TYPE-CURVE
p
W
>
7
u
bni
- •^^ DUILD -Up
--- FISSURE DATACURVE
SYST.

10
---
^
- ------ a
j
^

DRAWDDVJN TYPE,GURVE
homogaaous
10•^
10- 1 1 10 102 103 104
DIMENSIONLESS TIME, t D/CD

Fig. 17-Well 4 type-curve match of buildup data when 10 102 103 104
0".-IUiAt
drawdown stops in transition.

Fig. 18-Well 4 dimensionless Horner plot when drawdown


3 also can be written as: stops in transition.

w=1+ ' ............... (10)


(oV)m ( c,)m
TABLE 3-COMPARISON BETWEEN INTERPRETATION
(OV )f (cr).( RESULTS FROM TEST A (AFTER ACID) AND TEST B

• which clearly shows that w depends on the ratio of the


total compressibilities in both constitutive media. In
IN WELL 2

Well 2
Test A
Well 2
Test B
After Acid After Acid
the same way, X depends on k, which is very
k,h, md-ft 347,000 264,000
sensitive to gas saturation. C. bbl/psl 0.13 0.03
An example illustrating changes in w and X in the S -1.5 >, -0.7
same well is presented.in Fig. 19. Data in Fig. 19 W C0.06 0.43
come from Test B in the same Well 2 used for Figs. X 3.6x10-5 >1.9x 10-s
c,,,, 1 3
14 and 15. Tests A and B in Well 2 are discussed in cif 1 36
detail in Ref. 26. k,,, 1 1/21
As can be seen by comparing Fig. 19 with Fig. 14,
the buildup log-log behavior of Well 2 has changed
drastically between Test A and Test B. The data in
Test B exhibit a two parallel semilog straight-line
behavior, evident on the Homer plot of Fig. 20, example, no change in w and X after reservoir pressure
whereas in Test A, only the last semilog straight line has dropped below the bubble-point pressure would
was present (Fig. 15). The first semilog straight line in indicate that gas saturation is uniform in the reservoir.
Fig. 20 lasts 14 hours. This change is attributed to the Testing at regular intervals is therefore advisable in
presence of gas in the reservoir. Complete analysis of reservoirs with double-porosity behavior.
the data was performed in Ref. 26 and results are
listed in Table 3. It was possible to obtain not only
Summary and Conclusions
kfh, C, S, w, and X from both tests but also, using
additional information to find the size of the matrix The ambition of this paper is to establish the state of
blocks, the change in total compressibility in the the art in the knowledge of double-porosity behavior.
fissures and in the blocks and the change in matrix The information presented can be summarized as
permeability from which the gas saturation in the follows.
blocks could be evaluated. Note that the well has 1. Fissured reservoirs and multilayered reservoirs

• become damaged, as evidenced from the increase in


skin and the significant decrease in wellbore storage.
Variations of w and X with time usually indicate a
change in the fluid characteristics. No change may
with high permeability contrast between layers exhibit
the same double-porosity behavior.
2. Double-porosity behavior can be diagnosed by
log-log analysis of the pressure change during a test or
also provide additional reservoir information. For by its derivative.

APRIL 1984 561



010^
0
--- TRANSITION CURVE 1.25_ 6 10-6

FISSURE SYST CURVE (Cot 25I, _ 40 -- TRANSITION CURVE


G
G FISSURE SYST CURVE
^ TOTAL SYST CURVE ICOe2SI1.,,,e17
APPROXIMATE START 0 TOTAL syS7 CURVE
r__ OF TRANSrnON PERIOD . . . OUILD-U7 DATA
010
¢
W
O
APPROXIMATE ENO
OF TRANSITION PERIOD

APPROXIMATE START 6
OF SEMI-LOG STRAIGHT
LINE ON FISSURE
SYSTEM CURVE

11.
102 103 104 105
DIMENSIONLESS TIME 1D/CD
102 103 104 105 106
11 P . _101-It

Fig. 19-Well 2 type-curve match for Test B with reservoir


pressure below bubble-point pressure. Fig. 20-Well 2 dimensionless Horner plot for Test B.

3. Analysis of tests in reservoirs with a double- indicates a fissured reservoir even if the pressure
porosity behavior using the double-porosity type curve behavior appears homogeneous. This may occur when
of Figs. .2 and 3 can provide all pertinent reservoir the test is too short, so that only the fissure
parameters (kfh, C, S, w, and X), even if these are not homogeneous behavior can be seen. A longer test is
accessible by semilog analysis, on the condition that
drawdowns and buildups are long enough to reach
total system behavior. In most cases, however,
matching must be done with buildup type curves.
required to extract all the additional information (w
and X) needed to describe the reservoir fully.
8. w and X may change with time for the same well
depending on the characteristics of the reservoir fluid.

Once co and X are obtained, the total compressibility in Testing at regular intervals is recommended to obtain
the most permeable medium and dimensions of the the information associated with such changes.
least permeable medium can be computed if additional 9. If the drawdown stops during transition, buildup
information is available, such as the geometry, total behavior in double-porosity reservoirs is similar to that
compressibility, and permeability of the least in homogeneous reservoirs with a boundary.
permeable medium. 10. Interpreting heterogeneous reservoirs in terms of
4. The two parallel semilog straight-line feature may "equivalent" homogeneous reservoirs with inner or
or may not exist, depending on the well condition and outer boundaries appears inadequate.
characteristics of each medium. When it does exist,
the first semilog straight line may last for many hours.
5. Nondamaged wells in a double porosity exhibit a References
pseudoskin of around -3. Acidized wells can have 1. Barenblatt, G.E., Zheltov, I.P., and Kochina, I.N.: "Basic Con-
skins as loa as -7, whereas a zero skin usually cepts in the Theory of Homogeneous Liquids in Fissured Rocks,"
indicates a damaged well. J. Appl. Math. Mech. 24, 5 (1960) 1286-1303.
2. Warren, J.E. and Root, P.J.: "Behavior of Naturally Fractured
6. Fissured reservoirs can be distinguished from Reservoirs," Soc. Pet. Eng. J. (Sept. 1963) 245-55; Trans.,
multilayered reservoirs only if the well is nondamaged AIME, 228.
or acidized. 3. Odeh, A.S.: "Unsteady-State Behavior of Naturally Fractured
7. In multilayered reservoirs, the wellbore storage Reservoirs," Soc. Per. Eng. J. (March 1965) 60-66; Trans.,
AIME; 234.
constant corresponds to the volume of the wellbore,
4. Romm, E.S.; "Filtrasionnie Svoistsa Teschinovatich Porod (Flow
whatever the well condition. On the contrary, Phenomena in Fractured Rocks)," Nedra, Moscow, in Russian
nondamaged or acidized wells in fissured reservoirs (1966).
exhibit a very high wellbore-storage constant that 5. Kazemi, H., Seth, M.S., and Thomas, G.W.: "The Interpretation
of Interference Tests in Naturally Fractured Reservoirs With
includes the volume of fissures intersecting the well.
Uniform Fracture Distribution," Soc. Pet. Eng. J. (Dec. 1969)
This wellbore storage is usually one or two orders of


463-72; Trans., AIME, 246.
magnitude higher than that due to completion alone. 6. de Swaan 0., A.: "Analytical Solutions for Determining Natural-
As a result, downhole shut-in tools are ineffective in ly Fractured Reservoir Properties by Well Testing," Soc. Per.
such wells. Wellbore storage in damaged wells in Eng. J. (June 1976) .
7. Streltsova, T.D.: "Hydrodynamics of Groundwater Flow in a
fissured reservoirs is normal-i.e., corresponds to the Fractured Formation," Water Resources Res. (1976) 12, 3,
wellbore volume. Conversely, a negative skin 405-14.
associated with a high wellbore storage usually 8. Najuricta, H.L.: "A Theory for Pressure Transient Analysis in

562 JOURNAL OF PETROLEUM TECHNOLOGY



Naturally Fractured Reservoirs," J. Pet. Tech. (July 1980) P= characteristic length of a matrix block
1241-50. m= absolute value of semilog straight line slope
9. Kazemi, H.: "Pressure Transient Analysis of Naturally Fractured
Reservoirs With Uniform Fracture Distribution," Soc. Pet. Eng. n = number of normal sets of fractures
J. (Dec. 1969) 451-62; Trans., AIME, 246. p = pressure
10. Boulton, N.S. and Streltsova, T.D.: "Unsteady Flow to a Pumped PD = dimensionless pressure
Well in a Fissured Water-Bearing Formation," J. Hydrol. (1977)
35, 257-69. pi = initial reservoir pressure
11. Gringarten, A.C.: "Flow Test Evaluation of Fractured Reser- p`= extrapolated pressure from Homer semi-log
voirs," paper presented at the Symposium on Recent Trends in straight line
Hydrology, Geological Soc. of America, Berkeley, CA, Feb. 8-9,
1979. bp = vertical displacement of the two parallel
12. Mavor, M.J. and Cinco, H.: "Transient Pressure Behavior of semilog straight lines
Naturally Fractured Reservoirs," paper SPE 7977 presented at the Ap = pressure change
1979 SPE Califomia Regional Meeting, Ventura, April 18-20.
13. Cinco-L., H. and Samaniego-V., F.: "Pressure Transient q = flow rate
Analysis for Naturally Fractured Reservoirs," paper SPE 11026 Sq = interporosity flow per unit bulk volume per
presented at the 1982 SPE Annual Technical Conference and Ex-
unit time
hibition, New Orleans, Sept. 26-29.
14. Serra, K., Reynolds, A.C., and Raghavan, R.: "New Pressure
r = distance to production well
Transient Analysis Methods for Naturally Fractured Reservoirs," rw = wellbore radius
J. Pet. Tech. (Dec. 1983) 2271-83. S = van Everdingen-Hurst skin factor
15. Streltsova, T.D.: "Well Pressure Behavior of a Naturally Frac-
tured Reservoir," Soc. Per. Eng. J. (Oct. 1983) 769-80.
s = Laplace transform parameter
16. Muskat, M.: The Flow of Homogeneous Fluids 7hrough Porous t = time
Media, J.W. Edward Inc., Ann Arbor, MI (1946). to = dimensionless time
17. Bourdet, D. and Gringarten, A.C.: "Determination of Fissured tP = Homer production time
Volume and Block Size in Fractured Reservoirs by Type-Curve
Analysis," paper SPE 9293 presented at the 1980 SPE Annual V = ratio of total volume of one porous system
Technical Conference and Exhibition, Dallas, Sept. 21-24. to bulk volume

• 18. Uldrich, D.O. and Ershaghi, I.: "A Method for Estimating the In-
terporosity Flow Parameter in Naturally Fractured Reservoirs,"
Soc. Pet. Eng. J. (Oct. 1979) 324-32.
19. Warren, J.E. and Root, P.J.: "Discussion of Unsteady-State
a = block shape parameter
y= exponential of Euler's constant (=1.78)
X = interporosity flow coefficient
Behavior of Naturally Fractured Reservoirs," Soc. Pet. Eng. J. µ = viscosity
(March 1965) 64-65; Trans., AIME, 234. p =. fluid density
20. Crawford, G.E., Hagcdom, A. R., and Pierce, A.E.: "Analysis of
Pressure Buildup Tests in a Naturally Fractured Reservoir," J. 0 = porosity of one system
Pet. Tech. (Nov. 1976) 1295-1300. w = storativity ratio
21, Gringarten, A.C. et al.: "A Comparison Between Different Skin
and Wellbore Storage Type Curves for Early-Time Transient
Subscripts
Analysis," -paper SPE 8205 presented at the 1979 SPE Annual f = fissure
Technical Conference and Exhibition, Las Vegas, Sept. 23-26. m = matrix
22. Gringarten, A.C., Ramey, H.J. Jr., and Raghavan, R.:
f+m = total system
"Unsteady-State Pressure Distributions Created by a Well With a
Single Infinite-Conductivity Vertical Fracture," Soc. Pet. Eng. J. t = total
(Aug. 1974) 347-60; Trans., AIME, 257. D = dimensionless
23. Gringarten, A.C., Ramey, H.J. Jr., and Raghavan, R.: "Applied
Pressure Analysis for Fractured Wells," J. Per. Tech. (July 1975)
SI Metric Conversion Factors
887-92. bar x 1.0* E+05 = Pa
24_ Bourdet, D. et al.: "A New Set of Type Curves Simplifies Well bbl x 1.589 873 E-O1 = m3
Test Analysis," World Oil (May 1983). cp x 1.0* E-03 = Pas
25. Bourdet, D. et al.: "Interpreting Well Tests in Fractured Reser-
cu ft x 2.831 685 E-02 = m3
voirs," World Oil (Oct. 1983).
26. Gringarten, A.C. et al.: "Evaluating Fissured Formation ft x 3.048 E-01 =m
Geometry From Well Test Data: A Field Example," paper SPE in. x 2.54 E+00 = cm
10182 presented at the 1981 SPE Annual Technical Conference md-ft x 3.008 142 E+02 =gm2•m
and Exhibition, San Antonio, Oct. 5-7. x 6.894 757
psi E+00 = kPa
Nomenclature ' Conversion factor is exact.

B = formation volume factor APPENDIX


c = fluid compressibility Double-Porosity Solutions
c, = rock compressibility Warren and Root's solution2 was derived under the
^ c, = total compressibility assumption of pseudosteady-state interporosity flow and
C = wellbore storage can be written in the Laplace domain as
CD = dimensionless storage constant
KO[rDVii
s
h = formation thickness . (A-1)
PD(ro,s)=
k = permeability s,1sfts) K I [,[s-
fls) ]
APRIL 1984 563

for a finite well radius, and as for horizontal slab blocks, and

P.D(rD,s)=Ko[rD s] ................ (A-2) f(s)=w+

for a line-source well. In Eqs. A-1 and A-2, the Laplace


variable is based on the usual dimensionless time based 1 X 15(1-w)s 15(1-w)s
on total reservoir storativity. In engineering units: 1 oth 1l
5sC X X - J
0.000264kjt
(tD)f+m = . . . (A-3) V 15(1 V 15(1-(oo)s -11
1+25,,,,,D [ w)SCOth
(OfiCt)f+mµrw
X X J
pD is the dimensionless pressure, given by
.......... ...... .(A-10)
kfh
PD = OP j . . . . . . . . . . . . . . . . . . . . . (A-4)
141.2 qBµ
for spherical blocks, where S,,,,,D represents an inter-
and rD, the dimensionless distance to the production porosity skin.*
well axis: The effect of skin and wellbore storage on double-
porosity behavior was investigated by Mavor and Cin-
rD =r/rw . . . . . . . . . . . . . . . . . . . . . . . . . . . . . . (A-5) co. 12 Their solution was obtained in the Laplace domain
for pseudosteady-state interporosity flow and reads:
Ko and K) are the modified Bessel functions of the sec-
ond kind of zero and unit order, respectively. f(s) is in- PD(s)=
troduced by Warren and Root as
W(I -w)s+X Ko [ sf(s) J+S sf(s) K, sf (s) ]
./(s) _ . . . . . . . . . . . . . . . . . . . . . . (A-6)
(1-w)s+x s(-4flS)Ki [^[SASA +sCD[KO[ sj(.r)]+S sf(s)Ki[v'sfls)]j)

Using de Swaan's approach,6 Warren and Root's2


............. ....... (A-I1)
solution for pseudosteady-state intcrporosity flow can be
extended to transient interporosity flow by simply
replacing in Eq. A-I or A-2 thefts) function given in with f(s) given by Eq. A-6. CD is the dimensionless
Eq. A-6 by (7 wellbore storage constant, based on total reservoir
storativity and given in engineering units by:
3(1-w)s
f(s)=w+ tanh (A-7)
V Kw) 3s 0.8936C
X 2 . ......... (A-12)
CD=(G'D)f+m=
for horizontal slab blocks, and (0Vct)j+mhrw

X C 15(1: W)s 15(1- w)s


f(s)=W+-- -c.oth 1J As before, the corresponding solution for transient inter-
5 porosity flow is simply obtained by using f{s) from Eqs.
.........................(A-8) A-7, A-8, A-9, or A-10, as appropriate.
for spherical blocks.
In the case of interporosity skin, these become
'Cinco-Ley, H.: Personal communication (Oct. 28, 1983).

1 3(1-w)s
tanh
f(s)=w+ VE3s: 1\ Distinguished Author Series articles are general, descriptive presentations that
summarize the slate of the art in an area of technology by describing recent
V 12(1-w)s 3(l -w)s developments for readers who are not specialists in the topics discussed. Written by
I +SmQD tanh individuals recognized as experts in the areas, these articles provide key references
^ ^ to more definitive work and present specific details only to illustrate the technology.
Purpose: To inform the general readership of recent advances in various areas of
..............................(A-9) petroleum engineering.

564 JOURNAL OF PETROLEUM TECHNOLOGY


Chapter 7
0

GAS WELL TESTING

I. INTRODUCTION

The methods of pressure transient analysis discussed in


previous chapters were developed for liquid systems. These
same methods can be applied, with some modification, to gas
wells. Actually, we will find that most gas well testing
technology has evolved from techniques developed for oil
wells. For many years, there was little interest by the
^ petroleum industry in natural gas exploration and development;
this was due primarily to unrealistic price regulations placed
on natural gas. Accordingly, gas well testing technology
has developed only in the recent past. With increased energy
requirements and more attractive wellhead prices, current
interest in natural gas is very high.
The objective of this chapter is to present those
methods of pressure analysis most commonly applied to natural
gas wells. Further, we will discuss methods of predicting
the deliverability of gas reservoirs.
It was shown in Chapter 2, Eq. 2.42, that the radial
diffusivity equation for gas can be written in the form

a2pz + 1 ^P2 _ 1uct ^P2


(7'1)
r Dr 0.000264k at '
ar 2

It is instructive to compare this equation with the radial


diffusivity equation for liquids which was previously
presented as Eq. 2.26, i.e.,

^ + 1 9p _ ^uct DP
^^
(7. ^+ )

Dr ' r Dr 0.000264k at

^ Notice that the gas and liquid equations have exactly the
same form except pressure is squared in the S_as equation.
This is important because it suggests that these equations
might have similar solutions. At the same time, however,
the equations are vastly different. Whereas ct is
essentially constant for liquid systems, it is very
dependent upon pressure in gas reservoirs; this is because
of the high compressibility of natural gas. Also, gas
viscosity is very dependent upon pressure, whereas liquid
viscosity is essentially constant.
The liquid diffusivity equation is linear and can be
solved analytically; the line source solution, Eq. 2.31,
was developed in this manner. However, because of differ-
ences between the liquid and gas equations previously
noted, Eq. 7.1 is strongly non-linear. Mathematically, is

this is important because it means that Eq. 7.1 cannot be


easily solved using analytical methods; consequently, gas
flow equations cannot be developed in the same manner as
for oil.
Several early investigatorsl'2 showed, using finite
difference solutions, that the pressure response of gas
reservoirs is much like that of a liquid (oil) reservoir.
Because of this observation, and because of the similar
forms of the liquid and gas diffusivity equations, it was
proposed that, by analogy, liquid equations could be
modified to yield predictive equations for gas systems.
Accordingly, most of the equations presented in this chapter
are written by analogy from the liquid equations developed
in previous chapters.


7_ ^
I:I. PRESSURE nRllWnOtUN TEST

^
The equations used to analyze pressure drawdown tests
in gas wells are based on the same ideal model used to
analyze the pressure behavior of oil wells. Gas well behavior
is somewhat more complicated, however, becausP nf th^ h;t7h_
compressibility of j;as. Several methods of analysis have
been proposed by engineers and researchers, each varying
in the manner by which pressure dependent gas properties are
accounted Cor in the analysis. Three methods of analysis
widely used in industry today are the p2-method, p-method,
and real gas pseudopressure, m(p), method. Each of.these
methods will be discussed.

1. p2 - Method
The gas diffusivity equation, Eq. 7.1, is basically
analogous to the liquid diffusivity equation with p2 replacing
p. Carter2 solved Eq. 7.1 numerically and showed that for
infinite-acting reservoirs
^• ^= CanstQh^

2 5. SxlO"qj^zTpsc z^n^r'ni}e ^^}^,,^, (traasierl)


p2 kt
wf _ pi 7.sc 2 3. P-P^' @ t
[lo g 4u^trw

- 3.23 + 0.87s] (7.3)

where: q = gas flow rate, Mscf/D


T = reservoir temperature, °R (°F + 460)
psc = pressure at standard conditions, psia
Tsc = temperature at standard conditions, °R.

The fluid properties, u, z and ct are evaluated at pressure


p, where

[pi2 _ + 2
pwf
Z
( )
^ = ^ 7. 4
is

7-3
Experience and numerical studies indicate that the last
measured value of pwf should be used in Eq. 7.4 to compute
P.
If standard conditions are specified to be Tsc = 60 °F

and psc = 14.7 psia, Eq. 7.3 reduces to the commonly used
form

G)S p2 = p 2 _ 1637quzT [ log t+ log k


V^I wf i kh ^uE r2
tw

- 3.23 + 0.87s^. (7.5)

A. Limitations of p2 - Method
Equations 7.3 and 7.S are subject to three major
limitations:

1) It is assumed by the p2-equations that


pressure gradients around the we11h__re
cjy of the test well are small. Accordingly,
use of this method to analyze data from
wells with large pressure drawdowns can

result in significant errors. High
drawdowns can result from high flow rate
and/low permeability.
2) Laminar flow is assumed by the p2-equations,
whereas most gas wells experience turbulent
flow to some degree. Modifications for
turbulent flow will be discussed in a
subsequent section.
r1.\
3) The uz product is assumed constant at
the pressure defined by Eq. 7.4. This
effectively limits the applicability of
this method to p < 1500 psia. The
reason for this pressure limitation is
evident if the typical pressure behavior
of the uz product is observed; this
9tit3 ►^ behavior is illustrated by Fig. 7.1
which shows a plot of uz versus p for
a 0 . 66 grav i ty gas at a part i cu l ar
^ temperature. It is noted that uz is
approximately constant only for p < 1500
psia; other gases at different temperatures
will give slightly different curves, but
this pressure limitation holds true for
all gases.
q
7-4
i• 0.05

GAS GRAVITY = 0.66


REDUCED TEMPERATURE = 1.6 ,/
^
/

0.04

0.03
c_
CJ

0.02

0.01
CONSTANT
p
/
/
0
0 2000 4000 6000 8000 10000

p, I) s i.a

I;ig. 7.1: Relationship between uz and p


for a 0.66 gravi.tv, gas (After
Ref. 3) .

B. Permeability Determination
Equation 7.5 suggests that a plot of pwc vs t on
semilog paper will yield a straight line of slope

m 1637quzT
kh (7.6)

so that,
k 1637qu"zT
mh (7. 7)

This plot, and the required slope, is illustrated by Fi.g.


7.2.

E
7-5
•i
N
^•.
N S

SLOPE = m, PS12/CYCLE

0.1 1 10 102 103

t, hrs

Fig. 7.2: Typical p2 - plot for drawdown


test data.

C. Skin Factor
•i
Equation 7.5 can be rearranged to solve for the total
skin factor:

z 2
s= 1.151 pwfmpl - log kt 2+ 3.23 . (7. 8)
^uctrw

If pWf is evaluated from the semilog straight line at


t = 1 hr,

[ 22
#
s= 1.151 pl pi _ log k Z+ 3.23 . (7.9)
^ u c t rw

D. Evaluation of Turbulence Effect


Many gas wells experience turbulent flow. It is
recalled, however, that the gas diffusivity equation, as
L...^

7-6
well as Eqs. 7.3 and 7.5, are valid only for laminar flow.
^ The effect of turbulent flow is to cause an additional
pressure loss in the formation which is not accounted for
in the ideal pressure analysis equations. Therefore,
this pressure loss shows up in the test analysis as a
positive skin ' factor, i.e., st. Further, the pressure
loss due to turbulence and the corresponding skin factor,
increase as flow rate increases, i.e., as the degree of
turbulence increases.
The total skin factor was shown by Eq. 3.31 to be a
composite of several skin factors:
^ T(yow r ate)

s= sd + sr + sp + st + s f+ ssw' S (7.10)

The right-hand-side of this expression can be separated


into rate-dependent and rate-independent skin factors:

s = s' + st 'A (7.11)

where s' is the sum of the rate-independent skin factors.


Ramey`` has proposed that Eq. 7.11 can be rewritten
as

s = s' + Dq (7.12)

1^a=5-SE'. .. ,
where D is the turbulence coefficient. Inspection of
Eq. 7.12 indicates that s is a linear function of q, i.e.,
that the skin due to turbulence is a linear function of
rate. Consequently, if a gas well with turbulence is
tested at multiple rates, different values of s should be
obtained when the tests are analyzed. This observation
has led to the following suggested procedure for evaluating
turbulence:

01) Determine s from two or more independent


flow tests;

7-7
+ kr _ Sa=S^- . _ .. (wl^o St^

2) Plot s versus q on cartesian paper; this


plot is depicted by Fig. 7.3;


SLOPE = D
f,;

^./

5'

q, MSCF/D

^ Fig. 7.3: Plot of s versus q


to evaluate turbulence.

3) Compute the slope of the skin plot.


From Eq. 7.12, it is obvious that

D = slope;

4) Determine the intercept value of s;


this value is equal to s'.

The gas turbulence factor can also be measured in the


laboratory or predicted theoreticallys. However, these
methods are considered inadequate for the purpose of
evaluating pressure transient data.

E. Flow Efficiency
Flow efficiency is defined as
2 - ^2 - Q 2
pi I wf ps
E
2 2
pi Pwf

7-8
where pi represents the stabilized shut-in pressure
^ immediately before the test begins, and pwf can be
approximated by the flowing pressure at the beginning of
pseudosteady state. If the well does not achieve pseudo-
steady state during the test, the last measured value of
"pwf can be used in Eq. 7.13. The pressure change due to
..skin can be computed as

ops = - 0.87ms. (7.14)

F. Pseudosteady State Flow


The pseudosteady state gas equation is often used
for deliverability predictions. This equation can be
written in the following form for a cylindrical drainage
shape:

kh(pR - pwf)
a r (7. 15)
14 24^zT ( ln r^ - 0.75 + s' + Dq)
w

If a well experiences significant turbulence, it is


necessary that the turbulence skin effect be included in
the deliverability equation. It can be observed from
Eq. 7.15 that omission of this term will result in calcu-
lated flow rates which are too large. Application of
Eq. 7.15 to gas deliverability forecasting will be
discussed in a subsequent section.

Example 7.1: Analysis of pressure drawdown data using the


p2-method

Problem. A gas well has been shut-in, and the pressure


stabilized at 3150 psia. The well was placed on production
at essentially a constant rate of 1000 Mscf/day for 10 hours.

7 -9
The recorded bottomhole flowing pressures are tabulated
below. It is necessary to compute the formation skin and
permeability. 0

r = 0.33 ft
w
T = 195 °F
h = 25 ft
S = 70%
g
= 15%
Gas Gravity = 0.70

t, hrs. pwf, psia pwf, psia2

1 3095 9.579x106
2 3092 9.560x106
3 3091 9.554x106
4 3090 9.548xl06


5 3089 9.542x106
6 3089 9.542x106
8 3088 9.536x106
10 3087 9. ^30x106

Solution. The pressure in this reservoir is too high to


use the p2-method; however, for purposes of illustration,
we will analyze the data using this method.
A semilog graph of pwf versus t is presented in Fig.
7.4. As is evident, the data form a straight line. The
values of i, u and ct must be evaluated at reservoir
temperature, T, and pressure, p, where

p = (31502 + 30872^2
2

3119 psia.

7-10
I*
. . . . . . - - . . 1 ,

9.59 -------
T

--- -
p2 = 9.5751x106 PSIA2
9.58 lhr

9.57
I Ii i, ,^.11- * Iii i i i i

m = -4.63x10'` PSI2/CYCLE

^D
0
9.56

Ln

9.S5
N3

i •
9.54

9.53

9.52
• 1 2 4 6 8 1C

t, hours

Fig. 7.4: Semilog plot of drawdown data, Ex. 7.1.


7-11
Gas properties were evaluated at this pressure to be:

0.88
0.0215 cp
cg = 2.68 x 10psi-1.

Thus, ct = Sgcg =(0.7)(2.68 x 10-4) psi-1

ct = 1.88 x 10-4psi-1

The slope of the drawdown plot is m =- 4.63 x 104


psi2/cycle. Therefore, from Eq. 7.7

k = - 1637q5zT
mh

k = - (163-1)( 1000) ( 0.021S) ( 0.88)(195 + 460)


(-4.63 x 104) (25)

k = 17.5 md.

From Eq. 7.9,

r 2
s= 1.151 plhr_pi _ log
2
k
2
+ 3.23
m tw
r

(9•S751 - 9.9225) x 106


S 1.151
-4.63 x 10``

17.5 + 3.23
- log
(0 .15) (0.0215) (1.88 x 10 4) (0 .33) 2

s = 2.7


7-12
2. p - Method
When the reservoir pressure is greater than 3000

is psia, it is generally best to use a p - plot rather than


a p2 - plot. It is observed in Fig. 7.1 that for p > 3000
psia the plot of pz versus p forms an approximate straight
line for which uz/p is essentially constant. Under these
conditions, the gas equation is written as:

162.6x103quBg k
pwf = pi - kh log t+ log ^uc r 2
t w

- 3.23 + .87s] (7.16)

where fluid properties are evaluated at pressure, P.


defined as

P _ pi + pwf (7.17)
2

The gas formation volume factor, B, must also be computed


,. g
at p, i.e.,

z
BBg = psc RB/scf. (7.18)
5.615Tscp

A. Limitations of p-Method
This method is similar to the p2-method in that it
assumes pressure gradients to be small, and flow to be
laminar. However, this method should only be used at
pressures greater than 3,000 psia.

B. Permeability Determination
The analysis of a drawdown test using this method is
exactly analogous to liquid systems. According to Eq.
^ 7.16, a plot of pwf versus t on semi-log paper will yield
a straight line of slope

7-13
16 2. 6x10 3quB
m = - kh (7.19)

is
so that,

k = - 162.6xl03quBg
(7.20)
mh

C. Skin Factor

pi - log k + 3.23 (7.21)


s = 1.151 plhr m ^uCtrw2

D. Flow Efficiency

E = pi pwf Aps (7.22)


pi pwf

Lp
s

3. Real Gas Pseudo-pressure, m(p)


The pz - and p - methods are only approximate solutions
to Eq. 7.1. These solutions do not properly account for
changes in z and cg with pressure. Further, the methods
assume that pressure gradients are small. With increasing
gas prices and better fracturing techniques, some of the
most attractive gas producing regions have very low perme-
ability. In order to produce these wells at economic rates,
pressure drawdowns must be large. Under these conditions,
the methods discussed thus far can give misleading results.
In Chapter 2, we combined the continuity equation with
Darcy's Law to obtain the following relationship which
describes the radial flow of any fluid in a homogeneous,
constant thickness porous media:
0

7-14
1a Frkp jp 1= a (^p)
(7.24)
ar ^p arJ at
• This expression can be combined with the equation of state
for gas density, Eq. 2.36, and the coefficient of gas
compressibility, Eq. 2.40, to obtain

cpc p
(7.25)
r ar ^r ^ ^] = kZ ^

In 1966, Al-Hus
_ sainy, et al.6'' introduced the concept
of the real gas pseudo pressure, m(p). This function is
defined as

m(p) = 2 ^ dp, psi2/cp (7.26)


f p
p
pb

where Pb is an arbitrary low base pressure which, for


convenience, is usually selected to be zero.
For an isothermal system, u and z are functions
only of p; consequently,

am) = 2p
ap uz . (7.27)

Moreover, from the chain-rule of differentation,

am(p) = am(p) 3p
(7.28)
ar 3p ar

so that,

ap uz am(p)
ar ar (7.29)
2p


7-15
The time derivative of the pseudo-pressure function
is
0
am(p) dm(p) ap (7.30)
at ap at

Combining Eqs. 7 . 26 and 7.30,

ap = uz am(p) (7.31)
at Zp at

Equations 7.29 and 7.31 can be substituted into Eq. 7.25


to obtain

a2 m(p) + 1 am(p) = ^uct am(p) (7. 32)


r ar k at '
a2

Notice that Eq. 7.32 has exactly the same form as the
liquid diffusivity equation, Eq. 7.2, except it is written
in terms of m(p) rather than p. It is also important to ^
observe that it does not contain the limitation that
pressure gradients must be small. Further, since u and z
are integrated as a function of pressure, there are no
Limits on the pressure range to which the equation is
applicable. It is apparent therefore, that Eq. 7.32 is
a more rigorous equation than either the p- or p2-
diffusivity relationships.
Before considering the solutions of Eq. 7.32, we will
determine how to convert p to m(p).

A. Evaluation of m(p)
The real gas pseudo-pressure is not widely applied
because most engineers, after one look at the integral in
Eq. 7.26, decide it is too complicated to use. This is not
true; use of the m(p) method requires only one additional
step -- the conversion of p to m(p). The objective of this
section is to show how this is accomplished.

7-16
For a particular gas gravity and reservoir temperature,
^ the relationship between p and m(p) can be obtained using
the following procedure:

1) Determine u and z as functions of pressure


for the entire range of pressures involved
in the test analysis. Pressure increments
of 50-100 psi are normally adequate.
Correlations relating u and z to p are
presented in Appendix A.
2) Compute 2p/uz for each pressure in Step 1.
3) Compute m(p) as a function of pressure
using numerical integration. Figure 7.5
depicts a plot of 2p/pz versus p. In
order to compute the value of m(p) at some
pressure pl, it is necessary to compute
the area under the curve between Pb and pl.
This area, A1 is equal to

p
A1 = 1 ^ dp ( 7 . 33)
f
Pb

• If the pressure increment, pl-p , is


sufficiently small, the area caR be
assumed to be a trapezoid. The values
of m(p) at other pressures can be
determined in a similar manner. Pro-
cedures to compute m(p) are presented
in the literature3'e
4) Plot m(p) versus p. This plot is depicted
by Fig. 7.6.

A second method of evaluating m(p) utilizes pseudo-


reduced pressure and temperature. Tables of m(p) versus
pseudoreduced temperature and pressure are presented in
the literature? '6'9 This method is time consuming and
should only be used if a computer program is not available.


7-17
=^1 - f
pl
^ ap •
b

Pb p1
p, psia

Fig. 7.5: Determination of m(p) by


Numerical integration.


U
T = Constant
N
.^
^ yg = Constant

0 1

p, psia

Fig. 7.6: Typical plot of m(p) versus p.

7-18
0 Example 7.2: Calculation of real gas pseudopressure, M(p)7

Problem. Given below as a function of pressure are the gas


deviation factor, z, and viscosity, u, of a natural gas.
Compute the value of m(p) corresponding to each pressure.

p, psia z u, cp

0 1.00 -
400 0.95 0.0117
800 0.90 0.0125
1,200 0.86 0.0132
1,600 0.81 0.0146
2,000 0.80 0.0163
2,400 0.81 0.0180

Solution. Table 7.1 contains a summary of calculations


necessary to compute m(p). A plot of (2p/uz) versus p is
presented in Fig. 7.7 along with a plot of m(p) versus p.
Pressure increments of 400 psi were used to compute the
area under the curve of (2p/uz) versus p; it was assumed
that this pressure increment was sufficiently small that the
area defined by each pressure interval could be approximated
as a trapezoid.

0
7-19
Table 7.1: Calculation of m(p) versus p, Ex. 7.2

2p/pz MEAN Ap (2p/uz)MEAN m(p)


p p
(psia) z (cp) (psi/cp) 2p/pz (psi) x (Ap) (psi2/cp)

0 1.0 - 0 - - 0 0

400 0.95 0.0117 71,975 35,988 400 14.4x1.06 14.4x106

800 0.90 0.0125 142,222 107,099 400 42.9x106 57.3x106

1,200 0.86 0.0132 211,416 176,819 400 70.7x106 128.0x106

1,600 0.81 0.0146 270,590 241,003 400 96.5x106 224.5x106

2,000 0.80 0.0163 306,748 288,669 400 115.5x106 340.0x106

2,400 0.81 0.0180 329,218 319,000 400 127.6x106 467.6x106

^ ^ ^
• • •
5

c o

X x 3
r-. a.
U U

N • r-{
•^ ^
^ Q

^++

[^3
2
p, \
h--^

0 0 400 800 1200 1600 2000 2400

p, psia

Fig. 7.7: Plot of m(p) and 2p/pz versus pressure, Ex. 7.2.
B. Pressure Drawdown Equation
Equation 7.32 can be solved' for the transient pressure
behavior of a constant rate well as

5 . 8 x 10 "aTp log kt
sc 2
m(pwf) = m(pi) khT
sc ^uictirw

- 3.23 + 0.87s] (7.34)

where s = s' + Dq. Notice that viscosity, ui, and total


compressibility, cti, must be evaluated at pi. When Psc
14.7 psia and Tsc = 60 °F, this expression reduces to

kt 2
m(pwf) = m(pi) - 16khqT log
^uictirw

- 3. 23 + 0. 87s] . (;

C. Permeability Determination
Equation 7.35 indicates that a plot of m(pwf) versus t
should yield a straight line on semilog paper with slope
equal to

m = - 16k7T
h (7.36)

The procedure for preparing this plot is:

1) Record the drawdown pressure, pwf, in


normal manner;
2) Convert measured pressures to corresponding
values of m(pwf)'
3) Plot m(pwf) versus log t.


7-22-
A typical drawdown plot using m(p) is illustrated by Fig. 7.8.
It is observed that this graph exhibits the same general shape
and behavior as a pressure drawdown plot constructed using p
or p2. After the slope is determined,

k = - 16s7cT
(7.37)
mh

••



a •
U


^
N

r"1

`..^
^

• SLUYL = m, Y51`/l:Y/l:YI;LL

0.1 1 10 102 10 3

t, hrs

Fig. 7.8: Semilog plot of m(pwf) versus t.

D. Skin Factor

m(plhr) -m(pi) k
s = 1.151 log + 3.23 (7.38)
m ^ultlrw2


7-23
E. Flow Efficiency

m(pi) - m(Pwf) - Am(P) s


E= (7.0
5
m(pi) - m(pw f)

Am(p)s = - 0.87ms (7.40)

F. Pseudosteady State Flow


In terms of m(p), the pseudosteady state radial flow
equation is:

6
19.9 x 10 hkTsc[m(pR) - m(pwf)]
q= r (7.41)
TPsc [in re - 0.75 + s' + Da]
w

When psc = 14.7 psia, and T = 60 °F,

hk[m(PR) - m(Pwf)]
a= . (7.4J
r
1424T r ln re - 0. 75 + s' + Dc{^
L w

These equations are excellent for use in the prediction of


gas well deliverability. It is also possible3 to estimate
the drainage pore volume of a well when pseudosteady state
data are available.

G. Advantages of m(p) Method


It is recommended that the m(p) -method be used for all
gas well pressure analysis. This method is not subject to
any of the major limitations inherent in either the p2- or
p-method of analysis. In particular, this method does not
assume that pressure gradients are small in the reservoir,
and it does not require that gas properties be constant at
some specified pressure. Further, and perhaps the most
important advantage, the m(p) method is applicable to all
pressure ranges.

7-24
Example 7.3: Analysis of pressure drawdown test using real (f
gas pseudopressure, m(p)

Problem. A well producing dry gas was shut in and the


stabilized shut-in pressure was recorded to be 3,150 psia.
The well was placed on production for ten hours and the
rate maintained at a constant value of 2,000 Mscf/day with
the aid of an adjustable choke. Given the reservoir fluid
and rock properties, as well as the drawdown bottomhole
pressure data, it is necessary to determine the formation
permeability, total skin -factor, and flow efficiency. Figure
7.9 presents a plot of m(p) versus pressure for the reservoir
under study.

pi = 3,150
Formation Temperature = 195 OF
^ h = 25 ft.
= 0.115
yg = 0.70
r = 0.33 ft
w
Drainage area = 640 acres
Viscosity at pi = 0.0210
Total compressibility at pi = 2.81X10-4psi-1
z-factor at pi = 0.91

t, hrs pw f, psia m(pw f) , psi2/c p(Fig. 7.9)

1 3038 6.090 x 108


2 3033 6.079 x 108
3 3032 6.073 x 108
4 3030 6.069 x 108
5 3029 6.065 x 108
10 3027 6.054 x 108


7-25
n
r)

Ln
CIA

0
0
0
N Cb l^
.r{
N
x
w
C) ^a
0
^ ^
^
V)
In
a^

0 V)
0
^
^

F=
0
0
LO

bo
.^
o
h, \p LO d tN7 N o
Solution. Figure 7.10 presents a semilog plot of m(pwf)
versus time. As can be seen, a straight line exists with a
slope of -3.60 x 106 psi2/cp/cycle. The effective perme-
ability to gas can be estimated from Eq. 7.37 as:

k = - 1637 T
g mh

k = - (1637) (2000 MSCF/day) (655 °R)


g (-3.60x106)(25 ft.)

kg = 23.8 md.

Further, the total skin factor can be calculated from Eq.


7.38 as:

m(plhr) - m(pi) k + 3.23


s = 1.151 m
- log
lCtlrw2

6.090 - 6.490) x 108


s = 1.151 -
-3.60x106

23.8 _4
- log + 3.23
(. 115) (.021) (2. 81x10 ) (. 33) Z

s = 6.7

The flow efficiency can be estimated from Eq. 7.39. Although


m(pwf) should be analyzed at the start of pseudosteady state,
it is necessary in this case, since the test was not run
long enough to reach pseudosteady state, to use the last
measured value of pwf. Therefore,


7-27
6. 11

6. 10

6.09

6.08
U

^
N
k=-
6.07

E
- ^ - -t-..
•i
---^-
6.06

6.OS

6.04
1 Z 4 6 8 10

t, hrs

Fig. ?.10: Semi log plot of rn(p^-) versus t, Ex. 7.3.

7-28
m(pw f) = 6.054 x 108 psi2/cp

0 :^m(p)s = -0.87(m)s

= - (0.87)(-3.60x106)(6.7)

Am(p)s = 0.21 x 108 psi2/cp

and, from Eq. 7.39,

m(pi) - m(pw f) - Am(P)


E_
m ( pi ) - m pwf

(6.490 - 6.054 - 0.21) x 108


(6.490 - 6.054) x 10a

E = 0.52.


III. PRESSURE BUILDUP TEST

The most common pressure transient test used for reservoir


analysis is the pressure buildup test. As in the case of
pressure drawdown tests, three methods of analysis are commonly
used.

1. p 2 - Met ho d
This method is subject to the same limitations stated
previously for the pressure drawdown test. In particular,
you are reminded that this method is limited to pressures
less than 1500 psia. A pressure buildup test can be analyzed
^ by several different methods which include those developed by
Horner, NIi11er-Dyes-Hutchinson, Muskat and Agarwal. The Horner

7-29
method is the most commonly used, and is covered first.

A. Horner Analysis
The Horner buildup equation can be written, with the

aid of superposition, to be:

+At
z _ 2_ 16 37auzT lo t^' (7.43)
pws pi ch g At

where all gas properties are evaluated at

pwf(dt=0) + p*2
(7.44)
P 2

a. Permeability
Equation 7.43 indicates that a Horner plot of pk,s
versus (tp+At)/At should yield a straight line as long as
at is sufficiently small that boundaries do not affect the
data. This line should have a slope

0
m = - 1637quzT (7.45)
kh

from which the permeability can be computed as

k = - 1637quzT (7.46)
mh

b. Skin Factor
Equation 7.43 can be combined with Eq. 7.3 to obtain

[PfAt=oPs ktp4t
s = 1.151 m - log 0
^uctrw2 (tp+Ot)

+ 3.23] . (7.47)

When tp >> At such that tp + At = tp,



7-30
rPwf_0) -Pws At
s = 1.151 - log + 3.23 . (7.48)

• ^ m $uctrw
2

Finally, if pWS is evaluated from the Horner straight line


at At = 1 hour,

2 Ot=O
Pwf( m) plhr _ log k Z+ 3.23 (7.49)
s = 1.151
^uctrw

c. Average Reservoir Pressure


With appropriate modifications, the methods used to
determine PR in oil reservoirs can also be applied to gas
reservoirs.

1) Matthews-Brons-Hazebroek
The MBH correlations for determining PR
were presented in Figs. 5.8 through
5.11. These curves can also be applied
to gas well test data if the ordinate

• of the graphs are relabeled as

pDMBff = -2.303(p*2 - PR)/m.

The value of p*2 is obtained by


extrapolating the Horner straight
line to a time ratio of unity.
2) Odeh-Al Hussainy
While the MBH method applies to any set of
test conditions, this method is restricted
to wells which flowed to pseudosteady state
before shut-in. Other limitations are
discussed in Chapter S. The Odeh-Al Hussainy
correlations were previously presented for
liquid systems in Figs. 5.13 and 5.14. These
same correlations can be used for gas provided
the ordinate and abscissa of the graphs are
relabeled as:

P*2 - P?
ORDINATE: 1
m

. PR pi
ABSCISSA:
m

7-31
d. Flow Efficiency
Flow efficiency is computed using the same equation
presented in Chapter 5 for liquids except all pressures are
squared:

-2 2
pw f (At=0) - Ap
PR s
E (7.50)
-2 2
pwf(At=0)
PR

When pR is not known, an approximate value of E can be


obtained using p*, i.e.,

p*2 - pwf(At=0) - ^Ps


E (7.51)
p*L pwf(Gt=O)

The effect of skin in Eqs. 7.50 and 7.51 is accounted


for by the term

Op , = - 0.87ms. (7.5
s

Example 7.4: Horner analysis of a gas well buildup test


using the p2-method

Problem. A well producing gas and a small amount of


condensate was shut in for a buildup test and pressure was
recorded as given. Reservoir pressure is above the dew
point pressure and no condensate is believed to be present
in the formation. The flow rate of gas, plus the gas
equivalent of produced condensate, was approximately 7,500
Mscf/D prior to shut-in. Cumulative production since last
shut-in is 1.695 NNMscf, and the average pressure at last
shut-in was 2,250 psia. The well is located 3,960 feet from
each of two adjacent wells. Estimate the permeability, total
skin factor, and average reservoir pressure.


7-32
r 1,980 ft.
e
0.25 ft.
rw
^ 60 ft.
620 °R
0.65
Y9
.^ ^ 6.7%
S 7S%
9

t_AG p _ 1 , 695,000 Mscf 24


hr = 5 424 hours
p q 7, 500 Mscf/day day '

t +ot
At, hrs pWS, psia pWS, psia2x106
Apt

0 -- 1095 1.199
2 2710 2031 4.125
3 1810 2041 4.166
5 1086 2027 4.109
7 776 2033 4.133
10 543 2038 4.153
15 363 20S7 4.231
20 272 2076 4.310
24 227 2081 4.331
30 182 2098 4.402
36 152 2102 4.418
42 130 2107 4.439
48 114 2110 4.452
60 91 2114 4.469
72 76 2118 4.486
84 66 2121 4.499
96 58 212S 4.516
108 51 2128 4.528
120 46 2131 4.541
132 42 2133 4.550


7-33
Solution. From Fig. 7.11, the slope of the Horner semilog
straight line is 2.33 x 105 psi2/cycle. Extrapolation of th.is
line to a time ratio of unity cannot be read directly from
this graph; however,

*2 - 2 - m
p - pws
t +At
-^ = 10

p*2 = [4.693 x 106 - (-0.233 x 106)]

p*2 = 4.926 x 106 psia2

and,
p* = 2219 psia.

Thus, from Eq. 7.44,

P= p*2 + p2 z- (2219) 2 +(1095) 2 Z


2 2 ^

12
p = (4.926 + 1.199) x 106
2

p = 1750 psia.

At 1750 psia and 620 °R,

0.017 cp

z = 0.83
_ i
ca = O.OOOS26 psi
_4 -1
ct = (.75) (0.000526) = 3.95 x 10 psi .

Permeability is computed using Eq. 7.46, i.e.,


0

7-34
• 4.7

4.6

4.5

10
0

4. 4

^
N 3

4.3

4.L

4.1
1 1() 1()z 103

t +At
p
At
Fig. 7.11 Horner plot, Ex. 7.4.
k = 1637qUzT
mh

k = - (1637) (7500) ( 0.017) ( 0.83) (620)



(-2.33 x 105) (60)

k = 7.7 md.

We normally use the skin equation which requires plhr


at At = 1 hour, i.e., Eq. 7.49. For this example problem,
At = 1 hour corresponds to a time ratio of 5425; thus, plhr
cannot be readily evaluated from Fig. 7.11. However, pWs
can be determined at At = 10 hours (tp+At/At = 543.4) , and
Eq. 7.48 can be used to computes s:

wf10hrs p - log g + 3.23 .


s= 1.151 m ^uc trwz
[^-

From the straight line on the Horner plot,

pl0hrs '^• 2`^0 x 106 psia`

and

1.199 - 4.290) x 106


s= L 1S1 (
-0.233 x 106

(10) (7. 7) + 3.23


- log
(0.067) ( 0.017) ( 0.000395) (.25) 2

s = 8.1. V, I

This large value of skin suggests that the well may have
formation damage. However, recall that many other factors
can contribute to the total skin factor; it would be
necessary to evaluate these factors before it could be

7-36
concluded that the well is damaged.
In the absence of other information, it will be
assumed that the well is centered in a reservoir of square
geometry; accordingly, the drainage area is

a=(3960 ft) 2 = 1.568 x 10' ft2

The average pressure can be evaluated from the appropriate


MBH curve on Fig. 5.8. The value of the abscissa on this
plot is

_ 0.0002637kt
tDA ^uCtA

_ (0.0002637) (7.7) (5424)


(0.067)(0.017)(3.95x10- 4) (l.568x107)

t DA = 1.56.

• Assuming this well is centered in a square drainage area, it


is found from Fig. 5.8 that PDN1BH = 3.89; therefore,

PR - P*2 pR - 4.926x106
= 3.89
m -0.233 x 106
2.3 3 2.3 3

so that,

pR = 4.532 x 106 psia2

an d
PR = 2129 psia.

The static reservoir pressure at the time the reservoir

• was last shut in, pi, was 2,250 psia; thus,

7-37
p` = 5.063 x 106 psia2
i

and 0
p*2 pi _(4.926 - 5.065) x 106 = 0.59
m -0.233 x 106

Entering this value on the abscissa of the appropriate


Odeh-Al Hussainy graph, i.e., Fig. 5.13, the corresponding
value of the ordinate is

PR pi
m = 1.54

pR = pi + 1.54m

pR = 5.063 x 106 +(1.54)(-0.233 x 106)


PR = 2169 psia.

The difference in pressures obtained by the MBH and


Odeh-Al Hussainy methods is 40 psi. This could be due to
the fact that the drainage area used in the MBH method was
not accurately known. Moreover, reservoir parameters such
as porosity, permeability, fluid viscosity, and system
compressibility could be different from those used. It is
also possible that the value of pi used in the Odeh-A1
Hussainy method was not accurate, or that the drainage area
of this well had changed since that value was determined.

B. Miller-Dyes-Hutchinson Plot
The MDH plot can be used to analyze test data in lieu
of the Horner plot; this plotting procedure was discussed
in Chapter S. Since it offers no advantage over the Horner
plot, it will not be discussed further.

7-38
C. Agarwal Method
The Agarwal method10, introduced in Chapter S for
^ liquid reservoirs, can also be applied to gas wells. This
method requires that all plots be prepared using a new time
function, Ate, referred to as equivalent time, and defined as

t At
Ate = (7.53)
+
P

Applications of this method to gas wells are discussed in


Ref. 10. The greatest advantage of this method over other
available techniques is in the analysis of data from wells
with short producing times.

2. p-Method
This method is subject to the same limitations as
previously discussed for drawdown testing. It is recommended
that this method not be employed when reservoir pressure is
less than 3000 psia.

A. Horner Analysis
The Horner equation, written in terms of p, is basically
the same as for liquid systems;

162.6 x 103QUB t +At


g log (7.54)
Pws pi kh At

where:

Bg = ZpscT RB/scf (7.55)


5.615pTsc

p = pwf(At'0) + p* .
(7.56)

^ Equation 7.54 suggests that a semilog plot of pws versus


(tp+At/dt) will yield a straight line during transient flow

7-39
with slope
^.
162.6 x 103quB 7
m = - ° (7.5
kh

a. Permeability
Permeability can be computed from Eq. 7.57 as:

k = - 162.6 x 103quB9(7.58)
mh

b. Skin Factor
Equations 7.16 and 7.54 can be combined to yield the
following relationship for total skin factor:

pf(at=0) - plhr k
s = 1.151 - log 2+ 3.23 . (7.59)
m
fluctrw

c. Average Pressure
Average pressure can be determined using the Matthews-
Brons-Hazebroek, Odeh-Al Hussainy, or Muskat methods using
the same procedures employed for liquid systems. These
methods are discussed in Chapter S.

d. Flow Efficiency
The flow efficiency equation is identical to that used
for liquid systems, i.e.,

PR pwf(Ot=O) - Aps
E _ (7.60)

PR pwf(At-0)

B. Other Methods of Analysis


The Miller-Dyes-Hutchinson and Agarwal methods of
analysis can be employed in the same manner, and subj ect
to the same limitations and advantages, as previously
discussed.

7-40
3. Real Gas Pseudopressure, m(p), Method
^ This method, because of its greater accuracy, and its
applicability to any range of pressures, is recommended for
the analysis of all buildup tests in gas wells. We will
consider only the Horner method of analysis; however, any
analysis technique can be adapted to the use of the m(p)
function.

A. Horner Analysis
The Horner buildup equation written in terms of m(p)

is:

+At
m(pws) = m(pi) _ 163 (7.61)
kh7qT log t 2 at

According to Eq. 7.61, buildup data plotted as m(pwS) versus


(tp+At/At) on semilog paper should yield a straight line
during transient flow with a slope equal to

• 1637qT
M = - kh (7.62)

a. Permeability
Permeability is computed from the slope of the Horner
straight line using Eq. 7.62, i.e.,

k = - 16^T (7.63)
mh

b. Skin Factor
Combination of Eqs. 7.35 and 7.61 results in the
following expression for s:

rm(pf) - m(plhr)
s = 1.151 - log k + 3.23 (7.64)
^u*C* r 2
t w

• where u and ct are evaluated at p*

7-41
c. Flow Efficiency

m(pR) - m(pw f) - ^m(P) s


E _ (7.65)
m(pR) - m(pw f)

If PR is not known, use p* as an approximation,


i.e.,

p)s
m(p*) - m(pw f) - am(
(7.66)
m(p*) - m(Pwf)

d. Average Pressure
The MBH and Odeh-Al Hussainy methods can be applied
using the following modifications:

1) MBH Method
Use Figs. 5.8 through 5.11 in the same way
as for liquid systems ex-cept the ordinates
of these graphs must be relabeled as

m(PR) - m(P*)

PDMBH m/2.303

2) The Odeh-Al Hussainy correlations


presented in Figs. 5.13 and 5.14
can be applied in the same way as
for 1-iquid systems except the
ordinate and abscissa of these
correlations must be relabeled as:

m(PR) - m(Pi)
ORDI:VATE:
m

m(p*) - m(Pi) 0 1

ABSCISSA:
m

B. Other Methods of Analysis


The real gas pseudopressure can be employed in either
the MDH or Agarwal methods to analyze buildup data.
q
7 -42
Example 7.5: Analysis of gas well buildup test using the
m(p) method

Problem. A gas well has produced 16,594 Mscf at an average


rate of 400 Mscf/day. The well was shut in, and a pressure
buildup test was conducted. Given the pressure and reservoir
data shown, find the effective gas permeability, skin factor,
average pressure, and flow efficiency, using the m(p) method
of analysis on a Horner graph. A plot of m(p) versus p is
presented in Fig. 7.12.

Gas gravity = 0.62


Reservoir Temperature = 120 OF
r = 0.25 ft
w
Distance between wells = 2,970'
c = 4 x 10-6 psi-1
VV
^ Dr 3inage shape = closed square
= 13.5%
S = 35.50
w
h = 10 ft
= 925 psia
pi

At, hrs pws, psia At, hrs pws, psia

0 514 362 828


6.8 60S 391 833
26.2 647 409 835
49.9 681 435 837
73. 7 710 506 844
98 733 557 849
168 770 577 850
193 782 602 851
221 794 673 857
240 800 696 858
21 0 806 720 858
339 822 747 859

7-43
m(p), psi2/cp x 106

N W .A u1 Q^ V Co
G G G G G O G G G

H• G

F--'

U,

^
a^
O
O

'C7

tT7
5C
v ^]

(n G
F-'
U^

00
G
G

^o
0
0

• G
Solution. Producing time, tp, based on the flow rate at
the time of shut-in, is:

t = (16,594 N9scf) (24hrs/D)


p 400 Ms cf D

tp = 996 hrs.

All measured pressures were converted to equivalent values


of m(p) with the aid of Fig. 7.12 and are presented in the
following table as a function of Horner time ratio:

At, hrs tp+At pws, psia m(p), psi2/cp x 106


At
0 - pwf = 514 21.0
6.8 147 605 29.7
26.2 39 647 34.1
^ 49.9 21 681 38.0
73.7 14.5 710 41.3
98 11.6 733 44.2
168 6.93 770 49.0
193 6.16 782 50.5
221 5.51 794 52.0
240 5.15 800 52.7
339 3.93 822 S5.7
391 3.54 833 57.3
43S 3.29 837 58.0
557 2.79 849 59.2
602 2.65 851 59.7
696 2.43 858 60.8
747 2.33 859 61.0

A Horner graph of these data is presented in Fig. 7.13.


The slope of t he Horner straight line is m = -24.9 x 106
psi2/cp/cycle; therefor e, from Eq. 7.63

7-45
80

70

ID 60
0

U
N

H
50
U)

Crl
40

30

?O
1 10 ]02 103
t +At
p-
At

Fig. 7-13: Ilorner plot, Ex. 7.S.

0 0 1*
k = - 16 37aT

• mh

k = (1637) ( 400 Mscf/D) (580)


(-24.9 x 106) (10 ft)

k = 1.53 md.

In order to evaluate the skin factor, viscosity and


compressibility must be evaluated at p*. From Fig. 7.13,
m(p*) = 70.3 x 106 psi2/cp. This is converted to p* using
Fig. 7.12, i.e.,

p* = 920 psia.

From Fig. A.7, the gas viscosity at 1 atm. pressure and 120
'F is

• Uga = 0.0114 cp.

From Fig. A.2, ppc = 670 psia and Tpc = 365 °F:

= P _ * 920
ppr ppc ^
ppc - 670 = 1.37

T 580
Tpr = Tpc = T6 -5- = 1.59.

Thus, from Fig. A.9,

ug/uga = 1.11

u* = ug = (1.11)uga = (1.11) (0.0114)


u* = 0.0127 cp.

7-47
The total compressibility is

ID
ct = SOc0 + S w C w + SgLg + cf.

From Fig. A.4, cf = 4.2 x 10 6 psi I. The pseudoreduced


gas compressibility from Fig. A.10 is

c pr = 0.82

c = C pr = 0.82
g 670 psia
ppc

3 -1
cg = 1.22 x 10 psi

Therefore,

ct =(0.355) (4 x 10-6) +(0.645) (1220 x 10-6)

+ 4.2 x 10-b

ct = 792.52 x 10-6 psi

The skin factor is computed as

m m(Pws) - log kAt + 3.23 .


s= 1.1S1 m(Pwf)
`^^` ciw
t

For convenience, evaluate m(pws) at At = 100 hours; this


corresponds to a time ratio of 10.96, such that m(Pws)100 hrs=
0 1
44.4 x 106. Therefore,

= 1.151 21.0 - 44.4


s -24.9

(1.53) ( 100) + 3. 23
- log -
(0.135) (0.0127) (792.52 x 10-6) (.25)2 0

s = -5.9 .
7-48
This negative skin indicates the formation has been
^ stimulated.
The average pressure will be computed using the MBH
method. The dimensionless producing time is

0.0002637ktp
tDA ^uctA

(0.0002637) (1.53) (996)


t DA _ (0.135) (0.0127) (792.52 x 10-')(2970)'-

t DA = 0.034.

Relabeling the ordinate of Fig. 5.8,

m(pR) - m(p*)
= 0.45
M / 2.303

m(pR) = m(p*) + (0.45) (m/2.303)

M(PR) = 70.3 x 106 + (0.45)(-24.9 x 106/2.303)

M(PR) = 65.4 x 106 psi2/cp.

From Fig. 7.12, corresponding to m(p),

PR = 887 psia.

Finally, the flow efficiency is computed using


Eq. 7.65:

m(pR) - m(pw f) - Am(p)s


E =
m(pR) m(pw f)

^m(p)s = -0.87ms

7-49
_ (-0.87)(-24.9 x lOb)(-S.1)

Am(p)s = -110.5 x 10 09

[65.4 - 21.0 - (-110.5)] x 106


E =
(65.4 - 21.0) x 106

E = 3.5.

Based on the total skin factor and flow efficiency,


this well is obviously stimulated. Also, the Horner plot
exhibits the typical behavior of a fractured well. Further
analysis of this data using methods applicable to fractured
wells is recommended.

IV. PROBLEMS OF INTERPRETATION


0

The behavior of pressure data from gas wells is basically


the same as data from liquid systems. Further, the same
basic problems of interpretation occur with both gas and
liquid. We will not discuss again all problems of interpre-
tation; you are referred to Chapter 5 for a review of major
problems and their effect upon pressure behavior. However,
the common problems of wellbore storage and boundary effects
will be discussed.

1. Wellbore Storage
The major cause of wellbore storage in gas wells is gas
compressibility. Changing fluid level will only be a factor
in gas wells when water or condensate accompany gas production;
even in this situation, however, gas compressibility is
generally the primary cause of wellbore storage.

7-50
^ A. wellbore Storage Factor
The wellbore storage factor, C, caused by gas
compressibility is, from Eq. 3.45,

C = cwbVwb, bbl/psi. (7.67)


5.615

This relationship is valid for drawdown or buildup tests.


The wellbore volume, Vwb, represents the volume within the
wellbore which is in communication with the formation
during the test, and cwb is the compressibility of wellbore
fluids. If a changing liquid level is important during a
test, the storage factor caused by this effect can be
approximated using Eq. 3.44.

B. Detection and Duration of Wellbore Storage Effects


Weilbore storage affects gas well data in approximately
the same way it affects liquid systems. The best tool to

• detect storage is the log-log plot. If the correct plotting


parameters are used, the log-log plot will give a unit slope
straight line for data completely controlled by storage.
Further, the time at which data deviate from the unit slope
line can be multiplied by 50 to approximate the time at
which storage ends. The ordinate and abscissa values
required to prepare a log-log plot for the various methods
we have studied are:

a. p2 -Method
(i) Drawdown test
Plot Gp2 = p?i 2 versus t
pwf
(ii) Buildup test
Plot Apz ° pws pwf(At=0) versus At

b. p-Method
(i) Drawdown test

• Plot ap = pi pwf versus t

7-51
(ii) Buildup test
Plot Ap = pws - pwf(At=0) versus At

c. m(p) -Method
(i) Drawdown test
Plot Am(p) = m(pi) - m(pWf) versus t

(ii) Buildup test


Plot Om(p) = m(pws) - m[pwf(At=0)] versus At.

For design purposes, the time required for storage


effects to end can be predicted using Eq. 3.72 for drawdown
tests, and Eq. 5.33 for buildup tests. The value of the
storage factor required by these equations can be computed
using Eq. 7.67.

C. Changing Wellbore Storage


Our expectation that a log-log plot will give a unit
slope line at early times when storage dominates test data
is based on the assumption that C remains constant during

the test. It is obvious from Eq. 7.67, however, that C
will not remain constant because of the change in gas
compressibility which occurs in the wellbore during a
drawdown or buildup test. If drawdowns are large prior to
a buildup test or during a drawdown test, a common occurence
in tight gas reservoirs, the storage factor can change by
two or three orders of magnitude. When this occurs, rules
of thumb for predicting the duration of storage, such as
the SOt rule, are no longer valid. It is recommended in
this situation that a new time functionli'12, commonly
referred to as reaZ gas pseudotime, be used for all
calculations. This time function, which accounts for changes
in gas viscosity and compressibility as a function of time,
will be discussed further in a subsequent section.


7-52
2. Boundary Effects
The time required for boundary effects to influence
^ wellbore pressures is directly proportional to hydraulic
diffusivity. Accordingly, the equations and correlations
previously presented for liquid systems can also be used
to predict the time at which boundaries will affect gas
well tests.

3. Short Producing Time


When production time prior to a buildup test is short,
the Agarwal10 equivalent time function, Ate, should be used
to prepare all graphs, and to make all calculations.
Equivalent time, defined previously by Eq. 7.53, can be used
in the analysis of any well, but offers its greatest
advantage when applied to buildup tests with short producing
times.

* 4. Changing Gas Properties


A major problem in gas well testing is that analysis
techniques are based on methods derived from the diffusivity
equation which describes the flow of slightly compressible
liquids. You will recall that the liquid diffusivity equation
is linear and, accordingly, easy to solve. The gas
diffusivity equation, however, is not linear; this is
because of the strong dependence of gas properties upon
pressure. The p2- and p-equations represent approximate
solutions to the gas diffusivity equation under the
assumption that the pz product remains constant (p2-method) ,
or that uz changes linearly as a function of pressure
(p-method).
The real gas pseudopressure, m(p), was introduced in an
attempt to linearize the gas equation while rigorously
accounting for changes in fluid properties. While giving a
^ more accurate solution than other methods, replacement of
p by m(p) does not completely linearize the gas equation.

7-53
Instead, it is still necessary to assume the pc product
to be constant in some calculations. This limitation causes
problems in two major areas 12: First, the m(p) function does
not adequately account for changes in the wellbore storage
factor which are caused by changes in gas compressibility;
this problem was discussed in the preceding section on
wellbore storage. This can cause problems in detecting
storage, predicting the length of storage, and in type curve
analysis. Second, large drawdowns and the associated large
changes in gas properties, are not totally accounted for in
the partially linearized m(p) equation. These inaccuracies
can cause significant errors in calculated values of
permeability and fracture length, especially in tight gas
reservoirs.
In a recent study, Agarwalll found that these problems
could be largely eliminated by replacing t (At in buildup
tests) in all plots and calculations by a pseudotime, ta,
defined as
DO nof use, ^or

t DraujJown Te.s fs
ta = d (7.68
C
0 t

As shown by Lee, et all; this function effectively linearizes


the gas equation under certain conditions and makes gas
behavior equivalent to liquid behavior. Examples showing
how to compute ta are presented by Agarwalil and Lee'2.
It is also possible to compute an equivalent pseudotime
as

t At
a
Attae = tpa+ota (7.69)
r I

This can be used as the plotting function for any buildup


test, but has particular importance where producing time
before shut-in is short. An example of using this function
in the analysis of a fractured, low permeability reservoir
is presented by Lee'2.

7-54
V. TYPE CURVE ANALYSIS

• Type curves developed for liquid systems can be applied


to the analysis of gas well tests provided appropriate changes
are made in the plotting functions. The Ramey type curve,
presented in Fig. 3.24, can be used if dimensionless pressure,
pD, is redefined as:

1. p'-Method

_ khA (p2)
(7. 70)
pD --
1424quzT

Drawdown Test: d(p2) = p? - pz


i wf

Buildup Test: A (p2) = pWS pj^f(At=0)

0 2. p - 'Met.hod

khAp _
(7. 71)
PD = 141.2 x 103aBU

Drawdown Test: Ap = Pi pwf

Buildup Test: Ap = pws pwf(Qt=O)

3. m(p) -Method

kh0 [m(p)]
(7. 72)
v 1424cIT

Drawdown Test: a[m(p)] = m(pi) - m(pwf)

^ Buildup Test: 4[m(p)] = m(pws) m [pwf(At=0)]

7-SS
Dimensionless time, tD, for all methods is defined as:

t _ 0.0002637kt ( 7. 70
D
ouictirw 2

Shut-in time, At, replaces t in Eq. 7.73 for buildup


test analysis.
Log-log plots used for type curve matching are
prepared using a pressure difference which is appropriate
for the method employed; the required log-log plots for
each method were discussed previously. With field data
prepared on the correct log-log plot, and with PD and tD
appropriately defined for the method used, the curve
matching procedure is identical to that for liquid
reservoirs.
It must be remembered that the Ramey curve is based
on drawdown theory and does not directly apply to buildup
data unless producing time is sufficiently long for
tp + At = tp. If this condition is not satisfied by the
well being tested, log-log plots used in the curve matching
process should be prepared using the Agarwal equivalent
time!' Ate, and all calculations should be made using this
time function. Further, if a well is subject to a large
pressure drawdown, which will probably be true in most
tight gas reservoirs, pseudotime;l tat should replace
real time in all plots and calcuations.
Other type curves which are widely used in the analysis
of gas well tests are the Earlougher-Kersch13 and Gringarten1`'
curves.

VI. FRACTURED GAS WELLS

Fractured gas wells behave in a manner similar to oil


wells. If a gas well is hydraulically fractured, and inter-
sects a high capacity fracture, test data will chronologically

7-56
• • •
EQUIVALENT VARIABLES IN GAS FLOW AND SLIGHTLY COMPRESSIBLE LIQUID FLOW

Liquid Gas

Pressure: p Pseudopressure:
p
m(p) = 2 pdp
uz
Po

Dimensionless Pressure: Dimensionless Pseudopressure:

khTsc
D p= kh
D 141.2qBp Ap m(p) D 50,300qPscT Am(p)

Time: t Pseudotime

dt
ta j t uct
t0

Dimensionless Time: Dimensionless Pseudotime:

t _ 0.0002637k t _ 0.0002637k
D taD 2 ta
^uctrw w
Liquid Gas

Radi.us: r Radius: r

Dimensionless Radius: Dimensionless Radius:

_ r r
rD r rD = r
w w

Wellbore Storage Coefficient: Modified Wellbore Storage Coefficient:


CI1
Cr% Tct VwbTct
w
C = Vwb`wb
Ce = C Twbcwb wb

Dimensionless Wellbore Storage Modified Dimensionless Wellbore Storage


Coefficient: Coefficient:

0. 8 9-1 V4y,b cwb 0.894VWbT


CD =
CDe ^hr 2,1,
^C t}lr^
^,yr
w wb

0 9 •
The implication of these equivalency relationships is

this: A value of pD can be found at fixed values of tD, rD,

^ s and C for a slightly compressible liquid. If for a gas


D
we replace the liquid variables with gas variables, i.e., taD

tD, ta = t, CDe = CD, the value of m(p) D will be identical to

the value of PD for liquid. For example, the Gringarten type

curve, which is a liquid solution, is a plot of PD versus

tD/CD for fixed values of CDe2s. This plot can also be used

to describe gas behavior if it is considered to be a plot of

m(p)D versus taD/CDe for fixed values of CDe and s. Gas data

will behave exactly like liquid data if it is plotted using

the appropriate equivalent variables.

is

q
7-S6C
HMP11'l..li 1'IZORLIiM: llppl icat.ion ol' l)seuclopressur-e and ptieudo-
time to pressure buildup analysis of a
low-permeability gas well

Problem. The following pressure buildup data, presented by
Lee and Holditch (Journal of Petroleum Technology, December
1982, 2877), are for a low permeability gas well which was
tested prior to fracturing:

= 0.075 h = 25 ft
rw = 0.25 ft tp = 1,200 hrs

Vwb = 131.1 bbls T = 202°F


Twb = 202°F
= 6500 psia
pi
yg = u.ns
pwf(At=o) = 707 psia
ui = 0.0282 cp
q = 190 Mscf/D
cgi = 8.28 x 10-5 psia-1 m(pwf) = 3.994 x 10' psiaZ/cp

psc = 14.65 psia tpa = 5.13 x 108 hr-psia/cp

Tsc = 60°F

At m(Pws)-m(p w f) Atae ote
pws
(hours) (psia) (psia2/cp) (hr-psia/cp) (hours)

0 707 0 0 0
0.0720 720 1.46x106 3.64x103 0.072
0.288 759 5.89x106 1.49x10" 0.288
0.936 872 2.03xl0' 5.16x10`' 0.935
2.23 1,088 5.31x10' 1.38xl0' 2.23
3.58 1,304 9.27x10' 2.44x1.05 3.57
4.97 1,521 1.39x108 3.70x105 4.95 0 i
6.41 1,739 1.91x108 5.18x10' 6.38
7.92 1,957 2.49x108 6.90x105 7.87
9.46 2,176 3.12x10' 8.84x105 9.39
11.0 2,395 3.80x108 1.10x106 10.9
12.7 2,615 4.52x108 1.35x106 12.6

7-56D
14.4 2,834 5.27x108 1.62x106 14.2
16.1 3,054 6.05x10' 1.92x106 15.9
• 17.8 3,272 6.86x108 2.24x106 17.5
19.6 3,491 7.68x10 8 2.60x10 6 19.3
21.5 3,707 8.51x108 3.00x106 21.1
23.4 3,922 9.35x108 3.43x106 23.0
25.4 4,136 1.02x109 3.91x106 24.9
27.6 4,346 1.10x109 4.47x106 27.0
29.9 4,556 1.19x109 5.09x106 29.2
32.3 4,760 1.27x109 5.76x106 31.5
35.0 4,961 1.35x109 6.56x106 34.0
38.0 5,158 1.43x109 7.48x106 36.8
41.4 5,348 1.51.x104 8.57x106 40.0
45.6 5,530 1.58x109 9.96x106 43.9
50.6 5,702 1.66x109 1.17x10' 48.6
57.2 5,861 1.72x109 1.40x10' 54.6
66.6 6,001 1.78x109 1..74x1.0' 63.1
81.6 6,118 1.83x109 2.28x10' 76.4
110 6,210 1.86x109 3.30x10' 101
^ 181 6,283 1.89x109 5.71x10' 157
301 6 , 3 34 1.92x10 9 9.30x10 ' 241
421 6,363 1.93x109 1.24x108 312
541 6,383 1.94x109 1.51x108 373
661 6,397 1.94x109 1.74x108 426
781 6,408 1.95x109 1.95x108 473
901 6,417 1.95x109 2.13x108 515
1,021 6,424 1.95x109 2.29x108 551
1,141 6,429 1.95x109 2.44x108 585
1,200 6,432 1.96x109 2.56x108 600

Pseudopressure, m(pws), was calculated by numerical


integration using the defining equation

m(pws) = 2 ^ Pws ^'dP


-P z
pJo

• 7-56E
where base pressure, po, was chosen to be zero. The term,
m(Pwr), is the pseudopressurc corresponding to thc
pressure, pwf(Ot=o), at the instant of shut-in.
Pseudotime during shut-in, ota, was calculated for each
shut-in pressure using the defining equation

t=At
ota = dt
Uct
t=o

Pseudotime was converted to an effective time basis, Atae'


using the equation

tpaAta
^tae t +At
pa a

where tpa was approximated as tp/uicti"

a) Graph Am(pws) versus Ate and Am(p5) versus Atae on log-


log paper. How does the graph based on effective pseudotime,
Otae, differ from the graph based on effective time, Ate?

b) Using the Gringarten type curve, analyze the pressure


buildup data for permeability, k, effective wellbore storage
factor, CDe, and total skin factor, s.

Solution.

a) A log-log plot of the test data, using both equivalent


time and pseudoequivalent time is presented in Fig. I. It
is observed from Fig. 1 that the graph of pseudopressure
versus actual shut-in time, ltte, begins with a slope greater
than unity, which suggests that changing weZZbore storage is
having asignificant effect on the test data. Because of the
changingstorage effect, type curve analysis of this data
1tiOLl: ;: L- :jlf i iClil^^ lj not i'LIY^t^le.r-, various rule";

7-56F
• • •
At , hrs
e

1 10 102 10' 10`' 105


101Q F

cd
-H 10Q

^n

108

10'

10" 1.05 106 10' 10" 109

Atae, hr-psia/cp

Fig. 1: Comparison of log-log plots based on equivalent time, At.,


and equivalent pseudotime, At ae.
of thumb used to determine where storage effects end, such
as the "50 At" rule, cannot he applied because it is not
possible to identify the unit-slope line.
The same test data plotted versus pseudotime, Atae'
exhibit the expected unit slope line at early times. Further,

it is observed that the slope is never greater than the theo-
retical maximum slope of unity. Since this data behaves in a
conventional manner, it will be shown that type curve analysis
of this data is possible.

b) The test data were plotted as Om(pws) versus Atae on


tracing paper and matched with the Gringarten type curve.
This match, shown by Fig. 2, resulted in the following match
points:

m(p) D= 1.38 at Am (p) = 10 8

^aD = 54 at Otae = 10'


De

C De e2$ = 10zo .
is
Permeability is related to dimensionless pseudopressure
by the equation

khT
m(P)D 50,300qpSCT Im(Pws) - m(pwf)1

so that

S0,300qpscT m(p)D
k - hTsc Zm p
MATCH

k=(50, 300) (190) (14. 65) (662) 1. 38


(25)(520)
108

k = 0.0983 md.


7-56H
• q •
Atae, hr-psia/cp

104 105 10° 10' 108 10'


10

C`.
U
10`
RJ

V)

r--^

v^

108
V)

10'

DIMENSIONLESS TIME o -°^0^^5 ^ c 2 3 . s e7 a 9


2 5 b ^89
TYPE-CURVE FOR A WELL WITH WELLBORE STORAGE AND SKIN (HOMOGENEOUS RESERVOIR) 3

Fig. 2: Match of test data on Gringarten type curve.


The effective wellbore storage factor is computed
as

CDe
_ 0.0002637k ^tae •
^r LtaD/CDCJ MATCH

(0.0002637)(0.0983) 10'
54
CDe (0.07S)(0.2 S) 2

CDe = 1,024.

The effective wellbore storage factor can also be computed


using the log-log plot of pseudopressure versus equivalent
pseudotime presented as Fig. 1. The equation of the unit
slope line is:

t aD
= 1
TDemTP

so that,

13.26qpscT Atae
CDe ^hrwTsc ^m p Unit Slope Line

From the unit slope line on Fig. 1,

©m(p) = 5x1.08 at At,ie = 1.3 x 106

Therefore,

C = (13.26)(190)(14.65)(662) 1.3x106
De i
(0,075)(25)(0.25)2 (S20) 5x10$

CDe = 1,042

which is in excellent agreemc,nt the value obtained


the Gri.ngarten type c_i,ir'Ve.
0

7-56J
The total skin factor is computed as

(C De e2s) MATCfI
s= 1 In
^ 2
CDe

1 (10201
s =
2 In L1024J

s = 19.6.


7-56K
exhibit linear flow, transitional flow, pseudoradial flow
and pseudosteady state flow, as was described for liquid
systems in Chapter 6. Accordingly, the square root plot
and log-log plot can be used simultaneously to detect high
capacity fractures. These plots can then be combined with
the appropriate semilog plot to analyze the data for
permeability, fracture length and skin.
Many gas wells are completed in low permeability
reservoirs which require very large fracture treatments to
stimulate production. The long fractures created by these
massive hydraulic fracturing treatments tend to create
fractures with limited conductivity. Data from wells with
limited conductivity fractures cannot be analyzed using the
same methods applied to wells with infinite conductivity
fractures. In fact, with existing technology, type curves
seem to offer the only practical method of analyzing data
from reservoirs with finite conductivity fractures.

1. Conventional Method of Analysis


The conventional method of analysis is analogous to
liquid systems and requires preparation of log-log, square
root and semilog plots of test data. The pressure function
used to prepare these plots will depend on the method used,
i.e. , p2 , p , or m(p). Time functions on each graph will be
dictated by whether the test is drawdown or buildup, and by
the possible need of using equivalent time, At e , or pseudo-
time, t a .
The primary purpose of the log-log plot in a conventional
analysis is to assist in defining the linear flow data.
Linear flow will result in a half-slope line on the log-log
plot provided the line is not masked by wellbore storage or
skin effects. Data required from the square root plot is
the slope,m LF, of the linear flow straight line. Data required
from the semilog plot is the slope,m , of the pseudoradial
PR
straight line.

7-57
^ Determination of permeability, k, and fracture half
length, xf, requires an iterative procedure analogous to
that described for liquid systems. The slope, mp., from the
semilog plot permits us to compute the apparent permeability,
ka, of the formation. The true permeability is computed
using the relationship

(7.74)
k = kaFcor

where Fcor is given by the correlation in Fig. 6.8. Fracture


length and the correction factor, Fcor, are determined
iteratively using Fig. 6.8 and one of the following relation-
ships for x f :

p2-Method:

2
x _ 1.012 mPRq 'T (7.75)
f mLF ^cthF cor
is

p-4lethod:
z
10.09 mPRaB
f - - (7.76)
mLF ^ct Fcor .

m(p) -Method:

x = 1 .01Z mPRaT (7.77)


f mLF hw^uctFcorJ

Equations 7.75 - 7.77 are to be used when permeability


is not known. If permeability is known prior to the test,
fracture half length can be computed using the slope of
the linear flow straight line from the square root plot
and one of the following equations:


7-58
p2-Method:

^
_ 40.925 zT u 1z
Yf mLF
TC_tkj

p-Method:

40 70qBg r
x f = m-^- [ck] (7. 79)

m(p) -Method:

40.925 T 1 1z (7.80)
x f = ---
m Tuctk

Notice that all calculations using the conventional


method of analysis require the slope, mLF, of the linear
flow straight line on the square root plot. If this line
is masked by wellbore storage, it will be necessary to
analyze the data using type curves.

2. Type Curve Analysis
Fracture type curves are of two basic classifications:
(i) infinite conductivity fractures, and (ii) finite
conductivity fractures.

A. Infinite conductivity Fractures


The type curve for infinite conductivity fractures was
presented in Fig. 6.16 for liquid systems. This same curve
can be used for gas well analysis provided that PD and tDxf
are redefined. Depending on the method of analysis used,
PD will be defined by Eqs. 7.70 - 7.72. Dimensionless time,
tpxf, is defined for fractured gas wells as

7-59
0.0002637kt
(7. 81)
tllxf - '
^^ 1 c. x _2
i ti j:
^

When analyzing buildup tests, At will replace t in Eq. 7.81.


The curve matching procedure for gas is identical to that of
liquid.
When working with buildup tests preceded by a short
flow time, it is recommended10 that equivalent time, Ate,
replace the shut-in time, At, in all plots and calculations.
Further, it is recommended that pseudotime replace real time
when analyzing any well with a large drawdown.

B. Finite Conductivity Fractures


Type curves for finite conductivity fractures include
solutions for uniform flux fractures, and solutions where
fracture conductivity is a variable.

1) Uniform Flux Fractures - The type curve for uniform


flux fractures is presented in Fig. 6.19 for liquid systems.
This curve can also be applied to the analysis of gas well
data provided PD is appropriately defined for the method being
used. Dimensionless time, tDxfI is defined by Eq. 7.81.
Uniform flux fractures have very high conductivity and
seem to match the data from a large number of wells which
have been hydraulically fractured.

2) Finite Conductivity Fractures - Figure 6.20 presents


the type curve for fractured liquid systems where dimensionless
conductivity, FcD, is a variable on the type curve. Again,
these curves can be applied to gas when PD and tDxf are
• appropriately defined. Dimensionless fracture conductivity,
FcD, is defined by Eq. 6.32.

The primary application of Fig. 6.20 is in the analysis


of data from tight gas reservoirs which have been stimulated
with massive hydraulic fractures. Tight gas reservoirs are

7-60
becoming increasingly important because of higher gas prices
and improved stimulation techniques in the form of massive
hydraulic fracturing (MHF). Because of the combination of long
finite conductivity fractures and low matrix permeability,
these reservoirs offer a challenging well test problem which
has been the subject of a significant research eff.ortll'12'ls-1A
The specialized procedures necessary to design, conduct and
analyze data from tight gas reservoirs is beyond the scope of
this course; however, the analysis of data from a well with a
finite conductivity fracture is illustrated by the following
example.

Example 7.6: Analysis of buildup test from a well with a


vertical finite conductivity fracture

Problem. The following buildup data were obtained from a


gas well following a massive hydraulic fracturing treatment.
Estimate permeability and the half fracture length using the
p-method of analysis.

tp = 11,059 hrs
q = 3,186 Mscf/D
h = 60 ft
pi =-8,900 psia
ui = 0.033 cp
c g i = S.4 x 10 4 psi
B gi = 5.65 x 10 bbl/scf
yg = 0.716
= 18%
0 1
S CY = 0.50
0
T = 300 ° F

I - -]
7-61
At, hrs pws, psia pwf(At=0), psi
P At pws
0 - 36 80 0
0.07 158,000 3740 60
0.15 73,700 3761 81
0.22 50,270 3776 96
0.29 38,100 3787 107
0.36 30,700 3796 116
0.44 25,100 3804 124
0.58 19,100 3818 138
0.73 15,200 3827 147
0.87 12,700 3836 156
1.02 10,800 3845 165
2.19 5,505 3884 204
4.37 2,530 3930 250
8.75 1,260 3986 306
16.03 691 4043 363
^ 31.31 354 4127 447
57.92 192 4216 536
100.48 111 4311 631
152.42 73.6 4411 731
220.72 51.1 4501 821

Solution. A Horner plot of the buildup data is presented


in Fig. 7.14. The shape of this plot is typical of wells
with deeply penetrating fractures. This data will be
analyzed using the finite conductivity type curve previously
presented in Fig. 6.20. The log-log data curve required for
the type curve match is presented in Fig. 7.15, and the final
match of the two curves is depicted by Fig. 7.16.
Since the p-method is being used for the analysis, PD
is defined as

kh [pws pwf (At=0)]


=
^ pt) 141.2q uiBi

7-62
4800

^ l ^.... „

. . : . , , . . ..,.. ,. ,. ^ . .. ... :, . ^
4600
^ 1{ ^

. ;^ . ^ . ^ l ^. . , ; ^ ^. . ,. . ..^ . :.
4400
t,. .,.
; ,..

^
t
.. ,. . . 1 1, ^
^ ., . i
4200 ' , , :. , . . ,., .., ,,,, ,.,i . . . . . . .. ..... ..
ut) ^
i 3 ; , ,
^. a ... - --
^
,_. ._. . :
:.. :., . :.. ,. ^,. :. . , ... ,.. .. ... ... .... .. .... ... . ... . . . . ,, ., . . . . ., , .: 1 . ,,. ; , ,
4000 ,... ,... ... , - -- _ _, ... .. ,. ,.. . ., , . . . , . ,
. ,. .:: , ^:. , , ;;^, ,,,. .... ... . ... . . .. . . . . . '^
. . .... .... ...: ... .. . . .. .... . . : . . . . ..:. ..:. .... ... . . . ... .... . .. .. . . . . , . . .... _ , ; . . ,... . : ; , :' ^.. ,' ,

.. :. ... ... . . . .. . :.. ,. ... _ .. . :.. .


3800 _ . .^
. ..... .... ... : . . .... ... .. . .:., ,. ..;. ... . . .. .... . . . . . . , . , , . . ... .., , . , . ,

.-
.
. , . .., , ., . . . . ..
. . . . .Ii^ ^ ^ ^, i. ^: .^ , ..., ^ ... .... .. . .... .... ^.. ... . . . . . . ..-. .... .., . .. . . ., .... .... .,.. _, . . ..^. . . . .. .._ ,.... .... , ,i.., . .
3600
10 102 103 10`' 105
t +At
P
At

Fig. ,. 14 Horner plot, Ex. 7.0.


i • •

103

102 ^,,.• • _ _ _. _ - -__

10
10 10 1 10 102 103

At, hrs

Fig. 7.15: Data Curve, Ex. 7.6.


C

N
N
u-^
N
W
w
CL
^-h
N
NW

Z
C) ^''
N

DIMENSIONLESS TIME, tDX


f
Fig. 7.16: Type curve match, 0 7.6.
is
The ordinate values, corresponding to the match point,
are:
I*
(PD) N1 = 0.04

(Ap)M = 43.

Therefore,

141.2 x 103Q1-ilBgl (pD)M


k= h (AP) m

k = (141.2x103)(3186 Mscf/d)(0.033cp)(5.65x10-4bb1/scf) 0.04


60 ft 43psi

k = 0.130 md.

The abscissa values of the match point are

^ M = 0 .002
(tDxf)

(At) M = 3.25 hrs

From the definition of dimensionless time in Eq. 7.81,

0.0002637k (At) M z
xf = "lctl tD )

The total compressibility is:

cti= cgSg =(5.4 x 10 5)(0.5) = 2.7 x 105 psi 1

and,
(0.0002637) (0.130) 3.25 2
xf (0.18) (0.033) (2.7 x 10 5) 0.002

^ x -- = 589 ft.
t

7-66
VI. GAS WELL DELIVERABILITY TESTING

S
Deliverability testing, sometimes referred to as
backpressure testing, has been used for more than fifty
years by the petroleum industry to determine the flow
potential of gas wells. Deliverability tests most commonly
run are flow-after-flow tests, isochronal tests, and modified
isochronal tests-. These tests basically consist of flowing
wells at multiple rates, with or without interim shut-in
periods, and measuring pressure as a function of time. From
the rate-pressure-time behavior of a test which is properly
designed, run, and analyzed, it is possible to determine the
flow potential of a well. The purpose of this section is
to introduce the reader to basic techniques of conducting
and analyzing commonly used deliverability tests. This
discussion is not intended to be comprehensive, and the
reader should be aware that testing options other than those
discussed are available! 11-24 Further, testing procedures
are regulated by state and federal regulatory agencies3'2o-za
and can vary significantly according to location.
Deliverability tests are run for a number of reasons which
include the following:

• Reguiired by state and federal regulatory


agencies for purposes of establishing
allowables and proration;
• Production forecasting to aid in (1) selection
of location and number of wells for field
development, and (2) design of gathering
system plant facilities;
• Negotiation of gas sales contracts;
• Establish present value of gas reserves;
• Reservoir characterization.

Most deliverability tests are run to satisfy regulatory


requirements; however, because of poor test design and
insufficient flow times, these tests are often not acceptable

7-b?
for production forecasting and reservoir characterization.

- 1-
-^" 1. Flow-After-Flow Tests UF4- '+ eS
The required procedure for conducting this test,
commonly referred to as a conventional backpressure test, is:

1) Shut in thc test wel^ sufficiently long


for the formation pressure to equalize
at volumetric average pressure, pR.
Measure PR'
2) Place the well on_production at a constant
rate ,^-,) After the well has flowed
suffici-efitly long to stabilize, i.e., to
reach pseudosteady state, measure the
flowing formation face pressure,
Pwfl
3) C`hange the rate to q2 and flow for
1 t e
same length of time sed in Step 2.
Measure the flowing formation face
pressure, pwf2 at the end of the flow
period.
4) Repeat Step 3 for rates q-,, q, ----- qN•
A test is conventionally run at four
^ different rates.
5) At the end of the test, the well can
remain on production or be shut-in.
If the well is shut-in, and pressures
are being recorded with a subsurface
gauge, a pressure buildup test could
be run.
6) Analyze the data with one of the methods
to be discussed.

The rate and pressure history of a typical four-point test


followed by shut-in is depicted by Fig. 7.17.
Figu re 7.17 illustrates a normal sequence of rate
changes w here the rate is increased during the test.
Tests may also be run, however, using a reverse sequence.
Experienc e indicates that a normal rate sequence gives better
data in m ost wells.
* The most important factor to be considered in running
^ this test is the length of the flow periods. It is
required that each rate be maintained sufficiently long

7-68
q4

a q 3

^?

tl t2 t4
t3

T I ME

PR
pwfl

pwf2

3 pwf3
•i
pwf4

tl t., t3 t4
^

TtME

^ Fig. 7.17: Typical rate and pressure


history of a flow-after-flow
test.

for the well to stabilize, i.e., to reach pseudosteady state.


The time required for stabilization was presented previously
as Eq. 3.15, i.e.,

tpss
^uct
0.0002637k (tDA) pss
(7 0
.

7-69
The dimensionless time at which pseudosteady state begins,

(tDA)pssI is presented in Table 2.2 for various drainage


configurations under the column heading "Exact for tDA '11'
The minimum time to reach pseudosteady state, which corre-
sponds to a well centered in a square or cylindrical drainage
area, is:

^uct
k (7.83)
(tpss)min = 380

Because of the long stabilization times required in low


permeability formations, this test is generally applicable
only in high permeability formations. Significant errors
can result when a test is not run sufficiently long; this
will be discussed in a subsequent section.

A. Empirical Method of Analysis


Based on the analysis of data obtained from a large
^ number of wells located in major gas producing areas of
the United States, Rawlins and Schellhardt25 proposed the
following empirical relationship: ^-----b cu,^d, w. f93os

-2 pwf)n
-^ q = C(pR 2
(7.84)

where q = stabilized flow rate, Mscf/D


pwf = stabilized flowing formation face pressure,
psia
= static formation pressure, psia
PR
C = deliverability coefficient, Mscf/D/psi2
n = deliverability exponent.

Taking the logarithm of both sides of Eq. 7.84 results in:

log q = log C + n log (pR pwf) (7' 83)

-2 2 1
log(pR pwf) = n log (^) + n log q. (7.86)^j

7-70
Equation 7.86 suggests that a log-log plot of pu Pwf)
versus q should yield a straight line of slope 1/n. This
plot, commonly referred to as the deliverability graph, is •
illustrated by Fig. 7.18.

103
PR(reflects pwf=0)

102
N3
SLOPE = 1
n

10

AO F •
1
10 102 103 104
q, Mscf/D

Fig. 7.18: Ideal gas well deliverability


graph.

a. Determination of n and C
The deliverability exponent, n, can be determined from
any two points on the straight Zine, i.e., (a1,Apl) and
(q2, Ap2), according to the following equation:

log q2 - log ql (7. 87)


n =
log (Ap2) - log (APi)

7-71
Zft^ k,'>es
'a'3c,
'

With n known from Eq. 7.87, any point on the straight


line can be used to compute the deliverability coefficient,
C, as

C = - q - (7. 88)
(,P2)n

^ b. Absolute Open Flow Potential (AOF)


Absolute open flow potential refers to the [Maximum rate
at which a well could produce against a zero backpressurel
Historically, many regulatory bodies have used the
(pwf-0).
AOF as a means of establishing allowables; for example, the
allowable might be established at 25% of the AOF. The
primary objective of deliverability tests required by
regulatory bodies is the AOF.
* The AOF of a well cannot be directly measured since it
is not physically possible to achieve a zero backpressure on
^ the formation face. However, this value can be obtained from
the deliverability plot. Assuming the deliverability curve
is an accurate measure of the relationship between q and pwf,
this line can be extrapolated to determine the flow rate
corresponding to pwf = 0; this is illustrated by Fig. 7.18.
With C and n known, the AOF could also be computed by setting
Pwf-0 in Eq. 7.84, i.e.,

AOF = C(p^)n . (7.89)

c. Prediction of Reservoir Performance Using Deliverability


Test
A major purpose of establishing the deliverability
relationship is to predict the performance of a reservoir.
For example, suppose we are working with a single-phase,
volumetric gas reservoir and it is desired to predict gas
deliverability versus time to the economic limit for the
purpose of gas contract negotiations. The relationship

7- 72
between cumulative gas production, Gp, and average reservoir
pressure, p^, can he established'`^ by material balance to be
0

PR _ (P) (pGz) i G
(7.90)
i p

where pi = initial reservoir pressure, psia


= volumetric average pressure, psia
P.
G= gas initially in reservoir, Mscf
G p = cumulative gas production at time
reservoir pressure is pR, Ms cf
z = gas deviation factor at PR
z i = gas deviation factor at pi

This equation suggest that a plot of (pR/z) versus Gp will


be a straight line; this graph, commonly referred to as the
"p/z curve", is illustrated by Fig. 7.19. Once this straight
line is established from production performance, or from
theory, cumulative gas production, Gp, can be predicted as a
function of average reservoir pressure PR-

P-)
zi

f (p)1

----^ ---
^ (z) 2

r•-- A Gp .

G pl Gp G

G^, Mscf
^

Fig. 7.19: p/z curve for a volumetric


gas reservoir.

7-73
After the deliverability relationship for this
reservoir is determined, and assuming that C and n remain
^ constant throughout the Life of the reservoir, Eq. 7.84 can
be used to predict q as a function of pR A typical plot
of q versus PR is depicted by Fig. 7.20.

p1
Mv
..;
f P2

q1
q2

Ll, '%15cf1ll
^

Fig. 7.20: Relationship between q and PR-

By combining the material balance relationship (Fig.


7.19) and the deliverability relationship (Fig. 7.20) it
is possible to predict rate and cumulative production at
any reservoir pressure. Further, it is possible to predict
the time required for this production to occur. For example,
suppose we want to know how long it will take for pressure
to decline from pRl to pR,^ ; then,

1) Read Gpl and Gp2 from Fig. 7.19 corresponding


to PRl and pR2.
2) Compute cumulative gas production between
pRl and pR2 as AGp = Gp, - Gpl.


7-74
3) Read al and q2 from Fig. 7.20 corresponding


to pRl and p R2, and compute the average
rate in this pressure interval as

clavg = (ql + q2) /2.

If contract rate or regulatory allowed rate


is smaller than Qavg, use the smaller rate
for this increment
4) Compute the time required to produce AGp
Mscf of gas using the relationship

AG
At = p
aavg

This process could be repeated using small pressure intervals


to predict reservoir performance to the abandonment pressure.
A plot of deliverability as a function of time could then be
prepared as depicted by Fig. 7.21. Finally, we have a rate-
time prediction which can be used for contract negotiations,
E
economic studies, etc.

PREDICTED
\ DELIVERABILITY

\
\
^
CONTRACT RATE^-, ECONOMIC
LIMIT

t, days

Fig. 7.21: Deliverability versus time


for a gas reservoir. is
7- 75
The production forecast depicted by Fig. 7.21 is only
^ as accurate as the gas deliverability equation and, accordingly,
the gas deliverability test. Therefore, we need to look further
at possible limitations of this test.

••d. Effect of flow rate on "n"


It is assumed in deliverability predictions that n is
a constant; however, n is a function of rate. The value of
n should lie between 0.5 and 1.0 according to the degree of
turbulence in a reservoir, i.e.,

Laminar Flow: n = 1.0


Fully Turbulent Flow: n = O.S.

Since the slope of the deliverability straight line is


equal to 1/n, we have that

Laminar Flow: Slope = n= 1.0

Full y Turbulent Flow: Slope = 1= 2.0.


n

The effect of turbulence on the deliverability graph


is depicted by Fig. 7.22. If a well is tested at low rates
to establish the deliverability curve, and this curve is
extrapolated to predict deliverability at high rates where
turbulence occurs, notice that predicted rates will be too
large. This suggests that wells should be tested at rates
which approximate anticipated producing rates if the data
is to be used for production forecasting.

e. Effect of Test Duration on "C"


It is assumed when predicting gas well deliverability
that C is a constant. The validity of this assumption can
be evaluated by considering further the theoretical basis
for the deliverability equation.

7-76
Notice that Eqs. 7.84 and 7.91 are identical if n = 1.0
and C = Cl; accordingly, it is noted from Eq. 7.93 that C
will remain constant if k remains constant (gas saturation
does not change), uz remains constant (true at low pressures),
the drainage radius, re, does not change, and flow is laminar.
This means that these conditions must be satisfied during
the deliverability test in order to obtain the correct C,
and they must remain true throughout the life of the
reservoir if this value of C is to yield accurate predictions.
Note, in particular, the effect of re on C: If the value
of C obtained from a deliverability test is to be indicative
of the long-term deliverability of a well, each flow period
during the test must last sufficiently long for the pressure
transient to reach all boundaries and stabilize, i.e., to
reach pseudosteady state. The error caused by not running
the test long enough can be evaluated by considering the
transient flow equation.
The drawdown equation during transient flow can be
^ written, from Eq. 7.5, as

- _ 1637quzT kt
pi pwf kh log 2

- 3.23 + 0.87s' + 0.87Dql (7.94)

(7.95)
pi pwf = Atq + Btq2

1637uzT kt
where At = - 3.23 + 0.87s (7.96)
kh [10 g ^uctrw 2

Bt = 1424uzT D
. (7.97)
Th


7-78
For laminar flow, Bt = 0, and

q (7.
(p pwf^ ' 0
t

Equation 7.98 is similar to the deliverability equation,


Eq. 7.84, if n = 1 and C = 1/At. Notice however that C is a
function of time during transient flow and that it will
decrease with time until the system reaches pseudosteady state.
This is illustrated by Fig. 7.23. This means that a test not
run sufficiently long to reach pseudosteady state will yield
a value of C which is too large, and which will result
in optimistic deliverability predictions. The effect of
test time on the deliverability plot is illustrated by Fig.
7.24.

10"


U 10 3

STAB ILIZED
VALUE OF C

P
10 2

10 102 10 3

TI ME

@I I
Fig. 7.23: Effect of flow time on value
of C obtained from
deliverability test.

7-79
10
tl < t2 < t3 < tpss
• Cl > C2 > C3 > C

3
10

N 3 K^ 4Z ^^ y1

C,n^
N z
^^.
2
10

10
102 103 10'' 105

• Q

® Fig. 7.24 Effect of flow time on


deliverability plot.

f. Effect of Wellbore Storage


All flow tests must last sufficiently long for wellbore
storage effects to end; otherwise pressures measured during
a deliverability test will have no meaning. Equation 3.72
can be used to predict the duration of storage for purposes
of test design.

Example 7.7: Analysis of stabilized backpressure test using


the empirical gas deliverability equation


7-80
Problem. A well drilled on a square 640 acre pattern was
tested using a flow-after-flow test where each rate was
maintained for 8 hours. Data recorded during the test, as
well as approximate rock and fluid properties are presented.

pwf, psia q, Mscf/D

0
2'800 R)
2,661 1,800
2,590 2,700
2,500 3,600
2,425 4,500

k 74 md ct = 3.57 x 10-4 psi-1


1Sa rw = 0.33 ft
0.021 cp

a) Compute C and n for the test well,


b) What is the AOF of this well?
c) After PR has declined to 2000 psia, what flowing
bottomhole pressure will be necessary for the well to
produce at a rate of SNlhlscf/D?
d) Did this well flow long enough at each rate to stabilize?

Solution. Calculations required to prepare the log-log plot


are summarized by the following table:

pwf' psia q, Mscf/D pu-pwf, psi2x105

2,661 1,800 7.59


2,590 2,700 11.32
2,500 3,600 15.90
2,425 4,500 19.59
2,000 ? 38.40
0 ? 78.40

7-81
A log-log plot of the test data is presented in Fig. 7.25.

0 10'

= pR = 7.84 x 106 PSI2

° 106
N

• = 16.8 MMsc

105

10 3 10 4 105

q, Mscf/D

Fig. 7.25: Deliverability graph for Ex. 7.7.

a) Select two points from the straight line:

q = 3,000 Mscf/D at pR - p^ f 1.3x106 psi2

q = 15,000 Mscf/D at ^R - pwf = 7x106 psi2.


7-82
log c12 - log ql
n =
log (Ap2) - log (Api)

log(15,000) - log(3,000)
IL =
log(7x106) - log(l.3x106)

n = 0.96.

The deliverability coefficient, C, can be computed using


Eq. 7.88:

q
C=
(Ap2)n

Using a point on the straight line, i.e. , q = 3000 Mscf/D at


-2_ pwf
2 = 1.3 x 106, and the value of n,
PR

C = 3000
(1.3 x 106)0•96

C = 4.05 x 10-3Mscf/D/psi2

Therefore, the deliverability equation for this reservoir


is

-3 -2 2 0•96
q= 4.05 x 10 (pR - pwf)

b) The AOF can be determined graphically from Fig. 7.25,


or calculated by the deliverability equation. From Fig.
7.25 at PR = 7.84 x 106 (pwf=0) ,

AOF = 16.8 MMscf/D

From the deliverability equation,


7-83
AOF = 4.05 x 10-3(p2R)0'96

S = 4.05 X 10-3(7.84 X 106)0•96

= 16,800 Mscf/D

AOF = 16.8 MMscf/D.

c) From the deliverability equation


1/0 •96
2 2 q
pwf _ PR
4.05x10-3

5 000 1/0•96
= 4.0 x lO 6 - ' _3
4.05 x 10

pwf = 1.785 x 106

* pw f = 1,336 psia.

d) The minimum time required for a well to stabilize is


given by Eq. 7.83:

OuctA
k
tpss = 380

-4
(380) (0.15) (0.021) (3.57 x 10 ) (640) (43,560)
- 74

t = 161 hrs.
pss

It is obvious from this result that this well did not stabilize
during the 8-hour flow periods used in the test. This means
that any rate predictions made using the deliverability
equation derived from this test will be optimistic.


7-84
B. Theoretical Method of Analysis
The pseudosteady state gas equation, Eq. 7.91, can be
rewritten as
LJ
r r
2 (7.99)
= 1424 ^ ln re - 0.75 + s' + Dq]
pR pw L
This is a theoretically based equation which, if a well
flows sufficiently long to achieve pseudosteady state, can
be used to predict deliverability as a function of PR and
Before it can be used, however, other parameters in
pwf"
the equation must first be determined. Equation 7.99 can
be rewritten as

Asq + Bsq2 (7.100)


PR pwf

where
r r
As = 1424 khT ln re - 0.75 + s'I (7.101)
L w

Bs = 14 24 khT D. ( 7.^)

Equation 7.100 can be further rearranged as

-2 2
PR pwf (7. 103)
= As + Bsq

which suggests that a plot of 2 22


pwf) /q] versus q
PR
should yield a straight line on cartesian paper with slope,
Bs, and intercept, As. This plot is illustrated by Fig.
7.26. Once as and Bs are determined from this plot, gas i 1

deliverability can be predicted using Eq. 7.100.

= As+C^s + q Ss CPa-Pw^)^`^z
^
28S

7- 85

cV
•'-I 4a
U) U
(A
:E,

t4.4 SLOPE = Bs
C-4 3

N
I r-4 INTERCEPT = As

q , Mscf/D

Fig. 7.26: Gas deliverability plot based


upon the theoretical stabilized
flow equation.


Example 7.8: Analysis of stabilized backpressure test using
theoretical deliverability equation

Problem. The following results of a four-point stabilized


backpressure test were reported by Lee 27 Use the theoretical
pseudosteady state flow equation to develop a deliverability
equation for this reservoir. Use this equation to compute
the AOF of the well.

-2_ 2 -2_ 2
pwf q PR pwf (PR pwf/q)
(psia) (Mscf/D) (psi2) (psi2/Mscf/D)
408.2 0 - -
403.1 4,288 4,138 0.965
394.0 9,265 11,391 1.229
^ 378.5 15,552 23,365 1.502
362.6 20,177 35,148 1.742
0 AOF=? 166,411 -
7-86
Solution. A plot of ( pR - pWf)/q versus q is presented in
Fig. 7-27. From two points on the straight line it is
determined that •

Bs = slope = 4.94 x 10 5 psi2/(Mscf/D)2

As = intercept = 0.753 psi2/Mscf/D.

Therefore, the gas deliverability equation is:

0.753q + 4.94 x l0-sa2


PR pwf

This quadratic equation can be solved to determine the flow


rate, q, corresponding to any value of PR and pwf'
The AOF can be obtained by substituting pwf=0 into the
deliverability equation and solving for q, i.e.,

pR = 0.753a + 4.94 x 10_ 5 q2

4.94 x 10-5a 2+ 0.753q - 1.666 x 105 = 0. ^

Solving this quadratic equation for q,

AOF = q = 50,949 Mscf/D = 50.95 MMscf/D.


7-87
_. :.--- ^ - - - -- ---- - - --_ _ . - --------- ^ ^ - - ^.
2.0
-- ------^- - -;-----^^------G--^
--- -

i• - ._. __.--
I
_^
^ ^
-i---^- ^ -
^- -
-- -- - _.--- -^ - -
-

1.8
---_-t--_. • . .

-`^

1.6

1.4

4; ^

N (1!

1.2

i• 1.0

0.8
0 S 10 15 20 2S

q, Mscf/D x 10 3

Fig. 7.27: Deliverability graph for Ex. 7.8.

is
7-88
C. Summary
It is clear from the foregoing discussion that
stabilized flow-after-flow tests apply only to high
permeability reservoirs which can achieve pseudosteady
state in a short period of time. If a well is not tested
long enough, production forecasts based on test results will
be optimistic. This is a common mistake which can have
disastrous results if the rate predictions are used for gas
contract negotiations, property acquisition, equipment and
pipeline design, or planning for field development.
When permeability is small, isochronal tests, or
modified isochronal tests, should be used to determine gas
deliverability.

2. Isochronal Tests
The primary limitation of the flow-after-flow test is
the requirement that each flow period must last sufficiently
long to achieve pseudosteady state. The objective of an
isochronal test is to obtain a stabilized deliverability
curve without having to flow at all, or any, rates long
enough to achieve pseudosteady state. The major application
of this test is in low permeability reservoirs where it is
impractical to achieve stabilized conditions during the test.
The isochronal test28 is similar to the flow-after-flow
test, but requires that the well be shut in between flow
periods. The procedure for conducting an isochronal test is:

1) Shut in the well and measure PR-


2) Flow the well at rate ql and record the
flowing pressure, pwf, at tl, t2, t3,-----tn.
3) Shut in the well sufficiently long for pressure
to build back up to PR-
4) Flow the well at rate q2 for the same length
of time used in Step 2. Measure pwf at
tilt t-), t3,-----tn which also correspond to
the times when pressure was measured in Step 2.

7-89
5) Repeat Steps 3 and 4 for rates q3 and q4.
b) At the end of the final flow period, leave
^ the well on production sufficiently long to
reach pseudosteady state. Measure the
flowing pressure after pseudosteady state is
achieved. This final step may not be possible
in very tight reservoirs. Further, if the
well is not connected to a pipeline at the
time of the test, it may be necessary to
delay this step until a pipeline connection
is available.

The rate and pressure history of an isochronal test is


depicted by Fig. 7.28.
It is important to observe that this test requires equal
flow periods and that pressure measurements must be made at
the same times during each flow period. Further, the shut-in
periods are not of equal length. The objective of the shut-
in period is to permit pressure to build back up to PR; this
will require increasingly longer shut-in periods as the test
proceeds.
The isochronal test is based on the observation that
the radius of investigation of a test depends upon diffusivity
and time, but not upon rate. This is shown clearly by Eq.
3.14. Consequently, values of pwf measured at time t1 during
each of the four flow periods will represent the same radius
of investigation, r1; pressures measured at time t2 will
represent a radius of investigation, r2; etc. Therefore, by
plotting pressures measured at a particular time during each
flow period, we obtain a deliverability curve that represents
the drainage radius associated with that particular flow
time. This is illustrated by the deliverability plot
presented in Fig. 7.29. Notice that the slopes of the lines
representing different flow times are equal; this means that
the correct slope of a stabilized deliverability curve can
be determined from short flow times, even though the times

7-90
q
a4
01
^ a2

a1

TI ME tpss

pR

pwfl

pwf2
•i
pwf3

pwf4

(pwf) pss

`pss

T IN1E

Fig. 7.28: Rate and pressure history of an


isochronal test.


7-91
are not adequate for the well to reach pseudosteady state.
The positions of the lines, however, are different; this is
^ because the gas deliverability coefficient is dependent upon
time until the well reaches pseudosteady state.

106
t
pss

t3
Jt 2

tl
105 v
4^~

ti
wv
N3

N
y^
^v
10`'

• tl<t2<t3<tpss

101
102 103 104 105

q, Mscf/D

Fig. 7.29: Deliverability curve for


isochronal test.

The objective of the isochronal test is to determine the


stabilized deliverability curve; this requires that we know
the correct slope and position of the curve. The deliverability
curves which represent short flow times give us the correct
slope. Since position can be established with one point, it
is necessary to extend only one rate long enough to achieve
• pseudosteady state, and to measure pressure at that time. A

7-92
line through this point of correct slope establishes the
stabilized deliverability curve. This curve is depicted ^
by Fig. 7.?J.

A. Empirical Method of Analysis


The procedure for analyzing isochronal data using the
empirical method is:

1) Plot (PR pwf) versus q on log-log paper as


depicted by Fig. 7.29.
2) Draw the best straight line through points
obtained at fixed values of time for each flow
rate.
3) Determine the slope of each line and decide
upon the best value of n.
4) Using the slope from Step 3, draw a line through
the single stabilized point obtained from the
extended flow period. This is the correct
stabilized deliverability curve.
5) Determine C for the stabilized cur^-c t%,ith
the same equation and procedure used for
flow-after-flow tests. C and n, when
substituted into Eq. 7.84, establish the
deliverability equation for the test well.

B. Theoretical Method of Analysis


This method is based on the theoretical equations
which describe transient flow and pseudosteady state. For
pseudosteady state, we have from Eqs. 7.100 - 7.102 that

-2 2 (7.100)
PR pwf Asa + Bsa2

where

r r
ln re - 0 . 75 + s'^ ( 7. 101)
As = 14 24 khT
L

7-93
Bs = 1424 khT D. (7/. .1.02)
^

When flow is transient, Eqs. 7.95 - 7.97 show that

(7.95)
PR pwf Ata + Btq 2

whe re

At = 16375H 11 og
kt 2_ 3.23 + 0.87s'^ (7.96)
^uctrw

B = 1424u2T D. (7.97)
t kh

The procedure for using the theoretical equations to


analyze isochronal data is:

1) Plot (P- 2 2
pwf)/a versus q for a fixed value of
R
t and determine the slope of this line. This
slope, as shown by Eq. 7.95 is equal to Bt.
Comparison of Eqs. 7.97 and 7.102 shows,
however, that this slope is also equal to Bs,
a constant in the stabilized flow equation.
2) Using the value of q and (P- 2 - pW f) obtained
from the extended flow period, compute As
using Eq. 7.100 in the form

- pwf) - Bsq2]/q. (7.104)


As = [(P 2R

3) With As and Bs known, Eq. 7.100 can be used


to compute gas deliverability as

-As + [AS + 4Bs ( PR 2 f)1 -2


Pw
cl = . (7.105)
2B
^ s

7-94
3. Modified Isochronal Test
The major disadvantage of the isochronal test is the
long shut-in periods required for pressure to stabilize at
The modified isochronal test is designed to overcome
PR'
this disadvantage by using equal shut-in periods having the
same duration as the flow period. This results in a shorter
test, but does not permit pressure to stabilize at PR during
the shut-in period. Consequently, the shut-in pressure
which precedes each flow period is different; these pressures
must be measured during the test as well as the pressure
at the end of each flow period. The rate and pressure
history during a modified isochronal test is depicted by
Fig. 7.30.
The method of analyzing the modified isochronal test is
basically the same as that of the isochronal test. The
primary difference is that the shut-in pressure, pws, pre-
ceding each flow period is used in place of PR to calculate
Ap2. For example, apz =(PWSl Pwf) at ql' Ap2
(pws2 Pwf)
at qZ, etc. After the deliverability plot is made using
these pressure differences, whether in the empirical or
theoretical method, the equations and procedures used to
analyze the data are identical to the isochronal method.
The modified isochronal test, unlike the isochronal
test, is not theoretically rigorous. However, experience
has shown that it is an excellent approximation to the
more rigorous isochronal test. Because of the savings in
time and money, this test is used most of the time in
preference to the isochronal test.

Example 7.9: Analysis of a modified isochronal test using


the empirical method

Problem. The following data are from a modified isochronal


test run on a well in the Crossfield East Field in Alberta,
Canada:

7-95
q4

i• a3

a2

Q1

0 I 1 I'll, pss

PR
pws2
pwsl pws3
pws4

pwfl

I • = pwf2

pwf3 pwA

(pwf)pss

0 TINIE

Fig. 7.30: Rate and pressure history for a


modified isochronal test.


7-96
Duration Pws or pwf q
(hrs) ( sia) Mscf/I)
Initial Shut In 371 2,874 -
Flow 1 8 2,777 1,478
Shut in 8 2,859 -
Flow 2 8 2,722 2,048
Shut In 8 2,849 -
Flow 3 8 2,664 2,587
Shut In 8 2,839 -
Flow 4 8 2,566 3,403
Extended Flow 58 2,463 3,268

Determine values of C and n for this well, and determine the


absolute open flow.

Solution. Calculations necessary to prepare the deliverability


plot are summarized in the following table.

q
2 _ 2
q pws pwf
pws pwf
(h9scf/D) (psia) (psia) (psia2x103)
1,478 2,874 2,777 548
2,048 2,859 2,722 765
2,5-87 2,849 2,664 1,020
3,403 2,839 2,566 1,476
3,268 2,874* 2,463 2,194

*Note that pR, the true average reservoir


pressure is used for the stabilized point
#i
A plot of these data is presented in Fig. 7.31. The slope
of the stabilized line can be calculated using Eq. 7.87:

log q2 - log q1
n =
log (Op2) - log (Api)
0

7-97
10

• -
---- _- -- _-._- _ -=
-- o '

- ----_r_-
--- ---
-- -- ^ -
- ^ 4
_
_- F =-- ^ i
^, - -- -
r-! -
- -r-- r-^ ^ -
^
_ -^-
-
r
- - ` - -'-^-
N -- Q '
.

10

N 3
n = 0.77
N= C = 0. 04 27 Ms cf/D/ps i
I P.
AOF = 9.05 MMscf/D

• 1

103 10`' 105

q, Mscf/D

Fig. 7.31: Deliverability curve for Ex. 7.9.

Two points from the stabilized straight line are:

Api = 8x105 psia2 at ql = 1.5x103 Mscf/D

Ap^ = 7x106 psia2 at q2 = 8.Ox103 Mscf/D.

These points, when substituted into Eq. 7.87, yield the


following result:

log(8.0x103) - 1og(1.Sx103
n =
11 log(7x106) - log(8x105)

n = 0.77.
7-98
With n known, C can be computed using Eq. 7.88. Substituting
a point from the straight line, i.e., ap2 = 8x105 psia2 at
q = 1.5x103 Mscf/D, into this equation, •
q
C=
(OP2)n

1500
(8x10s)o•77

C = 0.0427 Mscf/D/psi2

Therefore, the deliverability equation for this well is

q = 0.0427(p2R-Pwf)1•77

The AOF can be computed by substituting pwf = 0 into


the deliverability equation, i.e.,

AOF = q = 0.0427(p2R) o'" •


o . 7 7
= 0.0427 (8.26xl06)

AOF = 9,047 Mscf/D.

This value is in agreement with the AOF read directly from


Fig. 7.31.

4. Analysis of Data When Pseudosteady State Flow is Not


Attained
In reservoirs of very low permeability, it may not be
possible to obtain any data in pseudosteady state flow because
of the unreasonable time required. In this case, an isochronal

7-99
or modified isochronal test is run without the extended flow
^ period at the end, and the data are analyzed using the
theoretical method. It was shown previously that the pseudo-
steady state flow equation can be written as

pR - pwf = A s q + B s q 2 (7.100)

where

As = 142khzT [ln r re - 0.75 + s(7.101)


w

and

Bs _ 142khzT D
(7,1p2)

The constant, Bs, can be determined from a plot of


the isochronal data as (pR-pwf)/q versus q; the slope of
the straight line for a fixed value of time is equal to Bs.
Without a stabilized point, however, it is not possible
to determine As from the test. Instead, As must be
calculated from Eq. 7.101 using data from external sources.
It is observed from Eq. 7.101 that values of kh, u, z, T,
re, rw and s' are required for this calculation. Fluid
property data, temperature and wellbore radius will
probably not be difficult to obtain. Drainage radius, re,
must be estimated based on well spacing and knowledge of the
drainage geometry.
The most difficult data to obtain are kh and s'. The
value of kh can be measured using a drawdown or buildup
test in conjunction with the deliverability test.
Determination of s', however, will require that at least
two tests, buildups or drawdowns, be run at different rates.
Recall from Eq. 7.12 that s' is defined as

^ s' = s - Dq. (7.12)

7-100
Values of s, calculated from each of the two tests, can be
plotted versus q as shown in Fig. 7.3. Values of s' and D
can be obtained from the plot. A simple procedure for
determining kh, s' and D using a two rate flow test is
0
presented by Carter, et al?
With Bs known from the slope of the deliverability plot,
and A. calculated from Eq. 7.101, these values can be
substituted into Eq. 7.100 to obtain the stabilized deliver-
ability equation. If the isochronal test is not available
to measure, Bs, notice that it can also be calculated
theoretically from Eq. 7.102 using the value of D from the
two rate test.

S. Use of Real Gas Pseudopressure, m(p), In Gas Well


Deliverability Analysis.
All of the equations presented thus far to analyze
deliverability tests have been written in terms of p2.
Improved accuracy can be obtained in some tests by using
the real gas pseudopressure, m(p), previously defined by
Eq. 7.26 as •
p

m(P) = 2 L. dp. (7.26)


f Pz
Pb

For example, the pseudosteady state flow equation can be


written in terms of m(p) as

r r
24 in re - 0. 75 + s' + Dq] (7.10b)
m(PR) 14k^
m(Pw f)
L w

or m(PR) - m(pwf) = Asq + BSq2 (7. 107)

where
re - 0.75 + s'J (7. 108)
As = 1kh4T [ln r
w
^
B, = 1424T D (;'.109)
S kh
7-101
Equation 7.107 indicates that a plot of [M( PR) - m(pws)]/a
^ versus q will yield a straight line of slope BS, and
intercept, AS. Once AS and BS are known from the deliver-
ability plot, flow rate can be predicted using Eq. 7.107
in the form

-As + [A2 + 4BS {m(PR) - m(pw f) }1 2


a= J • (7. 110)
2B'
S

This procedure can be applied to stabilized flow-after-flow


tests, isochronal tests, or modified isochronal tests.

VII. SUMMARY

We have shown in this chapter how to analyze pressure


buildup and pressure drawdown tests for permeability, skin
^ factor, flow efficiency, average reservoir pressure and
fracture length. Both fractured and non-fractured wells
have been discussed, and both conventional and type curve
methods of analysis have been presented. Common problems
of interpretation such as wellbore storage and boundary
effects were discussed in detail as to their effect on test
design and analysis. Finally, methods commonly used to
ineasure gas well deliverability were presented for both
high and low permeability reservoirs.
Three methods were presented that can be used to
analyze pressure data from gas wells, i.e., the p2-method,
p-method, and m(p)-method. The advantages of using the
• m(p) method are clear; accordingly, it is recommended that
this method be used for the analysis of all pressure
transient data from gas wells.


7-102

0
REFERENCES

1. Aronofsky, J. S. and Jenkins, R.: "A Simplified Analysis


of Unsteady Radial Gas Flow," Trans., AIME (1954) 201, 49.

2. Carter, R. D., Miller, S. C., Jr., and Riley, H. G.:


"Determination of Stabilized Gas Well Performance from
Short Flow Tests," J. Pet. Tech. (June, 1963) 651.

3. Theory and Practice of the Testing of Gas Wells,


Energy Resources Conservation Board, Calgary, Canada
(1975) Third Edition.

4. Ramey, H. J., Jr.: "Non-Darcy Flow and Wellbore Storage


Effects in Pressure Buildup and Drawdown of Gas Wells,"
Trans. , AIME (1965) 234, 223.

5. Katz, D. L., et al.: Handbook of Natural Gas Engineering,


McGraw-Hill Book Company, Inc., 47.

6. Al-Hussainy, R., Ramey, H. J., Jr., and Crawford, P. B.:


"The Flow of Real Gases Through Porous Media," Trans.,
AIME (1966) 237, 624.

7. Al-Hussainy, R., Ramey, H. J., Jr.: Application of


Real Gas Flow Theory to Well Testing and Deliverability
Forecasting," Trans., AIME (1966) 237, 637.

8. Dake, L. P.: Fundamentals of Reservoir Engineering,


Elsevier Scientific Publishing Company, Amsterdam (1978).

9. Zana, E. T. and Thomas, G. W.: "Some Effects of


Contaminants on Real Gas Flow," J. Pet. Tech. (Sept.,
1970) 1157.

10. Agarwal, R. G.: "A New Method to Account for Producing


Time Effects When Drawdown Type Curves Are Used to
Analyze Pressure Buildup and Other Test Data," paper SPE
9289 presented at the SPE 55th Annual Fall Technical
Conference and Exhibition in Dallas, Sept. 21-24, 1980.

11. Agarwal, R. G.: "Real Gas Pseudotime - A New Function


for Pressure Buildup Analysis of MHF Gas Wells," paper
SPE 8279 presented at the 1979 SPE Annual Technical
Conference and Exhibition, Las Vegas, Sept. 23-26.

12. Lee, W. J., and Holditch, S. A.: "Application of


Pseudotime to Buildup Test Analysis of Low Permeability
Gas Wells With Long-Duration Wellbore Storage Distortion,"
J. Pet. Tech. (Dec., 1982) 2877.


7-103
13. Earlougher, R. C., Jr., and Kersch, K. M.: "Analysis
of Short-Time Transient Test Data by Type-Curve Matching,

14.
J. Pet. Tech. (July, 1974) 793-800.

Gringarten, A. C., B )urdet, D. P., Landel, P. A., and



Kniazeff, V. J. : "A Comparison Between Different Skin
and Wellbore Storage Type-Curves to Early Time Transient
Analysis," paper SPE 8205 presented at the SPE S4th
Annual Fall Meeting, Las Vegas, Sept. 23-26, 1979.

15. Cinco-L., H., Samaniego-V., F., and Dominguez-A., N.:


"Transient Pressure Behavior for a Well With a Finite-
Conductivity Vertical Fracture," Soc. Pet. Eng. J.
(Aug., 1978) 253-264.

16. Agarwal, R. G., Carter, R. D., and Pollock, C. B.:


"Evaluation and Performance Prediction of Low-Permeability
Wells Stimulated by Massive Hydraulic Fracturing,"
J. Pet. Tech. (March, 1979) 362-372.

17. Bostic, J. N., Agarwal, R. G., and Carter, R. D.:


"Combined Analysis of Postfracturing Performance and
Pressure Buildup Data for Evaluating an MHF Gas Well,"
J. Pet. Tech. ( Oct., 1980) 1711-1719.

18. Holditch, S. A., Lee, W. J., and Gist, S. R.: "An


Improved Technique for Estimating Permeability, Fracture
Length, and Fracture Conductivity From Pressure Buildup
Tests in Low-Permeability Gas Wells," J. Pet. Tech.
(May, 1983) 981-990.

19. Donohue, D. A. T., and Ertekin, T.: Gaswell Testing:
Theory, Practice and Regulation, International Human
Resources Development Corporation, Boston (1982).

20. Interstate Oil Compact Commission, IOCC Procedures, Box


53127, Oklahoma City, OK 73105.

21. Railroad Commission of Texas, Oil and Gas Division, Back


Pressure Test For Natural Gas Wells, Revised Ed. (1951T,
9. 0. Drawer 12967, Austin, TX 78711.

22. Oklahoma Corporation Commisson, Manual of Back-Pressure


Testing of Gas Wells, Thorpe Buil^ ing, Oklahoma City, OK
73105.

23. State of New Mexico Oil Conservation Commission, Manual


for Back-Pressure Testing of Natural Gas Wells, Order
R-333-F, Jan. 1, 1966, P. 0. Box 2088, Santa Fe, NM 87501

24. Louisiana Oil and Gas Handbook, Sixth edition, 1260


Havenwood Drive, Baton Rouge, LA 70815.

q
7-104
^ 25. Rawlins, E. L. and Schellhardt, M. A.: Back-Pressure
Data on Natural-Gas Wells and Their Application to
Production Practices, Monograph 7, United States Bureau
ot mines

... 26. Craft, B. C. and Hawkins, M. F., Jr.: Applied


Petroleum Reservoir Engineering, Prentice-Hall Book
Co., Inc., Englewood Cliffs, N. J. (1959).

27. Lee W. J.: Well Testing, Textbook Series, Society of


Petroleum Engineers of Dallas (1982) 1.

28. Cullender, M. H.: The Isochronal Method of Determining


the Flow Characteristics of Gas Wells." Trans., AIME
(1955) 204, 137-142.


7-105

0
• NOMENCLATURE - CHAPTER 7

A = drainage area of well, ft2


As = constant in Eq. 7.101
AS = constant in Eq. 7.108
At = coefficient in Eq. 7.94
AOF = absolute open flow of gas well, Mscf/D
Bg = gas formation volume factor, RB/scf
Bs = constant in Eq. 7.100
Bs = constant in Eq. 7.109
Bt = constant in Eq. 7.94
C = wellbore storage factor, ft3/psi
2
C = gas deliverability coefficient, Mscf/D/psia
C' = coefficient in Eq. 7.92
= formation compressibility, psi-1
Cf
cg = gas compressibility, psi
^ co = oil compressibility, psi
c p r = reduced gas compressibility 1 = cgppc, dimensio nless
ct = total compressibility, psi
cw = water compressibility, psi
C = compressibility of wellbore fluids, psi
wb
D = turbulence coefficient, (Mscf/D)-1
E = flow efficiency, dimensionless
permeability correction factor for fractured systems,
F cor
dimensionless
FcD = dimensionless fracture capacity
G = initial gas in place, Mscf
Gp = cumulative gas production, Mscf
h = net formation thickness, ft
k = volumetric average effective permeability of the
reservoir in the drainage volume of the test well, md
ka = apparent permeability of fractured reservoir, md
m = slope of semilog straight line
^ mLF = slope of linear flow straight line on square root
p lo t

7-106
m PR = slope of pseudoradial straight line on semilog plot
for fractured reservoirs
m(p) = real gas pseudopressure, psi^ /cp
n = gas deliverability exponent
p = pressure, psia
p = false pressure obtained from semilog buildup plot
at infinite shut-in time, psia
= base pressure used to compute real gas pseudopressure,
Pb
m(p), psia
= dimensionless pressure
PD
dimensionless Matthews-Brons-Hazebroek pressure
pDMBH =
pi = initial reservoir pressure, or stabilized pressure
at the beginning of a drawdown test, psia
p- = average reservoir pressure at time of last shut-in,
psia
pPc = pseudocritical pressure, psia
ppr = pseudoreduced pressure, dimensionless
= volumetric average reservoir pressure, psia
PR
Psc = pressure at standard conditions, psia

pwf = flowing formation face pressure, psia


=0) = flowing formation face pressure immediately
pwf(At
preceding shut-in of well for buildup test, psia
pressure from semilog straight line at At = 1 hour,
plhr =
psia
q = gas flow rate, Mscf/D
r = radius, ft
re = drainage _radius, ft
rw = wellbore radius, ft
Sg = gas saturation, fraction
S = oil saturation, fraction
0
Sw = water saturation, fraction
s = totoal skin factor, dimensionless
i 1
s' = component of total skin factor which is independent of
rate, dimensionless
sd = skin due to permeability alteration, dimensionless
sf = skin due to fracture, dimensionless
sp = skin due to perforations, dimensionless
sr = skin due to restricted entry, dimensionless
s = skin due to slanted well, dimensionless
sw

7-107
st = skin due to turbulence, dimensionless
T = reservoir temperature, °R
T pr = p seudoreduced temperature , dimensionless
T sc = temperature at standard conditions, OR
t = time, hrs
ta = pseudotime, psi-hr/cp
tD = dimensionless time based on r w
tDA = dimensionless time based on A
= dimensionless time to beginning of pseudosteady
(t DA ) pss
state
dimensionless time based on xf
tDxf =
tp = time well produced before shut-in, hrs
t pss = time required for well to reach pseudosteady state, hrs
v wb = volume of wellbore in communication with test formation,
ft,
z = gas deviation factor, dimensionless
AG p = incremental gas production, Mscf
Aps = pressure change due to skin, psi
At = shut-in time, hrs
ot ae = equivalent pseudotime
Ate = equivalent time, hrs
= gas gravity, dimensionless
yg
u = viscosity, cp
g as visc cp one atmosphere pressure and reservoir
uga emperature
= porosity, fraction


7-108
0

0
SUMMARY OF MAJOR EQUATIONS - CHAPTER 7

Equation Number Equation
in Text

p2 - Drawdown

7.5 p2 = p2 - 1637quzT [log t + log k


wf kh ct rw2

- 3.23 + 0.87s]

1637 -Z-T
7. 7 k =- m

2 2

7.9 S 1.151 plhrm pi - log k 2


uc t r w

• + 3. 231

2 - z - Q z
E _ pi pwf ps
7.13
pi p w2 f

p- Drawdown

162.6x103quB
7. 16 g[log t
pwf pi kh

+ log k - 3.23 + 0.87s]


2
^uetrw

162.6x103quBg
7. 20
k = - - mh

7-109
7.21 s = 1.151 plhrm pi - log k Z
^uotrw

^
+ 3.231

E = Pi pwf Aps
7.22
pi pwf

m(p) - Drawdown

p
7.26 m(p) = 2

Pb
J ^ dp

hr{T log kt 2
7. 3S m(pw f) = m(pi) - 16k7
$ui,tirw

- 3.23 + 0.87s^ ^

k = - 1637gT
7.37
mh

k
m(plhr) - m(pi) - log
7. 38 s = 1.151
L m otirw

+ 3. 231

M(Pi) - m(pw f) - AM(P)S


7.39 E --
M ( Pi ) - m pwf


7-110
p2 - Buildup

• 7.43
t +At
pw s = Pi - 163khuET log pQt

k = - 1637quzT
7.46 mh

[Pft=o- lhr
7.49 = 1.151 m

- log k + 3.23^
2
lac t r w

-2 2
pR pw f(At=0) - APs
7.50 E_
-2 _ 2 At-0
PR pwf( )

• p - Buildup

162.6x103qu^ t +Ot
7.54 g log p
Pws Pi kh

162.6x103quBg
7.58
k = - - mh

pwf(At=0) - Plhr
7.59 s = 1.151 m

- log ^k + 3.23]
^uotrw

pwf(At=0) - Aps
PR
7.60 E
PR p w f (At=0)

7-111
m(p) Buildup ^

t +^t
7.61 m(pws) = m(pi) - 16khaT log
4

k = - 1.637qT
7.63
mh

m(pw f) - m(plhr-)
7.64 s = 1.151 m

- log k + 3.231
^u^etrw
z

m(PR) m(pw f) - Am(p) s


7.65 E
M( PR) m(pw f)

Gas Well Deliverability

7. 84 q = C(pR - p^ f) n

7.100 PR - pwf = asq + BSa2

r
7.101 A = 142?T rln re - 0.75 + s
s
w

Bs = 142khzTD
7.102
. I

7.107 m(PR) - m(pw f = asq + BSq 2

r
7.108 as 1424T ^ln re 0.75 + s'^

7- 112
Bs = 14k4hTD
7.109
0

-AS + [(As)2 + 4BS{m(pR) - m(Pwf)1^ ,z


7. 110 q -
2B'
s

is
7-113


PROBLEMS

^ GAS WELL TESTING

l. The properties of a natural gas are presented as


a function of pressure in the table below. You are
requested to compute the real gas pseudopressure, m(p),
at each pressure.

p, psia z P" cp

0 1.0 -
400 0.955 0.0118
800 0.914 0.0125
1,200 0.879 0.0134
1,600 0.853 0.0145

^ 2. A new gas well produced 275 Mscf at a constant rate


of 135 Mscf/D and was shut in for a pressure buildup
test. With the buildup data and the reservoir rock and
fluid properties given, estimate the formation perme-
ability, total skin factor, and flow efficiency using
(a) the p-method, (b) the p2-method and (c) the m(p)-
method. Horner plots of pZ, p and m(p) are given along
with the m(p) versus p relationship for this reservoir.

= 18.5%
S = 301'0
w
h = 6 ft
rw = 0.17 ft
pwf(At=0) = 566 psia
m(pw f) = m(566) = 2.57 x 10'psi 2/cp
Gas gravity = 0.60
Well Spacing = 640 acres (assume well in
^ center of square)
Formation depth = 5,430 ft
T = 158°F

7-114
tp+At p ws m(p) Am (P)
At
(hours) At (psia) (psi`/cpx107) (psiZ/cpxl0')
0.00 - S66 2.57 -
0.07 699 655 3.41 0.84
0.11 445 681 3.72 1.15
0.14 350 711 4.07 1.50
0.18 273 744 4.45 1.88
0.22 223 764 4.68 2.11
0.25 197 1793 5.06 2.49
0.36 137 857 5.91 3.34
0.54 92 919 6.79 4.22
0.72 69 1031 8.51 5.94
0.90 55 1129 10.09 7.52
1.08 46 1246 12.48 9.91
1.27 40 1335 14.25 11.68
1.45 35 1427 16.10 13.53
1.63 31 1502 17.75 15.18
1.81 28 1583 19.75 17.18
1.99 25.6 1652 21.40 18.83
2.35 21.8 1778 24.40 21.83
2.71 19.0 1883 27.30 24.73
3.25 16.0 2007 30.80 28.23
4.16 12.8 2147 34.75 32.18
4.88 11.0 2211 36.70 34.13
5.61 9.71 2254 38.00 35.43
6.51 8.51 2289 39.1.7 36.60
7.59 7.44 2317 40.04 37.47
9.04 6.41 2335 40.61 38.04
10.13 5.83 2348 41.01 38.44
14.10 4.47 2371 41.74 39.17
18.08 3.70 2386 42.21 39.64
21.70 3.25 2395 42.50 39.93
27.12 2.80 2404 42.79 40.22
34.36 2.42 2412 43.04 40.47
50 . 63 1 . 97 2426 43 . 49 40 . 92
79.56 1.61 2437 43.84 41.'7

7-115
^ • ^
50 ^

PROBLEM 2

40 L

rl .•.I . r
0
. .. ,
X 30
^ ..^
V

. . . .t-. t 1 {

^l ¢ 20

_
y

- lfl-
10

t
..
.. 1

1 10 102 103

t +Ot
p
4t
6

4
10
0

3
^
U)

^
N

0
1 10 10.2 103

t +At
-P
4t

^
E • •
2600

2400

2200

2000

1800

^
1600

1400

00
1200

1000

800

60C
1 10 102 103

t +At
p
At
50

40

r--^

30
u

20

10

0 400 800 1200 1600 2000 2400 2800


pressure, psia

m(p) vs Pressure for 0.60 Specific Gravity Gas and 1580F

• • •
E C:

-
Q U
L 4
q L
.^ }
L

1 2 3 4 5 8 7 B 9 1 2 3 4 5 C 7 B O 1 2 3 4 5 6 7 B 9 1


7-120
3. Using the isochronal data given (adapted from
Ref. 3), you are asked to determine the absolute open
flow potential and the gas deliverability equation using
(a) the empirical method and (b) the theoretical method.

Duration pwf or pws q Remarks


(hours) (psia) (Mscf/D)
48 1,952 0 Initial shut-in
12 1,761 2,600 First flow
15 1,952 0 First shut-in
12 1,694 3,300 Second flow
17 1,952 0 Second shut-in
12 1,510 5,000 Third flow
18 1,952 0 Third shut-in
12 1,320 6,300 Fourth flow
72 1,151 6,000 Extended flow (stabilized)
100 1,952 0 Final shut-in


7-121
I` •

U
4

q L
C:
Z^.

U
U. 4

q ^
>
L
P


7-122
DIETZGEN CORPORATION
, .,, .•^,
NO. 340-10 DIETZGEN GRAPH PAPER
IO X 10 PER INCH MADE IN U.B.A.
• 4. The following data is for a low-permeability MIIF gas
well. which is producing at constant formation face pressure.
The permeability of the formation, determined using a
prefrac pressure buildup test, is 0.0081 md. Use the
Agarwal constant pressure type curve to analyze the test
data for fracture length, xf, and fracture conductivity,
kfw.

= 2,394 psia
pi
T = 260°F
h = 32 ft.
k = 0.0081 md
10.7%
-4
Cti = 2.34 x 10 psi (at p
ui = O.0176Cp (at pi)
m(pi) - m(pwE) = 396 x 10° psi'/cp
40

t, days q, MSCF/D 1/q, (MSCF/D) i

20 625 1.6 x 10-3


35 476 2.1 x 10-3
50 408 2.45 x 10-3
100 508 3.25 x 10- 3
150 250 4.00 x 10 3
250 208 4.81 x 10 ;
300 192 5.21 x 10 3

• I-'

7-123A

0
49
Society of Petroleern Fngirx3ees of AIME

0 8279
"REAL GAS PSEUDO-TIME" - A NEW FUNCTION FOR PRESSURE
BUILDUP ANALYSIS OF MHF GAS WELLS

by Ram G. Agarwal, Member SPE-AIME,


Amoco Production Co.

:6) Copyright 1979. Amenran institute of Mrnrnp Metallurgical. and Pctroleum Engineers. Inc

This paper was presented at the 54th Annual Fill TeChckal Conference and Exhibition Al tne Society of Pettoleum inqinC:rS of AIME neW in Las Yepas Nevada. September 2726. 1979. The nat?na1 is 5u-je[I to correction Ly the
autnor PermsSnn to copy rs renrxeed in an aostract al not more Wn 700 words Write 6209 N Cenlrai ExpY DallasTexas 75206

ABSTRACT and time variables are appropriately defined for gas


wells. For example, to use the liquid system type
A new time function has been defined which con- curves for an MHF gas well, dimensionless variables
siders variations of gas viscosity and compressi- are defined as follows:2
bility as a function of pressure, which in turn is a
function of time. This function appears to be Dimensionless pressure,
similar to the real gas pseudo-pressure, m(p) of
Al-Hussainy et al., which takes into account the
variations of gas viscosity and z-factor as a func-
_ kh 0[
tion of pressure. However, this is an approximate
pwD 142 qT (1)
function as opposed to m(p). This time function
will be referred to in this paper as the real as
eudo-time ta(p). This function has aided in -3
t-treatment pressure buildup analysis of frac- (In SI units, the numerical constant is 1-28x10 )
ed (including M}iF) gas wells by type curve anal-
ysis. Results of computer simulated pressure Dimensionless pressutre, for a gas well, may also be
buildup analysis indicate that the use of t (p) pro- expressed in terms of A(p2) or Op.
vides satisfactory values of computed fracture
lengths in fractured gas wells. Dimensionless time,

In this paper the real gas pseudo-time is


described and its application is demonstrated by
t 2 . 634 x 10-4 kt
means of example problems. Although the discussion Dxf =
2 (2)
in this paper is limited to pressure buildup anal-
O(Nct)i xf
ysis of vertically fractured gas wells, the utility
of this function is not meant to be restricted to
suc h we ll s on l y. (In SI units, the numerica l constant i s 3 . 6 x 109

INTRODUCTION The definition of dimensionless fracture


capacity remains the same.
In recent years, type curve analysis methodsl
have become well known in the petroleum industry for
analyzing both pressure drawdown and buildup data in kfw
oil and gas wells. These methods are meant to be F
CD kx (3)
used in conjunction with the conventional methods f
whenever possible. Exceptions appear to be P[HF gas
wells with finite flow capacity fractures where con-
ventional methods are not readily applicable and, at Note that in Eq. (1), the real gas
least to date, only type curve methods appear prac- pseudo-pressure, m(p) of Al-Hussainy et al .4,5
tical to determine fracture length and fracture flow has been used to take into account the variations of
capacity. Although the majority of published type gas viscosity and z-factor as a function of
curves,' including those for MHF we11s,2i3 are based pressure. In Eq. (2),,viscosity-compressibility
on the pressure drawdown solutions for liquid sys- (Nc t ) i is shown to be evaluated at the initial
tems, they can be used in an approximate fashion to reservoir pressure.
nalyze pressure data from real gas wells. The
at requirement is that the dimensionless pressure In analyzing pressure drawdown data from real
gas wells using a liquid system type curve, it is
eferences and illustrations at end of paper. recommended that the real gas pseudo-pressure is
"REAL GAS PSEUDO-TIME" -- A NEW FUNCTION FOR

used and 4[m(p)] to be used in Eq. ( 1) is defined as TYPE CURVES FOR VERTICALLY FRACTURED WELLS
follows:
During the past few years, type curves have

A[m(P)] = m(pi) - m(Pwf) (4)


appeared in the petroleum literature which can be
used to analyze pressure data from vertically frac-
tured wells. Gringarten et a1.6 presented type
curves for infinite flow capacity fractures. Since

It has been also established that in analyzing their type curves could not be used for MHF wells
pressure drawdown data from gas wells by type curve with finite capacity fractures, Cinco et al.3 and
matching, reasonable answers are obtained if the Agarwal et all published new sets of type curves
(Nct) product in Eq. 2 is evaluated at the initial (finite fracture flow capacity) for MIiF' wells. Con-
reservoir pressure.4 stant wellbore rate type curves of Gringarten
et al.6 and Agarwal et al.2 are shown in Figs. 1
In analyzing pressure buildup data using draw- and 2, respectively. Since all these type curves
down type curves, the additional restriction which are based on the pressure drawdown data in liquid
should be imposed is that the producing time, t systems, it may first appear that they may not be
prior to shut-in is significantly greater than Pthe used to analyze (1) pressure buildup data in oil
shut-in time, At that is (t + At)/At s-- 1). This wells, and (2) pressure drawdown and buildup data in
should apply to both oil andpgas wells. In the case gas wells. Results of this study indicate that the
of gas well buildup analysis, A[m(p)] is defined as above type curves can be used, at least in an
follows: approximate fashion, to analyze a variety of draw-
down and buildup problems provided certain restric-
tions are realized and a few modifications are made.
0[m(P)] = m(POt) - m(Ppt-()) (5) These restrictions will be examined next.

1. Use of Drawdown Type Curves for Buildup


However, in using type curves for analyzing
pressure buildup data especially from fractured gas Let us first examine the validity and limita-
wells (even if the effect of producing time is tions of liquid system drawdown type curves (see
insignificant), it is not clear as to which pressure Figs. I and 2) to analyze pressure buildup data in
level the (Nct) product in the dimensionless term oil wells. Before these type curves may be used, it
should be evaluated. The question arises whether is important to note the duration of producing time,
the (pc ) product should be evaluated at (1) the tp compared to shut-in time, At.
initial treservoir pressure, (2) the final flowing
pressure prior to shut-in, or (3) some average pres- a) Small producing time
sure. Results of this study indicate that the use
of any one of the above three pressure levels is
less than satisfactory. In general, the use of the
initial reservoir pressure resulted in a fracture
If the producing time, t, prior to shut-in is
relatively short such that itpdoes affect the pres-
sure transients due to the subsequent buildup, draw-

length value greater than the actual; the use of the down type curves should not be used. In this case,
final flowing pressure provided a computed fracture pressure buildup type curves need to be generated to
length which was smaller than the actual; and the include the effect of producing time. This aspect
use of an average pressure provided a fracture of producing time and its effect on type curve anal-
length value which was different than that used in ysis has been discussed recently by Raghavan.7
the computer model. Fig: 3, taken from his paper, presents buildup type
curves for a vertically fractured well with infinite
To overcome the above difficulty, a new time flow capacity fracture (for x/x f =^). Dimension-
function has been studied. This has aided us in less pressure rise pDs has been plo^^ed as a func-
post-treatment pressure buildup analysis of frac- tion of dimensionless shut-in time, Dxf. A family
tured (including MHF) gas wells. This function con- of tyq curves is shown with dimensionless producing
siders variations of gas viscosity and compressibil- time, Dx as a parameter. This figure clearly
ity as a function of pressure, which in turn is a shows theflimitations of drawdown type curves for
function of time. This function is named real gas analyzing pressure buildup data collected after
pseudo-time, t(p), in this paper. This function small producing times. The effect of small pro-
is analogous toathe real gas pseudo-pressure, m(p) ducing time will not be considered in the subject
of Al-Hussainy et al.,4iS which includes the study. However, a future paper is planned to cover
effects of pressure dependent gas viscosity and this aspect.
z-factor. It should be emphasized that t(p) is an
approximate function as opposed to m(p). aHowever, b) Long producing time
it provides reasonable values of fracture lengths in
pressure buildup analysis of vertically fractured If the producing time prior to shut-in is sig-
gas wells and should be most useful for MHF gas nificantly long (that is (t + At)/At c-_ 1) so
wells. that it does not affect theptransients due to pres-
sure buildup, drawdown type curves may be used to
The discussion in this paper will deal with the analyze pressure buildup data. This should be
applicability and limitations of liquid system draw- obvious from Fig. 3. The basis for this is depicted
down type curves in analyzing pressure buildup data schematically in Fig. 4 in which the pressure
from gas wells. However, type curves for only ver- behavior is shown during both constant rate drawdown
tically fractured wells will be considered. and pressure buildup periods. According to the
pressure transient theory Ap during pressure draw- •
The discussion will also include a description down should be equal to that during the pressure
of the new time function, its computational proce- buildup (for t=At) provided they are defined as fol-
SPE 8279 RAM G. AGARWAL 3

(6a) a) Gas well drawdown data


(AP)drawdown - Pi pwi
_ (initial reservoir pressure) To use the liquid system type curves for gas
-(wellbore flowing pressure) well drawdown data, it is recommended that real gas
pseudo-pressure, m(p) is used in the dimensionless
pressure (pD) term and (pct) evaluated at the ini-
(AP)buildup - PLt Ptp+At (6b) tial reservoir tpressure is utilized in the dimen-
sionless time ( Dxf) term. Fig. 6 shows the
_ (shut-in pressure) comparison between the drawdown type curve for
-(wellbore flowing pressure liquid and real gas systems. Drawdown data were
in absence of shut-in) generated using the M1F simulator and reservoir data
shown in Table 1 and gas properties data shown in
Table 2. Results indicate that the use of a liquid
However, since Pt +At is not readily avail- system type curve for gas well drawdown data is rea-
able, ( AP) buildup is no6ally defined as equal to sonable provided that the above mentioned conditions
are met.
(PAt - PAt=o)'
If one questions the applicability of these
Thus there is a difference between the (Ap) drawdowa type curves for a particular gas reservoir because
of unusual gas properties and/or reservoir pressure,
and the way (AP) buildup are calculated. This dif- as suggested in a recent paper2, type curves could
be generated using an appropriate (PC ) function,
ference is equal to (POt=O - Pt +At) and is shown as temperature, and pressure ranges specific to the
the cross hatched area in Fig. 4P reservoir under study. These type curves should
then be used for pressure transient analysis of data
To further investigate this difference, an MHF from that reservoir.
simulator2 was used to simulate pressure buildup in
an i1HF gas well, using the liquid system analogy b) Gas well buildup data
(Nct = constant). Reservoir and fracture data are
shown in Table 1. The well was allowed to produce To use the drawdown type curves for gas well
at a constant rate for 180 days followed by a pres- buildup data, considerations regarding the duration
sure buildup test for 14 days. Pressure drawdown of producing time, as discussed earlier, should also
and buildup data expressed as A[m(p)]/q as function apply. Consequently, in this study it will be
of time (t or At) in days are plotted on coordinate assumed that the effect of producing time on pres-
graph paper and are shown in Fig. S. 0[m(p)] is sure buildup data is insignificant. The effect of
tL-fined as foll.ows: the (Nc.) product on pressure buildup data will be
mainly considered. As mentioned earlier, for pres-
sure buildup analysis it is not clear as to the
(7a) pressure level at which the (PC ) product in the
A[m(P)]drawdown - m(Pi) - m(Pwf)
dimensionless time term should be evaluated.

and To study this problem, the NNF simulator was


utilized to generate pressure buildup data on an PllIF
gas well using real gas properties. Reservoir data
- m(pAt-0) (7b) and gas properties used are shown in Tables 1 and 2.
A[m(P)]buildup - m(Ptt)
As was done for the liquid case, the well was pro-
duced at a constant rate for 180 days followed by a
Note that there is virtually no difference between pressure buildup for about 14 days. Pressure draw-
the drawdown and buildup data at early times. How- down and buildup data expressed as A[m(p)]/q as
ever, the difference gets bigger as time increases. function of time in days are plotted and shown on
This difference is due to the way A[m(p)]hildup is Fig. 7. Note that this figure is similar to Fig. 5
calculated. Also shown on Fig. 5 as the cross presented earlier for the liquid case. .A[m(p)] has
been appropriately defined. Also shown on Fig. 7,
hatched area is [m(PAt=O) - m(ptp+At)], which is
as the cross hatched area, is [n'(PAt=O) - m(ptp+At)].
equal to the above difference. 'fhus there is a
basic difference between the drawdown and buildup Notice that there is a marked difference between the
type curves. However, in many cases this difference drawdown and buildup type curve. This difference is
is•not significant. This fact should also be much greater than that shown by the cross hatched
apparent from Fig. 3 where the difference between area, mentioned above and discussed earlier.
the drawdown and buildup curve for long producing
times is shown to be small. This indicates that in The big difference in the drawdown and buildup
liquid systems, for large producing times, pressure curves for gas wells is due to large variations of
buildup data can be analyzed using drawdown type (PC ) or (^act) product as a function of pressure.
curves to obtain reasonable answers. Forggas wells, (Nct) is approximately equal to
S•(Nc ). Fig. 8 shows the graph of (pc ) vs pres-
2. Use of Liquid System Type Curves for Gas Wells sfire Ar the simulated gas well buildup gcase. Note
that the variations in (pc ) are much larger in
Next, we will. examine the applicability of the low pressure range (saygbelow 2000 psi) than
uid system type curves for analyzing pressure those in the high pressure range (say above 3000
pwdown and pressure buildup data obtained from psi). This indicates that during the pressure
real gas wells. buildup, changes in the value of the (PC 9 ) product
"REr11, GAS PSI{UDO-TTMP." -- A NEW FIINCTTON FOR
4 Mil,

will be inure severe at a lower pressure than at a where p and c are functions of pressure. This
higher pressure. In the absence of the effect of function is referred to as real gas pseudo-time,
producing time, this should explain the large dif-
ference between the drawdown and buildup curves.

The effects of variations of (Nc t ) product as a


t a (p), in this paper.

If time and pressure are assumed to vary


linearly with each other, over small time incre-

function of pressure on drawdown data are well known ments, Eq. 10 can be approximated as
in the petroleum industry. As early as 1962,
Carters proposed a method to find formation flow
capacity based on dimensionless time correlation n
(tj - tj-1) p dp (tl)
wherein viscosity and compressibility were point ta(P)
time functions. Al-Hussainy et a1.4 established j-1 (Pj - pj_1 - N(P)ct(P)
that solutions for the flow of real gases should - j 1
correlate as functions of dimensionless time based j
on initial values of viscosity and compressibility.
Unfortunately, there is only limited work done to Note that in Eq. 11, t- represents flowing time for
investigate the effect of (Nc ) variations on pres- a drawdown test and shAt-in time for a buildup test.
sure buildup data in gas wells. Recently, Scott9 Eq. 11 may be rewritten as
suggested consideration of the variations of (Nct)
as point time functions in pressure buildup analysis
of MHF gas wells. (t. t.
t(p) ^ ^-1 1I(p^) - I(pj-1 (12)
During the course of this study, a new time (p• p- )
j-1 i J-
function has been developed which takes into account
the variations of (pc ) product as a function of
time and pressure. This function appears to provide where an integral
excellent engineering answers when applied to gas
well buildup data. This function will be discussed
next. P
I(P) f dp (13)
N(P)ct(P)
REAL GAS PESUDO-TIME, t ,(p)
Po

Al-Hussainy et a1.4 defined real gas pseudo-


pressurez

P
can be evaluated beforehand using p and c as func-
tions of pressure. In Eq. (13), po is aiow base
pressure and p is the maximum pressure of interest.
The above integral, expressed in graphical or

m(p) - 2 r N(p)z(p) dp (8) tabular form, can be used in conjunction with Eq. 12
to compute real gas pseudo time, ta(p). Since c
3Po rather than c is normally ,iv,iilrihle is a functiong
of pressure, the following relationships may be
utilized.
which takes into account the variations of gas vis-
cosity and z-factor as a function of pressure with
Po as a low base pressure. ct = S9 cg + So co + Sw cw + cf (14)

In this paper an analogous function is defined


as follows: For a gas well, Eq. 14 is usually approximated as

t (15)
ct ;Z^ S9 cg
a(t) dt (9)
t N(t)ct(t)
0 Going back to Eq. 11, it should be noted that
during the time interval, At. = t. - t. 1 and the
pressure change, Op. = p. -jp•-1^ the viscosity- •
where P and c are used to denote viscosity and compressibility prod^ct (act)j3is defined by
system compressibility as a function of time rather
than p and c which are usually expressed as func-
tions of pressure. If "t (t) is redefined as a P-
1 _ 1 J dp (16)
function of pressure, a new function is obtained as
(Nct ^P^fpj-1 Y(P)ct(P)

P rdt

ta(P) =
J
Po
N(PJC t ( p) dp
(10) where, j = 1, 2, ..., n


SE'E 8279 RAM G. AGARWAL S

This definition in Eq. 11 gives Eq. 19 appears very similar to Eq. 2 where time
(t) has been replaced by [(Nct)•ta(p)]. This sug-
gests a correspondence between the real time and the
n At. pseudo-time. This also implies some correspondence
ta(p) (17) between the flowing time for a drawdown test and the
E (N t . shut-in time for a buildup test as well be shown
j=1
next.

Eq. 17 clearly indicates that the units in Correspondence between flowing and shut-in times
t(p) consist not only of time but a combination of
time, viscosity, and compressibility. The subject correspondence is being established
based on certain observations rather than rigorous
COMPUTATION OF REAL GAS PSEUDO-TIME, t_(p) mathematical solutions.

Although the use of the real gas pseudo-time,


Since Eq. 12 for t(p) contains an integral, t(p) is not meant to be restricted, this study
I(p), given by Eq. 13, computation can be performed points out that the dimensionless time (see Eq. 18
using either graphical or tabular data. Simply, it or 19) for gas well buildup data correlates much
can be accomplished by means of a computer or a desk better as a function of ta(p).
calculator. Trapezoidal or Simpson's rule can be
used. Integration can be performed by reading mid- Based on the earlier work of A1-Hussainy et al.4,
point values of (1/pc ) from the table or graph and it appears reasonable to assume that the dimension-
multiplying by Ap. Computations of I(p) utilizing less time for pressure drawdown data should corre-
gas properties in Table 2 are outlined in Table 3. late as a function of the (pc ). product evaluated
Fig. 9 shows a graph of (1/pc ) and 1(p) as a func- at the initial reservoir pressure as shown in Eq. 2.
tion of pressure. Although not shown, it is also
useful to prepare a similar graph for real gas pseu- A comparison between Eq. 2 and Eq. 18 or 19
do-pressure, m(p). Thus I(p) and m(p) curves pre- imply the following correspondence between the
pared for the gas in a specific reservoir can be flowing time, t, and the shut-in time, At:
used as master graphs for future wells in that res-
ervoir. The I(p) curve is used in conjunction with
Eq. 12 to convert real times to corresponding pseu-
t
do-times for the specific application. (20)
(Nc t) i - ta(p)
CERTAIN USEFUL ASPECTS OF ta(p)

or,
Before discussing the application, let us con-
sider certain aspects of real gas pesudo-time,
ta(p): t ^`[(Nct)i ' ta(p)] (21).

Definition of dimensionless time


It should be noted that the shut-in time, At,
If ta(p) is used to express the dimensionless is already included in t(p). The use of the above
a
concept appears to provide a number of practical
time term, tDaxf, then benefits.

1. This allows us to use the same definition of


the dimensionless time (see Eqs. 2 and 19) in
tDaxf - 2.634 x210- 4
k ta(p) (18) type curve analysis for both pressure drawdown
and buildup data.
Oxf
2. In performing type curve analysis of the com-
Since the viscosity-compressibility product is bined pressure drawdown and buildup data from
already included in t a (p), it does not appear in the same well, it is possible to plot drawdown
Eq. (18). To express an analogy between Eq. 18 and data as a function of time (t) and buildup data
Eq. 2, the above equation can be multiplied and as a function of [(Nct)i•ta(p)] using the same
divided by (Nc ). evaluated at the initial reservoir graph or tracing paper. This concept, along
pressure. This provides with the superposition principle, was utilized
in a companion paper by Bostic et al.lo

3. This also permits us to compare drawdown and


tDax f = 2.634 x 10 k[(pct)i , t a(p)] (19) buildup data from the same gas well, as shown
0(pct)ixf2 4
in Fig. 10. This will be discussed next
showing the application of ta(p).

It should be noted that in Eqs. 18 an^ 19, a APPLICATION OF REAL GAS PSEUDO-TIME, ta(p)
0 eneral definition of dimensionless time, Daxf has
n used. Accordingly, it may represent dimension-
s drawdown time or dimensionless buildup time To illustrate the application of real gas pseu-
epending on whether t(p) in Eq. 12 has been calcu- do-time, let us consider the pressure drawdown and
a
lated using the flowing time, t, or shut-in time, buildup data, shown in Fig. 7, for the gas well
At.
"REAL GAS PSEUDO-TIME" -- A NEW FUNCTION FOR
PRESSURE BUILDUP ANALYSTS OF MHF GAS WELLS SPE 8279

case. These data have been replotted in Fig. 10 and type curve, provided a computed fracture length
are shown as A[m(p)]/q as a function of time (t or which is smaller than the actual. A third data
At) in days. In this figure, solid line with cir- curve is also shown which utilized Eq. 18 to incor-
cles represent the drawdown data whereas the solid
line with triangles is for the buildup data. As
mentioned earlier, there is a considerable differ-
ence between the two curves. These buildup data,
porate the concept of real gas pseudo-time function
Results of curve matching, using the new time func-
tion, gave results which are close to the actual
fracture length.

being so much on the right side of the drawdown
curve, imply that the use of (PC ) at the initial Table 4 provides a comparison of fracture
reservoir pressure will result in computed fracture lengths, computed by type curve analysis, using the
length which is much greater than the actual. This (Nct) product in the dimensionless time term at var-
aspect will be investigated later by means of type ious pressure levels. Results of four sets of simu-
curve analysis. lated gas well buildup data are shown, where both
the value of fracture length and the level of final
To use the concept of real gas pseudo-time, flowing pressure p were varied. Inspection of
Eq. 12 was used in conjunction with Fig. 9 to con- Table 4 reveals tha^-Oresults are affected by the
vert shut-in time, At, to t(p). Eq. 21 was uti- (PC ) product used and the level of the final
lized to express these data ain terms of equivalent flowing pressure at the instant of shut-in. The use
flowing time, t or (PC ).•t (p). It enabled us to of (PC ). provided values of computed fracture
compare drawdown and builhupadata on an equivalent length twLch are too optimistic. The effect is
time basis. Pressure buildup data plotted as a further exaggerated at a lower value of the flowing
function of (PC ).•t (p) are shown as the dotted pressure. The use of the (Nct) product at pAt=0
line with triangles.a Note that the result was to provides a low but reasonable value if pAt is rela-
move the buildup data (shown as triangles) horizon- tively high, otherwise it provides pessimistic
tally from the solid line on the right to the dotted values of fracture length. The use of real gas
line on the left. Also the modified buildup curve pseudo-time provided computed fracture length values
came very close to the drawdown curve. This figure similar to those entered in the simulator. Based on
also indicates that real shut-in times, At, are a number of computer runs, it appears that the con-
equivalent to only about 60% of the equivalent cept of real gas pseudo-time function is useful in
flowing times, t. For example, the real shut-in analyzing post-treatment buildup data from fractured
time of 6 days is equal to only about 3.75 days of (including MHF) gas wells.
the equivalent flowing time. It is possible to plot
shut-in time, At as a function of equivalent flowing Steps Used in Applying Real Gas Pseudo-Time Function
time, (PC ).•t (p). This is shown in Fig. 11. The for Type Curve Matching
solid line represents the gas case and the dotted
line is for liquid case. This figure clearly shows
that shut-in times for liquid case are equal to the
equivalent flowing times, whereas they are much less
for the gas case_ This indicates that a graph
The following step-by-step procedure should be
useful in applying the concept of real gas pseudo-
time to gas well buildup data for type curve
matching purposes.

similar to Fig. 11 should also prove useful in the
design of a pressure buildup test on an M1fF gas Step 1
well. For example, if a pressure buildup test is
required to be run for an equivalent flowing time of Prepare a table of gas properties as shown in
6 days to obtain the desired information, it may be Table 2. Compute real gas pseudo-pressure, m(p) and
necessary to run the test for about 10 days, which integral, I(p) as a function of pressure and plot
is almost twice as long. them on coordinate graph paper.

Let us next consider the effect of (PC t ) pro- Step 2


duct evaluated at different pressure levels on the
type curve analysis of gas well buildup data. The Tabulate pressure buildup data, At vs at.
values of computed fracture lengths will be compared
against the actual total fracture length of 1000 Using the above figure, convert ptit to m(p,t) and
feet used in the simulator. Pressure drawdown and
buildup data presented in the preceding example will compute fl[m(p)] = m(pAt) - m(Ppt=O)'
be utilized for this purpose. Fig. 12 shows a sem-
ilog graph of pressure buildup and drawdown data Step 3
expressed in dimensionless quantities. Dimension-
less pressure, pwD, has een plotted as a function Using the figure for I(p) and Eq. 12 convert At
of dimensionless time, Dxf: For drawdown data, to t(p). It should be noted that t(p) already
(PC ) product in the dimensionless time was evalu- contains the (Nct) product. a
atetd at the initial reservoir pressure. This curve
will be considered as the reference type curve. Step 4

Data curves for buildup data have been plotted Plot A[m(p)] vs t(p) on a tracing paper uti-
using the (Nct) product at the initial reservoir lizing the appropriateatype curve. Type curve
pressure, p., and the final wellbore flowing pres- matching should be done in the usual manner. For an
sure prior ^.o shut-in, pA Since the data curve P41F well, if formation flow capacity is known a
using (PC t). is on the righ?-hand side of the draw-
down type curve, matching will provide computed
fracture length which is greater than the actual.
On the other hand, the data curve using Nc at
priori, the vertical position of the data plot may
be fixed on the y-axis of the type curve. Otherwise
the matching should be done by sliding the tracing
paper parallel to both x and y axes.

pAt=0' being on the left-hand side of the reference
SPE 8279 RAM G. AGARWAL 7

Step 5 pressure ranges and then used for the spe-


cific application.
Once a match is obtained and a match point is
lected, the fracture length is calculated as b) For gas well buildup data, the use of (Nct)
product at the initial reservoir pressure
provides optimistic values for fracture
lengths whereas the (Nc t ) at the final
/ -4 [t (P)IM pressure provides pessimistic
x/ 2 . 634 x1 0 k a (22) values for fracture length. The use of the
f VV t real gas pseudo-time provides satisfactory
[ Daxf],i
values of fracture length. I

In regard to the term in the above equation, the 6. Due to the effect of variations of (Nct)
following should be pointed out: on gas well buildup data, it may be neces-
sary to run a buildup test twice as long as
If the (Nc t ) product in the real gas pseudo- it is normally run. This aspect should be
time, t(p), is based on the system compressibility, considered in the design of pressure
c t = S ac +S c o +S c w, +c f , then ^ should be the buildup tests on MHF gas wells.
total for g si g y in ^^ he system.
7. Although the discussion in this paper is
If gas compressibility, c, has been used limited to pressure buildup analysis of
instead of c, then ^y should begreplaced by hydro- vertically fractured gas wells, the utility
carbon porosity, Os of the real gas pseudo-time is not meant to
9- be restricted to such wells only. For
Once the value of x is determined, the frac- example, this function was also found very
ture flow capacity can be determined by Eq. 3 as useful for gas wells in analyzing wellbore
storage data, linear flow data, etc., to
name a few.
(kfw) = (FCD)(kx f ) md-ft (23)
NOMENCLATURE

CONCLUDING REMARKS cf = formation compressibility, psi-1

As a result of this study, the following (Pa- I)


remarks appear warranted:
cg = gas compressibility, psi -1 (Pa -1 )
^ A new time function [real gas pseudo-time,
t(p)] has been developed which has aided in c = oi l compress ibilit y, ps i-1(P a -1
post-treatment pressure buildup analysis of o
fractured (including MHF) gas wells. ct = system compressibility, psi - 1 (Pa -1 )

2. This function is analogous to the real gas = = system compressibility as a func-


pseudo-pressure, m(p) of A1-Hussainy el al."
Although it is not a rigorous function, it pro- tion of time, psi - 1 (Pa -1
vides excellent engineering answers for verti-
-1 -1
cally fractured gas well buildup analysis. c = water compressibility, psi (Pa
w

3. There is a basic difference between the draw- F = dimensionless fracture flow


down type curve and the buildup type curve CD capacity (see Eq. 3)
because of the different ways Z^[m(p)] or Ap are
calculated for pressure drawdown and buildup. h = formation thickness, ft (m)
However, drawdown type curves may be used for
buildup data provided the producing time prior I(p) = integral in Eq. 13, psi 2 /cp (Pa 2 /s)
to shut-in is long (that is (t p + pt)/3t^_•1),
k = formation permeability, md
4. For small producing time prior to shut-in,
2
buildup data should not be analyzed by drawdown (10 -3 Nm
type curves.
kf = fracture permeability, md (10 -3 Nm 2
5. To use liquid system type curves for gas wells,
the following points should be noted: m(p) = real gas pseudo-pressure, psi 2 /cp
(MPa 2 /pa s) (see Eq. 8)
a) For analyzing gas well drawdown data, the
use of liquid system type curves appears b[m(p)] = difference in real gas pseudo-
reasonable provided that the real gas pseu-
do-pressure is used in the dimensionless pressures psi 2 /cp (MPa 2 /Pa•s)
pressure term and the (lie ) product in the
^ dimensionless time is evaluated at the ini- p = pressure, psi (MPa)
tial reservoir pressure. However in cer-
tain cases, because of unusual gas = initial pressure, psi (MPa)
pi
properties or reservoir pressure, type
curves should be generated using the appro- pnt = shut-in pressure, psi (MPa)
priate (^c t. ) function, temperature, and
"REAL GAS PSEUDO-TIME" -- A NEW FUNCTION FOR
8 PRESSURE BUILDUP ANALYSIS OF MHF GAS WELLS SPE 8279

PAt=O = shut-in pressure at the instant of (Nct) = viscosity-compressibility product,


shut-in, psi (MPa) cp/psi(Pa•s/Pa)

pDs

PwD
= dimensionless shut-in pressure
rise (see Ref . 7)

= dimensionless pressure or pressure


(Nct)i = viscosity-compressibility product
at initial reservoir pressure,
cp/psi(Pa•s/Pa)

drop (see Eq. 1) (Nc t ). = viscosity-compressibility product
^ in a given interval,
Pwf = wellbore flowing pressure, psi cp/psi(Pa-s/Pa) (see Eq. 16)
(MPa)
p = delta or difference
Op = pressure change, psi (MPa)
formation porosity, fraction
Ap. = pressure change, p. - p._1, psi
^ (MPa) (see Eq. l1) ^ sigma or summation

A(p2) = diff2erencg in squares of pressures, Subscripts


psi (MPa )
cD = dimensionless flow capacity
q = flow rate, MCF/D ("standard" in3/d)
D = dimensionless
S = saturation, fraction
Dxf = dimensionless based on X. and time
S = gas saturation, fraction
9 = dimensionless based on xf and pseu-
Daxf
So = oil saturation, fraction do-time

Sw = water saturation, fraction a = apparent or pseudo

t = flowing time, hours e = external boundary

ta(p) = real gas pseudo-time, hours-psi/cp f = fracture or formation


(hours-Pa/Pa•s) (see Eq. 11)
g = gas
[ta(p)]M

^ a (t)
= real gas pseudo-time for match
point, hours-psi/cp (hours-Pa/Pa•s)

= a function defined by Eq. 9


i

j
= initial

= index for summation



tDx f = dimensionless time based on x f (see M = match point values
Eq. 2)
n = index for summation
Atdxf = dimensionless shut-in time (see
Ref. 7) o = oil

tDaxf = dimensionless time based on ta(p) p = producing


(see Eqs. 18 and 19)
t t = total
[ Daxf]M = dimensionless time based on ta(p)
for match point (see Eq. 22) w = water

tp = producing time prior to buildup, wf = wellbore flowing


hours

at = shut-in time, hours


1. Earlougher, R. C., Jr.: Advances in Well
At = time interval, t j - t j-1 , hours Test Analysis, Monograph Series, Society
i of Petroleum Engineers of AIME, Dallas
T = reservoir temperature, °R (K) (1977) 5, 264.

xe = distance from well to the reservoir 2. Agarwal, R. G., Carter, R. D., and Pol-
boundary, ft (m) lock C. B.: "Evaluation and Performance
Prediction of Low-Permeability Gas Wells
xf = fracture half length, ft (m) Stimulated by.Massive Hydraulic Frac-
turing," J. Pet. Tech. (March, 1979)
z = real gas deviation factor 362-372.

N
= viscosity, cp (Pa•s)

= viscosity as a function o f time,


3. Cinco-L., H., Samaniego-V., F., and Domin-
guez-A., N.: "Transient Pressure Behavior
for a Well With a Finite Conductivity

cp (Pa•s) Vertical Fracture," Soc. Pet. Eng. J.
(August, 1978) 253-264.
SPE 8279 RAN G. AGARWAI, 9

4. Al-Hussainy, R., Ramey, If. J. Jr., and Carter, R. D.: "Solutions of Unsteady-
Crawford, P. B.: "The Flow of Real Gases State Radial Gas Flow," J. Pet. Tech.
Through Porous Media," J. Pet. Tech. (May, (May, 1962) 559-554; Trans., AIME, 225.
1966) 624-636; Trans., AIME, 237
Scott, J. 0.: "Application of a New
5. A1-Hussainy, R. and Ramey, H. J. Jr.: Method for Determining Flow Characteris-
"Application of Real Gas Flow Theory to tics of Fractured Gas Wells in Tight
Well Testing and Deliverability Fore- Sands," paper presented at 1979 SPE Sympo-
casting," J. Pet. Tech. (May, 1966) sium on Low Permeability Gas Reservoirs,
637-642; Trans., AIME, 237. Denver, Colo., May 20-22, 1979.

6. Gringarten, A. C., Ramey, If. J. Jr., and Bostic, J. N., Agarwal, R. G., and
Raghavan, R.: "Unsteady-State Pressure Carter, R. D.: "Combined Analysis of
Distribution Created by a Well With a Post-Fracturing Performance and Pressure
Single Infinite-Conductivity Vertical Buildup Data for Evaluating an MU Gas
Fracture," Soc. Pet. Eng. J. (August 1974) Well," paper SPE 8280 presented at SPE
347-360; Trans., AIME, 257 . 54th Annual Fall Meeting, Las Vegas, Nev
Sept. 23-26, 1979.
7. Raghavan, R.: "The Effect of Producing
Time on Type Curve Analysis," submitted to
SPE of AIME.

TABLE 1

Reservoir and Fracture Data for Simulated MHF Well

Reservoir Data

• Reservoir pressure, pi
Reservoir temperature, T
5000
720
psi (34.5 MPa ►
OR (400°K)
Formation thickness, h 50 ft (15.2m)
Formation permeability, k .01 md (9.9 x 10^um2
Formation porosity, 0 .07 fraction
Initial gas saturation, Sg .50 fraction 3
Production Rate, q 500 Mcf/D (14,158m / D)
Fracture Data
Total fracture length, 2xf 1000 ft (305m)
Fracture flow capacity, k.fw 50 md-ft (15 x 103 4m2m)
Dimensionless fracture
capacity, FCD 10

0
TA B LE 2

Gas Properties for Simulated MHF Gas Well

Pressure Viscos ity z-factor cg m(p)


(psi) (c P) (fraction) ( psi-1) (psi2lcp)

0 -- 1.000 -- --
600 .0147 .971 170. 6 x 10-5 2.5 x 107
1200 .0155 .951 86.0 x 10 -5 10. 0 x 107
1800 .0166 . 940 56. 0 x 10 -5 21. 8 x 107
2400 -. 0180 .939 40. 9 x 10-5 37.3 x 107
3000 .0197 .947 31.1 x 10 -5 55. 6 x 107
3600 .0216 .%4 24. 3 x 10-5 75.6 x 107
4200 .0236 . 986 19. 5 x 10-5 96, 9 x 107
4800 .0255 1.014 16. 0 x 10-5 118. 9 x 107
5400 .0275 1.045 13. 3 x 10-5 141. 3 x 107

TABLE 3

t Using G as Properties
^^p,
Com putation of 1^ 1 and I ( p °o ^ u^D
t
Mean Mean
Pressure 14ct1` (l/µcti 11luct) 6D 11luctI Xpp I(pI• '
Ipsil Icplpsil Ipsi'cpl Ipsi!cp^ (psi) (psi21cpl (psi2/c p)

0 - - - 0.00
600 12.54 x 10 7.98 x 10 3.99 x 10 600 2.39 x 10 2. 39 x 10
1200 6.67 x 10'6 15.00 x 104 11.49 x 104 600 6.89 x 107 9.28 x 107
18DO 4.70 x 10'6 21.29 x 104 18.15 x 104 600 10.89 x 107 20.17 x 107
2400 3.68 x 10-6 27.17 x 104 24,23 x 104 600 14,54 x 107 34.71 x 107
3000 3.06 x 10-6 32-64 x 104 29.91 x 104 600 17.95 x 107 52.66 x D 7
3600 2.62 x 10^ 38.14 x 104 35.37 x 104 600 21.22 x 107 73.88 x 107
42DO 2.30 x 10-6 43.46 x 104 40,78 x 104 600 24.47 x 107 9& 35 x 107
4800 2.04 x 10-6 49.02 x 104 46.24 x 104 600 27.74 x 107 126.09 x 107
5400 1.83 x 10-6 54.68 x 104 51.85 x 104 600 31.11 x 107 157. 20 x 107

- (uctl S9lµcgi
" Ilpl vs. pressure is shown in Fig, 9
Remarks: Real gas pseudo pressure, ta1p, can be computed using E. 12 and
I(p) in conjuncti on with the de sired pressure vs. tim e data.

0
JAB-, -'

Con parisoo of Con%ted Fracture Len:t^s'ror Tvoe Cur^e Anal-Os


of Gas 1'eli BuilJ.ip Data Usin; 1 Prx.ut at Vario.,s Pressure LP'.e's

Total Fract.;re tPnct~ if;


Initial Final Floxinc
Run Flow Rate Pressure Pressure (uc 1• (u '^
!vo. btiiCF'D tpsi^ ipsi, Sinulatar t D, 1. 0 a ari l zi,s i s
LA 500 5000 Z374 1000 1370 95; 102`
19 650 5000 1334 1000 1500 862 995
2a 500 5000 2245 15^.'t3 2020 143C 1515
23 650 5000 1163 15•71- 2466 1295 15 1,
10
102 0 kfw
3
6 FCD ^ kxf
.,1
W 1
N

Ce
co tl.

^1D
O
t/1

Z
2xe 10
10
10-2 10-1 1 10 102 if
DIMENSIONLESS TIME, tDx
10
f
Fig. 1- Constant rate drawdown type curves for infinite flow DIMENSIONLESS TIME, tDx
f
capacity vertical fracture - liquid system (after Gringarten
Et Al.') Fig. 2 - Constant rate drawdown type curves for finite flow
capacity vertical fractures - liquid system (after Agarwal
Et A1.2 )

N
rCe
pi CONSTANT RATE DRAWDOWN BUILDUP
V,
c7t

N •
PRESSURE
z
rpOf•o- ptp+^t^
0 %fi
V1
2

t ► -d

DIMENSIONLESS SHUT-IN TIME, -'St Dx TIME


f
Fig. 3 - Effect of producing time an buildup data for infinite Fig. 4 - Schematic of pressure behavior during constant rate
flow capacity vertical fracture - liquid system (after drawdown and buildup periods.
Raghavan7 ).
6.0

5.0 1

.? LIQUID
4.0 DRAWDOWN DIMENSIONLESS
tLL
^ U FRAC. CAPACITY
GAS
FCD
BUI 10
10-1
3.0

r-x

2.0 I pm ty
mtat -m I011
TIME

t Draraoan i
g
mlp,ti -m At BuiMup 102
1.0 ', l'
L,1.0) -In 61 Dillmnl,
1D 1D 10 10
DIMENSIONLESS TIME, fDxf
OIFFERENCEti
0.0
Fig. 6 - Comparison between drawdown type curves for liquid and
gas systems (constant well rate).
-1.0

TIME (DAYS)

^ Fg, 5 - Pressure drawdown and buildup data vs time (liquid


system - ( v ct ) is constant).
6.0

28 ; ,
DRAWDOVJN
5 .0
24

4.0

• d
BUILDUP/
20
q
n u 30 6
^. /
16
,r pm ^p TIh1E
• m Ip^l t Dr^wOaen
it 1
2.0
pt^ -1° (p pt4!
^t Buildup 12
F
u
'^1 cinererKe
I 1 Ot^^-m ^pt D • ^V
4
1.0 8

DIFFERENCE 4
0.0

-1.0 ,
01 1000
1
2 00() 3(Xl0 4000 5000 6000
0 1 2 3 4 5 6 7 PRESSURE, fpsil
TIME (DAYS) Fig. 8 - Variations of gas viscosity-compressibility (Pct)
Fig, 7 - Pressure drawdown and buildup data vs time (gas system -
( . t t ) is function of pressure). product as a function of pressure.

o, u

DRAW DOWN
5,0 -
P

4, 0 -
a
ILDUP
^

•i
6 ^
O `

3.0 -
C.
9^u x pmlpI TIME
a 1
mlpii - m IV t Dravdown
I u
n ^Q 2. 0 tnID011 mkplp\ Buildup
"t
m(p6i) •ml^p^ fpcl),tilpi Buildup w11,0
pt•DI)

l.o At Difference
mlp Ot 0^ m^qp• ^t}

Fig. 9 u) and I(p) =f p dp 'as a function of pressure.


t 0 .,ct
1
12 3
TIMEIDAYSI
Fig. 10 - Application of real gas pseudo-time to gas well buildup
10 data.
0.3
>
08
^
^
LA
4/1 W 0.2
6
^
Ln
= 4
^ Ln 0.1
WC
2

0
0 2 4 6 8 10 12
EQUIVALENT FLOWING TIME, t or (µc t)1 ta(p) (DAYS) DIMENSIONLESS TIME
.• i
Fig. 12 - Drawdown and buildup data plotted using (0 ct) at different
Fig. 11 - Shut-in time vs equivalent flowing time. pressure levels (gas case-(U ct) is a FN. of pressure).

Application of Pseudotime to Buildup Test


Analysis of Low-Penmeabiliry Gas Wells With
Long-Duration Wellbore Storage Distortion
W. John Lee, SPE, Texas A&M U.
Stephen A. Holditch, SPE. Texas A&M U.

Summary
This paper presents an improved method of analyzing When there has been a drawdown from, say, 5,000 to SO
pressure transient tests in low-permeability gas wells us- psia in a flow period, the wellbore storage constant dur-
ing the recently proposed pseudotime concept. The ing a subsequent buildup test will change by two orders
method is particularly helpful in providing less am- of magnitude during the test. (Large drawdowns of this
biguous type-curve analysis in gas wells with large magnitude are not unusual in closed-chamber drillstem
^ pressure drawdowns. tests and in other tests in tight gas reservoirs.)
2. Large drawdowns also cause inaccuracies remain-
Introduction ing in the partially linearized flow equation to grow,
The purpose of this paper is to present an improved even in situations in which wellbore storage distortion
method of analyzing pressure transient tests in gas wells. ceases at early times. These inaccuracies can cause er-
A fundamental problem in analyzing these tests has been rors of more than 100% in type-curve-derived fracture
that analysis techniques have been based on methods length estimates, for example.
derived from solutions to the diffusivity equation In a recent paper4 Agarwal proposed an intuitive
describing flow of slightly compressible liquids. The liq- pseudotime, tQ, defined as
uid equation is linear, but the corresponding equation
describing gas flow is not linear. This fundamental dif- ta _ Jrt dt . . . . . . . . . . . . . . . . . . . . . . . . . . . . . . . (2)
ference has continued to cause problems and t o Act
misinterpretation.
One important step in adapting solutions of the liquid- Agarwal found empirically that buildup tests in tight gas
flow problem to gas flow was introduction of the reservoirs yielded much more accurate formation and
pseudopressure concept.' Replacement of pressure by fracture properties when to replaced t as a plotting func-
this variable almost (but not completely) linearizes the tion in type-curve analysis. No theoretical basis for this
gas flow equation. At least two problems remain: conclusion was provided, and the problem of changing
1. In tests with wellbore storage distortion of early wellbore storage constant was not presented.
test data, no published type curves2,3 provide much This paper presents a theoretical analysis showing
assistance in test analysis because all these curves are that, under certain conditions, pseudotime effectively
based on solutions to the liquid-flow problem. More linearizes the flow equation for gas. It also shows that a
specifically, the liquid-flow solution characterizes plot of pseudopressure changes vs. elapsed pseudotime
wellbore storage distortion with a wellbore constant, C, should fit a type curve developed for a slightly com-
which is constant throughout the test. In a gas well, the pressible liquid, with unchanging wellbore storage con-
wellbore storage constant, C, is the product of wellbore stant in some cases. These theoretical conclusions are
volume, V,yb, and compressibility of gas in the wellbore, supported by example tests generated on a computer
reservoir simulator and by field examples. The paper
cwb
also presents an example calculation showing how the
0 C-VwbCwb • ............................ •(1)
proposed techniques are applied.

Results
O1 49-2136l8210012-9888$00.25
Copyright 1982 Society of Petroleum Engineers of AIME The Appendix shows that solutions of the diffusivity
DECEMBER 1982 2877
TABLE 1-EQUIVALENT VARIABLES IN GAS AND SLIGHTLY COMPRESSIBLE LIQUID FLOW

Liquid Gas'
^-^

Pressure,p Pseudopressure, 0(p) = 2Ip pdp


AZ
PD
pc = kh (p; -pr)
Dimensionless pressure, Dimensionless pseudopressure, tGp =
141.2q8µ
kh Tsc
50, 300qp ^ T
dt
Time, t Pseudotime, ta(p)= I -
Act
t0
0.0002637kt
Dimensionless time, to = 2 Dimensionless pseudotime,
oµc,rw 0.0002637kta
t,p = orw z

Radius, r Radius, r
Dimensionless radius, ro =r/r, Dimensionless radius, r p =r/r,
Skin factor, s Apparent skin factor, s' = s+ Dlql
Wellbore storage constant, C= VwbCwb Modified wellbore storage constant =
`i(TC I)J(T wbC wb) a V wb TC rlTwb

Dimensionless wellbore storage constant, Modified dimensionless wellbore storage


0.894Vwb T
Co =0.894Vwbcwb/rbcthrw 2 constant, CN =
4shrw2Twb
In most applications, m can be replaced by mo and c, by co.

equation describing flow of a slightly compressible liq- known as "Ramey's type curve"). Another is a
uid can be used as semirigorous models of gas flow. The republication of the same data plotted in a different way
variables and parameters summarized in Table 1 have by Gringarten et al. 3 Ramey's type curve can be con-
equivalent roles in the liquid and gas flow problems. sidered a plot of pD (at rD = 1) vs. tD for fixed values of
The implication of these equivalency relationships is
this: A value of PD can be found at fixed values of tD,
rD, s, and CD for a slightly compressible liquid. For
CD and s; it also can be considered a plot of 1P D vs. taD
for fixed values of Coe and s'.
The Appendix makes clear the theoretical equivalence

taD=tD, s'=s, and CDe=Co, the value of fora gas between liquid and gas solutions. However, because the
flow problem will be identical to the value of PD from theory is not rigorous, it is also important to establish
the liquid flow problem. Values of pD as functions of the that the theory can be verified with synthetic (computer-
other variables and parameters are published in generated) pressure transient tests and that actual tests
numerous places; one convenient and familiar place is appear to be consistent with this theory. Accordingly, we
the type curve published by Agarwal et al. 2 (commonly calculated pf vs. t with a gas well simulator, reduced it

IoE

e• •
e o• A
e o •

10 •• S=0
Co = 0 •• A a
LEGEND
LIOUID SOLUTIONS
PD Symbol
GAS SOLUTIONS
0 ^ Symbol Test
1.0 0 Buildup 0
Go Go • 102
e •• " 103
106

• Druwdam 0
V^ • " 102
• "" 103
I • 106
► 1
to
102 lo3 In4 Inb Ing in7 IC

t0

Fig. 1-Comparison of dimensionless pressures for slightly compressible liquids and for gas.

2878 JOURNAL OF PETROLEUM TECHNOLOGY


to the form ^D vs. taD, and compared it with PD vs. tD Drawdown test data for gas wells match the type
from the Agarwal et al. solutions for a slightly com- curves for each value of s and DD, quite well except
pressible liquid. We also analyzed, qualitatively, (1) a when CD =0, s=20, and ID < 104 The reason for
pressure buildup test from a closed-chamber drillstem discrepancy here is that the type-curve solution was
q test that had changing wellbore constant and that shows
the difference in interpretation when one attempts an
developed for concentrated wellbore damage (zero-
thickness skin), whereas the finite-difference solution
analysis of >G vs. t and then 0 vs. t,,, and (2) a longer used for the gas well had reduced permeability to a
duration pressure buildup test that also was affected by radius of approximately 6 ft in the formation. With no
changing wellbore storage constant and in which the wellbore storage dominating early-time data for the
qualitative difference in interpretation resulting from use CD=0 case, the early-time data simply reflect the lower
of actual time and pseudotime is illustrated. These permeability near the wellbore and do not indicate a flaw
checks of theoretical expectations are discussed in the in the theory.
following paragraphs. Drawdown data for gas wells fitting the theoretical liq-
uid curves so well is excellent verification of the theory.
Comparison of Flow Equation Solutions for Gases Buildup test data match the type curves as well as
and Slightly Compressible Liquids drawdown test data. This result was obtained because
Fig. I is a comparison of flow equation solutions Agarwal's5 "effective time,"
(dimensionless pressures) for slightly compressible liq- At
uids and for gases. The solid lines are the Agarwal et al.
solutions for pressure drawdown of liquid wells with
^te= (l+AtltP)
wellbore storage constants, CD, varying from 0 to 105 was used in constructing the abscissa instead of using
and skin factors, s, varying from 0 to +20. The triangles shut-in time itself. Effective time, of course, was con-
are drawdown data from gas well simulations, and the verted to a pseudotime basis.
circles are buildup data for gas wells. Our conclusion is that the use of pseudotime and
Gas-well drawdown and buildup test data were pseudopressure result in gas-well test data that are
simulated for effective wellbore storage constants, CDe, modeled adequately by type curves constructed for
of 0, 102, 103, and 105; at each value of CDe, tests slightly compressible liquids. This is true for both
were simulated with values of skin factor, s, of 0 and 20. drawdown and buildup tests. There is one important
In this way, the entire range of data reported for qualification, however: Buildup tests must follow pro-
unstimulated liquid wells was studied for gases. (Data duction periods such that there are considerable
for negative skin factors reported by Agarwal et al. were drawdown data beyond the end of wellbore storage
generated using fractured well solutions. We will ad- distortion (as discussed by Agarwal5 and verified in this
dress application of pseudotime to fractured gas wells in study). Failure of type-curve matching when this condi-
a future study. For given values of s and CD, the liquid tion is not satisfied is not a failure of the use of
solutions are graphs ofpD vs. tD; the corresponding gas pseudotime and pseudopressure for gas wells; analysis of
solutions for the same values of s' and CD, are graphs of tests in oil wells fails under this condition for the same
OD Vs. tuD. reason.

SHUT•IN TIME, MINUTES


10 CO
1091

Op^
^

ACTUAL TIME /PSEUDO -TIME GRAPH

N I I

a3
^

a^
3
10

• 108
102 IOs
I64
tn6

PSEUDO -TIME, HR • PSIA/CP


ins

Fig. 2-Type-curve graph for closed-chamber drillstem test.

DECEMBER 1982 2879


ACTUAL TIME, MINUTES
in 10 IDO 1000 10000


10

eeepeAEmem °° pOOO° °uo°

0
0

N I
109 ACTUAL TIME GRAPH °o

O.i ^

3
e NIT
top SLOPE
NE

/ /:EUDCTME GRAPH

107
10V
10' 10° 10° 101 10°
PSEUDO - TIME, HR - PSIA/CP

Fig. 3-Type-curve graph for long-term buildup test.

Application to Actual Tests The a0 vs. Ate graph appears to lie totally on a unit-
Use of pseudotime instead of actual time can affect the slope line-i.e., a line characterized by an afterflow rate
qualitative interpretation of a buildup test significantly, equal to rate before shut-in. Such a test cannot be used to
as in type-curve matching. Two examples illustrating estimate formation properties. The graph of O>y vs. At,,,
this point follow.
Fig. 2 is a log-log graph of the buildup period for an
actual closed-chamber drillstem test. Plots of
G+]/=>y( pWS)- >y(p„f) vs. both Ate and Otae are
shows not only that the O>G vs. Ote graph is incorrect,
but that the test is, in fact, interpretable. Data at late
times actually deviate sufficiently from the unit slope
line to provide a possibility of a type-curve match with
0
shown. As is characteristic of many drillstem tests in the Gringarten3 type curve (Fig. 6). We did not attempt
low-productivity gas wells, all the data are distorted by a complete analysis, but note that all the test data are fit-
wellbore storage (as comparison with either curve to a ted reasonably well by a type curve characterizing ex-
conventional type curve shows); accordingly, Homer tremely severe damage. It is possible to find a match
analysis is not possible, and type-curve analysis provides point, and thus to estimate formation permeability and
the only chance of estimating formation properties. skin factor.

•---4--b--•-+-5^

MATCH POINT
109 Wp=13.8,GY'=109 +
oI
¢E^ 2CURVE FOR ► I
"01) =5.4, At^ 10^ ^I^I^E
Coe C" e as. 10
^

3
10°

10^
I04 q6
UNIT SLOPE LINE

106 107 10 e Ic

Ataa , HR - PSIA/CP

Fig. 4-Simulated pressure buildup test (AO vs. tstae)

2880 JOURNAL OF PETROLEUM TECHNOLOGY


Fig. 3 is a log-log graph of a 64-hour pressure buildup TABLE 2-DATA FROM SIMULATED PRESSURE BUILDUP TEST
test in a low-productivity gas well. The graph of
Well, Rock, and Fluid
pseudopressure vs. actual shut-in time has an extended

• period (200 to 800 minutes) with a slope greater than the


supposedly theoretical maximum unit slope, suggesting
that changing wellbore storage constant is influencing
the test data significantly. Further, the test data fit no
k=0.1md
s=20
0g = 0.075
rw = 0.25 It
Vw=131.1bbl
h=25ft
T=202°F
tP = 1,200 hours
at^ = 1,200 hours
Twp=202°F
single Gringarten type curve over a wide time span, even p; = 6500 psia yy = 0.65
pw, = 707 psia µ; = 0.0282 cp
if we approximate all data up to 800 minutes' shut-in q = 190 Mcf/D Darcy flow
time as lying on a unit slope line. Accordingly, type- cg, = 8.28 x 10 -5 psia -' ^(p^,) = 3.994 x 10' psiaz/cp
curve analysis of these data would be difficult or tpe = 5.13 x 108 hr-psia/cp
impossible.
Buildup Data
The same test data graphed as A>y vs. pseudotime have
At pwa 0(pws)-0(pwl) Atae(pws) At.
a conventional form; the slope never exceeds unity; and, (hours) (psia) (psiaz/cp) (hr-psia/cp) (hours)
importantly, the later data are fit well by a type curve for 0 707 0 0 0
a moderately damaged well. Match points can be found; 0.0720 720 1.46x 106 3.64 x 103 0.072
accordingly, it is possible to estimate formation 0.288 759 5.89 x 10 6 1.49 x 10 4 0.288
permeability and skin factor for this well. 0.936 872 2.03 x 10 7 5.16 x 10 4 0.935
2.23 1,088 5.31 x 107 1.38x 105 2.23
In summary, these two examples illustrate that use of 3.58 1,304 9.27x10' 2.44x105 3.57
pseudotime instead of time in actual test analysis can 4.97 1,521 1.39 x 108 3.70 x 105 4.95
make an otherwise uninterpretable test interpretable, and 6.41 1,739 1.91 x 108 5.18 x 105 6.38
7.92 1,957 2.49X108 6.90x105 7.87
it certainly can change the interpretation of other tests. 9.46 2,176 3.12x108 8.84x105 9.39
This will be particularly important in many closed- 11.0 2,395 3.80x10$ 1.10x106 10.9
chamber drillstem tests on low-productivity gas wells 12.7 2,615 4.52x 108 1.35 x 106 12.6
14.4 2,834 5.27 x 108 1.62 x 106 14.2
because there appears to be a high probability that these 16.1 3,054 6.05 x 10 8 1.92 x 10 8 15.9
tests will be distorted over much of the time range of 17.8 3,272 5.86x108 2.24x106 17.5
wellbore storage with changing wellbore storage 19.6 3,491 7.68x 108 2.60x 106 19.3
21.5 3,707 8.51 x 108 3.00 x 106 21.1
constant. 23.4 3,922 9.35 x 108 3.43 x 106 23.0
25.4 4,136 1.02x 109 3.91 x 106 24.9
Example Test Analysis 27.6 4,346 1.10x109 4.47x106 27.0
29.9 4,556 1.19 x 109 5.09 x 108 29.2
A simulated pressure buildup test (with properties given 32.3 4,760 1.27 x 10 9 5.76x 106 31.5
^ in Table 2 input into the simulator) now is analyzed us- 35.0 4,961 1.35 x 109 6.56 x 106 34.0
ing the Gringarten type curve to illustrate application of 38.0 5,158 1.43x109 7.48x108 36.8
41.4 5,348 1.51 x 109 8.57x 108 40.0
the principles developed and discussed in this paper. 45.6 5,530 1.58 x 109 9.96 x 108 43.9
Pressures and shut-in times from the simulated test also 50.6 5,702 1.66x109 1.17x107 48.6
are given in Table 2. The following steps were taken in 57.2 5,861 1.72 x 109 1.40 x 10' 54.6
66.6 6,001 1.78 x 109 1.74 x 107 63.1
the type-curve analysis. 81.6 6,118 1.83x 109 2.28x107 76.4
1. Pseudopressure, >Ji, at each shut-in bottomhole 110 6,210 1.86x109 3.30x107 101
pressure (BI-1P), pw,., was calculated by numerical in- 181 6,283 1.89 x 109 5.71 x 10' 157
301 6,334 1.92 x 109 9.30 x 107 241
tegration of the defining equation: 421 6,383 1.93x109 1.24x100 312
541 6,383 1.94 x 109 1.51 x 108 373
661 6,397 1.94 x 10 9 1.74 x 10 B 426
Pwspdp 781 6,408 1.95 x 109 1.95 x 108 473
2 901 6,417 1.95x 109 2.13x 108 515
0 µZ 1,021 6,424 1.95 x 10 9 551
2.29x108
1,141 6,429 1.95x109 2.44 x 10 8 585
1,200 6,432 1.96x 109 2.56 x 108 600
where base pressure, po, has been chosen to be zero.
2. For type-curve plotting, A>y=,^( pWS}-,^( pWf)
was calculated; ^( p4) is pseudopressure at the instant We have obtained best results by approximating tPa as
of shut-in. tP/µ;ct;, but more thorough investigation is required on
3. Pseudotime during shut-in, Ata, was evaluated for this point.
each shut-in BHP from the relationship 5. a,y vs. Otae was graphed on log-log paper (Fig. 4)
and match points were found on the best fitting curve
r=41 dt from the Gringarten type curve (Fig. 6). The match
Dta= 1 - points are
t=0 Act
>GD =13.8 at 0>G=109
We prefer the direct numerical integration of the defining
equation for Ata; Agarwal4 suggests an alternative ap- and
proach that requires an assumption that dp/dt remain
^ constant over small intervals of time. t,DICD, =5.4 at At',= 106.
4. Pseudotime is converted to an effective time basis,
Atae, by the equation The parameter for the matching type curve is
Cpe2s'=1020.
At,, =Ata/(1 +Ata/tpa). 6. A unit-slope line appears on this graph; such a line

DECEMBER 1982 2881


1010

MATCH POINT
Wp=10,6W=5.Ox le
tp
= IO,Ate=49
GRINGARTEN TYPE CURVE FOR
Cpe2S = 10 30

NI G
^
V
109 CD ^,.

^
3 l08

UNIT SLOPE LINE

I O^
al LO 10 100 1000 10000
Ate, HR
Fig. 5-Simulated pressure buildup test (AO vs. dta).

has the property that taD/CDe0 D=1 or From data input to the simulator, results should have
been k=0.1 md, s=20, and CDe=1,000. The agree-
13.26qpS,T Ata ment is excellent.
CDF 08hr Z - Q^ )unit slope line
To illustrate the improved accuracy possible with use
w Tsc \ of pseudotime, we now repeat the analysis with the con-

(13.26)(190)(14.65)(662) /2.55? 104


(0.075)(25)(0.25) (520) ` 10 7
ventional A>G vs. Ate log-log graph (Fig. 5).
Two distributing features of this graph become ap-
parent immediately when we attempt a type-curve
match: (1) early data fall on a line with greater than unit

slope (in theory, impossible for a properly modeled
=1,022. reservoir), and (2) no single type curve fits all those data,
and about the best we can do is fit all the late data
7. From the pressure match point, (Ot>40 hours). (Both these features have been observed
on a number of actual tests; their occurrence, in fact, led
50,300qpSc ^ OD \ us to the study described in this paper.)
k=
hTs, /)MP With the curve match chosen, match points are
AO <
(50,300)(190)(14.65)(662) / 13.8 \
0 1) =10 at A^ =5.0x 108
(25)(520) 1v 09 /J
and
=0.0983 md.
ID/CD = 10 at ate = 4.9 hours.
8. As a check, from the time match point (this is a
necessary step when no unit-slope line is present), The parameter, CDe2s, of the matching curve is 1030

_ 0.0002637 k (_&,, 1 2. From the pressure match,


CDe 2 JMP
4) g rw taD/CDe 50,300 9 PScT Od
(0.0002637)(0.0983)(106) k= h TSC ^ MP
_ = 1,024.
(0.075)(0.25)2(5.4)
_ (50,300)(190)(14.65)(662) ^ 10
9. From the matching parameter, CDe 25,
(25)(520) 5.0x 108
CDee2' = 1020

and
=0.143 md. •
3. Since no unit slope line is present, we must
s=19.6. establish dimensionless wcllbore storage constant from

2882 JOURNAL OF PETROLEUM TECHNOLOGY


IOz
APPROXIMATE START Ps
OF SEMI-LOG ^°
STRAIGHT LINE 1030

0
c
^ 1020
tC)'e
L Q domaged well
Y N I
a
10^
0 APPROXIMATE END in 5
OF UNIT SLOPE
OF M° r
W LOG-LOG 5.6
STRAIGHT LINE b^ d W% ry o-2
^O ice 51o-
0•3
iE
0 f lo-s
^ 10p ,P
W 6
J
z
Q
N

e •
[^O

I
1010 1
1& Id I01 l03 IC

DIMENSIONLESS TIME °a =O.OOO295 uh Ct_

Fig. 6-Type curve for a well with wellbore storage and skin.

the time match point. analysis can be misleading or impossible if


pseudopressure and actual time are used.
0.0002637 k ( Ote Nomenclature
CD = 2 \ MP
0gl.LiCgirx (DICD ) Bg = 5.04 Tz/p, gas formation volume factor,
(0.0002637)(0.143) RB/Mscf (res m3/103 std m3)

• (0.075)(0.0282)(8.28 x 10 -1)(0.25) 2
4.9
)=1,688.
c1 = pore space compressibility, psi-' (kPa- I)
cR = gas compressibility, psi-1 (kPa - I)
Ce; = gas compressibility evaluated at p; , psi - t
(kPa-t)
10
C, = csSB+c,SK,+c^, total compressibility,
4. From the matching parameter,
psi - t (kPa - )
CDe 2s = 1030 = 1,688 eZs c,,,,b = total compressibility evaluated at p,,,6,
psi-I (kPa-1)
and c,,, = water compressibility, psi - ^ (kPa - ^ )
c,,,b = gas compressibility evaluated at p,,,b, psi -1
s = 30.8
(kPa-t)
In summary, note that this method of analysis led to C = Cx;bV,,,b, wellbore storage constant, bbl/psi
values for k and s somewhat different from those input (m 3 /kPa)
into the simulator. More importantly, though, the match CD = 0.894 V,,,yc,,,b/0c,hrµ,2, dimensionless
was over a limited time span and was, thus, most uncer-
wellbore storage constant
tain. No match at all would have been possible if only
CDe = 0.894 VH,bT/0hr,,,2T,,,b, modified dimen-
data for at<40 hours had been present. This region of
sionless wellbore storage constant
the curve is frequently all that is present in a drillstem
test run on a low-permeability gas well. D = non-darcy flow constant, D/Mcf
(d/f03 m3)
Conclusions h = net pay thickness, ft (m)
The results of this study lead to the following k = effective formation permeability, md
conclusions. M = molecular weight of gas, lbm/lbm-mol
1. Use of both pseudotime and pseudopressure can (kg/kg, mol)
result in linear equations modeling gas flow in
p = pressure, psia (kPa)
reservoirs.
p; = original formation pressure, psia (kPa)
2. Type curves developed for slightly compressible
liquids with unchanging wellbore storage constants can p,,, = standard condition pressure, psia (kPa)
^ be used for gas well tests following large pressure p„,b = average pressure in wellbore, psia (kPa)
drawdowns if analysis is performed using pseudotime p,,,f = flowing BHP at shut-in, psia (kPa); also
and pseudopressure. This result is helpful in both flowing BHP in drawdown test, psia
qualitative and quantitative analysis, since both types of (kPa)

DECEMBER 1982 2883


p,,,, = shut-in BHP, psia (kPa) 2. Agarwal, R.G., Al-Hussainy, R., and Ramey, H.J. Jr.: "An In-
vestigation of Wellbore Storage and Skin Effect in Unsteady Liq-
po = base pressure, psia (kPa) uid Flow: I. Analytical Treatment," Soc. Pet. Eng. J. (Sept.


q = gas flow rate at surface, Mcf/D (103 m3/d) 1970); Trans., AIME, 249.
qsf = gas flow rate at sandface, 3. Gringarten, A.C. et al.: "A Comparison Between Different Skin
and Wellbore Storage Type-Curves for Early Time Transient
Mcf/D (103 m3/d) Analysis," paper SPE 8205 presented at the 1979 SPE Annual
r = distance from center of well, ft (m) Technical Conference and Exhibition, Las Vegas, Sept. 23-26.
rD = r/r,t„ dimensionless radius 4. Agarwal, R.G.: "'Real Gas Pseudo-Time'-A New Function for
Pressure Buildup Analysis of MHF Gas Wells," paper 8279
rDe = re/r,,,
presented at the 1979 SPE Annual Technical Conference and Ex-
re = drainage radius of well, ft (m) hibition, Las Vegas, Sept. 23-26.
r,y = wellbore radius, ft (m) 5. Agarwal, R.G.: "A New Method to Account for Producing Time
Effects When Drawdown Type Curves are Used to Analyze
R = real gas law constant, psia-ft3/Ibm-mol-°R Pressure Buildup and Other Test Data," paper 9289 presented at
(kPa'm3/kg'mol' °C) the 1980 SPE Annual Technical Conference and Exhibition,
s = skin factor, dimensionless Dallas, Sept. 21-24.
tP = producing time before buildup test, hours APPENDIX
tPa = producing time converted to pseudotime,
hr-psi/cp (h • kPa/Pa • s) Derivation of Flow Equation for Gas Well
to = base time, hours in Terms of Pseudopressure
T = formation temperature, °R and Pseudotime
T. = standard-condition temperature, °R This Appendix presents a derivation of a special form of
Tti,b = average temperature in wellbore, °R the diffusivity equation, complete with initial and boun-
V,ti,b = wellbore volume, bbl (m3) dary conditions, for a gas well. Pseudopressure is used
as the dependent variable and pseudotime as an indepen-
At = shut-in time, hours
dent variable. As a result of using these variables, the
Ota = pseudotime elapsed since shut-in,
diffusivity equation is linearized (in those cases in which
hr-psia/cp (h' kPa/Pa' s) an error term is negligible) and the inner boundary condi-
Atae = effective shut-in time converted to tion, which characterizes wellbore storage, has the same
pseudotime, hr-psia/cp (h • kPa/Pa' s) form as for slightly compressible liquid flow. The im-
At, = effective shut-in time, Atl(1+At/tp), hours plication is that solutions developed for slightly com-
z = gas-law deviation factor evaluated at reser- pressible liquid flow model flow in gas wells quite well
voir conditions
zs, = gas-law deviation factor evaluated at stan-
dard conditions
when the error term is negligible.
For radial flow of gas in a formation with constant and
isotropic permeability and porosity, the mass balance,
with Darcy's law and pseudopressure introduced, is

z,,,b = gas-law deviation factor evaluated at
wellbore conditions 1 a (r a>G \ - Ouct
(A-1)
yg = gas gravity (air= 1.0) r ar ar J 0.0002637 k at
µ = gas viscosity, cp (Pa's) Eq. A-1 may be written more completely as
µ; = gas viscosity evaluated at pi, cp (Pa's)
1 a ( a>G ^µct
µ,,,b = gas viscosity evaluated at wellbore condi-
tions, cp (Pa's) r ar L r\ ar 0.0002637 k
p = density, Ibm/cu ft (kg/m3)
0 = porosity, fraction -t ^^ /)r . ...........................(A-2)
Og = OSg =gas porosity, fraction
Eq. \A-2 is not linear because µ and ct are functions of
pressure and thus of ^. In hope of completing the
linearization, we introduce pseudotime, ta, defined as
ty = 2^pp dp , pseudopressure, psia2/cp
po µz (kPa2/Pa's) rt dt
ta = - . ............................ (A-3)
to Act
kh TSC(yr-^) ,
OD dimensionless Now tG can be expressed either as a function of r and t, or
50,300 q p,, T pseudopressure r and ta. Thus, the total differential of ^ is

>G; = pseudopressure evaluated at p„ psia2/cp dt/i= \ a4 Jrdt+( ^^ )tdr=C at^ /rdta


(kPa/Pa's)
C a,^
>G,,,b = pseudopressure evaluated at p,,.b, psiaZ/cp
+ J t a dr . ..................... (A-4)

(kPa/Pa • s) ar
If we differentiate Eq. A-4 with respect to t, the result is
References
1. Wattenbarger, R.A. and Ramey, H.J. Jr.: "Gas Well Testing a>G a>G dta
With Turbulence, Damage, and Wellbore Storage," J. Pet, Tech. ..............(A-5)
j
(Aug. 1968) 877-87; Trans., AIME, 243. at r at q r dt

2884 JOURNAL OF PETROLEUM TECHNOLOGY


Then, if we differentiate Eq. A-3 with respect to t, the Using Eq. A-14,
result is

1r=r f
^ dta
dt
1
licr . ........................ . . . . (A-6) ^ ar [r^ ar aQ R310^r ( 2ar--

Thus, combining Eqs. A-5 and A-6, +^ ar Ir ( ^a'ar) , + atal ar)'


+r
O ker \ a^ / r k
-)r . . . . . . . . . . . (A-7)
\a^ +^ ar }ra^ ^a [ () r` ar^r
Now, if we differentiate Eq. A-4 with respect to r, the
result is + ( ^,
] }. ..... ..... ...........(A-17)
Expanding, rearranging, and dividing by l/r,
alk dtR +
(A-8)
( -- )
ar atQ dr ^ ar'
Since
r ^ Or [r ^ Or ^r r a^ r ^
( 3241 \
tQ=tQ(µc,, t) and µc,=tic, (p)=µcr(0), ... .. (A-9) 2 ( atQ l 2
+ '
( -ar- '
2 ) ,, + rar lr +2
and since

^=^(r,t) .

then
. . . . . . . . . . . . . . . . . . . . . . . . . . . . (A-10)
C aralp 1'a,'\atrl'+ r (ata)r (ar)r
..................... ......... (A-18)
tQ=ta(r,t) . .... . ..... .. ....... . . . ..... (A- Il)

and Combining Eqs. A-2, A-7, and A-18, the result is

• dta- \ ar /Idr+( at )rdt .


............ (A-12) 192o at ,
Differentiating Eq. A-12 with respect to r, the result is r I
la r Lr\ aIr )ta]}ta+(at?)r (v):

dt1, ata ( at a 8 41. ) at"


....................... (A-13)
dr ar ' +2 `ara0 \+ r ( ar r ` ar /'
Combining Eqs. A-13 and A-8, a
) ................ (A-19)
0.002637 k atp '

C a^ I'=`a00 arlr+( a'r l'u The diffusivity equation will have been linearized if the
second, third, and fourth terms on the left side of Eq.
................... ........... (A-14) A-19 are negligible compared to the first term-i.e., if

As used in Eq. A-14, ,G is a function of either r and t, or \


r and tp. Therefore, we can write
a [ 8r[r1 ar /'Q] ] ^atQ^ J, r

at,
• ^ a ^r^ a^ at, Lr^ ar
a
C atr/`2+2 Caa^ /`a•'\a r/`+ rI (at, /'

\
.....(A-15) a t,
ar
Expanding,
+ [ ar E r \ a^ I' ]}...
( - ),. ...........................(A_20)*
Assuming that Eq. A-20 is satisfied, we may write
{ 8 [ /3 )]] 1 a a,^ to a
- r (A-21)
arr\ ar atQ \ ar r Or ar 0.0002637 k ata
^ atQ Eq. A-21 is linear, and thus, there is hope that solutions
" ),+r[ -
' ( a' )j" to the linear equation describing flow of a slightly com-

'We thank J.E. Chappelear and R.H. Roach of Shell Oil Co., Houston, for their sug-
......... ..................... (A-16) gested derivation of Eq. A-20.

DECEMBER 1982 2885


pressible liquid can be used rigorously for gas wells for Thus,
suitable initial and boundary conditions. 0.1348T«. V,,.n dw,,.n


Initial and boundary conditions of major importance qsf=q+ - . . . . .... . . . (A-31)
include the following: uniform pressure in the reservoir 2Twb Psc dta
before flow; no flow across the outer boundary of a A significant simplification resulted from use of >G and to
cylindrical reservoir; and constant surfhce flow rate in Eq. A-31. The coefficient of d^,,,b/dt, is a constant
(with changing sandface rate caused by wellbore (as in the case of the analogous equation for a slightly
storage). Our objective at this point is to state these con- compressible liquid), whereas the coefficient of dp,,,n/dt
ditions in terms of the variables, >y, t,, and r. in Eq. A-29 included the pressure-dependent term
The initial condition is stated simply as (PC K /Z) wn •
We now need to relate the sandface flow rate to the
^=^r at tQ=O . .. . . . . . . . ... . . . . .... . . . .(A-22) pressure gradient at the sandface:
k 3
The outer boundary condition (no flow) is stated as qsfBB =0.001127 (27rrwh) u aP ^ r v ...(A-32)
a,y r
=0 . . . . . . . . . . . . . . . . . . . . . . . . . (A-23) or
ar 11 = 1' e
The inner boundary condition is more difficult. First, aP qSfBbµ ... ...(A-33)
we note that a mass balance on the wellbore gives the ar 0.007081kftr,,,
following equations. Mass rate in (at sandface) is given Therefore,
by
a^ qsfBB(2P/z)
qsjM Psc
_ .......... ...(A-34)
. . . . (A-24) ar `w 0.007081khrw
Z.scR Tsc Since
Mass rate out (at sandfacc) is given by
q M P.c BY =178.1 " ^ ( RB/Mscf) . . . . . . . . . . . . . (A-35)
P T
................. ........... (A-25) Tsc P
z.scR Tse then
Rate of accumulation in wellbore is given by ay 50,300q,fpS,T
. . . . . . . . . . . . . . (A-36)
khrwTsc

is
d 0.1348p,,,nV,,.bM l 0.1348V,,,bM d
ar
Introducing Eq. A-31 into Eq. A-36,
dt z^^nRTb / RT,,,b dt

P aty 50,300p5CT 0.1348TSCVwn


wn . ..........................(A-26) (
z ar `w kh r, T,, q 2T„.nps,
These equations hold for a wellhoie in which the
aO wb
temperature of the gas is not changing with time. ................... (A-37)
The mass balance for the isothermal wel]bore may be atn
written as
TS` Vwb d P Simplifying,
qsf-q +0.1348 .-b atfi _ 50,300pSCT q + 3,390T Vwb a>1'wb
Twb Psr dt z
ar I'µ' kh rwTsc kh r,,,Twn atQ
........... ................... (A-27)
Now, .......... .................... (A-38)

d P dP,,-b d(P/z) ^, b_ c„•nP wn dP irb Implications of our results become more clear if we
dt x ^n = rewrite the differential equation and initial and boundary
z dt dp µ,b z,,,n dt
conditions in terms of dimensionless variables and com-
.
pare them with similar equations for a slightly com-
........ ...................... (A-28)
pressible liquid.
Define
Thus, the mass balance becomes
rp =r/r,v . . . . . . . . . . . . . . . . . . . . . . . . . . . . . (A-39)
qsf=q+0.1348TSCVsvbPwbcwb dPwb
(A-29)
T1,,bP.sczvt,b dt kh T S^.
^LD= 4i -0) . .............. (A-40)
Now, 50,300q p.sc

dP wb
dt
I
(Ficr)wb
dP „-n
dta
µZ

2P
J wb
I
(Ycr)wb
d>G
dta
and

taD =
0.0002637kt.
(A-41)
is
Or w 2
..............................(A-30) In terms of dimensionless variables, the differential

2886 JOURNAL OF PETROLEUM TECHNOLOGY


equation, Eq. A-21, becomes Then the inner boundary condition can be written as

1 a Ilk D a>^D

• - - (rD = . ........... (A-42)


rD arp arD ataD OD I rD_1 = - I+CDe ( d4D 1 w,b. .. .. ..(A-47)
arD \ aD J
The initial condition, Eq. A-22, becomes The analytical solution presented in the literature by
Agarwal et al. 2 is for precisely the same equation and
OD =Oat taD=O . ....................... (A-43) conditions. Thus, when the assumption that the
temperature of the gas in the wellbore is constant
The outer boundary condition, Eq. A-23, becomes throughout a test is adequate, we should expect the Agar-
wal et al. solutions to model gas well tests in those cases
ND I =0 . . . . . . . . . . . . . . . . . . . . . . . . (A-44) in which Eq. A-20 is satisfied.
arD rDe
SI Metric Conversion Factors
The inner boundary condition, Eq. A-38, becomes
acre x 4.046 873 E+03 = m2
bbl x 1.589 873 E-01 = m3
a>GD 0.894 T V,,,b (df,D ^ cp x 1.0* E-03 = Pa's
D r=) =-1 + Z wb
ar cu ft x 2.831 685 E-02 = m3
T,b
OF (°F-32)/1.8 = °C
ft x 3.048* E-01 =m
..............................(A-45)
gal x 3.785 412 E-03 = m3
Ibm x 4.535 924 E-01 = kg
Define an effective dimensionless wellbore storage
psi x 6.894 757 E+00 = kPa
constant, CD,,, as
'Conversion factor is exact. JPT
Original manuscript received in Society of Petroleum Engineers office March 30,
0.894 T V,,,b 1981 Paper accepted for publication April 16, 1982. Revised manuscript received
CD, ...................(A-46) Oct. 15. 1982. Paper ( SPE 9888) first presented at the 1981 SPE Low-Permeability
oFtr,,.'` T wb Symposium held in Denver May 27-29.


DECEMBER 1982 2887
^ A= Ah^ = 1.54x107 ft
h^ (30 ft)(0.11)(43560 ft2/acre)
A = 107 acres.

Further pressure transient analysis is possible to determine the reservoir shape factor
and the drainage geometry of the well, but these computations are beyond the scope
of this manual.

3.4.2. Highly Compressible Fluids (Gas)


When a constant rate gas well achieves pseudosteady state flow, its
behavior can be expressed in terms of the real gas pseudopressure as:

-Vp R) - 50.3 qgSc T psC i!ln( 10.06A ) 3+ s (7.107)


*(P„;,f)
khTsC 2 Cr2 4
A W

F- 1
This equation can be rearranged to solve for flow rate:

khTsc [*(p R) - VPw)]


q = (7.108)
50.3 T psc I lln (
10. 06 A- 3 + s
2 ^ CArW2 4

For the classic case of a well centered in a circular drainage area of radius, r., the
shape factor is determined from Table 7.3 to be CA = 31.62, and the drainage area
is A = 7tr. 2 . Equation 7.108 reduces in this case to the following form:

khTsc [^^ R) - ^(p,^)J (7.109)


q^^ = r 3
50.3Tpsc In e - - + s
P-j rW 4

W"
If the µz product can be assumed constant, which is approximately true when ^
p < 1500 psia, Eq. 7.108 reduces to the following form:

khTs^ [p R - p^.]
q gsc = 1 10.06A 3 (7.110)
50.3 T µzpSG 21n Z - 4+s
CArw

Also, the radial form of the pseudosteady state equation, Eq. 7.109, reduces to:

-2 2
khT
q gsc = Sc [p R p`fl ! 7.111

50.3Tµzpsc In e -3+ s
rw 4

Because the pressure-squared forms of the pseudosteady state equation are


approximate, it is recommended that Eq. 7.108 be used for reservoir engineering
applications.

EXAMPLE 7.11: Prediction of gas flow rate under pseudosteady steady flow
conditions using real gas pseudopressure

Problem. A gas well is located in a large reservoir which was developed using a
square drilling pattern and 320-acre spacing. The well is an interior well and is
believed to be approximately centered in a square drainage area. The well is being
placed back on production after being shut in for a pressure buildup test which was
used to measure the values of kg, s, and PR that are provided. Calculate the flow rate
this well should produce at the following values of p,,,f: 3000, 2500, 2000, 1500,
1000, and-.500 psia.

kg =6.3md T =170°F
h = 20 feet TSC = 60 °F
rW = 0.35 feet pSC = 15.025 psia •
PR = 3200 psia yg = 0.72
s = 5.5

7-78
PROBLEM PTXLHGAS: PRESSURE BUILDUP TEST, TIGHT
• GAS WELL, SKIN & STORAGE

File PTXLHGAS.PAN contains the pressure buildup data for a tight


gas well. Analyze this test using both type curve and conventional methods
and verify your final analysis using Quick Match.

0
-b

q • q

PTXLHGAS: Type Curve Match (using pseudo time)


Match Results
Radial homogeneous
Infinitely acting
K = 0.1029 md
Cs = 0.0117 bbUpsi
1000 Cd = 1030.5216
V -144.2371 bbl
S = 19.5'S69
P, =0 1/(Mscf/day)

^100
^

10

0.1 1 • 10 100 1000


Equivalent Time (hours) - Tp=1200.0
PTXLHGAS: Lo g - oPlot Quick Match usin p seudo time
Q uick Match Resutts
Radial homogeneous
1000 Infinitely acting
Constant compressibility
Cs -0,0109 bbl/psi
K = 0.1044 md
S =20
D =0 1/(Msc(/day)
Pi -6493.9658 psFa

O
W 10
1
100
Jr,

...i
E
ed
..+

10

0.01 0.1 1 10 100


Equivalent Time (hours) - T =1200.0

0 0 0
• • •

2000 . PT XLHGAS: Semilo Plot, Quick Match using p seudo time

Quick Match Results


Radial homogeneous
Infinkely actinD
Can"t
l coff wrmsbw
U 0.0109 bbUpW
1500 K = 0 . 1044 md
S =20
0 -0 1!(fNscUdrA
Pt - 6493.965E psia
0
W
^
01000

o.
E

500

0
1 10 100 1000 10000 100000 1e+
Homer Time Function - Tp=1200.0
zsoo
- PTXLHGAS: Cartesian Plot, Quick Match usi n pseudo time)

2000

.-^

1 Soo Oukk Match Results


a dal homogeneo us
l^ink esY sdi^D
^ C4nstwt cwrpvssUNty
-0.0109 bbUpai
^ K = 0.1044 md
1000 S °20
CL D -0 1/(Mscf/day)
-6493.9658 Psia

500

0 0 500 1000 150


Elapsed Time (hours)

0 0 0
• PROBLEM GBUNDAH2: PRESSURE BUILDUP TEST - GAS
WELL

File GBUNDAH2.PAN contains the data for a buildup test on a gas well.
You are requested to analyze the test for permeability and skin factor using
both conventional and type curve methods, and comment on possible
boundary effects. Verify your results using Quick Match.

0
• • •

2WO GBUNDAH: Semilo Horner Anal sis ( usin g pseudo time

1800

1600 T n n lit
1400 n
,-.
co
W 1200
:.. '
n
1000
2V) •
Q BOO n
^ n I
600
Model Resufts
400 K = 1.9874 md
kh = 41.7344 md.ft
Rinv = 1073.1372 ft
FE = 0.1731
200
ps = 4342.1484 psi • .
W1 OWA OMMUOt
S =37.3177 •n n •
0 P' = 6240.3242 psia
1 10 100 1000 10000 100000
Homer Time Function - Tp=142.705002

This is the conventional semilog analysis of the data which the horizontal derivative on the log-log plot, and type curve analysis,
indicates is radial transient flow. The large positive skin factor in noted; it was found by nonlinear regression analysis (see
accompanying graphs) that most of this skin is probably due to turbulent flow.
• • •

2000 GBUNDAH: Quick -Match, Homer Results ( using seudo time

1800

lsoo

1400
' '.

W 1200
v n

I :: n

600

400
Quick Match Results
s = 0.0073 bbVpsi
K = 1.9874 md
200
S = 37_3177

0
1
D =0 11(Mscffday)
Pcalc = 6245_7095 psfa
10
I J J_ L _ I I I I I f
100 1000
I I I^]^
M It7 10000
n•

100000
Homer Time Function - Tp=142.705002

This is the semilog plot showing the actual test data overlayed with the theoretical buildup behavior that should be expected from the
results computed with the conventional Homer straight line analysis. This is not a good match and, accordingly, implies that the semilog
analysis may not be correct, or at least that it may not account for all of the variables which are affecting the test. It clearly says to us
that we need to go back and take another look at the data.
• • •

This is the log-log plot of the actual test data overlayed with the theoretical buildup behavior that should be expected from the results
computed with the conventional Homer straight line analysis. This is not a good match and, accordingly, implies that the semilog
analysis may not be correct, or at least that it may not account for all the variables affecting this test. It clearly says to us that we need
to go back and take another look at this data. _
• • •

This result was obtained by first manually type curve matching the data. The results of the manual type curve match were then used as
a starting point for the program to perform an automatic type curve match using nonlinear regression techniques; when doing this, the
turbulence coefficient, D, was included as a variable. The results of the automatic type curve match were then manually fine tuned
using the quick match capability of the program to obtain this final match. The derivative curve indicates a possible boundary effect in
the late time data; insufficient information is available about the reservoir to try and model this behavior.
• • •

2000 G BUNDAH : Semi lo Plot , Quick M atc h (using seudo ti me


nn TP1 Pressure
- Generated

1800
RRR

1600

1400

CO
uj 1200
...

! b000
a
800
a
E
600

400
Quick Match Results
Cs = 0.0062 bbUpsi
K = 1.985 md
200
S =8
D = 0.0191 1 !(Mscf/day}
0 Pcalc = 6236.8105 psia
1 10 100 1000 10000 100000
Homer Time Function - Tp=142.705002

This result was obtained by first manually type curve matching the data. The results of the manual type curve match were then used as
a starting point for the program to perform an automatic type curve match; when doing this, the turbulence coefficient, D, was permitted
to be a variable. The results of the automatic curve match were then manually fine tuned using the quick match capability of the
program to obtain this final match. It is noted that the skin factor is 8. This is not the total skin factor; this does not include the skin
factor due to turbulence. The total skin factor is: Stotal = S + Dq = 8+(.0191)(1530 MSCFD) = 37.2. This is almost exactly equal to
the total skin factor of 37.3 given by the Homer semilog analysis. It appears, therefore, that the large skin factor in this well is caused
primarily by turbulent flow.
• PROBLEM 10-11T: BUILDUP TEST, HYDRAULICALLY
FRACTURED GAS WELL

This is a pressure buildup test from a hydraulically fractured gas


well in Alberta, Canada. Use both conventional and type curve methods
to determine permeability and fracture length. Was this fracture treatment
effective?

PanSystem File: 10-11T.PAN

qg = 1.3 MMSCF/D

rW=0.24ft

h=25ft

(k = 0.094
^
p; = 2,100 psia

T = 158 °F

SW = 0.3

yg = 0.735

0
SOLUTION: 10-11T

SOLUTION:

It appears from the analysis of this data that the fracture treatment on this well was not very effective.
Attempts were made to match this data on infinite conductivity, uniform flux, and finite conductivity type curves.
While computed values of k and Xf were essentially the same regardless of the model used, the finite
conductivity type curve appeared to best match the data.
FINITE CONDUCTIVITY FRACTURE SOLUTION:
k = 0.47 md
Xf = 18 ft
• Fcd=20

The data were also analyzed using conventional semilog analysis, and by matching the data on the Bourdet,
et al., derivative type curve for homogeneous reservoirs. This solution seems to match the data better than the
fracture solution, but gives similar values of reservoir parameters. Only the semilog analysis results are
presented.
SEMILOG ANALYSIS:
k = 0.46 md
s = -3.5
Xf = 16.6 ft (based on effective wellbore radius equation)

The early data in this test appeared to be affected by changing wellbore storage. This could have been
caused by phase redistribution since the well was producing some liquids. The late data on the derivative
curve could also be showing a boundary effect; since no geological information was available regarding
possible boundaries, no attempt was made to model this behavior.

0
Prob. 10-11T: Type Curve Match (using pseudo time)
Match Results
Vertical fracture - finite cond
Inftnkely acting
K = 0.44726 md
t z 17.6836 It
pr = -3.5372
Fcd = 20
Cs = 0.01 bbl/psi
f =0
=0 1/(Mscf/day)
100

O
W
...
CL

E
cc
• ♦N^
- ♦♦♦
(D • ♦♦
^ ♦
• ♦♦

n ©

10 ♦

0.01 0.1 10
^- ---- -- - Equivalent Time (hours) - Tp=64.0 _
This plot shows the test data compared to the theoretical behavior using k, Xf, and Fcd determined by manually matching the data on the
Cinco-Ley finite conductivity fracture type curve. The wellbore storage factor was then manually adjusted using Quick Match to obtain a better
fit of the early data. Similar values of k and Xf are obtained using the infinite conductivity and uniform flux type curves, but the finite
conductivity solution seems to give a better match.

0 0 0
• •
350 Prob. 10-11T: Semilo Anal ysis ( usin g seudo t ime
Model Results
Radial homogeneous
Infinitely adkV
K = 0.4641 md
Ich = 11.6021 md.ft
300 Rlnv -328.579 ft
FE =2.1621
pS = -817.4211 psi
--3.5437
P' = 2099.3354 psfa

250 SL

m
0
uJ

a 200 SN'
NCIS
CL
v
C
C

150

100

. .
.
.
50 1
10 100 1000 1000
Homer Time Function - T p=64.0

The semilog straight line shown on this plot corresponds to those data which best approximate a horizontal derivative on the log-log plot.
Prob. 10-11T: Log-Log Quick Match ( usin g pseudo time )

100

^o
0
w
« n
Q
V
'
N ♦
cc
y ♦
Q ♦
..i
Q ♦ Quick Match Resutts
E 10 . 'dia l homogeneous
rrvitey acWq
= 0.015 bbl/psi
Q = 0.4641 md
S --3.5437
0 =0 11(Mscfrday)
PI -2103.3428 psfa

0.001 0.01 0.1 1 10


Equivalent Time (hours) - Tp=64.0

This plot shows a comparison of the test data with the theoretical behavior expected using the values of k and s computed from the semilog
straight line. The wellbore storage factor was adjusted manually using Quick Match to match the early time data.

0 0 0
• • q

350 Prob. 10-11 T: S emilo Qui ck Match usin pseu do time )


Quick Match Results
R a di a l h omogeneous
Infinitely acUng
= 0.015 bbUpsi
K = 0.4641 md
300 ` -3•5437
D 60 1!(Mscflday)
Pi ^ 2103.3428 psta

250

^
0
W

CL
a 200
.T

150,

100

50
1 10 100 1000 10000
Homer Time Function - Tp=64.0

This plot shows the test data compared to the theoretical behavior predicted using k and s computed from the conventional semilog analysis.
The wellbore storage factor was adjusted manually using Quick Match to match the early time data.
• PROBLEM SCH10-30: POST-FRACTURE BUILDUP TEST,
TIGHT GAS

This is a buildup test from a hydraulically fractured well in a tight


gas reservoir in South Texas. The well showed little response to the
fracture treatment, and the pressure in the well declined rapidly when
placed on production. This well is currently developed on 80-acre spacing
and is surrounded by other producing wells. Water influx is believed to
be negligible in this reservoir.

(a) Analyze this test for formation permeability and fracture length, and
evaluate the effectiveness of the fracture treatment.

(b) Does this test offer any explanation for the apparent rapid depletion
of this well?

(c) One of the owners of this property believes that the formation is too
tight for the wells to effectively drain 80 acres and wants to reduce the
spacing to 40 acres. Based on this test, can this well effectively drain 80
acres?

PanSystem File: SCH10-30.PAN S, = 0.3

qg = 0.5 MMSCF/D 'Yg = 0.727

rW 0.328 ft tp = 160 hrs

h 96 ft Ots,,uli, = 168 hrs

0 0.09 pWXat=0) = 613 psia

Pi 2,400 psia

T 190 OF

^
V
SOLUTION: SCH10-30

Perforated Interval Bottom

SOLUTION:

The major objectives of this test were to evaluate the effectiveness of the fracture job on this well, and to
determine why the pressure in the well declined so rapidly after the well was placed on production. A
secondary objective was to determine if the well could effectively drain the reservoir with the 80-acre spacing
which was present at the time of the test.
As the attached plots show, the data can be analyzed using both homogeneous and fracture models.
Regardless of the model used, a fracture half-length of only 6 feet is indicated. It appears from this test that the
fracture stimulation treatment was unsuccessful.
[ iJ This test has the classic fingerprint of a limited reservoir. It was possible to model the test with a well located
in the center of a square drainage area with no-flow boundaries only 88 feet away; this represents a drainage
area of less than one acre. While this should not be regarded as a unique solution, it offers a possible
explanation of why the pressure in this well is rapidly declining. This conclusion has to be tempered with the
observation that this test has a radius of investigation of only 103 feet.
This test does not answer the question of whether a well in this reservoir can effectively drain 80 acres;
again, it should be noted that the radius of investigation is only 103 feet. If the 80 acres is in pressure
communication with this well, the well will eventually drain it. The real question is whether the well will
"economically" drain 80 acres; a well test cannot independently answer this question, but will provide data
which can be used in other reservoir and economic models to address this problem.

0
SCHIO-30: Infin. Cond. Fracture Model usin seudo time
Match Results
Vertical fracture - infinite conductivity
Cbsed system
K = 0.0432 md
f = 6.1271 ft
pr = -2.2343
CdXf = 0.2
Cs = 0.0214 bbUpsi

OWA

100
n
n

M ♦
^7 * ♦♦ M
Cl
♦ ♦
_a ♦
E ♦
:° ♦

10

0.01 0.1 1 10 10
Equivalent Time (hours) - T-160.05

This is a match of the test data using the infinite conductivity fracture model. The well is assumed to be in the center of a square drainage
area with no-flow boundaries only 88 feet away. While the boundary model should not be regarded as a unique solution, it does offer a possible
explanation of the rapidly declining pressure in the well.
As expected, this data can be matched using a fracture model. However, the fracture half-length is only 6 feet. Although the well is
stimulated, it is obvious that the fracture treatment was not successful.

0 0 0
• •

SCH10-30: Homo eneous Model ( usin g seudo time
Quick Match Results
Radial homogeneous
Closed system - [L:L:L:L]
Cs = 0.024 bbUpsl all
K = 0.0422 md
S = -2.26
D =0 1 /(Mscf/day)
L = 88 ft
100 Pi = 2377.7185 psia

co
W •
,• . •
CL •.
a fAl •

ca
m
q •
.
10

0.01 0.1 1 10 1
Equivalent Time (hours) - Tp=160.05

This is a match of the test data using a homogeneous reservoir model. The well is assumed to be in the center of a square with no-flow
boundaries only 88 feet away. While this boundary model should not be considered a unique solution, it does offer a possible explanation of the
rapidly declining pressure in this well.
This plot has the fingerprint of a stimulated well. However, the skin factor does not reflect the degree of stimulation expected in a well which
has been hydraulically fractured. If this skin factor is used in the equation for effective wellbore radius, a fracture half length of only 6 feet is
indicated_
350 SCHIO-30: Horner Semilo Analysis( usin pseudo time

300

250

CO
0
W 200
^
v
d
V
N
c4
^
'Ilk
° 150
CL
E
%
n^
100
Model Results n
Radial homogeneous n
Infinitely acting oi l
K = 0.0416 md n
50 kh = 3.9929 md.ft '
n
Rinv = 102.8079 It
FE = 1.6994
pS = -613 psi
S = -2.3169
0 P' = 2333.5549 psia
1 10 100 1000 1000
Homer Time Function - Tp=160.05

This is the conventional straight line analysis of the data which appear from the horizontal derivative on the log-log plot to be radial transient
flow.

9 0 •
PROBLEM GAS-JCPT: GAS BUILDUP TEST, CHANNEL
0 SAND

This gas buildup test is from a well located in southern Alberta,


Canada. The well is completed in a long channel sand which has a width
estimated to be 700 - 1000 feet, and is believed to located close to one
side of the channel. No other wells were producing nearby at the time of
the test. Analyze the test for k and s, and determine any possible
information about the drainage limits of the well.

PanSystem File: GAS-JCPT.PAN

qg = 13,168 MMSCF/D

tp = 65.356 hrs

Atsn„t;n -316.246 hrs


• pW,XAt=0) = 1274.4 psia

p; = 1523 psia

T=131°F

rW = 0.229 ft

h = 49.2 ft

0 =0.22

-yg = 0. 665

0
GAS-JCPT

SOLUTION:

The behavior of this well test Is consistent with the geology of the reservoir. Permeability and skin were
determined to be:

k = 200 md
s=22.5

History matching of the late time data with a parallel fault model resulted in a channel width of 950 feet with the
• well located 200 feet from one side of the channel. The late data also exhibited linear flow; this data was
analyzed on a tandem square root plot to obtain a channel width of 935 feet.
Although the results are not presented, it is possible to match the test data with other boundary models. As
is the case with most well tests, what gives us confidence in this model is that it is consistent with the geology
of the reservoir.

0
GAS CPT: Log-Log Plot usin pseudo time
-41
IL

.
Quick Match Results
Radial homogeneous
• ra{bl faults
10 s: 0.042 bW/psi
p K = 200 md
o S = 72.5
W D =0 1 /(Mscf/day)
1 = 200 ft
a = 750 It
U
a Pi -1447. d633 psia
2N
CL
CL
E

, • ..

0.001 0.01 0.1 1 10


Equivalent Time (hours) - Tp=65.356 I

This is a match of the test data using a parallel fault model; the parallel boundaries are no-flow boundaries, and it makes no difference
whether the boundaries represent sealing faults or the lateral limits of a channel. The theoretical model assumes the reservoir to be infinite
along the channel; this is not true in reality, but is an adequate theoretical assumption since no other wells or boundaries are nearby. This
solution is consistent with what is known about the geology of the reservoir.

9 0 is -
• • •
185
GAS -JCPT: Se mil o Analysis (using pse u do time)

180
n^
n
' n n n ^
n .

175 n

n n

0
W 170 ^

CL
c)
N
_cE

165

160,

Model Results
Radial homogeneous
Parallel faults
K = 182.0826 md
155 kh = 8960.7598 md.ft
Rinv = 2484.8425 ft
FE = 0.314
pS = 109.6838 psi
S = 19.9151
150 tP'* = 1432.0201 psia
50
1 10 100 1000 10000
Homer Time Function - T p=65.356

The semilog straight line used in this analysis corresponds to those data that form a horizontal derivative on the log-log plot.
I
GAS-JCPT: Log-Log Plot (using pseudo time)
° ^. TPt Pressure
♦^ Aga. Radial Der
Aga. Unear Der

100 °

o °
nnnn • n n n n n n n n n n n n n n n n n n n n
W ^ . n a nn . n n n n n n n n nn .
^. n -
CL

Mc
Q. 1o , . °

.,
^ •
E
m • °°
° •

^ . ...- '.•'• ° ° °
° °
• ♦ • ♦ °

♦ • • • • • ♦ ♦

0.01 0.1 1 10 100


Elapsed Time (hours) - T-65.356

This is a log-log plot of the test data using "elapsed time". Notice that the radial derivative plots as a half-slope straight line at late time, and
that the linear derivative plots as a horizontal line; this is indicative of linear flow, and is to be expected since linear flow will develop at late time
in a long narrow channel. This data will also form a straight line on a square root from which the channel width can be approximated; this plot is
attached.
It is important to observe that "elapsed time" rather than "equivalent time" is used on this plot. Recall that equivalent time is based on infinite
acting radial flow and, when boundaries affect a test, this is no longer appropriate. "Equivalent time" will often cause data affected by
boundaries to tum up sharply resulting in shapes that are mathematical artifacts and are not related to the derivative behavior that we normally
associate with classic geometries such as linear flow, bilinear flow, etc. This effect is more exaggerated when producing time is short relative to
the shut-in time. "Elapsed time" is also not perfect in this situation because it does not account for the production history of the well; it does
however eliminate the compression of the data caused by equivalent time. It is only when the simulated response of the test matches the actual
data that we have confidence that our model is correct.

9 0 •
• • •
185 GAS-JCPT: Linear Flow Pl ot ( usin g pseu do time)

18 0

175
n^

CO n
0
W 170

,B165
CL
E

160

Model Results
Radial homogeneous
Parallei faults
155 = 934 . 7099 ft
l.1 = 139.9697 ft
Sc = 7.2676
K = 200 md
S = 22.3
P' = 1447.161 psia
1501
2 3 4 5 6 7 8
Tandem Square-root Function - Tp=65.356

The purpose of this plot is to compute the width of the channel using linear flow data. The straight line used for the analysis corresponds to
those derivative (linear flow derivative) data that form a horizontal line on the log-log plot. It is observed that the channel width, W, is in good
agreement with the model results shown on the attached log-log plot.
GAS-JCPT: Log-Log Plot using p seudo time )
-•--•-^.^-

100

Quick Match Results


Radial homogeneous
Parallel faults
• s -0.055 bbUpsi
K -237.5 mod
=5
• = 0.002 1 /(Mscf/aay)
t =105 ft
L3 =750 ft
a + Pi -1446.3376 psla

CL
as
=
m •
o •

10

^ . ..
. . • • , .
. +
. •
. . • . ' . • '
. '
0.01 0.1 1 10
Equivalent Time hours - Tp=65.356

This shows an alternative match of the test data where turbulence has been considered as a possible variable in the model. While this gives
a better match of the early data than is obtained using a laminar flow model, it should be noted that this does not offer 100% proof that this well
has turbulent flow; it is possible that non-idealities in the early data are caused by changing wellbore storage due to phase segregation. The
possibility of turbulence could be checked by running a multirate test.

9 0 0
• PROBLEM DS TS-HPl : GAS DRILLSTEM TEST,
PROCESSING AND ANALYSIS OF
RAW DATA

PanSystem file DST5RAW.HP1 contains the raw data for a cased hole
drillstem test from a gas zone in a discovery well. This is the upper zone in a
five-zone sequence where the top three zones contain gas and the bottom two
zones contain oil. After importing the raw gauge data and defining the
various flow periods, you are requested to analyze this test for permeability
and skin. Also, it is not clear from logs and PVT data whether the five zones
are in communication and represent a common reservoir, or whether each
zone represents a separate reservoir. Is there any evidence in this test that the
test zone is in communication with one or more of the zones below it?
The test sequence for the drillstem test was:

Start First Flow 24 March 94 21: 23: 00 (21.383 hrs)


^ Start First Buildup 21: 33: 00 (21.550 hrs)
Start Second Flow 23: 33: 20 (23.556 hrs)
Start Second Buildup 25 March 94 08: 22: 10 (32.369 hrs)
End Second Buildup 20: 19: 30 (44.325 hrs)

Other reservoir and test data are:

h44ft
p; = 4,348 psia
T = 223 OF
qe = 24.41 MMSCFD ( main flow period preceding final buildup)
rw=0.41ft
SW 0.25
yg - 0.808
^ = 0.25

Gas flow rate is not available for the first flow period.
0
• • •

4800 DST5-HP1 : Pressure and Rate Hi tory 24.8982

4600 21 . 2716

4 4 00 17 . 6449

"a

4200 14 . 0183 M
a
ro ° m
cc
7 °
4000 ° 10.3917
CL ^
° y
cc
^
3800 6.76506

3600 3 . 13843

340 ^-0.4882
ZO 25 30 35 40 45
Time (hours)
1100 DST5-HP1: Semilo Plot, Final Buildu ( using seudo time

1050

n n
n -

,-.1000 n
[O
9
UJ
s
Q.
0 950
CL
CL
E9w

Results
K = 16.4956 md
850 = 725.8077 md.rt
Rim = 468.7637 ft
FE = 0.6974 n
S = 266.3328 psi
S = 2.9169
P• = 4332.8589 psia
800
1 10 100 1000
L Homer Time Function - Tp=8.817331

The semilog straight line on this plot corresponds to the horizontal derivative on the log-log plot.

• I ^, •
• • •

iioo DST5-HP1: Semilo Plot , Quick Match ( usin g pseudo time

1050 -

r.10oo

w
.
a
0 950
^
n
.^
CL
^.
E 900

uidc Match Results


Cs = 0.007 bbNpsi
850 K = 16.8942 md
S = 3.1115
= 0.0188
Lam = 3.786e-004
D =0 1/(Mscf/day)
Pcalc = 4331.6269 psia
800
1 10 100 1000
Homer Time Function - Tp=8.817331

This analysis was performed using nonlinear regression techniques (sometimes referred to as automatic type curve matching) as
opposed to using conventional straight line graphical methods. The points represent the measured pressures and the solid line is the
theoretical pressure response predicted using the reservoir parameters shown in the results box. These results were achieved by using
a dual porosity reservoir model. One explanation of dual porosity behavior is that the test zone could be in communication with another
zone in the reservoir; in this case, there is a good possibility that this zone communicates with another gas zone immediately below.
The values of kh and s are in very good agreement with those obtained from conventional straight line analysis on the Horner plot.
DST5-HP1: Lo -Lo Plot, Quick Match usin pseudo time

Quick Match Results


s -0.007 bbUpsi
K = 16.6942 md
100 = 3.1115
9 -0.0188
W Lam = 3.786e-004
=0 1 /(Msct/dtyj
Q. Pcalc = 4331.6289 psis

.^ •
E
o •
nn TP1 Pressure •
••
•• Radial Deriv
• ^
- Generated • •
- Gen derfvative •
10

0.01 0.1 1
Equivalent Time (hours) - Tp=8.817331

This analysis, also shown on the accompanying semilog plot, was made using nonlinear regression techniques. A dual porosity reservoir
model was used in the analysis. It is believed that dual porosity behavior is due to communication with a gas zone which is located
immediately below the test zone.

0 0 0
• PROBLEM PTX7-7: STABILIZED FLOW-AFTER-FLOW
TEST, SURFACE PRESSURES

A well centered on a 640-acre drilling pattern was tested using a four-


point flow-after-flow test. All pressures were recorded at the surface. It was
assumed that pseudosteady state flow was achieved during each 8-hour flow
period. Use PanSystem to analyze the following data for C, n, and AOF:

k = 74 md
(^=0.15
h = 27 feet
T=196°F
r,,, = 0.33 feet
'yg0.75
cf = 12 x 10' psi''
SW = 0.35

• . ..2 . .

♦ ykrrt
y: £ i
* ^. .^ ^1.x
141 (PR) 0
^ ^

0 2,800
8 2,661 1,800
16 2,590 2,700
24 2,500 3,600
32 2,425 4,500

0
• • •

10 PTX7-7: Sandface C&n Plot

27
zo^^
Model Results
Radial homogeneous
Infinitely acting
n = 0.9542
C = 4.434e-006 MMscf/day/psi2n
^ OF -16 .8026 MMscflday

rA
C^

CL
'0
72,

000,

1 10
Flow Rate (MM scf/da y)
• PROBLEM PTA7-3: MODIFIED ISOCHRONAL TEST

The following data are from a modified isochronal test. You are
requested to analyze the test using the C&n method and the LIT method.

rW=0.33 ft T=178°F
h=19ft SW 0.26
c^ = 0.224 X9 0.69

Flow Period Duration, hrs p, psia q, MMSCF/D


Initial Shut-In 48 1,952 0
First Flow 12 1,761 2,600
First Shut-In 15 1,952 0

• Second Flow
Second Shut-In
12
17
1,694
1,952
3,300
0
Third Flow 12 1,510 5,000
Third Shut-In 18 1,952 0
Fourth Flow 12 1,320 6,300
Extended Flow 72 1,151 6,000

0
3. A summary of calculations required to prepare

• the deliverability graphs is presented in the follow-


ing table:

q PR Pw f PR pw f) /q
(Mscf/D) (psi2) (psi 2/Mscf/D)

2,600 709,183 272.8


3,300 940,668 285.1
5,000 1,530,204 306.0
6,300 2,067,904 328.2
6.000 2.485.503 414.3

(a) Empiricial Method


The plot required by the empirical method is
presented in Fig. 7P.S. A best fit straight line is
constructed through the four points which represent
a transient flow time of 12 hours. It is assumed
• that this line has the correct slope; however, it
is in the wrong position because the 12 hour duration
of each flow period was insufficient to achieve pseudo-
steady state. Therefore, a parallel line is drawn
through the stabilized point to establish the correct
position of the deliverability curve.
Two points are selected from the stabilized
curve:

q = 2,600 Mscf/D at p'R-p^f = 9x105psi2

q = 10,000 Mscf/D at pR-pWf = 4.5x106psi2

The empirical deliverability equation is, from Eq. 7.84,

n
q
C (P R p2wf)

7-140
, 1
, I

10'

STABILIZED
DELIVERABIL ITY ,

pWf = 0 CURVE

N TRANSIENT FLOW
CURVE (t = 12 HRS)

N 3 106

AOF = 8 , 700 Mscf/D


• i
105
103 10" 105

q, Mscf/D

Fig. 7P.5: Empirical gas deliverability plot,


Prob. 3.

From Eq. 7.87,

log q2 - log ql
n =
log (Op2) - log (api)

log (10,000) - log (2,600)


n log(4.5x106) - log(9x105)


n = 0.837.

7-141
Y

From Eq. 7.88,

• C=-q
(,Pz)n

C= 10,000
(4.5x106)0.837

C = 0.027 Mscf/D/psiZ

Therefore, the empirical deliverability equation is

pwf) 0. 837
q= 0.027 (PR

The absolute open flow potential can be calculated,


or determined graphically, at pwf = 0. From the equation,

AOF = 0.027 1 (1952)2 - (0)2I0.837


0

AOF = 8.7 x 103 Mscf/D

AOF = 8.7 MMscf/D.

This is in agreement with the AOF read from Fig. 7P.5.

(b) Theoretical Method


According to Eq. 7.100, the pseudosteady state gas
equation can be written as

Pp _ pw f_ Asq + Bsa2

-z 2
or PR q pwf = As + Bsq

q
7-142
which suggests that a plot of (PR pwfya versus q
should be a straight line. This plot is presented
in Fig. 7P.6 for the test data. The slope of the
S i
best fit straight line on Fig. 7P.6 is

Bs = slope = 0.0138 psi2/ Mscf/D

c.f
^

N
G3+

^
31
0 1
N
^04
N C4

2
2 3 4 5 6 7

q, Mscf/D x 103

Fig. 7P.6: Theoretical gas deliverability


curve, Prob. 3.


7-143
,

From Eq. 7.100, the intercept, As, is


- B 2
-2_ 2

A = (PR pwf ) sa
s
q

The intercept, which reflects the position of the


deliverability curve, must be computed using a point
on the stabilized curve. From the test, we know that at
a stabilized rate q = 6,000 Mscf/D, pR-pwf = 2,485)1503;
therefore,

A = 2,485,503 - (0.0138)(6 '000)2


s 6,000

As = 331.45.

The theoretical deliverability equation, therefore, is

pwf = 331.45q + 0.0138q2


• PR

This quadratic equation can be solved for q as

-331.45 + [1.099x105 + 0.0552(p2_ p2 )]2


q R wf
0.0276

The absolute open flow can be computed by


substituting pwf = 0 into the deliverability equation:

-331.45 + [1.099x105 + (0.0552) (1952) 2]1


AOF =
0.0276

AOF = 8,494 Mscf/D

AOF = 8.5 MMscf/D.


7-144
• • •

PTA7-3: Sandface C&n Plot

FAodel Results
Radial homopmeous
klfinitely aCtinp
W n -0.7496
:^. C = 9.653e-005 WAscf/day/psi2n
AOF = 8.2802 MMscUday
.^J
N

'L7

10
Flow Rate scf/da
0.5
PTA7-3: San dface LIT Plot, p- Squared

Model Results
Radid homogeneous
4diradY sdirv
B - 239M.8556 PcITJcW(MscUCaY)
F -1.0686 pst2M.p/
OF -6.4719 MAsctlday
.-^

0.4

^O
0

.-r
^

CY0.9

0'?^ 3 4 5 6
Flow Rate (MMscf7day)

0 0 0
• • •

35 PTA7-3: Sandface LIT Plo t m

-^
U

1O

25

c^.

^1.
11{Odd R^^wltf
a Radial homopeneous
11-N
04. kMWtdY scfk^p
'2a
8 = 24354.8964 pst2/cp/(Msct/day)
F =12455 psi7Jcp1(Mscf/day
OF = 8.573 MMsct/day

^ 52 3 4 5 6
Flow Rate seflda 7j
• PROBLEM ISOTEST: Modified Isochronal Test with Effects
of Non-Darcy Flow

PanSystem file ISOTEST.PAN contains data for a simulated modified


isochronal test for a gas well. You are requested to analyze this test for
permeability, non-Darcy coefficients, Darcy and non-Darcy components of the
skin factor from the "S vs q" plot, and to generate IPR curves using both the
"C & n" and LIT methods.

0
• • •

8000 ISOTEST: Pressure & Rate Histo ry


7. 1604

7900 ° 6 . 3492
8 ^
°
°
7800 5.538

7700 4.7268

W E
y
CL 7600 3,9156

027500 3.1044
a °° b
U_
7400 2.2932 cc

7300 ° 1 .482

7200 0.6T08

710-5 U LL f
0 25 50 22^ 1404
75 100 125 150 17 5 200
Time (hours)

This is a simulated modified isochronal test with four flow and four shut-in periods, all of equal duration, followed by an extended flow
period.
• • •

ao ISOTEST: Radial Flow Plot - Non-Darc anal ms


m TPI Propure
^ TP2 Pressure
TP3 Prom"
*0 TP4 Proem"
cc
35
ti

30

2
d 25

a>
^
^
a
E
cc 20

15
1 10
Equivalent Time (hours)

This semilog plot presents data from the four equal length transient flow periods. A semilog analysis of each flow period was made to
determine permeability, and then these permeabilities were averaged. Parallel lines with slope defined by the average permeability,
were then constructed through data from each flow period to compute the skin factor associated with that rate; these are the lines
shown on this graph. A plot of total skin factor versus flow rate can now be made to determine the non-darcy flow coefficient, O, and a
skin factor which is not affected by non-darcy flow.
• • •

-0 8 - ISOTES T: Skin vs Flowrate Plot

Model Results
RsdMdhomopeneous
-1 InHnRaly actlnD
S --2.9686
D - 2.80&-004 1 /(Mscf/day)
F - 2.0624 ps12/cp/(Msci/dsy)

0
-1.6

-1.8

-2

.5 3 3.5 4 4.5 5 5.5 6 6.5 7 7.


Flow Rate (MMscf/day)

NOTE: The Darcy skin factor, S, shown in the results box, is defined as S' in your course notes; this represents that part of the total
skin factor that is not affected by non-Darcy flow. The coefficient, F, shown in the results box, is defined by the symbol B's in the
course notes (see Eq. 7-109).
• • •

26 ISO TEST:, Radial Flow Plot - Non-Darcy analysis F=2.06238


an TPI Pressure
^ TP2 Pressure
24 TP3 Pressure
M TP4 Pressure r •
cis

M 22
n

n
W
L 20

c^ n
cc
18

16
cc n

W
a

_-14

E
CIO
12

10
1 10
Equivalent Time (hours)

This semilog plot has been corrected for rate dependency by changing the y-axis plotting function. It should be noted that data from all
four flow periods now plot as a common straight line. This plot was generated by selecting ANALYSIS on the menu bar, and then
selecting CORRECT FOR RATE DEPENDENCY.
• • 0

100 ISOTEST: Sandface C&n Plot

Model Rasults
Radial homogeneous
Inflt*ely eding
n =0.7814
C = 1.968e-005 MMacf
AOF -24 . 5153 NNulsct/dt y
.^-.

11-10
11,00
C,4 10

10000-

1 1 10
Flow Rate (MMscf/day)
• • •

9000 ISOTEST: IPR - C & n METHOD


- Ea«,ded
- sab^
8000

7000

6000

4000

3000

2000

1000

0 0 5 10 15 20 25 30 3
Flow Rate (MMscflday)
• • •

50 ISOTEST: Sandface LIT Plot


Model Results
Radial hmvgwmmm
infinkdy acdnD
e = 34902.875 psClJcp/(Mscf/day)
F = 1.8®96 pe12/cp!(MscfJday)
45 AOF -28.9604 MM^cUds^r

^40

67

_ 35
a
...
ar

E
v
30

52L
-L
3 4
-L 5 6 7
Flow Rate MMscf/da

This plot is based on Eq. 7-107, page 7-101, Smith. However, the variable B used by PanSystem is equivalent to A's in Eq. 7-107,
and the variable F used by PanSystem is equivalent to B's in Eq. 7-107.
• • •

9000 ISOTEST: IPR - LIT METHOD


IPR-Trans Test
- IPR-UT

8000

7000

6000

Cu
a 5000
...

4000
a

3000

2000

1000

0 0 5 10 15 20 25 3
Flow Rate MMscf/da
BILINEAR FLOW - GAS RESERVOIRS

• I. BILINEAR FLOW EQUATIONS

A. Constant Formation Face Rate

Dimensionless Pressure:

kh [m(pi) - m(Pw f)] (1)


PD I424ciT

Dimensionless Time:

t _ 0.0002637kt (2)
llxf
Wtxf

Dimensionless Fracture Conductivity:

k w
(3)
I CD kx
r-

I3ilinear Flow Equation:

2 .45 ,
(4)
PD = tDxf
FCD z

NO - m(pw f) _- 444.6c1T - t4 (5)


h (k fw; Z &uctk) °

Bilinear Slope (graph of am(p) versus t4):

444.6q"I' (6)
bf
m h (kiw) ' Nuctk) '
^

GBF- 1
B. Constant Formation Face Pressure

Dimensionless Rate:

a 1424qT (7)

kh[m(pi)-m(Pwf)^

Bilinear Flow Equation:

1 ` 2.72 t a
(8)
qD FCD 2 Dxf

1 = 493.6T t% (9)
q
h(kfW) 2(^uctk) 4 [m(pi)-m(pwf)]

Bilinear Slope (graph of 1/cl versus t°) :

493.6T
m __ (lUi ^
bf
h(kfIV)^Wctk)'[m(pi) -m(pwf

NO'1'L: The equations presented in this section are


written specifically for pressure drawdown
tests. These equations can be modified for
pressure buildup tests by replacing the
pseudopressure differential, Am(p), and the
producing time, t, with appropriate values
as shown in the following table:

Pseudopressure
Test Differential Time
Drawdown Am(p) = m(pi)-m(pwf) t

Buildup Am(p) = m(pWs)-m(pwf) At or 6te


---- - - ---
^

GBF-2
II. BILINEAR FLOW GRAPHS

^
A. Constant Formation Face Rate
When the rate of a gas well is maintained constant, the
pressure change at the formation face is described by Eq.'5.
This equation indicates that a plot of m(pi)-m(pwf) versus
t` for drawdown tests, or m(pws)-m(pwf) for buildup tests,
will yield a straight line with slope, mbf, predicted by Eq.
6. This plot is illustrated by Fig. 1. When bilinear flow
ends, the straight line will end and the plot will exhibit
curvature which is concave upward or downward depending upon
the value of the dimensionless fracture conductivity, FCD.
When FCD < 1.6, the curve will be concave downward; a value
of FCD > 1.6 will cause the curve to be concave upward.

• F CD > 1.6

. < 1.6
,-. FC.ll
SLOPE = mbf
^

END OF
BILINEAR FLOW

^. ,.
t nr s 4

Fig..l: Bilinear flow graph for a constant


rate gas well.


GBF-3
When FCD > 1.6, bilinear flow ends because the fracture
tip begins to affect wellbore behavior. If a pressure
transient test is not run sufficiently long for bilinear

flow to end when FCD > 1.6, it is not possible to determine
the length of the fracture. When FCD < 1.6, bilinear flow
in the reservoir changes from predominantly one-dimensional
(linear) to a two-dimensional flow regime. In this case, it
is not possible to uniquely determine fracture length even
if bilinear flow does end during the test.
A more diagnostic plot to recognize bilinear flow is the
log-log plot. From Eq. 5

4 4 4. 6 q '1' ^
log [m (pi) - m(pw f) ]= log
[h(kfw) ½ Pctk) ¼ j

log t. (11)

Equation 11 indicates that a log-log plot of m(pi) - m(pwf)


versus t will yield a straight line with a one-fourth is
slope; this is illustrated by Fig. Z.

B. Constant Formation Face Pressure


When formation face pressure remains constant, the
formation face rate will change with time as described by
Eq. 9. According to Eq. 9, a plot of 1/q versus t° should
yield a straight line with slope, mbf, defined by Eq. 10;
this graph is depicted by Fig. 3. Following the end of the
bilinear flow period, the curve for FCD < 2.8 will be
concave downward and the curve for FCD > 2.8 will be concave
upward. The straight line for bilinear flow ends for the
same reasons presented for the constant rate case.
Equation 9 also indicates that a log-log plot of 1/q
versus t should yield a straight line with a slope of one-
fourth:

C13F-4
• SLOPE = 1
4

r-.

t, hrs

Fig. 2: Log-log plot showing effect of ideal


bilinear flow for a constant rate gas
well.


I; CD > 2.8

SLOPE = mUf
2.8
I: CD
^la,

END OF
BILINEAR FLOW

^.
t4, hrs4'

Fig. 3: Bilinear flow graph for a constant


0 pressure well.

GBF- 5
493.6T
log (1)
q = log , ,.
1i(k fw)"($uctk) M(pi) -m(pwf)
q
+ 4 log t. (12)

This plot, illustrated by Fig. 4, is the primary diagnostic


tool by which bilinear flow can be recognized.

SLOPE = 1
4

^0 U,

is

t, hrs

Fig. 4: Log-log plot illustrating the effect


of ideal bilinear flow for the
constant pressure case.

.
III. END OF BILINEAR FLOW

A. Constant Formation Face Rate


The relationship between ( tDxf)ebf and FCD is depicted
graphically by Fig. S1 This relationship can be approximated
as:
LJ

(.^ B F - 6
I

10 1

w _z
10

;^ - 3
^ 1-0

- 4
10

10
10-1 1 101 102

i• Fig. 5:
FCD

Dimensionless time to the


end of the bilinear flow
period versus dimensionless
fracture conductivity,
constant rate case (Ref. 1).

(13)
FCD > 3: (tDxf)ebf _ 0.1
2
FCD

_1.53
1.6 < FCD < 3: (tDxf)ebf - 0.020S(FCD - 1.5) (14 )

4.55
F CD < 1.6: (t Dxf)ebf ^ (FCD 2.5) 4 (15)

For the case where FCD > 3, the dimensionless pressure


at the end of bilinear flow is


GBF - ?
l . 3 <7
(P^^^cl^l^ = l 7;^i

Therefore,

1.38
(17)
FCD ^ b£
(pD e

and,

= _ 1965.lqT
^+CD . (18)
kh [m(pi) - rn(pwf)l
.lebf

B. Constant Formation Face Pressure


The relationship between (tDxf)ebf and FCD is presented
graphically by Fig. 6? This relationship can be approximated
by the following equations:

-2
x 10 (1J) ^
FCD > 5: (tDxf)ebf - 6.94
F^Ii

2 < FCD < 5: See Fig. 6

0.5 < FCD < 2: (tDxf)ebf - 1.58 x 10-3FCD1'6 (20)

For the case where FCD > 5,

1 1.40
( 21)
q D) ebf = F CD

Therefore,

FCD = 1.40(qD)ebf (ZZ)


GBF-8
• 10
CONSTANT PRESSURE
PRODUCTION

=5
44p
Q)

4-1
103
+J
LJ

FRACTURE
TIP
i •
164 RESE R

'1KELL FRAI
1
r xf
^
i
^
105
- L
io'' I z•s 10 102

FCD

Fig. 6: Dimensionless time to the end of


bilinear flow for constant pressure
production (Ref. 2).


GBF-9
and,

ICD
1.988Tcl cbE
(23)
J
kh[m(pi) - m(pwf)]

IV. ANALYSIS OF BILINEAR FLOW DATA

The conventional analysis of bilinear flow data requires


two plots -- A log-log plot of the appropriate rate or
pressure function versus t, and a cartesian plot of the
appropriate rate or pressure function versus t°.

A. Gas-Constant Rate
The following procedure can be used to analyze bilinear
flow data for fracture conductivity and fracture length.
When rate is constant:

1) Make a log-log plot of m(pi) - m(pwf)


versus t.

2) Determine if any data fall on a straight
line of quarter-slope.
3) If there are any data with a quarter-slope
in Step 2, plot m(pi) - m(pwf) versus
t° on Cartesian paper and identify those
data which form the bilinear flow straight
line.
4) Determine the slope, mbf, of the bilinear
flow straight line.
S) Using the slope, mbf, from Step 4, compute
the fracture conductivity, kfw, using Eq. 6:

2
kfw = 444.6qT (24)
mb f1i (^uctk) °

It should be noted that this calculation can


only be made if k is known from a prefrac test. •
GBF-10
G) If the bilinear Clow straight line ends
and the data rise uLove the stT'alb'I11: li.ilc,
^ cletermine the value of Am(p) , i.e. ,
[Am(p)] ebr, at which the line ends. The"
from Eq. 18, FCD can be computed as

196S.Iq'I' (18)
^ CD -
kh[m(pi) - m(pwf)I ebf

With FCD known, the fracture length can


be computed us-ing }:q. 3:

kfw
xf = -^- (25)

It should be noted that Eq. 18 assumes


FCD > 3. If enough data are available
beyond bilinear flow, a type curve match
^ should be attempted to verify that this
is true.

B. Gas-Constant Pressure
When formation face pressure remains constant during
a test, the following procedure can be used to analyze the
bilinear flow data for fracture conductivity and fracture
length:

1) Make a log-log plot of 1/q versus t.


2) Determine if any data fall on a straight
line of quarter-slope.
3) If any data form a quarter-slope in Step
2, plot 1/q versus t4 on cartesian paper
and determine the slope, mbf, of the
bilinear flow straight line.
4) Using the slope, mbf, from Step 3, compute
the fracture conductivity, kfw, using Eq. 10:


GBF-11
J-w
493.6T
^ 12
Lm})i^}1(^uctk) ''[m(1) i)-lit (pwf)]J

5) If the bilinear flow line ends and the data


rise above the straight line, determine the
value of q where the line ends, i.e., aebf'
Then, from Eq. 23, FGD can be computed as

- 1988Tqebf
F^D = (23)
kh[m(pi)-m(p,,f)]

With FCD known, the fracture length can be


computed using Eq. 25:

kfw
xf = kF (25)
CD

Equation 25 assumes FGD > 5; accordingly,


if enough data are available beyond
bilinear flow, a type curve match should
be attempted to verify that this is true.


GB F: -12
REFERENCES

is Cinco, H. and Samaniego, F.: "Transient Pressure


Analysis for Fractured Wells," J. Pet. Tech. (Sept
1981) 1749-1766.

Bennett, C. 0., Reynolds, A. C., and Raghavan, R.:


"Performance of Finite Conductivity Vertically
Fractured Wells in Single-Layer Reservoirs," SPL•'
11029, presented at the S7th Annual Fall Technical
Conference and Exhibition of Society of Petroleum
Engineers, held in New Orleans, Louisiana, Sept.
26-29, 1982.


G E3F* - 13
^ Chapter 8

INJECTION WELL TESTING

The performance of injection wells is of major


importance in waterflood operations and in tertiary flooding
operations. Accordingly, it is important that a thorough
testing program be used before and during injection to
determine the condition of the reservoir adjacent to each
injection well, and to monitor the performance of the
injection wells. Various injection well tests are commonly
used for the following reasons:

^ • Determine effective reservoir permeability


to the injected fluid; this information is
needed for injectivity and production
performance predictions.
• Check for formation damage; wells should be
tested on a regular schedule to check for
formation damage due to plugging, clay
swelling, precipitate formation, etc.
Formation damage reduces near-well perme-
ability and causes a corresponding
reduction in injectivity.
• Estimate average reservoir pressure; this
information can be used to determine if
water is being lost to thief zones, and
to plan additional enhanced oil recovery
projects.
• Measure formation parting pressure; this
pressure is needed prior to, and during,
the injection project to permit maximum
possible injection rates without fracturing
the formation.
• Detect fractures and estimate fracture
length to properly design and monitor
flooding operations.
In general, the major objectives of an injection well
testing program are to detect formation damage so it can
be minimized, maximize pressure difference between injection
and producing wells, and make sure the formation parting
pressure is not exceeded. A testing program used by a major
petroleum company to accomplish these objectives has been
described by Robertson and Kelml
Injection wells are generally tested using falloff or
injectivity tests. The pressure falloff test is used almost
exclusively and, accordingly, will be emphasized in this
discussion. Injectivity tests are seldom used but can be
important in special applications. Most injection wells are
water injectors. Consequently, water will be used as the
basis for discussing injection well tests.

I. PRESSURE FALLOFF TEST IN UNIT-MOBILITY RATIO RESERVOIRS

The most commonly used injection test is the pressure


falloff test. It is run by shutting in an injection well and
recording the pressure response of the well as a function of
shut-in time. Falloff testing is analogous to pressure build-
up testing in a producing well. The theory used to analyze
falloff data contains the assumption that injection into the
well is at a constant rate q for a time t prior to conducting
the falloff test. This idealized rate history is depicted by
Fig. 8.1. When significant variations in rate occur before
shut-in, it is possible to apply the multi-rate analysis
methods previously discussed in Chapter 4.
When fluids are injected into a reservoir, it is common
for two or more fluid banks to be present. The existence of
these banks must be recognized and accounted for in the
analysis of pressure data. When the mobility ratio between
injected and reservoir fluids is approximately unity, the
analysis is straight-forward. This is because the reservoir
behaves as if it contained only one fluid of constant mobility.

8-2
• SHUT- IN

INJECTING
Q

-^-
tp(ot=0)

-•' pwf (At=0)

At

• tp(At=0)

Time

Fig. 8.1: Ideal rate history and pressure


response for pressure falloff
- testing.

However, when the various fluid banks differ significantly


in mobility, the analysis becomes difficult, and sometimes
impossible. The unit mobility ratio system will be considered
first.
When mobility ratio is effectively equal to unity,
pressure falloff tests are analogous to pressure buildup
tests conducted in producing ivells. The assumption of unit
mobility ratio will be satisfied under two conditions:


8-3
1) Injected fluids and in-situ fluids have
approximately the same mobilities. For
oil-water systems, to which this
discussion will be restricted, this means
that (kw/uw) = (ko/uo) •

2) Injected and in-situ fluids have
different mobilities, but injection has
occurred long enough that the outer
radius of the injected fluid bank is
sufficiently removed from the injection
well that the falloff test will not
investigate beyond this radius. In other
words, if the test is sufficiently short
that the pressure transient remains
within the first bank during the test,
banks further removed from the test well
will not affect the pressure data.
The outer radius of the water bank can
be determined by material balance to be

TrrW2 bh^ASw
W. (8.1)
5.615

where: Wi = cumulative water injected, bbls


rwb = radius of water bank, ft
h = formation thickness, ft •
AS w = increase in water saturation
in the water bank (can predict
from frontaZ advance2 calculations).

Therefore, the radius of the water bank is

5.615^ti`. 2
i (8. 2)
rwb = Th4 1^11 5`"

If the pressure transient created by the 0 1


pressure falloff test is assumed to be
radial in geometry, the radius of investi-
gation of the falloff test during time At
can be approximated using Eq. 3.14, i.e.,

rd = 0.029 k^t z (8.6


[ct]

8-4
where: k = effective permeability, md
At = shut-in time, hrs
• u = water viscosity, cp
ct = total reservoir compressibility
.
in the water bank, psi_1,

If rd < rwb, the test is influenced only by


the water bank and the assumption of a unit
mobility ratio is valid.

The pressure behavior at the formation face during a


falloff test can be described by the equation

t +At
pws = p + m log P At (8.4)

= - 162.6qBu (8.5)
where: in
kh

q = injection rate (use negative number), STB/D


El tp = time of injection prior to shut-in, hrs
_ cumulative injection since last shut-in
constant injection rate prior to shut-in
At = shut-in time, hrs
= pressure measured during test, psia
pws
p* = false pressure (equal to pi for an
infinite acting system), psia
B = formation volume factor, RB/STB
k = effective permeability, md
h = formation thickness, ft
}S = viscosity, cp.

Equation 8.5 indicates that a plot of p`as versus


log [(tp+-,It)/At] will yield a straight line of slope m; this
is illustrated by Fig. 8.2. Note on Fig. 8.2 that the
abscissa scale increases from right to left; this means that
At increases from left to right. Although the slope of the

• curve appears to be negative, it is actually positive because


of the reverse scale.

8-S
• I_--- IVELLBORE 5TOR.aG1;
• •
0 0
0 SLOPE

• /
P*

i^.

/ •
BEGINNING OF 0
INTERFERENCE

1()' 103 102 I() 1

t '}At
p
At

Fig. 8.2: Horner plot of pressure falloff


data.

1. Permeability
Using the slope o.f the Horner falloff plot, permeability
can be computed by rearranging Eq. 8.5 as

k 162.6qBu (8.6)
mh

This value of k represents the effective permeability to


water in the water invaded portion of the reservoir. Again,
you are reminded that injection rate, q, is used in all
calculations as a negative number.

2. Skin Factor
The skin factor equation can be developed in a manner
analogous to that previously presented for buildup testing,

_.e.,
8-6
1
s= 1. 151 'pwf ("t=0) - plhr log + 3. 23 . (8.7)
^ m g Ouctr2w
^

:Again, it is emphasized that m is a positive number.

3. Flow Efficiency
The pressure loss due to skin is:

a = - 141.2aBu s. (8.8)
ps kh

The effect of this pressure loss on injection rate can be


expressed in terms of the flow efficiency, E, as

pwf(Lt-0)
PR Aps
E= . (8.9)
PR pwf (At=0)

• If a well is stimulated to remove the damage indicated by


Aps, the injection rate after stimulation can be approxi-
mated as

1 (8.10)
aafter stimulation E "before stimulation'

The flow efficiency in Eq. 8.10 should be based on


sdam, not s. Further, if pR is not known, p* can be used
as an approximation to Pp in Eq. 8.9; however, unlike
production well testing, p* may be a poor approximation

to PR-

4. hliller-Dyes-Hutchinson Method
The Miller-Dyes-Hutchinson method' can also be used to
analv:e falloff data provided tp is significantly larger
than the maximum shut-in time. The equation on which this

8-7
method is based is

m log At lS•0
pws plhr -

where m is the slope of the straight line on a plot of


pws versus log At. Once the slope is determined, perme-
ability, skin and flow efficiency can be computed using
Eqs. 8.6 - 8.9. Note that the slope of the IN1DH plot is
a negative number; however, it will be necessary to use
it as a positive number in Eqs. 8.6 and 8.7.

S. Average Reservoir Pressure


The determination of volumetric average reservoir
pressure in a waterflood project is generally more difficult
than in comparable production system because of the nature
of the pressure distribution between injection wells and
producing wells. A producing well surrounded by other
producers effectively has a no-flow drainage boundary; this
was illustrated clearly by Figs. 3.4 and 3.5 in Chapter 3.
However, an injection well surrounded by producing wells
is best approximated by a constant pressure boundary. The
pressure distribution between an injection well and
producing well is depicted by Fig.'8.3. When an
injector is adjacent to both producers and injectors, the
pressure distribution can be very complex3 In the absence
of detailed knowledge about the pressure distribution
within the area of influence of an injection well, it is
recommended that a constant pressure boundary be assumed.
Average pressure is generally not the primary objective
of an injection well test. However, this information can
be useful in monitoring flood performance, or in planning
additional enhanced oil recovery projects.


8-8
PRODUCTION INJECTION

/ 1
CONSTA.N'f PRESSURE
BOUNpARY

Fig. 8.3: Schematic showing pressure


distribution between adjacent
injection and producing wells.

• a. Alatthews -Brons -Hazebroek Method


The MBH methods requires that we first determine p*.
The false pressure, as in the case of buildup tests, can
be determined by extrapolating the semilog straight line
on the Horner plot to a time ratio of unity. This is
illustrated by Fig. 8.2. In new wells, or in wells where
injection time is short, p* is approximately equal to the
reservoir pressure at the time injection began. For other
wells, p* must be corrected to pR.
For producing wells, PR is related to p* by the
equation

_ mpDMBH (8.12)
pR p * + 2.303

Graphs expressing the functional relationship of Eq. 8.12


^ were presented in Chapter 5 for various drainage geometries
and well locations. Note, however, that the t1,4BH graphs

8-9
developed for producing wells will not work for injection
wells because of the differences in pressure distribution
and boundary conditions. This is shown clearly by Matthews
and Russel 16.
For the case of liquid-IiZZed five-spot waterfZoods,
Matthews and Russe116 developed a MBH-type correlation that
relates PR and p*. This correlation is presented in Fig. 8.4
as a plot of pDMBH versus tDA where

2.303(pR - p*)
(8. 13)
pDMBH m

0.0002637kt
and tDA = ^ucta p . (8.14)

The area A in Eq. 8.14 is the area associated with the in-
jection well; this is equal to one-half the five-spot
pattern area. The procedure for using Fig. 8.4 is the
same as for previously discussed MBH graphs. Please note
again, however, that this plot cannot be used before fillup,
or for systems where the ratio of bank mobilities differs
significantly from unity. Similar correlations for other
flooding patterns are not available.

b. Hazebroek-Rainbow-Matthews Method
Before fill-up occurs, it is best to use the method
proposed by Hazebroek, Rainbow and Matthews'. This method
is basically the same as the Muskat Method which was
previously discussed for production wells. Plots of log
(pws - pe) versus At are made for various values of pe
until a value of pe is found that will give a straight
.

line at large values of At. This procedure is illustrated


by Fig. 8.5. The pressure pe is actually the pressure at
the outer radius of the oil bank, but is a good approxi-
mation to PR' This method is subject to the same
limitat:ions previously discussed for the Muskat method.

8-10
• • •

^ s

ac

O
M
3
W

F-^
F--^

^o : v .

0.0002637kt.
tDa ^uetA

Fig. 8.4: A4I311 type correlation for a five-spot waterflood pattern


(After Ref. 6).
10'

kX,
0 1,01V P
S Correct p.
e •
102

10

1
't, hrs

Fig. 8.5: Muskat-type plot to determine p


before fillup in unit mobility e
waterflood systems.


c. Miller-Dyes-Hutchinson Method
Average pressure can be estimated from the MDH plot
in the same manner as for buildup tests. This method
requires that PR be read directly from the semilog straight
line at a shut-in time equal to

1688^uctke
(^t) ^ k (8.15)
R

Before gas fillup, xe can be approximated as the radius


of the oil bank, rob, which surrounds the injector, i.e.,

S . 615IV. 2
_ i (8.16)
x e - r ob Trh^Sg
i


8-12
where
S^Ji = gas saturation at the beginning
of the flood, fraction.

After fillup,

,1 2 ^
= L2i (8. 17)
Xe

where A is the area served by the injector. This area


can be approximately defined by a line which passes
through adjacent producers and points half-distance to
adjacent injectors. Figure 8.6 illustrates this area for
an irregular flood pattern. For a symmetrical five-spot
pattern, this area would form a square.

• A ^
A A

---^----^^ ^_ 1 _^

i *. ^•^'.•" • •i
A
^ • ' • ./
-----^-'-^c,,

i
i
i

Fig. 8.6: Flood pattern showing approximate area


served by an iniector.
0

8-13
6. Wellbore Storage
Weilbore storage, when present, affects early-time
falloff data in basically the same way it affects pressure 0
data in producing wells. Storage is caused by the flow of
water from the wellbore into the formation after the well is
shut in. Data completely controlled by storage will form a
unit-slope line on a log-log plot of [pwf(ot=0) - pws] versus
At; further', the end of wellbore storage will occur at
approximately SOAt*, where At* is the time where the unit-slope
ends.
It is not uncommon for a falloff test to experience
a change in wellbore storage8 after the test begins. This
will occur in any well which goes on vacuum during the test.
An injection well will go on vacuum when the bottomhole
pressure decreases to a value which is insufficient to
support a column of water to the surface. Prior to going
on vacuum, an injection well will experience storage due to
water expansion; after going on vacuum, the storage will be
due to a falling fluid level. This change in storage will
generally exhibit itself as a decrease in the rate of pressure
decline. Accurate pressures are obtained only after the
storage effect ends. If storage effects- are severe, it may
be necessary to use a bottomhole shut-off tool.

II. PRESSURE FALLOFF ANALYSIS IN NON-UNIT


MOBILITY RATIO SYSTEMS

When the conditions necessary for a unit mobility ratio


system fail, the reservoir must be treated as a composite
system having fluid banks of different mobilities. An ideal
composite reservoir with radial fluid banks is illustrated
by Fig. 8.7. Prior to gas fillup, three separate mobility
zones will be present. After fillup, and before water
breakthrough, only two zones will exist. Finally, after
water breakthrough, only the water invaded zone will be

8-14
present; the saturation distribution within this zone,
however, will not be uniform.

, UNAFFECTED REGION


Fig. 8.7: Idealized composite reservoir
(After Ref. 3).

1. Two-Bank System
The assumption of a two-bank system is applicable if
the reservoir is liquid filled, or if the maximum shut-in
time of the test is such that the radius of investigation
of the test does not exceed the outer radius of oil bank.
The ideal behavior of a falloff test in a two-bank system
is illustrated by Fig. 8.8.
The slope, ml, of the first line will yield the skin,
s, and the effective permeability to water in the flooded
zone. It is commonly believed that the slope of the second

• line, m21 will yield the permeability of the oil bank

8-15

1 V 1 V 1 V i

t +At
_p
At

Fig. 8.8: Pressure falloff behavior in a


two-bank system.


according to Eq. 8.5; however, this is not necessarily true.
In fact, it has been shown by Kazemi, et al?, that the slope
m2 can be used in Eq. 8.S to determine oil zone mobility only
if rf2 > lOrfl and (^ct)1 = (^ct)2.
A method developed by Merril, et a110, can be used to
determine (k/ P)2 and rfl subject only to the conditions
that rf2 > lOrfl, and that (^ct)1 and ("t)2 be known. Figure
8.8 illustrates a Horner plot of falloff data and the infor-
mation needed from this plot, i.e., ml, m2, and Atfx.
The mobility of the first zone is computed as

k162.6qBw
= - - m^- (8. 18)
1]l 1


8-16

I
To compute the mobility of the second zone requires knowledge
• of m2/ml and (^ct)1/(^ct)2. With this information, we can
determine

al (k/u)1
(8.19)
x2 (k u 2

from Fig. 8.9; then,

[kl (k/u) 1
u 2 = 1 ^2 ( 8.20)
J
The radius of zone 1 can be computed according to the
relationship

0.0002637(k/u) At
r = 1 fx
(8.21)
fl ^ct 1 AtDfx


where AtDfx can be obtained from Fig. 8.10.
For those wells where only a small volume has been
injected, Ref. 7 gives an equally good method of analysis.

2. Three-Bank Svstem
When falloff data are affected by three zones, it
is not possible with conventional methods of analysis to
determine anything about the third zone10. The first two
zones can be analyzed using previously described methods.

is
8-17
Iq

12

IC
/4
0

\ \0
N
^
e
0
a
ac
w °
a
0J
N
4

IQ > 1O
rf l
0
0 2 4 6 S 10 12
MOBILITY RATIO, XI/X2

Fig. 8.9 Relationship between mobility


ratio, slope ratio, and storage
ratio (After Ref. 10).

lo'

( C )I
C 6
((Oct)?
4
o 3-30
O 1.0
2
t= O 0.80
0 0.58
Q o C 0.30
~ 6
v 0.10
U
W
W {

~ 2
2
THI S LINE IS
USEFUL FOR
fN I ^ AIR /N3ECT/ON
W
J
Z 6
0
4
N
Z
W r
^^ > I 0
N
G
10-1
2 4 6 e 2 4 6 e 2 4 6 9
10'' I 10 102
SLOPE RATIO, rn2/mj

Fig. 8.10: Dimensionless intersection


time, tDfx, for two-zone
composite reservoirs (After 11
Ref. 10).

8-18
• Example 8.1: Analysis of a pressure falloff test.

Problem. Pressure falloff data, and other reservoir


"characteristics, are presented for a water injection well'
-located in the Forties Field of the North Sea. All
pressures are adjusted to the midpoint of the perforations.

Wi = 32,000 bbls
q = -5162 bbls/D
AS = 0.59
w
h = 27.6 ft.
= 0.26
pwf(At=0) = 4310 psia
T = 200 'F
r = 6 in
w
uo = 0.8 cp

• uw
r
e
= 1600 ft
= 1.15 cp

Total compressibility in water bank = 9.8x10-6 psi 1


Total compressibility in oil bank = 12.8x1.0-6 psi-1
Depth to midpoint of perforations = 6,585 ft

t +ot
Ap, psi
At, hrs p At Pwsl psia
0.00042 3.54x105 4231 79
0.021 7.09x103 3758 552
0.038 3.97 3638 672
0.054 2.75 3568 742
0.071 2.10 3518 792
0.088 1.70 3480 830
0.104 1.43 3451 859
0.121 1.23 3430 880

q 0.204 7.30x102 3351 959

8-19
0.288 5.19 3312 998
0.371 4.02 5290 1020
0.454
0.621
3.29
2.41
3276
3253
1034
1057

0.788 1.90 3249 1061
0.954 1.57 3240 1070
1.454 1.03 3225 1085
1.954 7.71x10 3217 1093
2.454 6.16 3205 1105
2.954 5. 14 3197 1113
3.621 4.21 3191 1119
3.871 3.94 3175 1135

a) Compute the effective permeability to water in the


water bank.

b) Is there any evidence of formation damage?

c) Based on the amount of water injected, compute the


radius of the water bank. •
d) Is there any evidence of banking effects in the falloff
data? If so, does the time at which these effects occur
agree with your knowledge of the water bank radius from part
cc)?

e) Did this well go on vacuum during the test?

Solution. A log-log plot of the test data, while not presented,


showed no effects of wellbore storage or fracturing. A semilog
plot of the data is presented in Fig. 8.11.

a) Two straight lines are formed by the falloff data in


Fig. 8.11; since wellbore effects are negligible, it is
concluded that the first line represents the water bank.
The slope of this line is 348 psi/cycle; accordingly, from
LJ

8-20
0 3800

3700

3600

3500

3400

3300

3200

3100
104 103 1 O2 1 O
Eq. 8.6 ,

kl _- 162.6 m^^ h
0
1

k (162.6) (-5162 STB/D) (1.0 bb1/STB) (1.15 cp)


1 (348 psi/cycle) (27.6 ft)

k1 = - 100.5 md.

b) From the first straight line at a time ratio corresponding


to At = 1 hr, Plhr = 3110 psia. The total skin factor is, from
Eq. 8. 7,

[Pwf(At=o) - lhr _ log k


s= 1.151 + 3.23
m ^Pctr2

4310-3110 ^
s = 1.151
348

100.5 + 3.23
- log -
(. 26) (1. 15) (9. 8x10-6) (0.5) Z J
s = --1.7.

It appears, therefore, that this well is slightly stimulated.

c) If it is assumed that water flows radially away from


the injection wellbore, the radius of the water bank can be
approximated using Eq. 8.2, i.e.,

5.6156d. 2
_ i
rwb 1rh^ASw


8-22
i,
r = (5.615)(32000) 2
• 1"b Tr) (27.6)(0.26)(0 .59)

rwb = 116 ft.

d) A second straight line begins on. the semilog plot at a


Horner time ratio of approximately 250. This line is
probably caused by the oil bank. The time ratio at which
the two straight lines intersect is

t +At
= 460
p At

At = 0 . 32 hrs .

The approximate distance moved by the pressure transient


in this period of time is, from Eq. 8.3,

1-
rd = 0.0 29 k^t 2

12
(100.5) (0.32) 6
rd = 0.029
(0.26) (1. I5) (9.8x10 )

rd = 96 ft.

This agrees favorably with the radius of the water bank


computed in (c) and gives support to the conclusion that
the second straight line is caused by the oil bank.

e) A well goes on vacuum when the bottomhole pressure is


no longer high enough to support a fluid column to the
^ surface. If the hydrostatic gradient of the injection
water is assumed to be 0.465 psi/ft, i.e., the normal

8- 23
gradient of salt water, the well should go on vacuum at an
approx iirate hot tor^hoLe pressure equa i

(6585 ft) (0.4b5 psi/ft) = 3062 psi


(pws)vac

Since the last measured pressure was 3175 psia, it is evident


that this well did not go on vacuum.

III. PRESSURE FALLOFF ANALYSIS IN FRACTURED RESERVOIRS

Pressure falloff data from fractured reservoirs can be


analyzed using the same basic equations and procedures
previously presented for buildup testing. The conventional
method of analysis requires the slope, mLF, of the linear
flow straight line on the square root plot, and the slope,
mpR, of the pseudoradial straight line on the semilog plot. •
Fracture length can be computed as

= 0.319 mpRaB z
8.22 }
xf mLF ^cthFcor

The correction factor, Fcor' is related to the true matrix


permeability, k, and the apparent permeability from the
pseudoradial straight line, ka, by the relationship

(8.23)
k = Fcorka'

A correlation to obtain Fcor for wells with a no-


flow boundary was previously presented in Fig. 6.8.
Injection wells, however, are best approximated as having
a constant pressure boundary. Figure 8.12 presents Fcor
for wells with a constant pressure boundary!' Equation
8.22 and the correlation in Fig. 8.12 can be solved

8-24
simultaneously using an iterative procedure to determine
xe and Fcor' Finally, permeability can be computed using
Eq. 8.23.

\ \\ `^^,^ >o _

\ \\ \^ ^ ^^>o . i
oe
\ \ ^
^Q , 0 4

\ ^\ 0.02

^ 10-i
^ o 000

0 , 0 0 2 -

DIMENSIONLESS
TIME, tDt1
HORNER _
- - %I1DH

10-21 t 1
0 0 0.4 0.b 0.8 1

Xf/Xe

Fig. 8.12: Permeability correction factor for


fractured well in a constant
pressure boundary (After Ref. 11).


8-25
Type curves can also be used to analyze falloff data.
Infinite conductivity, uniform flux, and finite capacity
type curves were presented previously by Figs. 6.16, 6.19,
and 6.20 for fractured systems. These curves can be applied
to the analysis of falloff data using the same procedures
and equations previously presented for pressure buildup tests.

Example 8.2: Analysis of pressure falloff test from a


hydraulically fractured well

Problem. A 96 hour pressure falloff test was conducted in a


water injection well which has served as an injector for
several years. For seven days prior to the test, the rate
was approximately constant at 1,110 STB/D. The injection
formation face pressure at the time of shut-in was 3,713
psia. The well is located in a portion of the reservoir
which is believed to be watered out; consequently, the only
fluids present in the area influenced by the test are water
and residual oil. Using the rock and fluid properties
presented, and the falloff data, determine the effective
permeability to water, fracture length, and pressure loss
due to formation damage.

q = 1,110 STB/D
T = 104 °F
Bw = 1.0 RB/STB
ct = 7.7 x 10 6 psi
= 19%
uW = 0.81 cp (at Res. Temp)
Depth = 4,695' (4,044 ss)
Effective injection radius = 1,250 feet
h = 15 ft


8-26
LZ-8

809 SOIL 08'6 96


E
109 TITS 6S'6 Z6
L6S 9TH 8S'6 88
Z6S TZTS LT'6 t8
98S LZIS tr6'8 08
SLS 8STS ZL'8 9L •
S9S 8bT^ 6t8 ZL
6SS tSH SZ8 89
6bS b9IS 0'8 V9
82S SL TS SL ' L 09
HS .08H 8t*L 9s
LZS 98T2 S2 'L tS
ZZS 16T2 TZ'L ZS
ZTS IOZS LOL OS
90S LOZS S69 8t
DOS 60ZS 98'9 LV
2 OS OTZS 8L'9 9ti
96t LTZS 29'9 Vt
6Lt tiSZS 82* 9 Ot
69t titZS 0' 9 92
LbV 99ZS 99'S ZS
OZt 26ZS 6Z'S 8Z
662 VTSS 06' t7 VZ
ZLS IbSS Lttr OZ
9t2 z 9H 00 V 9T
802 SODS 9V 'S ZT
S9Z 8vbS 2 8Z 8
62Z VL VS Sti Z 9
96T LTSS 0Z t
IST Z9S2 TtiT Z
sL Ob92 TL0 S'0
0 SILL 0 0

zsd ` (s"'d-^r'd)=do ^zsd ' sMd siq Z ^o siu `^o


^
Solution. Log-log and semilog plots of the test data are
presented in Figs. 8.13 and 8.14, respectively. It is noted
on Fig. 8.13 that the initial data points form a half-slope
line, thereby indicating the probable presence of a fracture
in the reservoir. Those points which fall on the half-slope
line also define a straight line on the square root plot
presented in Fig. 8.15.
The slope of the pseudoradial semilog straight line is

mpR = 360 psi/cycle;

accordingly, the apparent permeability is computed using


Eq. 8.5:

k 162.6qBu
mh

k (162.6) (-1110) (1.0) (0.8?)


a 15

k = 29.1 md.
a

This value must be corrected to account for the presence


of the fracture.
Fracture length and true permeability can be determined
simultaneously using Eq. 8.22 and Fig. 8.12. The slope, mLF'
from the square root plot is

mLF = 96 psi/hr -2

Assume xf = 200 ft:

Fracture penetration, xf/xe, is

xf _ 200 = 0.1b.
x 17 S0 ^

8-28
Ol • •
,i; 4:
1o3 _LL
L Ti-

r ^ t ^t{1,17! t 1, r t!^ 7l
1,f ht
i
^--'-L -'^-^t-C . , t i , , + ;- ..• , . ^, :.
t'
.^ , i
t
^-{-
_ r I .
: ^ 1 {}I }: ^
1

, j;tt r„ 111t, , . +
^^

r
: I^I i ^, •'
I.l^ ^'. L^
A'l

1
,•
, T7 _
ii ,
I {^ ,,. , l;
^
a {.^ ^ , 1ai
}^
t`
, ^ ^ t , t `,,^^ ^^,. ^ , ^, ^ , ',
r: t r ^ ^ i :,.: 1 r i 1 r ^ I r ( • i. 1 3 j1
A
I ` !r^'
^ ^^ ^ ^ , 1 . 1 i. I ^ .II 1 • `1 ; I _ . '. ^_ i t ., I I . ; i ^ , .^. ^^ ^f .^i { l ^ ^{" 1 I ^^ ^;,
I
I ( I ^ ; '.
Half-s lop e line it
1 II, ,i
^ ;r l ft, ,l :
it ^1j ^
T.
;
'^^ ^, Approximate end
102 1fi .i
of linear flow hn
I I i t i
.
'
^
__ ♦ t. i , I i :,
rt'^l +^ ^ 111 t1 ' fl t; l^_-7 ^_i 1 i Il^
[`J '^ ^Jt ^,'I'^' ^I ,I,'
(4-4
t ' 1 il 1t : ; :r , ^'f ^^:I• ^
^'it ^ i ! : 1 1 1',' '', ! '? ; ' 1 ^ (I^ r
F
,,

t 1 I^, , ^ r a, ^
} I t` t 4+ {^ t ^ , 1- } { t1 ti ll ^i
t
It i ^ • - C `^
? ^ r ir , 1 ,a #I -t , h ,
I '

Il ^- ,.i , , ,i 3} r .t ,; ;.. ,.,, i+`r ,i I . : { ., ; ,•


{' 1 tl
r ^ , , .^
ri I4 'nt ' t
j
f ,
•i ,,I

^I ;', ^,, .,t, E, , • t Y


,I. l ,
I ; I;1^ ^^
^
jr ,^ , ^ ; fi^ , ^
^
.,
lo 1 1 I
io-1 10 102
Gt, hrs

Fig. 8.13: Log-log plot, Ex. 8.2.


3700 ^.,.:
.^^ ^^;^,.; t,.. , ^ ^ •1i1^t^{I I^II^111^ it^^^^l^fl, ^^.

1 1 ^
4 ^+ ; 1 ^. ,. . ,;; :;;i..,tr .{ ' ` ^^^^ i t^^^ ltilil ^hi ►:: ^
3600
SLOPE = m^,^ = 3G0 PSI/CYCLE
...,... .... .... .. .:.
....... ,. .. .,.
,
.. . ,,.i ^
3500

. . . ... ..:...... . .. .. . !11^^


.rct.,
3400
. , ,
. .. !,; .
,. i 1i1 ; ;• ^^^,
0

3300
:: .. .^.I :^4
;.

r^ --^-.. ...

3200
... . ... ,. , , . ,-
... ;t ,
i
,... , . ..,,,; .. :11 f I
. .:. .. ......:.. .^
3100
• ;^
lU 1 10 10`
At, hrs

Fig. 8.14: Semilog plot of test data, Ex. 8.2.

^ ^ ^
^ 3640

3600

3560

3520

3480

3440

3320

3280

3240

3200

3160
(^ 1 2 3 4 5 6 8 9
,/At, (hr)

Fig. 8.15: Square root plot, Ex. 8.2.


8-31
From Fig. 8.12,

Fcor
1.0. E
From Eq. 8.22,

m RqB ?
_ 0.319
xf mLF ^cthF cor

_ 0.319 _ (360)(-1110)(1.0) z
xf 96 (0.19)(7.7x10-6)(15)(1.0)

xf = 448 ft.

Since calculated and assumed values of xf are not equal,


this procedure must be repeated.

Assume xf = 460 ft:



xf = 0.36
x
e

Fcor = 0.95

xf = 460 ft.

Therefore, it is concluded that

F = 0.95
cor

xf = 460

and that

^
8-32
^ k = Fcorka

= (0.95) (29.1)

k = 2;'.6 md.

The pressure loss due to skin, exclusive of sf, is


obtained from the square root plot; from this graph,

pint - 3708 psia

and Ops = pwf(`^t=0) - Pint =-';13 - 5708

Aps = 5 psi.

If it is assumed that other skin effects are negligible,


this value represents the pressure loss due to damage.
Accordingly, damage is negligible in this well.

IV. INJECTIVITY TESTS

An injectivity test is run by injecting fluid into a


well which was previously shut in. Formation face pressure
is recorded as a function of injection time. This test is
analogous to a pressure drawdown test in producing wells;
however, pressures will increase during this test whereas
they decrease during a drawdown test. The ideal rate and
pressure history associated with an injectivity test is
illustrated by Fig. 8.16.


8-33
SHUT IN

:..,

INJECTING

.-. ,..]

TIME

Fig. 8.16: Ideal rate and pressure history


associated with an injectivity
test.

The formation face pressure during constant rate


injectivity can be computed using the following equation 3:

plhr + m log t (8.24) 1


pwf

where
162.6qBu
m kh (8.25)


8-3)4
Recall that q is a negative number for injection.
Equation 8.24 indicates that a semilog plot of pwf
versus t will yield a straight line of positive slope, m,
during transient flow. This plot is illustrated by Fig. 8.17.
Once the slope is determined from the semilog plot, perme-
ability and skin can be computed using the following equations:

k = 16 (8.26)
mh

s= 1.151 [PlhrPi log k 2 + 3.23 . (8.27)


^ucrw
t

An example of an injectivity test is presented by Earlougher3

• SLOPE
7=ffl

-- --- •


1•

• I
• •

10-' 1 10 102

t , hrs

Fig. 8.17: Semilog plot of injectivity


data.


8-35
V. STEP RATE TESTING


1. INTRODUCTION

The step rate test (SR'I') is the primary method used to


measure the maximum pressure at which fluid can be injected
into a formation without fracturing the reservoir rock.
Commonly referred to as the formation or fracture parting
pressure (FPP), this is the pressure that will initiate a
fracture in an unfractured well or extend an existing fracture.
This is the same pressure that is commonly referred to in
the hydraulic fracturing literature as the fracture extension/
propagation pressure. The formation parting pressure is
generally greater than the "closure pressure" (defined as the
pressure required to initiate the opening of an existing
fracture) determined from pump-in/flowback tests. 12,13
Knowledge of the formation parting pressure is very
important in designing and operating efficient waterflood or
tertiary recovery projects. The fracture or fracture extension
created by exceeding the parting pressure can cause premature
breakthrough and can significantly reduce areal sweep
0f f':iciency i f" the Cr;leture iti oriented tow,']r•cl procluctiOil
wells. If the fracture extends out of zone, the costly loss
of injection fluids can occur. On the other hand, injecting
at pressures substantially below the parting pressure will
result in lower injection rates than are safely possible and,
consequently, low oil production rates. Generally, injection
policy is to operate injectors at a predetermined incremental
pressure below the last measured parting pressure; a common
pressure increment is 50 psi.
Step rate tests have been used in the petroleum industry
for over 40 years and today still represent one of the most
commonly used pressure transient testing procedures. It is
surprising, therefore, how little technology is currently
available regarding the proper design, implementation and
analysis of these tests. This is reflected by the lack of

8-36
references in the literature and by the simplistic semi-
empirical methods currently used by most engineers when
designing and analyzing SRT's. Understanding SRT's is
made difficult by the fact that the test involves multiple
rate flow while simultaneously fracturing the formation.
"'A thorough understanding of this subject must include an
,-understanding of fracture mechanics, fluid flow in
fractures, and multirate flow analysis.
The purpose of this discussion is to document the
state-of-the-art for step rate test design and analysis.
Emphasis will be placed upon the fluid flow aspects of the
problem as opposed to considering issues involving rock
mechanics.

2. TEST PROCEDURE AND ANALYSIS

A step rate test is conducted by first shutting in the


well or by stabilizing the well at a reduced constant rate.
If the well is shut in, ideally it should be shut in long
enough for the bottomhole pressure to approach the static
reservoir pressure. Alternatively, if the well is stabilized
at a reduced constant rate, this rate should be maintained
constaiit sufficiently long to achieve a steady state or
pseudosteady state condition. Following stabilization of the
well, the rate is increased in a series of constant-rate
steps, all steps being of equal length. Injection rates
and pressures are recorded for each step. The pressure and
rate history for an ideal step rate test is qualitatively
depicted by Fig. 8.18.

A. Pressure Versus Rate Plot

Historically, the method most commonly used to analyze


step rate tests is based on a plot of injection pressure
versus injection rate. The use of this method is discussed
by Felsenthal14. While this is still the most commonly used
^ method today, it will be shown in subsequent discussions that
better technology currently exists.

8-37
I ^ q5
q6
•i
I I
^
q4
rr q3
c
0

^ q2

t1 t2 t3 4 . `6
`5
Time

^
•i
^^
^
^^
^^
^{

At At
Time
Fig. 8.18: Rate and pressure history of an ideal
step rate test.


8-38
The p versus q plot is originally based on theory which
assumes steady state flowl". If a well has not been previ-
ously fractured, then at pressures below the parting pressure
the step rate test will be described by the radial form of
Darcy's law. For single phase, radial, steady state flow,
the injection pressure, piwf, is related to injection rate,
q, according to the equation

hk(pe piwf)
q = 0.00708 (8.28)
r
uB (ln re + s)
w

or

r
piwf = pe - [141.2 kh (In r^ + s)^q (8.29)
w

Recall that injection rate, q, is assigned a negative


value in all equations. If steady state flow is achieved
during each constant rate period of a step rate test, the
distance to the outer boundary, re, and the pressure at
the Outer boundary, pe, w i l l remain constant. There fore,
according to Eq. 8.29, a linear relationship will exist
between pressure at the end of each constant rate
injection step, piwf' and the corresponding injection
rate, q, for the pre-parting portion of the test. If steady
state flow is achieved during each injection step, the slope
of the straight line will be independent of the time step
size. Figure 8.19 illustrates a cartesian plot of p versus
q as predicted by Eq. 8.29.
When pressure at the formation face exceeds the parting
pressure, the resulting fracture causes the fluid conductivity
of the formation to increase. This results in a decrease in
the slope of the p versus q plot. The point at which data
deviate from the initial straight line defines the parting
pressure; this is illustrated by Fig. 8.19. A common practice

8-39

p P^ P
PRS^^G

FI'I' ^
------- +

z
a
^ I I I
U
F-D
^ ^ti^^ I I ^

al Q2 q3 q4
INJECTION RATE
a5 c{6 q7

Fig. 8.19: interpretation of step rate test using
p versus c{ plot.

in industry is to draw a second straight line through the


late (post-parting) data and to define the parting pressure
as the point where the early- and late-time lines intersect.
There is no theoretical basis for another straight line on
this plot after parting occurs; in fact, subsequent discussions
will show that the slope should continually decrease after
parting occurs. Attempting to draw a second straight line
through the post-parting data generally results in a parting
pressure which is too large.
The length of each constant-rate period is not adequate
in most step rate tests to achieve steady state flow. In
this case, the pressure disturbance created by each rate
change will not reach the drainage radius of the well, re; •
8-40
instead, the radius of investigation, rd, will depend upon
the length of the constant rate period. Equation 8.29 in
this case is modified by replacing re with rd. We learned
in Chapter 2 that radius of investigation depends not upon
level of flow rate but is determined by the length of the
flow period. Therefore, if superposition effects can be
neglected, and if the length of all injection periods are
equal., the values of rd for all injection periods should be
approximately equal. Even if small changes in the drainage
radius do occur, ln(rd/rw), is not sensitive to small changes
in the drainage radius; practically, this term will remain
constant. Equation 8.29 indicates that pre-parting data
should still yield a straight line on a p versus q plot
even if steady state is not reached during each step.
If time step sizes are small, flow will be transient
rather than steady state. In this case, the pre-parting data
are best described by the single phase, transient, radial,
liquid flow equation previously presented as Eq. 3.22.
;ti9odi_l:iecl for injection, this equation is:

piw[' - pi -q 162.61311 (log kAt - 3.23 + 0.87s) (8.30)


t 2w
cpucr

If the time step sizes, At, are equal, and if superposition


effects are ignored, Eq. 8.30 indicates that pre-parting
data should result in a straight line on a cartesian plot of
p versus q.
It is concluded that pre-parting step rate test data,
when using equal. time step sizes for all constant rate
periods, and in the absence of nonidealities, should yie'id •
a straight line on a cartesian plot of p versus q. The
intercept of this line should be equal to the stabilized
pre-test pressure. This should occur regardless of the
time step size and regardless of whether flow is steady state
or transient. It is again emphasized, however, that all

8-41
constant rate injection periods must be of equal length.
Nonidealities and problems of interpretation will be
discussed in subsequent sections.

Example 8.3: Analysis of a step rate test using the p versus


q plot

Problem: A 2800 ft. water injection well located in a


developed waterflood in the Permian Basin of West Texas
yielded the step rate test data presented below. Prior to
the test the well was shut in for seven days. The shut-in
surface pressure immediately preceding injection was 355
psia. Analyze the test and determine the surface injection
pressure at which this formation fractured.

Time q Surface Pressure


hrs bbls/day psia

0 0 355
24 100 423
48 175 46S
72 250 SUO
96 325 552
120 400 564
144 475 597
192 ? ?
216 550 610
240 625 620
264 700 641
288 77S 640

Solution: The step rate data are presented by Fig. 8.20.


It is observed that early data form an approximate straight
line which extrapolates to an intercept near the pro-test

8-42

• • •
650

(FPP)2 = 603 psia


600 - ^ --- -L==--^-
t
H ^^_ ! ^ ;- -1-^-- :
(FPP)1 = 578 psia
I-rI -I-i- t - ^-I-I- }- i ^ -=-4-i-i

V)
550

r-
0
- ^ ---+ -
^ ----
500
.r, - --^ -'^ -.
1-r--
._-1^__^^.
a)
U
cd
4-I

450

400 L
0 100 200 300 400 500 600 700 800
Injection Rate, bbls/day

Fig. 8.20: Pressure versus rate plot, Ex. 8.3


stabilized pressure; the late data deviate from this line with
a decreasing slope. This behavior is consistent with theo-
retical expectations of a step rate test when pressure exceeds
the formation parting pressure during the test.
Because of scatter in the data, it is not obvious in this
example where to draw the post-parting straight line;
accordingly, two possible interpretations are shown. While
there is no theoretical basis for a post-parting straight line,
we have fit a straight line through the late data to assist in
selection of formation parting pressure. Observe that the
post-parting straight line extrapolates to an intercept
pressure that is much greater than the stabilized pre-test
pressure. Depending upon which of these interpretations is
used, the surface injection pressure at which this formation
fractures varies from 578 psia to 603 psia. If the point of
departure from the pre-parting straight line is used, rather
than the intersection of the two lines, the parting pressure
could be as low as 550 psia.

-- ---- ^

Multiple possible interpretations are a common problem


when using I ) versus cl J 7 1 0 t . 5 . T his i s causCcl primari ly by tlic
fact that this is a semi-empirical method. While there is a
theoretical basis for a pre-parting straight line, there is
no basis for a post-parting straight line. Attempts to draw
a post-parting straight line lead to confusion as where to
place the line and may even result in the construction of
multiple post-parting straight lines for which various inter-
pretations have been proposed in the literature. This
confusion points out the need for a more exacting method of
analyzing step rate tests. This method, based on multiple
rate analysis, is presented next.


8-44
B. Multirate Analysis

The p versus q plot is the most commonly used SRT


analysis tool in industry today. This plot is normally
constructed using pressures recorded at the surface. It is
also common practice to measure pressure only at the end of
each constant rate step. If pressures are continuously
recorded, preferably downhole, the more accurate multirate
techniques presented in Chapter 4 can be used to analyze
step rate tests.
Multirate analvsis, based on the method of superpositiori,
is most commonly applied in the form presented by Odeh and
Jones16. Application of the Odeh-Jones equation to SRT
analysis has been documented in the literature3'13'1'`'1'''a
The most notable study of SRT analysis using multirate
techniques was by Singh, et all; and is the basis for most
of this discussion.

a. Odeh-Jones Equation

The Odeh-Jones equation, previously presented as


Eq. 4.20, is modified for injection as

pi piwf __162.6Bu a' a-


^a log (t-tj 1)
qn kh
n
j=1

k
+ 162.6 Bp [log - 3.23 + 0.87s^
(8. 31)
^ucrt 2w

where piwf is the bottomhole injection pressure which is


continuously measured as a function of time, t. This can
be written more explicitly in the form of a straight line
^ as

8-45
n •
p i pi.w f _
(a La?-1)
^ log (t-t--1) + b1^1R (8. 32)
qn mMR
In ^

j=1

where the slope, in MR' is

162.6Bu (8.33)
^MR - kh

and the intercept, bMR, is

k z- 3.23 + 0.87s (8.34)


bMR mMRClog
^uctw

Equation 8.31 assumes transient radial flow in the


reservoir during each constant rate period. Theoretically,

Eq. 8.31 predicts that all pre-parting SRT data will plot
as a single straight ] ine when graphed on cartesinn paper is

pi piwf )log (t-tj 1)


a versus ( a
n n

j=l

Using the slope of this line, the reservoir flow capacity,


kh, can be computed as

kh = 162.6Bu
(S• 3 S
mMR
0

8-46
and the total skin factor, s, as


s= 1.151 bM1R - log -k z + 3.23 (8.36)
L P9R t w
qUcr

Alternatively, the Odeh-Jones equation can be written


as

n q j aj
pipiwC
a kh
_ 166Bp {log q
ff(t-t 1) ) }
j=1

+ 16k }i6Bu
k 2 - 3.23 + 0.87s (8.37)
[log
^uc tw 1
or

n aj aj 1
J( )
p i pi w f (t-t } (8.38)
^ cl mMR {log j-1 qn + bMR
n
j=1

Equation 8.38, which is equivalent to Eq. 8.32, indicates


that a plot of

n Qj aj
pi piwf ( )
versus log ^(t-tj-1 an
q
n
j=1
will yield a straight line of slope mMR and intercept t) MR'

b. Understanding SR'I' Behavior Above the Parting Pressure

It is important to understand what happens when injection


pressure exceeds the parting pressure. Does the fracture
length remain constant during the entire post-parting segment
oC the SRT, or does the fracture continue to increase in

8-47
length as injection pressure and rate increase? Singh,
et all; addressed this q uestion by simulating two six-step
SRT's with equal time step sizes and rate increments, and
with no wellbore storage and skin. The first three steps
in each study represent transient radial flow below parting
pressure, and the last three steps are above parting
pressure.
In the first case a fracture of arbitrary length, S.0
feet, is introduced at the beginning of the fourth step and
is maintained constant as the rate increases through Steps
5 and 6. The purpose of Case 1 is to determine if a
simulated SRT with constant post-parting fracture length
behaves like field data. A plot of pressure versus time for
this case is presented in Fig. 8.21. Notice that a reduction
in the rate of pressure change occurs when the fracture is
introduced. A p versus q plot is presented in Fig. 8.22.
The pre-test stabilized point, 1000 psi at zero rate, and
the data for the first three pre-parting steps, form the
first straight line. Data for post-parting steps 4, 5 and 6
form another straight line which is shifted downward from
the first and has a smaller slope. It is important to observe
that this plot does not have the characteristics we observe
in the field, i.e., it does not look like Figs. 8.19 and 8.20.
First, the two straight lines do not intersect near the part-
ing pressure. Further, the second line extrapolates toward
the pre-test stabilized pressure; in the field, the pre-
parting line extrapolates toward the pre-test pressure but
the second line extrapolates to a pressure much greater than
the pre-test pressure. Therefore, comparing these simulation
results for a constant fracture length to field observations,
it does not appear that fracture length remains constant
after parting occurs.
The Odeh-Jones plot for Case 1 is presented in Fig. 8.23.
As predicted by Eq. 8.37, data for the first three pre-parting
steps form a single straight line. When the fracture is
introduced a do^ni:ard s};i Tt in the data occurs and a second

8-48
2750 7

^..J
2500 -

5
2250-.
Steps 1-3:
Radial flow
S 4
;l^ 2000-

17507
2
Steps 4-6:
1500- Flow w/const frac length
(xf = 5 ft)
1250-

• 1000 0
10 20 30 40 50 60 70
t, hours

Fig. 8.21: Pressure-time data for simulated step


rate test13 (CD = 0, s= 0, xf = 5 feet).

line is formed for steps 4, 5 and 6. It is observed that this


line curves with increasing slope at the beginning, a charac-
teristic of fracture behavior on semilog plots that was
discussed in Chapter 6, and then approaches a straight line
as pseudoradial flow is achieved. Whereas post-parting data
in steps 4, S and 6 all fall on the same line, increasing
iniection rates in the field cause data to continually shift
downward. This difference between simulated and field
behavior supports the conclusion that fracture length does

• not remain constant as rate increases after formation parting.

8-49
2750-
6

Ste p s 1-3:
/ •
2500-
Radial flow
/
2250-
4
2000-
/

1750-
^.. 2
^ Steps 4-6:
1500- Flow w/const frac length
j (x f = 5 ft)
1250 -

^
1000
0 2 4 6 8 10 12 14 q
q, MscfD

Fig. 8.22: Pressure versus rate for simulated step


rate test13 .(CD = 0, s = 0, xf = 5 feet).

It is important to understand why the post-parting data


on an Odeh-Jones plot shift downward from the pre-parting
radial flow data. Observe from Eq. 8.34 that the intercept,
bMk, of the Odeh-Jones plot is inversely related to the
wellbore radius, r`v. Fracturing a well causes an increase in
the effective weZZbore radius. For example, Prats, et al.19
showed that the effective wellbore radius of a well with an
infinite conductivity fracture is one-fourth the total fracture
length, i.e., rwa = xf/2. During the pre-parting radial flow
period the wellbore radius, rW is a constant; accordingly,
the intercept of the Odeh-Jones plot, bMR, is single-valued.

8-SO
• • ^
2*10 -' -
Steps 1-3:

0.175 -
Radial flow 2 3
1
Steps 4-6:
U
0.155 - Flow w/const frac length
N
(xf = 5 ft)
00 6
0.135 -
0 4
L4-
Co U.115-
^
(-n
O O

0.095 -

7*10 -2 i
2*10 , 4*10 '
10 10° 10'

n (aj aj
TT (t-tj -1) an , hours

i=1

Fig. 8.23: ^,lultirate Odeh-Jones plot for simulated step rate


test13 ( CD = 0, s= 0, xf = 5 feet).
The effective wellbore radius increases, however, when
the formation parts; this causes a corresponding decrease
in the intercept of the Odeh-Jones line which results in a
downward shift of data at the parting pressure. NOTE: The
pressure at which SRT data begin to shift downward is the
formation parting pressure.
In Case 1 of the simulation study by Singh, et al.,
the fracture length after parting was maintained constant
and, accordingly, the effective wellbore radius remained
constant during steps 4, 5 and 6. Therefore, as illustrated
by rig. 8.23, all post-parting data fall on the same line.
It should be clear from Eq. 8.34, and from the concept of
effective wellbore radius, that any further increase in
fracture length should result in an additional downshift
of data on the Odeh-Jones plot.
In the second case considered by Singh, et al.,
fracture length is increased in steps 4, 5 and 6 to simulate
fracture extension. An arbitrary fracture length of 2.5
feet is introduced at the beginning of step 4 and is then
increased to 5.0 feet and 7.5 feet in steps 5 and 6,
respectively. The p versus q plot of this data is presented
by Fig. 8.24. The pre-part ing straight 1 ine is the same as
in Case 1. A straight line through the post-parting data
intersects the first line near the parting pressure, i.e.,
near the pressure at the end of step 3 where the fracture was
introduced in the simulator. Observe also that this line
extrapolates to a pressure much greater than the pre-test
pressure. While the fracture lengths used in this study
were arbitrary, it is significant that this plot exhibits the
same behavior observed in the field. Qualitatively, this
similarity between simulated and field behavior on the p
versus q plot indicates that increasing rates and pressures
above the parting pressure result in fracture extension.
The Odeh-Jones plot for this case is presented in
Fig. 8.25. It is observed that each post-parting step results

8-52
2750-.
Steps 1-3:
q Radial Flow 6
2500-

5
2250- J3/^^7- 4
FPP

M 2000-

1750-

7-150
Step 4: Fracture length = 2.5 ft
Step 5: Fracture length = 5.0 ft
1 Step 6: Fracture length = 7.5 ft
1250-

• 1000 0 ..................................................................

2 4 6 8 10 12 14

q, MscfD x 103

Fig. 8.24: Pressure-rate plot for simulated step


-rate test 13 fracture extension case
(CD = 0, s = 0).

in a downward shift of data as fracture length increases.


Notice that the curves for steps 4, 5 and 6 exhibit increasing
slopes in the early data, which is typical of fracture data,
and then approach a straight line as pseudoradial flow is
achieved. This behavior becomes more exaggerated as fracture.
length increases. It is possible to back-calculate the
fracture lengths with reasonable accuracy from the skin
factor and effective wellbore radius associated with each
curve; these calculated lengths are shown on Fig. 8.25.

• The strong similarity between the simulated data depicted by

8-53
2*10 -'-
Steps 1-3: Radial flow 2 3 Pi = 1000 psi
Step 4: Fracture length = 2.5 ft
0.175- Itow
Step 5: Fracture length = 5.0 ft ^ _
Q Step 6: Fracture length = 7.5 ft 2.97 ft
^Xf )c alc -
0.155-
= 5.36 ft
, (Xf )calc
r^ 5
^ 0.1357

112 0
Legend
O TEST DATA - FLOW PERIOD 1
0.115-
O TEST DATA - FLOW PERIOD 2
00
x TEST DATA - FLOW PERIOD 3
v^
0.095 - v TEST DATA - FLOW PERIOD 4
f )calc - 7.76 ft o TEST DATA - FLOW PERIOD 5

7*10 -2 i,oo I (XI I I


, io I
4*10 ^
x TEST DATA - FLOW PERIOD 6

2*10 -Z 1p '

n (aj aj -1)
I (t-tj-1 qn , hours
j=1

Fig. 8.25: Multirate Odeh-Jones plot for simulated step rate test;3
fracture extension case (CD = 0, s= 0).

9 • •
Fig. 8.25 and field data indicates qualitatively that fracture

• extension does occur as pressures and rates increase above the


parting pressure.
The study by Singh, et all; establishes that fracture
extension does occur above parting pressure as injection rates
and pressures increase. Existing SRT technology does not
permit, however, a quantitative prediction of fracture
extension. If the injection pressure is stabilized at a
value greater than the parting pressure it is not known whether
the fracture length remains constant or continues to increase.
If the fracture length remained constant, and was of a magni-
tude and orientation that did not adversely affect areal sweep
efficiency, it would be possible to inject above the parting
pressure without having a detrimental effect upon waterflood
performance. Additional research is needed to develop a
model capable of describing and predicting fracture propa-
gation caused by fluid injection.

• Example 8.4: Analysis of a step rate test using the


Odeh-Jones Plot.

Problem: The following simulated step rate test is for a


water injection well in which the shut-in pressure was
stabilized at 1000 psia prior to starting the test. Informa-
tion about the well and reservoir are given along with
pressure-rate-time data. Determine the formation parting
pressure, effective permeability to water, and the total
skin factor for this formation.

Pi = 1000 psia rw = 0.3 ft


h = 80 ft uw = 0.7 cp
= 150 Bw = 1.0 bbl/STB
ct = 6 x 10-6ps1-1

8-55
Point t pi.wf
No. hours psia

1 0 1000

STEP 1: q= 1000 bpd

2 1.000E -03 1159.7


3 2.000E -03 1192.2
4 3.162E -03 1208.5
5 5.012E -03 1223.7
6 1.000E- 02 1245.9
7 1.585E -02 1260.6
8 2.512E -02 1275.3
9 5.012E- 02 1297.3
10 0.100 1319.3
11 0.200 1341.4
12 0.283 1352.7
13
14
0.366
0.450
1361.2
1368.0

15 0.500 1371.4

STEP 2: q = 2000 bpd

16 0.501 1531.2
17 0.502 1563.8
18 0.504 1587.8
19 0.506 1602.9
20 0.510 1617.9
21 0.520 1640.6
22 0.550 1671.9
23 0.600 1696.8
24 0.700 1723.8
25 0.783 1738.8
26 0.908 1755.6

8-56
Point t piwf
0 No. hours Psia

27 1.000 1765.4

STEP 3: a= 3000 bpd

28 1.001 1925.2
29 1.002 1957.9
30 1.003 1974.2
31 1.005 1989.6
32 1.010 2012.3
33 1.020 2035.3
34 1.040 2059.2
35 1.063 2076.1
36 1.100 2094.0
37 1.200 2123.8
38 1.325 2148.2
^ 39 1.408 2160.8
40 1.500 2172.6

S'I'lii' 4: cl = 4000 hpd

41 1.501 2041.7
42 1.502 2066.2
43 1.503 2081.0
44 1.505 2095.6
45 1.510 2117.7
46 1.520 2140.5
47 1.540 2164.7
48 1.563 2181.9
49 1.600 2200.5
SO 1.700 2232.3
51 1.825 2258.9
52 1.908 2273.0
^ 53 2.000 2286.5

8-57
Point t piwf
No. hours psia

STEP 5: q = 5000 bpd

54 2.001 2202.0
55 2.002 2216.7
56 2.003 2228.1
57 2.005 2240.5
58 2.010 2260.7
59 2.020 2282.6
60 2.040 2306.5
61 2.050 2315.1
62 2.100 2343.0
63 2.200 2376.1
64 2.325 2405.2
65 2.408 2419.6
66 2.500 2434.1

STEP 6: q = 6000 bpd

67 2.501 2375.3
68 2.502 2385.7
69 2.503 2394.6
70 2.505 2405.0
71 2.506 2410.7
72 2.510 2423.2
73 2.520 2444.0
74 2.532 2459.3
. 1
75 2.550 2476.0
76 2.600 2504.2
77 2.700 2538.3
78 2.825 2568.2
79 2.908 2584.6
80 3.000 2599.8

8-58
Solution: Pressure-time and pressure-rate plots of the SRT
data are presented in Figs. 8.26 and 8.27 respectively.
Using straight lines drawn through the pre-parting and post-
parting data, Fig. 8.27 gives an approximate formation
parting pressure of 2,130 psia.
The Odeh-Jones analysis requires that we prepare a
semilog plot of

q.-q.
n ( ^ ^-1)
pi piwf
q versus I (t-tj -1 qn
n I
j=1

A table presenting this pressure function and radiaZ Odeh-


Jones time is given.

Radial
Odeh-Jones Pressure
^ Point t Time Function
No. (hours) (hours) (psi/bpd)

1 0.000 0.000 0.000

S'1'1;11 1: q= 1000 bpd

2 1.000E-03 1.000E-03 0.160


3 1.995E-03 1.995E-03 0.192
4 3.1.62E-03 3.162E-03 0.208
5 5.012E-03 5.012E-03 0.224
6 1.000E-02 1.000E-02 0.246
7 1.585E-02 1.585E-02 0.261
8 2.512E-02 2.512E-02 0'.275
9 5.012E-02 5.012E-02 0.297
10 1.000E-01 1.000E-01 0.319
11 2.000E-01 2.000E-01 0.341
12 2.827E-01 2.827E-01 0.353
13 3.662E-01 3.662E-01 0.361

8-59
Radial
Odeh-Jones Pressure
Point t Time Function
No. (hours) (hours) (psi/bpd)

14 4.495E-01 4.495E-01 0.368


15 5.000E-01 5.000E-01 0.371

STEP 2: q 2000 bpd

16 0.501 2.238E-02 0.266


17 0.502 3.165E-02 0.282
18 0.504 4.479E-02 0.294
19 0.506 5.652E-02 0.301
20 0.510 7.141E-02 0.309
21 0.520 1.019E-01 0.320
22 0.550 1.660E-01 0.336
23 0.600 2.450E-01 0.348
24 0.700 3.736E-01 0.362
25 0.783 4.706E-01 0.369
26 0.908 6.085E-01 0.378
27 1.000 7.071E-01 0.383

STEP 3: q = 3000 bpd

28 1.001 7.946E-02 0.308


29 1.002 1.000E-01 0.319
30 1.003 1.169E-01 0.325
31 1.005 1.365E-01 0.330
32 1.010 1.726E-01 0.337
33 1.020 2.195E-01 0.345
34 1.040 2.816E-01 0.353
35 1.063 3.355E-01 0.359
36 1.100 4.041E-01 0.365
37 1.200 5.511E-01 0.375
38 1.325 7.077E-01 0.383

8-60
^ Radial
Odeh-Jones Pressure
point t Time Function
No. (hours) (hours) (psi/bpd)

39 1.408 8.048E-01 0.387


40 1.500 9.085E-01 0.391

STEP 4: q 4000 bpd

41 1.501 0.166 0.260


42 1.502 0.197 0.267
43 1.503 0.221 0.270
44 1.505 0.249 0.274
45 1.510 0.297 0.279
46 1.520 0.356 0.285
47 1.540 0.431 0.291
48 1.563 0.493 0.295
^ 49 1.600 0.570 0.300
50 1.700 0.730 0.308
51 1.825 0.897 0.315
52 1.908 0.999 0.318
53 2.000 1.107 0.322

STr;1' 5: cl = 5000 bpd

54 2.001 0.273 0.240


55 2.002 0.313 0.243
56 2.003 0.344 0.246
57 2.005 0.378 0.248
58 2.010 0.435 0.252
59 2.020 0.504 0.257
60 2.040 0.588 0.261
61 2.050 0.621 0.263
62 2.100 0.740 0.269
0.911 0.27S

8-61
Radial
Oc}eh -Joncs Pressure
Point t Time Function
No. (hours) (hours) (psi/bpd)

64 2.325 1.085 0.281


65 2.408 1.191 0.284
66 2.500 1.303 0.287

STEP 6: q = 6000 bpd

67 2.501 0.394 0.229


68 2.502 0.443 0.231
69 2.503 0.479 0.232
70 2.505 0.518 0.234
71 2.506 0.538 0.235
72 2.510 0.583 0.237
73 2.520 0.659 0.241
74 2.532 0.718 0.243
75 2 . 550 0. 785 0 . 24 6
76 2.600 0.912 0.251
77 2.700 1.091 0.256
78 2.825 1.273 0.261
79 2.908 1.382 0.264
80 3.000 1.497 0.267

An example calculation of the pressure function and the


radial Odeh-Jones time function will be presented for point
51. The rate history which preceded the pressure measurement
represented by this point is depicted by Fig. 8.28.
The pressure function for point 51 is:

pi piwf _ 1000 - 2258.9 =


0.315 psi/bpd.
q -4000
n


8-62
• • •
30^

2600
0

D C) 0

C) D
2200
0 p o

1800
00 0

1400
C) p o

1000 r-}. r,.,r ,T,


0 0.5 1 1.5 2 2.5 3 3.5

t, hours

Fig. 8.26: Pressure versus time plot, Ex. 8.4.


3000

2600

' I I
2200 FPP = 2130 psia
r-1 -----^------^-----
^

^ 1800
CIO
4^

1400

1000
0 1000 2000 3000 4000 5000 6000

q, bbls/day

Fig. 8.27: Pressure versus rate plot, Ex. 8.4.

• • •
• y4 = -4000

^
7$
V) q3 = -3000

cz q2 = -2000

0
.r.(
4-j
U
ql = -1000
.r,

q0 = 0
=n t-=n t..=, _ 0 t.=1_S
J
t= 5
U 1 L

time, hours

i• Fig. 8.28: Injection rate history for


point 51, Ex. 8.4.

The Odeh-Jones time function for point 51 is:

n q i aj - 1) 4 (aj ^j 1)
TJ(t-t1) qn U(t-tji) q 4
j=1 j=1

n (a a 1) (al q°) (a2 a1)


TT (t-tj_1) a n =(t-t0) a4 x(t-t1) a4 x
j=1

-
(q 3 a2) (Q4 q 3)
(t-t2) a4 x (t-t3) q4

Substituting the appropriate values of rate and time, as


defined by Fig. 8.28, we obtain

0
8-65
n qj qj
(t-tj-1) O hours.
j=1

These functions are computed in a similar manner for all


other points.
The Odeh-Jones plot is presented in Fig. 8.29. It is
observed that data recorded during the first three steps
plot as a straight line. Beginning with step 4, however, an
obvious downshift in the data indicates that formation parting
has occurred. The last measured pressure prior to the
downshift is the formation parting pressure. This pressure,
represented by point 69, is

FPP = 2173 psia.

The slope, mMR, of the pre-parting radial flow straight


line i s 0.0745 psi/bpd/cycle. The 1)ermeabllity is computed
by rearranging Eq. 8.35 as

k = 162.6Bu
mMRh

k = (162.6)(1.0)(0.7)
(0 .074S )(3 0 )

k = 19.1 md.

The intercept, bMR, which is the value of the pressure function


on the y-axis at a value of the Odeh-Jones time function equal
to unity, is 0.394. Therefore, from Eq. 8.36


8-66
• • •
0.50
Legend
Stop 1: 1000 bpd
Step 2: 2000 bpd
0.45 Stop 3: 3000 bpd

Step 4: 4000 bpd

Step 6: 5000 bpd

0.40- Ste p 6: 6000 b p d

th - 1628. (mD•ft), t 18.10 (mD), i^-1.682E-0

p• - 1000.0 ( psi); flow period 1 ---

0.35 -
^

0.30-
v ° o00

Or, ° '0 + x XXX


0.25-
x
.

0.20-

0.15 -

0.10 I -L
. ,t
' i i
103 10 2 101 10° 2*10 0

n (aj -aj -1)


^ (t-tj-1 ) a n , hours

j=l

Fig. 8.29 Odeh-Jones plot, Ex. 8.4.


s= 1.151 [m ``1R -].og k + 3.23
MR
^Uctrw

0.394 log 19.1


s 1.151
0.0745
(0.l5)(0.7)(6x10-6)(0.3)2

+ 3. 231

s = 0.

3. FACTORS AFFECTING SRT ANALYSIS

A step rate test is a multiple rate pressure transient


test. It should be expected, therefore, that this test will
be affected by problems of interpretation caused by wellbore
storage and skin as would any pressure transient test. The
purpose of this section is to discuss how a SRT is affected
by wel.lbore storage and skin, as well as other factors such
as time step size, rate increment and changing wellbore
st.orage. Additionally, selectioji of the correct .initial
pressure for SRT analysis, and necessary alterations to the
analysis when the well is not shut in prior to the test,
will be discussed.

A. Time Step Size

It is important when conducting a SRT that all injection


steps be of the same length. This can be easily seen if we
consider the equations that govern preparting SRT behavior.
The time step used in most SRT's is of short duration,
i.e., one hour or less. Accordingly, most SRT data will be
in transient flow. The transient radial flow equation which
describes pre-parting behavior, previously presented as
I;q. 8..-) 0, is:

8-68
I'ii - P. - 1162 .
khBu (log kt]t 2 - 3.23 + 0.87s) Q (8.30)
4uct r W

Neglecting superposition effects, this equation predicts that


pre-parting data will plot as a straight line on a p versus q
plot provided that At for each injection step is a constant.
If At is not constant, it is obvious from Eq. 8.30 that the
slope of the p versus q plot will not be constant, i.e., there
will not be a straight line.
To demonstrate the effect of changing the time step size
while running a SR'I', Singh, et al13 simulated a six-step
SRT where all steps were below the parting pressure. The
first three steps were 12 hours long and then the step size
was reduced to one hour for the last three steps. The p
versus q plot for this study is presented in Fig. 8.30.
While this represents an unusually large change in time step
size, Fig. 8.30 illustrates that changing At does result in
a change in slope. In this case the decrease in step size
results in a reduction in slope that could be misinterpreted
as formation parting. An increase in step duration would
result in an increase in slope.
It is concluded f-rom these ohservcit.ions thnt changing
the step size during a SRT could result in a false indication
of formation parting and an incorrect parting pressure. It
is reemphasized that all injection steps during a SRT must
be of equal length.
Another important consideration in designing a SRT is
the length of the time step to be used. It has been recom-
mended by Felsenthal'" that 30 minute steps be used for
reservoirs with air permeability 10 md or greater and 60
minutes be used when air permeabil.ities are about S md.
ideally, the time step must he sufficiently long to overcome
wellbore storage and to insure that transient radial flow is
achieved.


8-69
3500-

3250-
Steps 1-6:
Radial flow

6

3000-

5
2750-

2500 - 4
^
^
¢ 2250=.
3 .

2000-,

17507

Steps 1-3: A t = 12 hrs


1500 = Steps 4-6: At = 1 hr

12507

1000
0 2500 5000 7500 10000 12500 15000

q, MscfD

Fig. 8.30: Effect of Changing time step


size on a step rate test13
All injection steps are below
the formation parting pressure.

The time required for wellbore storage to become


negligible can be estimated using methods developed in
Chapter 3. According to Eq. 3.72, wellbore storage will
be negligible when injection time, At, meets the condition
that

0
8-70
At > Cu(-2oo,000 + 12,000s (8. 39)
0 kh

6Vellbore storage is caused by one, or both, of the


following problems: (i) Compression of fluid in the wellbore,
and (ii) a rising fluid level in the tubing and, if it is
open, in the casing-tubing annulus. If the hole is not full
of fluid when the test begins, the early SRT data will be
affected by a rising fluid level. The wellbore storage
factor caused by a rising fluid level can be computed using
Eq. 3.44. If the wellbore is filled with fluid when the
test begins, or if this occurs at some time during the test,
storage will be due to compression of fluids in the wellbore.
The storage factor due to fluid compression can be computed
using Eq. 3.45. For a water injection well, storage due to
water compression will generally be insignificant compared
to rising fluid level storage. It is possible that a SRT
will begin with a rising fluid level storage and associated
large factor and then, when the hole fills, change to a small
compressive storage factor. This change in storage, as will
be illustrated in a subsequent section, could result in a
false indication of formation parting.

Example 8.5: Calculation of wellbore storage duration in a


water injection well.

Problem: Using the information given for a water injection


well, compute the duration of wellbore storage for each of
the following situations:
1) Storage caused by rising fluid level
2) Storage caused by fluid compression
The casing-tubing annulus is not open in this well.


8-71
k = 5 md cf = 5X10 -6 psi -1

h
90
= 67 ft
pW
r
w
=
=
62.4 lbm/ft3
0.33 ft

uw = 0.65 cp A = 0.0278 ft2(tubing)
= 3x10 6psi 1 Depth = 4,300 ft
cw

Solution: The wellbore storage factor due to a rising fluid


level is computed using Eq. 3.44, i.e.,

C = 144A
5.615p

C __ (144) (0.0278)
(5.615 (62.4)

C = 0.0114 bbls/psi.

From Eq. 8.39, the time required for storage to become


negligible is
^
At = Cu(200,000 , + 12,000s)

In the absence of any knowledge about the skin factor, it


will be assumed negligible; therefore

At = (0.0114)(0.65)(200,000)
5 67)

At = 4.42 hours.

The storage factor due to fluid compression is, from


Eq. 3.45,

C = CwUwb
5.615


8-72
The volume of fluid in the wellbore, which in this case is
the tubing, is

Vwb = (4,300 ft) (0.0278 ftz)

Vwb = 119 . 5 ft,

Therefore,

C (3x106)(119.5)
S.61S

C = 6.4x10-' hhls/psi..

With this storage factor, Eq. 8.39 indicates that storage


will end at

^t = (6.4x10-s)(0.65)(200,000)
5 6;

At = 0.025 hours

At = 1.5 1II1I1LJt.eti.

If it is known that a rising fluid level and compressive


type storage will both occur during a SRT, the time step size
should be based on the larger of the two storage factors. As
illustrated by Ex. 8.4, the largest storage factor in water
injection wells will be caused by a rising fluid level. If
the time step size based upon this criterion is too large to
be practical, a smaller time step may be used. However, the
time step used should be long enough that data are in the
storage-transition period, i.e., data must not be completely
controlled by wellbore storage. While this is not ideal, it
will be shown in a later section that multirate analysis

8-73
seems to give a good value of parting pressure even when SR'I'
data are influenced (but not copn,)lctcly controlled) by
wellbore storage.
Even if the duration of ivellbore storage is very small,
such as the compressive storage in Ex. 8.4, it is recommended
that the minimum time step size be 30 minutes. This permits
stabilization and accurate measurement of the injection rate
and prevents possible errors that can occur if pressure
measurements must be made immediately following a change in
injection rate while pressure is rapidly increasing.

B. Rate Increment

It is important when designing a SRT to use an adequate


number of injection steps and to select properrate increments.
Felsenthall" suggested using rates of 5, 10, 20, 40, 60, 80
and 100 percent of the maximum planned injection rate, but
it is also a common practice to use equal rate increments.
Which is correct?
According to Eq. S.29, which provides the theoretical
basis for a pre-parting straight line on the p versus q
plot, and Eq. 8.31, which provides the theoretical. basis for
a pre-parting straight line on the Udeh-Jones plot., equal
rate increments are not required to obtain pre-parting
straight lines. It. must be remembered, however, that these
equations assume radial flow with no wellbore storage.
1Velibore storage and skin, when present, can cause anomalies
which may result in misinterpretation of test data. It is
not clear in some situations how changing rate increments
might impact this behavior. In order to avoid unnecessary
complications that might be caused by changing rate
increments, it is recommended that equal rate increments be
used.
The only criterion for selecting the number of injection
steps is that both pre-parting and post-parting behavior be
clearly. defined. It is rec.ommended13 that 4-5 steps be

8-74
planned below the expected parting pressure. This should
result in a well defined pro-parting straight line. Another
3-4 steps should be used after parting occurs to help
establish an accurate parting pressure.

C. Wellbore Storage

Wellbore storage can occur during a SRT due to a rising


fluid level or fluid compression. The multirate superposition
plots used to analyze SRT's are based. on theory which assumes
that pressure data during each constant rate step are in
radial flow, i.e., that storage is negligible. Unfortunately,
it is not always practical to use time step sizes that are
large enough to get beyond the effects of storage.
In order to determine the effect of wellbore storage
on multirate analysis, Singh, et a113 simulated the effect
of storage on pre-parting SRI' behavior for cases where the
data were (i) completely controlled by storage, (ii) partially
controlled by storage, i.e., in the storage-radial transition,
and (iii) in purely radial flow. Log-log and semilog plots
of Agarwal's multirate equivalent time20, i.e.,

J'iwl (ta 1'cwC^tn l)


versus (Ate)MR
n n-1

where

n-1 (qj aj -1)


Utn-l-tj-1
(at+t q n-1
) -q n
(ote)MR = _t At (8.40)
n-1 j-1
J=1

are presented for these cases.


Figures 8.31 and 8.32 present the log-log and semilog
plots, respectively, for a case with 12 hour time step
sizes. For the system chosen, this time step was long

8-75
C-)
4-I
U

.c.

H
N

r--^

41

CL

iJ Legend
1__^
Q TEST DATA - FLOW PERIOD I
q i[51 DATA - FLOW PERIOD 2
X TEST DATA - FLOW PER100 3
P TEST DATA - FLOW PERIOD 4

(Ate)MR' hours

Fig. 8.31: Agarwal's multirate equivalent time


log-log analysis, steps ending in
radial f1ow13

ca
U
N
3*10 -'

Stora g e Tran s iti on R adi a l


•i
.,^
^
0.20; T 4

l-,
ti 0.157

4-J

4-f A.107
3
s3. ^ Legend
cr 0.057 o TEST DATA - FLOW PERIOD 1
4-J
q TEST DATA - FLOW PERIOD 2
* TEST DATA - FLOW PERIOD 3
^ V TEST DATA - FLOW PERIOD 4
7*10 -' H r
-: ,
2*10 10 100 10' 5*10 0 1

(Ate)MR, hours

Fig. 8.32: Agarwal's multirate equivalent time


semilog analysis, steps ending in
radial flow! 3

8-76
enough for all storage effects to end and for radial flow
to be achieved. It is observed that data for all steps form
a unit slope line on the log-log plot (Fig. 8.31) and a
radial flow straight line on the semilog plot (Fig. 8.32).
As expected, radial flow multirate superposition is applicable
to this case.
The second case is for shorter, 1 hour time steps.
This time step size is adequate to get beyond complete
storage control, but not for all storage effects to end.
Log-log and semi-log plots for this storage-radial transition
case are presented in Figs. 8.33 and 8.34, respectively. It
is suprising that multirate superposition seems to work for
this case almost as well as -for pure radial flow. The true
semilog straight line is not reached on Fig. 8.34, however,
and it is not possible to determine kh.
The third case represents a SRT where all data are
dominated by storage. The time step size is 0.15 hours.
The log-log and semilog plots, presented in Figs. 8.35 and 8.36,
show that multirate superposition does not work in this case.
Singh, et all; have suggested a new plot to determine if
SRI' data are completely dominated by wellbore storage.
According to Eq. 3. 64, storage dominated data in ci sing]c- ratc,
test can be described according to the relationship

qBt
pi piwf 24C (8.41)

Applying multiple rate superposition to this equation,


it can be shown that multiple rate injection will yield

B
piwf pi = 24C Wi (8.42)

where Wi is cumulative water injected. Rewriting this as

' og (piwC pi) = log IV- + Iog BC (8.43)

8- 7 7
Q
4-i
U
N •
^
a

.-^ Legend

+-J Q TEST DATA - R0V PERIOD i


q TEST DATA - FLOW PERIOD T
l-^--1 X TEST DATA - FLOW PERIOD J
V TEST DATA - FLOW PERIOD 4
.^
0

(Ate)MR, hours

Fig. 8.33: Agarwal's multirate equivalent time


log-log analysis, steps ending in
storage-radial transition13

C^_
w
U
^.,., -1 •
^

^
U)

(i-

a. Legend
o TEST DATA - FLOW PERIOD t
i,
q TEST DATA - FLOW PERIOD 2

x TEST DATA - FLOW PERIOD 3

.^{ v TEST DATA - FLOW PERIOD 4


71
D

(Ate)Mk, hours

Fig. 8.34: Agarwal's multirate equivalent


time semilog analysis, steps
ending in storage-radial transition13

8-78
Q -1
w

• U

H
V)

r-,

^
w
3 I__
•^+ a'
a.
^ Legend
4-J 0 TEST DATA - FLOW PERIOD I
^ TEST DATA - FLOW PERIOD ]
^ TEST DATA -lLOW PERIOD S
V TEST DATA - FLOW PCRiOD 4

(Ate)MR1 hours

Fig. 8.35: Agarwal's multirate equivalent time


log-log analysis, steps ending in
storage-dominated flow13

E U
(A

Ln
R

1--,

4-)

4H
I,
H O'
Legend
O TEST DATA - FLOW PERIOD 1
+-) 0 TEST DATA - FLOW PERIOD 2

4-r X TEST DATA - FLOW PERIOD 3


V TEST DATA - FLOW PERIOD 4
f^.

(Ate)MR, hours

Fig. 8.36: Agarwal's multi.rate equivalent


time semilog analysis, steps
ending in storage-dominated flow13


8-?9
suggests that a log-log plot of piwf versus jUi will yield a
line of unit slope for storage dominated data. An example
of this method is illustrated in the next section.

D. Changing Wellbore Storage

Changing wellbore storage can occur if a SRT begins


with a rising fluid level and then the wellbore fills at
some time during the test. T he effect of changing storage
from a rising fluid level type ( large storage) to a
compressive type ( small storage) was investigated in the
simulation study by Singh , et a1.13; the results of this
study will be presented.
Six-step SRT's were simulated by Singh, et al., in
which CD was reduced from 10; for the storage dominated
first three steps, to 103 for the last three steps. Two
cases will be considered: ( i) Case 1 - no fracture for the
entire SRT, and (ii) Case 2 - a fracture of 2.5 feet half-
length introduced at the beginning of step 5 and then
increased to 5.0 feet at the beginning of step 6. The
purpose of Case 2 is to illustrate the effect of fracture
extension following changing we]lhore storage.
A log-1og plot ol Al) versus W 1 , as suggested by Eq.
8.43, is shown for Case 1 in Fig. 8.37. Observe that data
for the storage dominated first three steps form a unit
slope line. The rapid increase in slope during step 4 is
caused by changing ( reduced) weilbore storage. This plot
for Case 2, while not shown, is very similar.
The p versus q plots for Cases 1 and 2 are compared on
Fig. 8.38. Observe that pre-parting data do not form a
straight line but exhibit a concave upward curvature; a
straight line forced through this pre-parting data would
be incorrect. Also, observe that the behavior of steps
4-6 for both cases are very similar; if a straight line was
forced through the early data, it would appear that
f o I I l i a at i c,r l aT t i11_, not on],, cct^1 T r. i n (;ase. 2, hut also in

8-80
• 10,

id

^ id
4-,

Legend
O TEST DATA - FLOW PERi00 1
iU O TEST DAIA - FLOW PER 00 2

)( TEST DATA - FLOW PERIOD J


V TEST DATA - FLOW PERIOD 4

O TEST DATA - FLOW PERIOD 5


X TEST DATA - FLOW PERIOD 6

10-
2 1 . i 1d

1v . , Mscf
1.

Fig. 8.37: Ap versus cumulative injection


for SRT with changing wellbore
storage, no fracture13

• Case 1 which has no fracture. Because of this behavior, it


is possible that changing wellbore storage on the p versus
(t plot L:oulLl be misinterpreted as ('ormat_i.on hat'tins;. It is
important in this case to observe that the true pre-parting
straight line formed by steps 4, 5 and 6, for Case 1,
extrapolates back close to the pre-test pressure of 1000
psia. A straight line through points above the parting pressure,
i.e., steps 5 and 6 for Case 2, intersects the y-axis
at a pressure much greater than the pre-test pressure.
The Odeh-Jones multirate plots for Cases 1 and 2 are
presented in Figs. 8.39 and 8.40, respectively. Even though
radial Clow superposition is not rigorously applicable
during the storage dominated first three steps, it is noted
in both Cases 1 and 2 that changing storage results in a
much steeper slope at the time it occurs. In Case 1 with
no fracture, it is noted that the data for steps 4-6

11
8-81
4000
Steps 1-3: CD = 1x105 •
Steps 4-6: CD = 1x103

Case 1 - No Fracture
3000
• Steps 1-6: Radial Flow
^

..
^ 2000
^
^ Case 2 - Frac Extension
/ n Step 5: xf = 2.5 ft
'3 ^ Step 6: xf = 5.0 ft
Z* *2
1000 L
0 5 10 15

q, MN1scfD

Fig. 8.38: p versus q plot for SRT with changing


wcllbore storage: (1) no fracture

and (2) fracture extension!'

approach a common line at late times when storage begins to


diminish and radial flow is achieved. Finally, for the
fracture extension case in Fig. 8.40, observe that a downward
shift in data occurs above the parting pressure in steps S
and 6 as fracture length increases. This means that formation
parting pressure can still be determined if the change in
wellbore storage occurs before the formation parts. If
formation parting occurs at or before the wellbore completely
fills up, the exact parting pressure will probably be masked 0

by wellbore storage. It should still be possible, however,


to determine whether data collected after the pressure
response caused by changing wellbore storage are above or
below the parting pressure.

8-82
• Q
w
U
^

rn

4a ^
L.g.-d
O - ^
p .(D,D.,e-nD.nnesr
)( 1[11 D.,1 - rl0v.(.IDD )
p ror D.u - ^DD r
p rcn ..n - ^
X * • .
.r,r.r. •n1..r•J,.y (.,D(_.
A . ..........

to -2
id 4*10

q
n aj 1 )
^ (t -t. ) qn , hours
^ -1
j=1

Fig. 8.39: Odeh-Jones plot for changing


wellbore storage: Case 1- no fracture13

2' 10 -'

• `~
U o.,s
Steps 1-4: Radial flow (prefracture)
Step 5: Fracture length = 2.5 ft
Step 6: Fracture length = 5.0 ft
4
5

lox
D.12:

LA
¢ J.10i

0.071
.,1
0.051 l y 11 Leg.nd
o
3 p ^• - - •

^ Le 0
0.02: 2
' pp o p 1
0 00 0 A . u.,.r t..:(..^. . ^.. ,
0'10
2'10 10 10 10' 4*'0 '

Il Cj) -(Ij -1)


(t -t - qn , hours
^-1

j=1
Fig. 8.40: Odeh-Jones plot for changing hrellbore
storage: Case 2 - fracture extension13


8-83
E. Skin Effects
Pressure losses due to formation damage, perforations,
or other near-wellbore effects, reduces the pressure actually

applied to the formation. Therefore, the pressure at which
a formation parts, as measured in the wellbore, will be
greater in a well with skin by an amount which will be equal
to the total pressure drop, Apskiii' caused by skin. This is
illustrated by Fig. 8.41. Since the pressure drop due to
skin is a function of flow rate, the difference between actual
and measured parting pressures will depend upon the injection
rate at which parting occurred.

^
T * ^
---
n t ^ •-^ n '

Apskiri

I _
pressure
(with skiri)
^ dpsi:in

0
4J
U
n/
Pressure Q) •w0 5k1r1
\`. .r,
(without skin)
n Skin

r
Injection '.?ate

Fig. 8.41: Effect of skin pressure loss on a step rate test.

When a well has skin, the pressure in the wellbore at


the formation face is greater than the pressure applied to
the formation in the amount of Apskin' Accordingly, the
formation parting pressure recorded in the wellbore is
actually greater than the true parting pressure by 'pskin'
Should skin be removed subsequent to measuring the parting

8-84

pressure, continuing to inject at the parting pressure
which was measured prior to skin removal will result in a
pressure applied to the formation which is Opskin greater
than the actual formation parting pressure. To prevent
injection above parting pressure, it is important that SRT's
be rerun anytime a well is stimulated or the mechanical
• condition of the wellbore is changed.

F. Selection of pi for SRT Analysis

Reference to Eqns. 8.31 and 8.37 indicates that


multirate analysis requires a value of pressure and rate
at the beginning of a SRT. The ideal theory on which these
equations are based assumes that the entire reservoir volume
affected by the test was shut in and stabilized at pi before
the test began. If this is true, and provided that pi is
known, an Odeh-Jones plot of the data will result in a pre-
parting straight line and post-parting data that shifts
downward from this line as the fracture extends. Practically,
SRT's are normally run following a short shut-in or reduced
rate period which is seldom long enough for the pressure to
stabilize. The question arises in these situations as to
what pressure should be used for p.
The effect of using an incorrect
I value of p i is
illustrated by Figs. 8.42, 8.43 and 8.44 which present the
results of a six-step simulated SRT13 where the first three
steps were below parting pressure and the last three steps
were above parting pressure. Figure 8.42 is an Odeh-Jones
plot of the data using the correct pre-test stabilized
pressure of 1000 psi. As expected, the pre-parting data
fall on a straight line, whereas a downward shift occurs for
those steps which represent fracture extension. Figure 8.43
is an Odeh-Jones plot of the same data using a pi of 900 psi,
i.e., 100 psi too small. Notice that all six steps shift
downward, implying that all steps are above parting pressure.
Figure 8.44 is a multirate plot of the same data using a pi
of 1100 psi. The pre-parting data in this case shift upward

8-85
2*10 --
Steps 1-3: Radial flow 2 3
0.175-
Step 4: Fracture length = 2.5 ft 1 q
Step 5: Fracture length = 5.0 ft
r--) Step 6: Fracture length = 7.5 ft
(4-4
U 0.155- 4^

001P 5
0.135- Zee,
6

0.115
Q. ^
s^. I C I I Or-01
0.095 pi use d = 1000 ps i a 'i,
. 14
Correct pi = 1000 psia
7*10 -' if' I I I lX I I I / 1 I I I i
2*102 10 ' 10° 10' 4*10 '

n (gj aj -1)
ff(t-t1) an , hours
j=1

Fig. 8.42: Odeh-Jones plot of simulated SRT


data using correct value of pi.13 q

htit thc.n, stops above ^^r^rt in^ pressure result in n downward


shift. It is concluded from these results that misinterpre-
tation of SRT data can occur if an incorrect value of p i is
used in multirate analysis.
The purpose of this section is to discuss the proper
selection of pi, and to show how the pre-test shut-in, or
reduced rate period, should be accounted for in the SRT
analysis. Two cases will be discussed: (1) SRT following
a shut-in period, and (2) SR'I' following a stabilized injection
period. These two cases are illustrated schematically by
Fig. 8.45. It should be noted that the pressure at the
beginning of the SRT will be referred to as pref.


8-86
2*10 --
• 0.216-
Steps 1-3: Radial flow
Step 4: Fracture length = 2.5 ft
1

Step 5: Fracture length = 5.0 ft


Step 6: Fracture length = 7.5 ft 3
U o
v 0.196-
0 o
a
U)
0.n6-
w^ 0 Oc oo 4
0.156- 0
.^, U-
0.136- p i used = 900 psia 6
o Correct pi = 1000 psia

0.116 ^1---1---1_1lIIlII I
2*10 -2 10' 10° 10 4*10

n a aj -l
(t-tj pn hours

• Fig. 8.43: Odeh-Jones plot of simulated SRI'


data using smaller than actual
value of pi.13

a. SRT Following a Shut-in Period

It was shown in Chapter 4 that the analysis of a


pressure transient test must consider all rate changes which
are recent enough to cause changes in pressure that
significantly affect the test. Since the shut-in period
which precedes most SRT's is not long enough for the well
to reach stabilization, it is possible that the rate change
imposed by shutting in the well might affect the SRT. Also,
if the well does not reach a stabilized pressure before
starting a SR'I', what value of pi should be used in the multi-
rate equations?


8-57
2*10 -'
Steps 1-3: Radial flow
Step 4: Fracture length = 2.5 ft 2

0.148
Step 5: Fracture length = 5.0 ft 13C
IC 4
U Step 6: Fracture length = 7.5 ft °
U) 0.128
ti
0.108
0
00
0.088
.r.,

0.068
0 ° pi used = 1100 psia
0.048 Correct pi = 1000 psia

3*10 -'
2*10
I- 4*10 '
10- 1OJ 10'

n a aj -1)
17 (t-tj-1) n n hours

Fig. 8.44:
j=1

Odeh-Jones plot of simulated SRT



data using larger than actual value
o f 1) :i • i 3

To study the effect of shut-in duration preceding a


SRT, Singh, et all; simulated a reservoir with an initial
static reservoir pressure, pi., of 1000 psia. The well
was allowed to inject at a constant rate, qL (see Fig.
8.45), for 14 days and was then shut in. The shut-in
period was followed by a four-step SRT with 1 hour time
step sizes. Two cases with shut-in durations of 3 hours
i
and 48 hours were considered.
In the case of the 48 hour shut in, Odeh-Jones
multirate analysis was performed using pref (see Fig. 8.45)
for pi and ignoring any rate changes prior to the SRT.
Referring to Fig. 8.45, the Odeh-Jones time function was


8-88
► ► ► i a4
I ►
I ► q
^ ► ► 3

aL a2
^
0 al
.^,
^
U ur;e 0 ►
v
► --or ---

j ar=0 I j

Long Term Reduced


Injection Rate or Step Rate Test
S',
,_ut-in

i•
Ln


Pref ►
-------- ^ ------ , At At At At
L.
`0 `? Z `" 3 `4

Time

Fig. 8.45: Schematic of SRT following reduced


rate or shut-in.13


8-89
started at time, to. While not shown, all data on this plot
formed a single semilog straight line with the correct slope.
This same SRT, preceded by only a 3 hour shut-in, was analyzed

using the Odeh-Jones plot and is presented in Fig. 8.46.

2*10 -'- ^
Steps 1-4: '
Radial flow o^ °4
0.137- 3-
2
1
0.117- 0
^
OP

4q
0.097-

,
^
0.077 -

^
6*10 -2
2*10 -'
„ 1o Z 10
id'
---. 0
3#10

n qj qj-1
q hours
IT t - tj -l) n '
j=1

Fig. 8.46: Odeh-Jones analysis of SRT following


a short shut-in, ignoring the shut-in
period. 13

This time, however, a single straight line is not obtained.


It is concluded that a SRT preceded by a long shut-in period
can be accurately analyzed with multirate equations using
pref in place of pi, and the pre-SRT rate history can be
ignored. This method will not work, however, if the shut-in
duration is short.
The SRT preceded by the 3 hour shut-in was reanalyzed
with the shut-in period included in the analysis. Referring

8-90
to Fig. 8.45, the Odeh-Jones time function was started at
the time of shut-in and prof was again used in place of p
From the Odeh-Jones plot presented in Fig. 8.47, it is
observed that all data now fall on a single straight line
with the correct slope.

2*10 -'

0.1
u
^
,^.

0.'

0.0

S^.

0.0

0 6*10
iV ^U U

/ ({i -(I i -^
11
l )
11 (t-t J-1 qn hours
j=1
Fig. 8.47: Odeh-Jones analysis of SRT
following short shut-in, accounting
for shut-in period13

It is concluded that pref, the shut-in pressure at the


beginning of a SRT, can be used in place of pi for multirate
superposition analysis if the shut-in period is properly
accounted for in the analysis. If a well is shut-in
sufficiently long before a SRT for the pressure to stabilize,
the rate history prior to the test can be ignored. When the
shut-in duration is short, which is the most common situation
in the field, the shut-in period should be included in the

8-91.
superposition time function.

0
b. SRT Following a Stabilized Injection Period

When the injection rate is stabilized at a non-zero


rate, dr, prior to the test (see Fig. 8.45), the multirate
analysis must be slightly modified. Recall from Chapter 4
that pressure transients are caused by changes in rate and
not the rate itself. For example, the pressure transient
caused by the first step in a SRT will be due to a rate
change (ql-qr) and not q,. Therefore, the Odeh-Jones
analysis now requires a semilog plot of

I? ( )
aa vs U(t-tj1) an ar
n r
j=l

If the well is flowed sufficiently long prior to the


SRT to achieve steady state or pseudosteady state, any rate
history prior to the test can be ignored and the super-
position time function can begin at the start of the SRT.
Otherwise, the superposition time function must include
the flow period prior to the test. In either case, pref
can be used in place of p ...
L

4. STEP RATE TEST ON A FRACTURED WELL

The SRT analysis techniques discussed in the preceding


sections assume radial transient flow during the pre-parting
steps. When testing a well. that has been previously fractured,
the radial flow assumption is violated if time step sizes are
not long enough for pseudo-radial flow to be achieved during
each step of the SRT. If pseudo-radial flow is not achieved,
Singh, et al'; have shown that the p versus q plot and the
Odeh-Jones multirate plot may not be adequate to determine the
parting pressure. Instead, it appears that linear flow plots
better define the pressure at which fracture extension occurs.

8-9?
It was shown in Chapter 6 that linear flow can be
described by the equation.

^z
_ 4. 064n1> it
pi piwf xfh (8.44)
[ctkj

Applying multirate superposition to this equation yields

pi^piwf = 4.06hB c^c k' (q7aq7 l) (t tj -1) 2 (8.45)


n f t n
j=1

Equation 8.45 indicates that a plot of

n
pi piwf ^' a' 1 -2
versus ( ^^I ^ ) (t-tj _1) 2
q
n
n j=1.
^
should result in a single straight line for pre-parting
steps in linear Clow. 1•'ractul•c extension should result in
a downward shift in data from this line.

S. TWO-STEP RATE TEST

This section describes a stop rate test developed by


Singh and Agarwal2i that requires only two constant rate
steps during which pressure and time data are measured
continuously. Test data are analyzed for formation parting
pressure ( FPP) using multirate superposition methods. The
• 2-SRT can be run i n at least four different modes i llustrated
by Fig. 8.48. The test well can be either shut in or
stabilized at a constant rate before the start of the test.
In either case, the stabilized pressure before the test, and
all. pressures during the entire duration of Step 1, must be
below the FPP. The injection rate for Step 2 must be chosen

8-93
Mode t

Step 2
u
Mode Z

Step 2 •
0

' II c II
Step 1
t II I!
Time Time

Mode 3 Mode 4

L
Step 2 Step 2
d v
0 0
Cr (X
C C
°- ° Step 1
t; I! ^
Step 1 C

II 11
Time Time

Fig. 8.48: Schematic of 2-SR'I' procedure S21.

so that the injection pressure will exceed the FPP during


this step.

A. Analysis Methods 0

It is necessary to analyze this test using multirate


superposition methods. rlg arwal's20 multirzt.e equivalent time
plot is made of the 2-SR'I' data. Pressures recorded during
Step 1 provide a "baseline" pressure transient response of
the well/reservoir system below the FPP. Ideally, pressures
measured during Step 2 should also coincide with this baseline
as long as the FPP is not exceeded and wellbore effects do not
change. When the FPP is exceeded during Step 2, pressures
should deviate from the baseline with a smaller slope; the
pressure at which this deviation occurs is the FPP. The
superposition plot can be made assuming radial or linear flow.

a. Radial Flow Analysis

When a well/reservoir svstem is behaving like an infinite


radial system and the log approximation is valid, Agarwal'`0
has shown that multirate pressure data can be described as

8-94
_ n-1
khlpwfn('t) pwfn-1(tn-1)1 _ 12flnlTT
r
(At)D
141.2 a an Bu
-1
n -l j=1
^

X ( t n-1
n-1 ^ -1 )(aj - aj_1)/an-1 - an
At + tn-1 t j' 1

+ 0.80907 + 2S} (8.46)

where to = 0, qo = 0, and n > 2. If an injection well has


a multiple rate history, the well is considered to have a
constant rate q1 for time tl, rate c{2 for time t2, etc.
The final rate for the time period being considered is an,
this rate begins at time tn_1 and lasts for an incremental
time At. The pressures are denoted as pwfl(tl)' pwf2(t2)
and p`afn-1(tn-1) at the end of the first, second and n-1
time periods, respectively. pwfn(At) refers to the pressure
at any time At during the time period of interest.
When considering a 2-SRT, if the injection well is shut
^ in prior to the test (Mode 4), n=2, c{1 and tl refer to the
rate and length of the first step, and q2 is the rate during
the second step. If the rate is stabilized prior to the test
(Modes 1-3), n=3, and q,l and ti refer to the rate and time of
stabilization; the injection rate during Step 1 is q2 from
tl to t2, and the injection rate during Step 2 is q3.
According to Eq. 8.46, a semilog plot of radial flow
equivalent time, (Aten)R^ versus Ap/Aq should be constructed,
where

n-1 (a j - aj_1)/(an-1 -an)


t _ - t.
^At (8.47)
(Aten)R .
[if1 n-1 ^-1

Ap = pwfn(at) pwfn-l(tn-1)
(8.48)
Aa an-1 - an


8-95
Data which are collected during Steps ]. and 2 while pressures
are below the I:PP should coincide on a graph of Ap/Aq versus
When the FPP is exceeded during Step 2, data will
(Aten)R'
deviate from the pre-parting baseline with a smaller slope;
the pressure at which this deviation occurs is the FPP.

Example 8.6: Determination of formation parting pressure


using a two-step rate test.

Problem: Determine the FPP for the following 2-SRT which


was run using the Mode 1(see Fig. 8.48) procedure. The
injection rate was stabilized at 814 barrels per day for
one week prior to the 2-SRT. The test consisted of a two-
hour shut-in (Step 1) followed by one hour of injection at
2750 barrels per day (Step 2). The pressure-time data are
presented.

- - !
Point t iwf ten) R A p/Gcl
No. (hours) (psia) (hours) psi/bpd

STEP 1: cll = 0 bpd

1 0.000 3065 0.000 0.000


2 1.667E- 02 3037 1.667E-02 3.440E-02
3 3.333E- 02 3023 3.333E-02 5.160E-02
4 5.000E- 02 3012 4.999E-02 6.511E-02
5 8.333E- 02 2996 8.329E-02 8.477E-02
6 1.1671;- O1 2983 1.166E-01 1.007E-01
7 1.333F- 01 2977 1.332E-01 1.081E-01
8 1.500E- 01 2970 1.499E-01 1.167E-01
9 1.667E- 01 2965 1.665E-01 1.229E-01
10 2.000E- 01 2956 1.998E-01 1.339E-01
11 2.333E- 01 2947 2.330E-01 1.450E-01
12 2.667E- 01- 2938 2.663E-01 1.560E-01

8-96
^ Point At piwf (Aten R Ap/Aq
No. (hours) (psia) (hours) psi/bpd
13 3.000E- 01 2932 2.995E-01 1.634E-01
14 3.333E- 01 2924 3.326E-01 1.732E-01
15 3.667E- 01 2917 3.659E-01 1.818E-01,
16 4.000E- 01. 2911 3.991E-01 1.892E-01
17 4.333E- 01 2905 4.322E-01 1.966E-01
18 4.667E- 01 2899 4.654E-01 2.039E-01
19 5.000E- 01 2893 4.985E-01 2.113E-01
20 5.667E- 01 2883 5.648E-01 2.236E-01
21. 6.333E- 01 2873 6.309E-01 2.359E-01
22 7.000E- 01 2865 6.971E-01 2.457E-01
23 7.667E- 01 2855 7.632E-01 2.580E-01
24 8.333E- 01 2848 8.292E-01 2.666E-01
25 9.000E- 01 2841 8.952E-01 2.752E-01
26 9.667E- 01 2833 9.612E-01 2.850E-01
27 1.000 2830 9.941E-01 2.887E-01
28 1.167 2814 1.159 3.084E-01
40 29 1 . 333 2800 1 . 323 3.256E-01
30 1.500 2786 1.487 3.428E-01
31 1.(i67 27711 1. 651. 3. 5751;-0:1.
32 1.833 2763 1.813 3.710E-01
33 2.000 2752 1.977 3.845E-01
34 2.039 2749 2.015 3.882E-01

STEP 2: c12 = 2750 bpd

35 2.050 2827 1.098E-02 2.836E-02


36 2.067 2881 2.789E-02 4.800E-02
37 2.100 2956 6.048E-02 7.527E-02
38 2.133 3007 9.277E-02 9.382E-02
39 2.167 3047 1.257E-01 1.084E-01
40 2.200 3082 1.575E-01 1.211E-01
41 2.233 3112 1.889E-01 1.320E-01
42 2.267 3139 2.210E-01 1.418E-01

8-97
Point At piwf Aten R 4p/Aq
No. (hours) (psia) (hours) psi/bpd

43 2.300 3162 2.520E-01 1.502E-01


44 2.317 3174 2.678E-01 1.546E-01
45 2.333 3184 2.827E-01 1.582E-01
46 2.350 3192 2.984E-01 1.611E-01
47 2.367 3201 3.140E-01 1.644E-01
48 2.383 3208 3.287E-01 1.669E-01
49 2.417 3222 3.597E-01. 1.720E-01
50 2.467 3239 4.048E-01 1.7821"-01
51 2.533 3259 4.637E-01 1.855E-01
52 2.600 3275 5.226E-01 1.913E-01
53 2.667 3290 5.807E-01 1.967E-01
54 2.750 3308 6.516E-01 2.033E-01
55 2.833 3323 7.213E-01 2.087E-01
56 2.917 3338 7.909E-01 2.142E-01
57 3.000 3349 8.586E-01 2.182E-01
58 3.028 3353 8.813E-01 2.196E-01

Solution: The pressure-time data are depicted graphically


by Fig. 8.49. A sem:ilog plot of- radial equivalent time
versus Ap/Aq is presented in Fig. 8.50. It is observed from
Fig. 8.50 that data measured during Steps 1 and 2 form the
same curve prior to an equivalent time of approximately 0.3
hours; at this time, Step 2 data deviate from Step 1 data and
form a curve of smaller slope. This deviation occurs
approximately at point 46 during Step 2 at which time the
injection pressure was 3192 psia. Therefore, it is concluded
that

FPP = 3192 psia.

Ll
8-98
• • is
3400.0

0
0
3300.0 0
0
D
0
0
320 0. 0

0
0
,.z 3100.0-
co
0
4 0
Y) D
4 3000-0-

2900.0 • I-W
D 0 O
OD Op
0
2800.0- E)
0 0
1.4 Step 1 (q 1 = 0 bpd) Step 2 (q2 = 2750 bpd)
2700.0
0.0 0.5 1.0 1.5 2.0 2.5 3.0 3.5
At (hours)

Fig. 8.49: Pressure versus time, Ex. 8.6.


0.5

Legend
D Step 1(ql = 0 bpd)

0.4 q Stop 2 ( q2 = 2750 bpd)

° Step1(q1=0bpd)
0
0
v
^• 0.3 0

n 0

° Step 2 (q2 = 2750 bpd)


a 0..2
^ Of q
^

0 CS ) d' For m a tion Parting


CID CPO P re s s ure = 3192 ps i
0.1

q °
q o

0.0
10z 10" 10f' 3*10 V
(Lten)R, hrs

Fig. 8.50 Radial Odeh-Jones plot, Ex. 8.6.

^ • •
b. Linear Flow Analysis

If a well has a high conductivity vertical fracture


and the data obtained for each step of a 2-SRT are in the
linear flow regime, Singh, et a1.13 showed that multirate
superposition assuming linear flow should be used to analyze
the SRT data. The pressure response for constant-rate
linear flow can be described as

kh I_pi pwf (t)_1 ,z


141.2qBu (rtDxf) (8.49)

For multirate linear flow, Agarwal's superposition method


can be extended to yield the following relationship for
linear flow equivalent time 21:

n-1
tj 1) z
(Ateri)L (q1 Jql) l(tn-1
n-1 n

(At + tn-1 -] + Ati-5 (8.50)


tj - 1)'2

Therefore, 2-SRT data from a fractured well that exhibits


linear flow behavior should be plotted as Ap/Aq versus
(Aten)L where Ap/Aq is defined by Eq. 8.48. The data for
Steps 1 and 2 should coincide on this plot as long as the
FPP is not exceeded. The pressure in Step 2 at which data
deviate from this pre-parting trend is the FPP.

B. Design Considerations

When considering time step size, the duration of Step


1 should be sufficiently long for wellbore storage effects
to end. If this time is too long to be practical, the
duration must be long enough for the data to be out of the
storage dominated (unit slope on log-log plot) flow regime;

8-101
this is necessary for multirate superposition to be validl'
The duration of Step 2 relative to Step 1 i s very
important. It is required that these steps be of similar
length on an equivaZent time scale to ensure enough
baseline data in Step 1 for comparison to data collected in
Step 2. If the relative duration of Steps 1 and 2 is such
that the FPP is exceeded during Step 2 at an equivalent time
which is greater than the equivalent time corresponding to the
last data point in Step 1, there will be no baseline for
comparison purposes and determination of the FPP will not be
possible.
Once the rates for a specific test have been selected,
Eqns. 8.47 and 8.50 can be used to ensure that the duration
of Steps 1 and 2 will result in similar equivalent times.
Alternatively, the graphs presented in Figs. 8.51 and 8.52
can be used. The curves on Fig. 8.51 represent the relation-
ship between 4a1/Aq 2 and At1/Ot2 for obtaining equal equivalent
times for both steps for Modes 1 and 3; Fig. 8.52 gives this
relationship for Modes 2 and 4. These graphs assume that the
flow rate was stabilized prior to the test for a time that
is much longer than the duration of Step 1.
Add it i ona 1 cles ign c_oris iderat, i orns and examp] Cs ;ire
discussed in Ref. 21.


8-102
0.0

• -0.2-
Legend
Radial Row

Lineer Flow
------- -------- -------- -

/
-0.4.

/
/
Q
-0.6-

-0.8-

-1.0 -
0.0 0.2 0.4 0.6 0.8 1.0

Atl/At2

Fig. 8.51: Timestep size design for


Modes 1 and 3 procedures2l

i• 2.00 -

1.75 - Legend
/

Radial Flow
1.50 Linear Flow '-
--- /

125
ca
° 1.00
a
0.75

0.50

025

0 00 -
1.00 1.25 1.50 1.75 2.00 2.25 2.50

Dtl/At2

Fig. 8.52: Timestep size design for


Modes 2 and 4 procedures?'


8-103
REFERENCES

40 "Injection-Weli
1. Robertson, D. C. and Kelm , C. H.:
Testing To Optimize Waterflood Performance," J. Pet.
Tech. ( Nov., 1975) 1337.

2. Craig, F. F., Jr.: The Reservoir Engineering Aspects


of Waterflooding, Monograph Series, Society of Petroleum
Engineers o AIME, Dallas (1971) 3.

3. Earlougher, R. C., Jr.: Advances in Well Test Analysis,


Monograph Series, Society of Petroleum Engineers of
AIME, Dallas (1977) S.

4. Miller, C. C., Dyes, A. B., and Hutchinson, C. A., Jr.


"The Estimation of Permeability and Reservoir Pressure
From Bottomhole Pressure Buildup Characteristics,"
Trans., AIME (1950) 189, 91.

5. Matthews, C. S., Brons, F., and Ilazebroek, P.: "A


Method for Determination of Average Pressure in a
Bounded Reservoir," Trans., AIME (1954) 201, 182.

6. Matthews, C. S. and Russell, D. G.: Pressure Buildup


and Flow Tests in Wells, Monograph Series, Society of
Petroleum Engineers of AIME, Dallas (1967) 1.

7. Hazebroek, P., Rainbow, H., and Matthews, C. S.:


"Pressure Falloff in Water Injection Wells," Trans.,
AIME (1958) 213, 250.

8. I:ar,
loughcr, R. C., Jr., Kcrsch, K. M., and Ramey, H. J.,
Jr.: "Wellbore Effects in Injection Well Testing,"
J. Pet. Tech. (Nov., 1973) 1244.

9. Kazemi, H., Merrill, L. S., and Jargon, J. R.: "Problems


in Interpretation of Pressure Fall-off Tests in Reservoirs
with and without Fluid Banks," J. Pet. Tech. (Sept., 1972).

10. Merrill, L. S., Kazemi, H., and Gogarty, W. B.:


"Pressure Falloff Analysis in Reservoirs With Fluid
Banks," Trans., AIME (1974) 257, 809.

11. Raghavan, R. and Hadinoto, N.: "Analysis of Pressure


Data for Fractured Wells: The Constant-Pressure Outer
Boundary," S. Pet. Eng. J. (April, 1978) 139.

12. Nolte, K. G.: "Fracture Design Considerations Based on


Pressure Analysis," paper SPE 10911 presented at the
1982 SPE Cotton Valley Symposium, Tyler, Texas, May
20, 1982.

• 8-104
13. Singh, P. K., Agarwal, R. G., and Krase, L. I).:
"Systematic Design and Analysis of Step-Rate Tests To
Determine Formation Parting Pressure," paper SPE 1679S
presented at the 62nd Annual Technical Conference and
Exhibition of the Society of Petroleum Engineers, Dallas,
Texas, September 27-30, 1987.

14. Felsenthal, 1`1.: "Step Rate Tests Determine Safe


Injection Pressures in Floods," Oil and Gas J.
(Oct. 28, 1974) 49-54.

15. Staggs, I-l. _ M. , Jr.: "Injection Well Skin Effects:


Plus or Minus?," paper No. 851-41-B presented at the
API Mid-Continent District Meeting, Oklahoma City,
Oklahoma, March 29-31, 1967.

16. Odeh, A. S. and Jones, L. G.: "Pressure Drawdown


Analysis, Variable-Rate Case." J. Pet. Tech. (Aug. 1965)
960-964.

17. Bennett, C. 0. and Clark, T. J.: "Design and Analysis


of a Nitrogen Step-Rate Test, Anschutz Ranch East
Nitrogen Injection Project," paper SPE 14456 presented
at the 60th Annual Technical Conference and Exhibition
of the Society of Petroleum Engineers, Las Vegas,
Nevada, Sept. 22-25, 1985.

18. Williams, D. B., Sherrard, D. W. and Lin, C. Y.:


"Impact of Inducing Fractures at Prudhoe Bay,"
J. Pet. Tech. (October 1989) 1096-1101.

19. Prats, M., Hazebrook, P. and Strickler, W. R.: "Effect


K Vertical Fractures on Reservoir Behavior-Compressible
Fluid Case," Soc. Pet. Eng. J. (June 1962) 87-95.

20. Agarwal, R. C.: "A New Method to Account for Producing


Time Effects When Drawdown Type Curves are Used to
Analyze Pressure Buildup and Other Test Data," paper
SPE 9289 presented at the 55th Annual Technical Conference
and Exhibition, Dallas, Texas, Sept. 21-24, 1980.

21. Singh, P. K. and Agarwal, R. G.: "Two-Step Rate Test:


New Procedure for Determining Formation Parting
Pressure," J. Pet. Tech. (Jan. 1990) 84-90.
r I

8-10S

NOMENCLATURE - CHAPTER 8

A = area served by injection well, ft2


B = formation volume factor, RB/STB
ct = total compressibility, psi-1
E = flow efficiency, dimensionless

F cor = permeability correction factor for fractured


wells, dimensionless
h = net formation thickness, ft
k = effective permeability, md
ka = apparent permeability from pseudoradial semilog
straight line, md
ko = effective permeability to oil, md
kw = effective permeability to water, md
m = slope of semilog straight line, psi/cycle
mH = slope of Hall plot, psi-hr/bbl
mLF = slope of linear flow straight line on square root
^ plot, psi/hr-2
mpR = slope of pseudoradial straight line on semilog
plot, psi/cycle
= dimensionless Matthews-Brons-Hazebrock pressure
PI)Ml;ll
flowing injection pressure, psia
piwf =
pwf(At= 0) = injection pressure at time of shut-in for a
falloff test, psia
average pressure, psia
A =
p = shut-in injection pressure, psia
wS
p* = pressure at Horner time ratio of unity, psia
= pressure at t = 1 hr (or At = 1 hr) from semilog
plhr
straight line, psia
q = injection rate (negative number), STB/D
rd = drainage radius (radius of investigation), ft
rob = radius of oil bank, ft
r ``, = wellbore radius, ft
rwb = radius of water bank, ft
s = total skin factor, dimensionless
^ S S. = gas saturation at beginning of injection project,
fraction

8-106
t = injection time, firs
tllA = dimensionless time based on A
= injection time prior to falloff test, firs
th
6ti`i = cumulative water injected, bbls
xe = distance to boundary influenced by injection well, ft
xf = fracture half-length, ft
Aps = pressure loss due to skin, psi
ASw = change in water saturation in water bank due
to water injection, fraction
At = shut-in time, firs
(At)- = shut-in time at which average pressure is read
PR from MDH plot, firs
a = fluid mobility, k/p, md/cp
= porosity, fraction
= viscosity, cp
= oil viscosity, cp
PO
Pw = water viscosity, cp


8-107
SUMMARY OF MAJOR EQUATIONS - CHAPTER 8

Equation Number Equation


in Text

^z
_ s.615wi
8.2
rwb Llffh^AS

8.3 z
rd = 0.029 ^u tct
L"

t +At
8.4 pws = pk + in log P At

m = 162.6qBu
8.5
kh

8.6 k = - 162.6qBu
mh

[Pwf(t=o) - I'lhr
8.7 s = 1.151
m

- log k + 3.23 ^
2
^u^trw

8 . 8 0 =- 141.2qBp s
Ps kh

PR pw f (At=0) - APs
8.9 E =
pwf(At=0)
PR


8-108
0.319 m pRqB
8.,^2 xf
mLF L^cthF cori

8.23 k = F k
cor a


8-109
PROBLEMS

I :VJIic'I' [ ON 1ti1^L1, '1'ES'1'1NG

1. Water was injected into the Grayburg formation,


Ector Coun ty, Texas, at an avera ge rate of 3S0 bbls/day
for 8,540 hours befor e shutting the injection well in
for a 331 hour pressu re falloff test. The bottomhole
injection pressure pr ior to shut -in, pwf(Ot=O), was
measured t o be 2,528 psia. The well is located in the
center of an 80-acre five-spot p attern (wells are
drilled on 40-acre sp acing). Re cords indicate that
this well. was hydraul ically frac tured at the time of
initial co mpletion. Gas fillup had already been achieved
at the time of the te st. Additi onal data are:

= 8o r w = 0.33 ft
c t = 1Ox10 6 h = 5S ft
u W = 0.8 cp B w = 1.0 RB/STB

, , t +^t
At,hrs At 2 ,hrs 2 dp,psi
^
a p ws, psia
O - - 2528 -
0.25 0.5 341.61 2522 6
0.50 0.71 17081 2461 67
0.75 0.87 11388 2445 83
1.0 1.0 8541 2432 96
1.5 1.22 5694 2415 113
2.0 1.41 4271 2400 128
2.S 1.58 3417 2386 142
3.0 1.73 2848 2375 153
• 4.0 2.0 2136 2363 165
5.0 2.24 1709 2349 179
6.0 2.45 1424 2332 196
8.0 2.83 1069 2309 219
10 3.16 855 2287 241
12 3.46 713 2267 261

8-110
14 3.74 611 2251 277
16
20
4.0
4.47
535
428
2238
2213
290
31S

24 4.90 357 2192 336
28 5.29 306 2174 354
32 5.66 268 21S6 372
36 6.0 238 2140 388
40 6.32 215 2124 404
46 6.78 187 2106 422
52 7.21 16S 2088 440
58 7.62 148 2070 458
64 8.0 1.34 2057 471
70 8.37 123 2042 486
80 8.94 108 2021 507
90 9.49 95.9 2000 528
100 10.0 86.4 1981 S47
110 10.S 78.6 1965 563
120 11.0 72.2 1950 578
130
140
11.4
11.8
66.7
62.0
1936
1922
592
606

150 12.2 57.9 1892 636
160 12.6 ;4 .'1 1879 649
170 13.0 51.2 1869 659
180 13.4 48.4 1859 669
200 14.1 43.7 1840 688
220 14.8 39.8 1821 707
240 15.5 36.6 1804 724
260 16.1 33.8 1788 740
280 16.7 31.5 1774 754
300 17.3 29.5 1759 769
i 1
320 17.9 27.7 1755 773
331 18.2 26.8 1749 779

You are requested to evaluate this data for permeability,


formation damage, fracture length and average pressure.
Horner, Miller-ll^Ies-}lutchinson and square root plots
are presented.
8-111
• 2800
• •
, , : ^ a. ^-

^. ^.
PROBLEM 1

2600
^
^ - •
1
^ - • • 1 '.. 1
.
.1_`«t- -r l . _ _- .
r f a 1
+ 1 -4 I:, I :

^
-
^.; ^. ^' i
.

^ I II
2400 I

.:: _. .
. . ^ _>.
_; _._ .. ^.. . . _. I1' ;^, .t.... ,. ^_ • _ _
- - -1 4 ^ ^- - -- - ^ I
. . _. ...
2200 0- - ^:
^ - --
^ N ;. ^. . ,..... ;.. ;. r...
II 1. -
u L L-r
F--' ^ If

. . . { i^ ..1.
.__ _l ^ • } ^- ^ ^ ' ..^ ' ^. ^ r^^ ^
2000
.:: .. . _
1

'^.

'^ I I '1 f• `1 t- '^^1 , " t _ t . • . 1 .. .+ 1


; ., 1. ^.^ _ _
. . , _ = t I. i,t ,,l,• ,,i^ ,^ ,^.

L
-4•
^ '' ^+ ^^ ^. ^ '^.

i
^^
1800 i t .., :
r :.
,1 l
I l 1 i ^
_
fai{ I
.
d
If
il uI, I. _ I ! ^II .
I ^ l .: .I. ^ .illll T :'1 il {
tr ^ + µ, ^ 1 1

1600
10 102 103 104 105

t +At
P
At
2600 . .^.. . . .; _ ._.
-. . .. . ..

. _ ] ._ ..- .
, -- -
- ^. , -
2400 ......_..; *..
...... -^
i-- •-
-`•--^---...
;^---+----
^- --^ -----

2200 .... ... --


-T
------i-- ;--
- ._.- f ....-1 ^-

_ -. . . .

2000
_.
--- ---^-- ---
--;--^
... L_.
- -- ^-- - -- -
----
1800
_._i----
^-^

:^: ♦_
_,. --j ---
16.00
^---
-I
^---
_
.:::,.::
1400
10-1 1 10 102 103

At , l1I'S

0 0 0
^
2600
: ^ ^ - ----

^-^ ; • r =;^ • -; 1- - - - - --^- -- - ^ T


^, -
2500 PROBLEM 1 +^ -
r4

_ .{ f4l ..( ^ . ( I • • ... . . . ..^ ^ ,..


.__ . . -_ . _. - _ - - -
.. .}.. . ^, . , .. . .. ^ -. ^^ ^
..^ .
I

{ J- ,
2400 ( a .
T-
-- ` , . r^ 4 - _
_^ {I {- • -^_ , ( • ^ _! - _ - -( _-
T. ^ 1^-^
_ t_ .. .
I .. _^ .- ^•_
_^ -^'rt__+-_ f •- r^1 _ -{'-•1 , • ^ ,. . -_-

^^ ^ ^- r ^ ^ ^ • - r r i^ ..^ f .
,
_ _1
2300
} i • -,
1 1 -4
{-_t_ -^-
cz 1^' - - - - -- ( - - - -- -^ +
H _ _.. _ _ ..^._
-+_ ^ -`._• A -. _• • . ^ -^-1. Y__r- f.^ .^ .. ' I ^ '
tn
1- '^-- - r - - -t-- - - -' -^ -{ •
Cl.. ri ^ ^_^t^ _^_1 ^ (

f ^.^ i ._^ . . ^, . . . • T •. ^ '- _. ^ - - - ^


T^• -
.

• 2200 _ ^.

T^
^_ I

F
_ _«_.-

t r.
-^- .^ .. .. - . .
r T
. . •: . .

le
-
4 ^
^^ _ _
I
"i . -t_ _ __ I -I i _.

'_ _ ^ , N , • ^
, ^ - - - -
i l I i I ^ '
- 7- 4
-
2100
i
. . _^ -. - r.. 1 4 - 4-j
I
. . -- - - ^ • - Y - ^- - r - ( •

T ` 7_ -^- - -i-- - .- ^ 1 --1 -- - +t 4


^'- .^._ - , ^
- : --

2000 ^ _`^'_ '_ : ♦:_ ' ( _' ,^ ^^ ^ < < +^-^ ^ + -T.^ -; ' " :
- r---- •- . _ ., , ^-^ -^ ^ - -t - ; ^- ^ -:--+ , --- ^, . .. h
. _._ . , . i - , , , -^ ( . +^ i--!---- . ' ^* ~ ^ -- • . _
--- ;^ ,
.- _ ; , .
- -- -- -•- - -- - -1--
• , , , ,^ _ , . i '
- •--
- - ;
^
L ^
+
-,-
1900 ,. . , .. --i ^.; -_ ... ..-... ,_. .
.,
U L 4 6 8 10 12

At 2, hrs z


8-114

z
0
^

a`.
a N

0Z
U

za
w a
Ll
N
^
W

LL
Q W
^ J
L7
_ }
Z f U

0 F ^
N X

W
0 J
J u
U

1 2 3 4 5 6 7 H 9 1 2 3 4 5 6 7 B 9 1 3 4 5 6 7 B 9 1

is
8-115
2. Use the infinite conductivity, vertical fracture type
curve to analyze the data in Prob. 1 for permeability
and fracture length. Compare your answer to the values
obtained in Prob. 1.

3. Analyze the following pressure falloff data for fracture


length, xf, and fracture conductivity, kfw, using:

a) Bilinear flow plot


b) Agarwal type curve.

Log-log, fourth root, and semilog plots of the test


data are provided.

At At '4
pws p w f (At-0) -Pws
hours psia psi hours 4

0.10 2578 56 0.562


0.15 2574 60 0.622
0.20 2568 66 0.669
0.25 2566 68 0.707
0.50 2553 81 0.841
0.75 2543 91 0.931
1.00 2536 98 1.000
2.00 2515 119 1.189
3.00 2500 134 1.316
4.00 2489 145 1.414
5.00 2479 15S 1.495
6.00 2469 165 1.565
7.00 2461 173 1.627
8.00 2452 182 1.682
• 9.00 2447 187 1.732
10.00 2439 195 1.778
12.00 2428 206 1.861
15.00 2411 223 1:968
20.00 2387 247 2.115

^
8-116
25.00 2368 266 2.236
30.00 2349 285 2.340
40.00 2318 316 2.515
50.00 2290 344 2.659
60.00 2268 366 2.783
70.00 2245 389 2.893
80.00 2226 408 2.991
90.00 2208 426 3.080
100.00 2192 442 3.162
110.00 2178 456 3.239
120.00 2166 468 3.310
130.00 2153 481 3.377
140.00 2140 494 3.440
150.00 2129 505 3.500
160.00 2120 514 3.557
170.00 2110 524 3.611
178.00 2103 531 3.653

T = 106 ° F co = 7.15 x 10 -6 psi


h = 78 ft c 1a = 3.0 x 10-6psi-'
0.115 c1- = 4.2 x 10-6psi-1
5a = 0.35 JA W = 0.7 cp
S La = 0.65 BW = 1.0 RB/STB
rW = 0.208 ft pw = 66.1 lbm/ft3
qja = 136 bbls/ day pW f(A t=0) = 2634 psia
tp = 730 days k 0.27 md (pre-frac test)

PR = 2000 psia (est.)


8-117
, ..

pa
0
C4
aq N
O
r-1

O
r-i
^


r--q

O N

r.--q CD 0
r'.q
-1

sM ^:ni
tsd ` d-^0=^0) d

• 8-118
stT - s

• Pwf^vt=u)-Pws' Psi

f - N W .P Ui 0'^
O O O O O O
O O O O O O O

. , 1 L t
#^

'
, ---
, ^ ,^ ^^ ^ ;_^._;_-^ ^- ^• ^ ^-+-; ^^-^ - -
^ -.-
^_ r.*+I---^-
i - - -- --- - -
,, !-- --^ - - - -^-!-^--± • - - . - - - - a_ • -
. - - - -! --- -- - -- - - -
_ _ - -- -{- .-- - - ^_ t II
f

i I* ^- ^ 4-#-;-
, - - - __ -

rt { - - - - - - - --- - -

^
v N

^
^ -- - -
^ 77

,- 1 •^ - I . _ - +

a
? •. ^_ i r----- -- W ..-
^ { , ^- - ^- '-^-'-^ - --
--
, ' _- . --
- -- -- ^- . -- - - -^--- ,-^ ---
,- -}- ! - ,-; --^-1 -
^-- i - - - -- ^- - f I - -
41, -
_
w
L
_ __ ....^

I-f ^ - '-- - - -
-- - :, -
--

i• -P.
-
^ ,

^ ^
-

-^ ^
- - -
r-

- - --
- -- - -

. -{
- -

^
- - - {- - ^ -

-
^ • ^
2600
P-
RO BL EM 3
u T
.^ i 7
_ ^ . i _.. .._}:. .. ,^ -- -^- • -^ ^ . _ . ---
- _. . . ,._.
,. .^. -
_ . . . . . . .. ,_ . .. .... # • ^ .^ - -- - - -
2500 ;.. ; --
• - ^ . -- - L- ^. _. _ _ ._ .- ^ __ _.._.. .^ ..
'.
.. ._ _... _. ._ -,- . . __.7.. . ._. ._ :-t- .. . ^ ,I ... . . . ..
^ I . ^. 41 . . . .-_ .-^
.
- .
,:.:. _._ _^ ._.. ...: :...
. .. . . ..
:_: ^--- - -
l ..
. ..I . . ..I . ; . ^ ,... . . • . .
►, . .. '
f: ^
^ -^-- - -
2400 J_... ...,. . .. . .. . ' ^
... .,.. .. . . _ ^.
-- . . „ - ,.. ^- . ... . ^ ... .. .
_ ^ . -.--- -- ---
^ ^ ^ - - - - _- i
- -- -
1. .._ ._. , ;, , -. . - -- -
:. ^
..^.. _- - -

74-
-
,- - ..
.
. ....

-
-• --_ .. .

^
-- T- t - _-
.
- .. .:1^.

2300 . :_: , - -- , . , ,. ^_ , , __+ ^_, ♦_ - ^T .

. .
- , _^ ! ..
^ t -- . a.. .. .. .. ... i - ... . ... ._, _ - - • ^+' -^ - - - . ^ : ..
:..' .., ^ .. . .
-^ -.- . .. . .-. .^: .._
T
... ....^... . . , ^^ 1-. _... J J , . , .- .. .^ ._ ` - •- ,_ . . ^. ,. . .. - _
2200
, ...
.. ._._. _. . ^^-' _:-- -'^- .
-.- _ ;^, ,,i .. r.. ^ _. _.._.^-:_. __,. ... ..:. _ -•. _-^ '-_--

. ._. . ; . . _. ... ..'. __}T


-^ 11 ,
.^^ _ -
_. i i . } .- , -..-
•_ . I..- ril I .^ ^ f 1 , . __.. ' ._:I i. .._ ^' . . . . . ,^ _ _-_
_ _ ^
i _
- ,
,-__^ ^,{ . •Y ^ '1-. .{. .:.1 ,, , ^ .. ___ - f^ , _-. . ^ .. . + t _ ^ _
2100
10-1 1 10 102 103

At, hrs
4. The following SRT data are from a well in the
Valhall field whic i is located in the Norwegian sector
of the North Sea. Prior to the test, the injection rate
was stabilized for 24 hours at 1,065 barrels per day.
The test consisted of five 3-hour steps. A plot of
injection pressure at the end of each step versus injection
rate is provided. Also provided is a radial Odeh-Jones
semilog plot of

n
pi piwf (aj -aj )/(
vs q n -q r)
qn-qr (t-t ^
. -1 )
j=1

where c{r represents the stabilized rate prior to the test.


You are requested to determine the FPP using the following
methods:

a) Pressure versus rate plot;


b) Radial t)de.•h-,?oncs plot.

Radial
Odeh-Jones Pressure
Point t piwC Time Function
No. (liours) (i) sia) (hours) (psi/bpd)

1 0 4823.9 0 0

STEP 1: q 2000 bpd

2 0.011 4933.6 1.0565E-02 0.1173


3 0.051 4997.8 5.0953E-02 0.1860
4 0.080 5015.3 8.0304E-02 0.2047
5 0.102 5024.3 1.0272E-01 0.2143
6 0.202 5049.8 2.0416E-01 0.2416
7 0.302 5065.2 3.0656E-01 0.2581
8 0.402 5077.1 4.0991E-01 0.2708
9 0.502 5086.2 5.1421E-01 0.2805
10 0.611 5094.2 6.2828E-01 0.2891
11 0.711 5100.5 7.3457E-01 0.2959
12 0.811 5106.5 8.4181E-01 0.3023

8-121
^ Radial
Odeh-Jones Pressure
Point t piwf Time Function
No. (hours) (psia) (hours) (psi/bpd)

13 0.911 5111.6 9.5001E-01 0.3077


14 1.011 5116.3 1.0592 0.3128
15 1.200 5123.4 1.2686 0.3204
16 1.411 5130.8 1.5054 0.3282
17 1.611 51.36.8 1.7342 0.3347
18 1.844 5142.8 2.0061 0.3411
19 2.011 5148.0 2.2035 0.3466
20 2.511 5164.7 2.8118 0.3645
21 2.761 5176.7 3.1250 0.3773
22 3.000 5189.6 3.4307 0.3911

STEP 2: a= 3100 bpd

23 3.005 5246.7 1.0059E-01 0.2078


24 3.011 5281.2 1.5083E-01 0.2247
25 3.030 5331.7 2.6610E-01 0.2496
26 3.051 5356.8 3.5512E-01 0.2619
27 3.080 5380.0 4.5602E-01 0.2733
25 3.102 53 ()3.2 5 .22571i-O1 0.279 8
29 3.205 5434.3 7.7423E-01 0.2999
30 3.305 5460.4 9.7523E-01 0.3128
31 3.405 5480.2 1.1547 0.3224
32 3.505 5497.0 1.3209 0.3308
33 3.611 5510.9 1.4866 0.3376
34 3.711 5525.2 1.6372 0.3446
35 3.811 5536.2 1.7829 0.3500
36 3.911 5544.0 1.9250 0.3539
37 4.011 5550.6 2.0641 0.3571
38 4.111 5559.0 2.2009 0.3612
39 4.211 5566.4 2.3359 0.3649
40 4.311 5572.4 2.4693 0.3678
41 4.411 5579.0 2.6014 0.3711
42 4.511 5585.5 2.7325 0.3742

8-122
Radial
Odeh-Jones Pressure
Point t piwf Time Function
No. (hours) (psia) (hours) (psi/bpd)

43 4.744 5592.9 3.0353 0.3779


44 5.011 5600.5 3.3777 0.3816
45 5.261 5606.7 3.6966 0.3847
46 5_.527 5610.6 4.0356 0.3866
47 5.727 5609.0 4.2895 0.3858
48 6.000 5611.0 4.6361 0.3868

STEP 3: q 4220 bpd

49 6.005 5634.4 4.1040E-01 0.2569


50 6.011 5636.8 5.3567E-01 0.2577
51 6.030 5636.0 7.7867E-01 0.2574
52 6.052 5642.1 9.5170E-01 0.2593
53 6.072 5646.6 1.0685 0.2608
54 6.102 5652.3 1.2185 0.2626
55 6.205 5662.3 1.5874 0.2657
56 6.305 5667.1 1.8584 0.2673
57 6.405 5670.5 2.0887 0.2683
58 6.505 5072.8 2.2948 0.2691
59 6.611 5675.3 2.4951 0.2700
60 6.711 5678.7 2.6731 0.2709
61 6..811 5681.6 2.8426 0.2718
62 6.911 5684.4 3.0056 0.2727
63 7.011 5686.9 3.1633 0.2735
64 7.211 5690.1 3.4667 0.2745
65 7.411 5692.4 3.7581 0.2753
66 7.761 5697.7 4.2488 0.2770
.
67 8.011 5698.3 4.5890 0.2772
68 8.511 5702.1 5.2532 0.2784
69 9.000 5708.8 5.8906 0.2802

8-123

^ Radial
Odeh-Jones Pressure
Point t piwf Time Function
No. (hours) (psia) (hours) (psi/bpd)

STEP 4: a= S075 bpd

70 9.005 5714.4 1.3053 0.2221


71 9.011 5720.0 1.5323 0.2235
72 9.030 5725.8 1.9208 0.2249
73 9.052 5728.2 2.1700 0.2255
74 9.086 5729.2 2.4246 0.2258
75 9.102 5729.7 2.5255 0.2259
76 9.202 5733.1 2.9700 0.2267
77 9.302 5733.7 3.2888 0.2269
78 9.402 5734.8 3.5516 0.2272
79 9.502 5735.8 3.7824 0.2274
80 9.611 5737.5 4.0090 0.2278
81 9.711 5739.1 4.2032 0.2282
^ 82 9.811 5740.2 4.3867 0.2285
83 9.911. 5740.6 4.5620 0.2286
84 10.011 5742.8 4.7308 0.2292
85 10.211 5746.7 5.0535 0.2301
86 10.411 5748.7 5.3613 0.2306
87 10.677 5751..7 5.7556 0.2314
88 10.844 5752.9 5.9950 0.2317
89 11.011 5755.0 6.2303 0.2322
90 11.261 5756.2 6.5769 0.2325
91 11.511 5758.3 6.9176 0.2330
92 11.761 5760.3 7.2538 0.2335
93 12.000 5762.0 7.5723 0.2339

STEP 5: q 6080 bpd

94 12.005 5766.3 1.7467 0.1879


95 12.011 5769.4 2.0306 0.1885
96 12.030 5773.8 2.5099 0.1894
97 12.052 5775.3 2.8135 0.1897
98 12.080 5776.2 3.0763 0.1899

8-124
Radial
Odeh-Jones Pressure
Point t piwf Time Function
No. (hours) (psia) (hours) (psi/bpd)

99 12.102 5776.6 3.2411 0.1900


100 12.202 5777.6 3.7666 0.1902
101 12.302 5779.1 4.1371 0.1905
102 12.402 5780.1 4.4385 0.1907
103 12.502 5780.6 4.7002 0.1908
104 12.611 5781.6 4.9548 0.1910
105 12.711 5782.2 5.1711 0.1911
106 12.811 5782.6 5.3742 0.1911
107 12.911 5783.5 5.5669 0.1914
108 13.011 5784.2 5.7514 0.1915
109 13.211 5785.2 6.1016 0.1917
11.0 13.411 5786.6 6.4327 0.1920
111 13.677 5788.7 6.8533 0.1924
112 13.844 5789.4 7.1070 0.1925
113 14.011 5790.6 7.3551 0.1928
114 14.177 5791.8 7.5985 0.1930
115 14.377 5793.0 7.8856 0.1932
116 14.511 5793.6 8.0743 0.1934
117 14.844 5795.1 8.5386 0.1937
118 15.011 5796.1 8.7673 0.1939


8-12S
• • •
5800

PROBLEM 4

5600

^ 5400
r^

00

^ 5200

5000


4800
0 1000 2000 3000 4000 5000 6000 7000

Injection Rate, bpd


0.5

Legend PROBLEM 4
o STEP 1: 2000 BPD

q STEP 2: 3100 BPD


0.4-
STEP 3: 4220 BPD

v STEP 4: 5075 BPD o


q o
o STEP 5: 6080 BPD po
0.3-
H
U) q q
X )(X
q

q vv vw
•^
D.2 -
0 00 0
•^
C

0.1-

0.0
102 101 10° 9*10 °

n (a3 q3

Tj (t-tj -1) an a r ^ hrs


j=l

0 • •
^ SPE
Soc,'iety of Petroleum Engineers

SPE 16798

Systematic Design and Analysis of Step-Rate Tests To Determine


Formation Parting Pressure
by P.K. Singh, R.G. Agarwal, and L.D. Krase, Amoco Production Co.
SPE Members

Copyright 1987, Sociely of Petroleum Engineers

This paper was prepared for presentation at the 62nd Annual Technical Conlerence and Exhibition of the Society.ot Petroleum Engineers held in
Dallas, TX September 27-30, 1987.

This paper was selected for presentation by an SPE Program Committee following review of information contained in an abstract submitted by the
author(s). Contents of the paper, as presented, have not been reviewed by the Society of Petroleum Engineers and are subject to correction by the
author(s). The material, as presented, does not necessarily reflect any position of the Society of Petroleum Engineers, its officers, or members Papers
presented at SPE meetings are subject to publication review by Editorial Committees of the Society of Petroleum Engineers. Permission to copy is
restricted to an abstract of not more than 300 words. Illustrations may not be copied. The abstract should contain conspicuous acknowledgment of
where and by whom the paper is presented. Write Publications Manager. SPE, P.O. Box 833836. Richardson, TX 75083-3836. Telex, 730989 SPEDAL

ABSTRACT Determination of FPP is of critical importance


for efficient operation of waterfloods and tertiary
Step rate testing (SRT) has been used for sev- recovery projects. Injection above parting pressure
eral years to determine the formation or fracture can result in premature breakthroughs, poor sweep,
parting pressure (FPP). Unfortunately, little is reduced oil recovery, and loss of costly injection
vailable in the literature regarding SRT design and fluids due to uncontrolled fracture extension. On
alysis. Further, the available guidelines are the other hand, injection at pressures far below the
adequate and may lead to questionable values of FPP implies injecting at a lower rate than an allo-
the FPP. wable maximum and thereby a reduced rate of oil
recovery.
A systematic investigation of several signifi-
cant factors affecting SET design and analysis is Unfortunately, technology development in the
presented. The analysis of SRT data influenced by area of step rate tests has not kept pace with field
wellbore storage and changing wellbore storage is operations even though these tests have been common
investigated. The proper application of multirate place in our industry for over 40 years. This is
analysis methods to SRT data is outlined. A new reflected by a surprising lack of references in the
method is proposed for determining parting pressure literature on the subject and continued problems
from SRT data on fractured wells. Field examples with the interpretation of such tests. To our
are included. This work should significantly knowledge, there have been only two papers published
enhance our ability to successfully design and ana- in the literature,2'3 which directly address step
lyze the step rate tests. rate test (SRT) design and analysis. A brief dis-
cussion is also presented in SPE Monograph 5.4 Our
INTRODUCTION investigation shows that the previously available
guidelines are inadequate and may lead to question-
Step rate testing (SRT) is the primary method able values of the FPP.
used to define the maximum safe injection pressure
without fracturing the reservoir rock. This pres- The entire subject of multiple rate flow into
sure is referred to as the formation or fracture the reservoir during a SRT, coupled with reservoir
pairting pressure (FPP). It is the pressure that fracturing, is a complex one since it requires a
will initiate a fracture in an unfractured well or knowledge of both rock mechanics and fluid flow
will extend an existing fracture in a fractured aspects of the problem. A recently published paper
well. The "FPP" determined from step rate tests is provides some understanding regarding waterflood
equivalent to the term "fracture extension/ induced fracture propagation.5 The analysis of SRT
propagation pressure" useJ in the hydraulic frac- data is also subject to the type of problems which
turing Literature.' The FPP is generally greater are common to other pressure transient tests.
than the "closure pressure" (defined as the pressure
required to initiate the opening of an existing This paper presents (1) a discussion of the
cture) determined from pump-in/flowback tests.l test procedure and the theoretical basis for the
conventional and multirate analysis methods; (2) the
influence of several parameters such as time step
duration, rate increment, wellbore storage, and
changing weilbore storage on test data; (3) the
References and illustrations at end of paper.

491
SYSTEMATIC DESIGN AND ANALYSIS OF STEP RATE TESTS TO
DETERMINE FORMATION PARTING PRESSURE SPE 16798

proper application of muLtirate analysis methods;


(4) a method for determining parting (propagation)
pressure from SRT data on a vertically fractured
well; (5) application of the developed analysis
techniques to field data; and (6) guidelines to
r
pw = pe- [141.2 kh (in re + s)] q ......... (1)
w

design and conduct successful tests in the field.

To aid this study, a single-phase, two-


dimensional, finite-difference gas reservoir model If a steady-state condition is achieved during
was used in a liquid mode to simulate SRT on a each injection step, r and p will be constant. A
single well in an infinite reservoir. Fracture linear relationship wigl therefore exist between
half-lengths were increased arbitrarily in discrete pressure (p) at the end of each constant-rate injec-
lengths to approximately simulate fracture extension tion step and the corresponding injection rate (q).
above the FPP. This limitation, however, does not
alter the conclusions of this study. The analysis For most SRTs, stei time lengths are seldom
methods presented in this paper are not meant to long enough to reach a steady-state condition. In
characterize the fracture propagation mechanisms but this case re can be replaced by r d the drainage
to demonstrate the influence of various parameters radius as defined by Aronofsky an ,d Jenkins. If
on preparting SRT data and on identification of the time step size is constant, a linear relationship
FPP. should still exist between p and q for the test data
below parting pressure. This is because the logar-
REVIEW OF TESTING AND ANALYSIS METHODS ithmic term, In r d /r , will be insensitive to small
changes in rd duringwthe test.
Testing Procedure
A cartesian plot of p vs. q is made for the SRT
The SRT procedure is schematically shown in data. When the formation face injection pressure
Fig. 1. Generally, the test well is either shut-in exceeds parting pressure, the resulting fracture
or stabilized at a reduced but constant injection acts as an additional fluid conductor. This changes
rate prior to the start of the SRT. Ideally, the (reduces) the slope of the p vs. q curve accord-
shut-in period should be long enough so that the ingly. This is illustrated by the p vs. q plot of
bottomhoLe pressure is near the static formation the simulated data as shown in Fig. 2. Since a gas
pressure. Alternatively, if the well is stabilized reservoir model, in an equivalent liquid mode, was
at a reduced injection rate, the stabilization utilized for this study, gas units ( Mcf) and liquid
period should be long enough to achieve a steady-
state (ss) or a pseudosteady-state (pss) condition.
The SRT that follows, consists of a series of
constant-rate injections with rates increasing from
units ( Bbls) have been interchangeably used in
Fig. 2 and elsewhere in this paper. The preparting
data, including the pretest pressure of 1000 psi,
falls on a straight line. Normally, another

low to high in a stepwise fashion. Each constant- straight line is drawn through the points above the
rate step is normally of equal time length. Injec- parting pressure as shown in Fig. 2. The pressure
tion rates and pressures are recorded for each step corresponding to the point where the two lines
and analyzed to determine the FPP. intersect is interpreted as the parting pressure.
This method provides an approximate estimate of
Analysis Method parting pressure. Since the fracture length con-
tinues to increase above the FPP, there is no theor-
To validate the model and demonstrate the etical basis for drawing a second straight line
assumptions and limitations of the existing analysis through the points above the parting pressure. How-
techniques, a six-step SRT with equal length of time ever, on the vertical axis, this line extrapolates
steps (also referred to as time step size) and rate back to a pressure point which is much higher than
increments and no wellbore storage or skin was simu- the pretest pressure. The value of the intercept
lated. The first three steps represent transient with respect to the pretest pressure provides a qua-
radial flow below parting pressure. To simulate litative indication that the pressure points corre-
fracture extension, an infinite conductivity frac- sponding to the second straight line are above the
ture of arbitrary half-length, 2.5 ft, was intro- parting pressure.
duced at the beginning of the fourth step and
subsequently increased to 5.0 ft and 7.5 ft at the (ii) Multirate Analysis
beginning of the fifth and sixth steps, respec-
tively. Note that these fracture lengths were If pressures are recorded with a continuous
chosen arbitrarily simply to demonstrate the anal- readout device and accurate injection rate data is
ysis methods and establish fracture propagation obtained, multirate pressure transient analysis
during steps above the FPP. Actual fracture lengths techniques can also be applied to SRTs. This tech-
attained during the field operations are anticipated nique is based on the principle of "superposition."
to be substantially longer. This, however, should Application of the Odeh and Jones superposition
not affect the interpretation methods for the FPP as method8 to SRT data has been presented in the liter-
outlined in this study. ature.2r4 The Odeh and Jones method assumes tran-
sient radial flow into the reservoir during each
(i) p vs. q Plot

The conventional analysis method assumes a


steady-state Darcy flow into an injection well,6 and
constant rate period. A plot of


is based on Eq. 1.

492
SPE 16798 PRAMOD K. S1NGH, RAM C. ACARWAL AND LOREN D. KRASE 3

(q, q_1)
Pi pwf n2 3 log (t-t._ )
vs 1 qj q^-1)
qn j=1 qn
n-1 - t.-1 qn-1 _ qn
t n-1 ^
At
j^l (^t + n-1 - j-1)
is made of the SRT data. Note that the rates are
considered negative for injection. Theoretically,
this should provide a single straight line of slope
m' and intercept b' for all data below parting pres-
sure, where: Multirate analysis has found limited applica-
tion in the analysis of actual SRT data due to the
lack of continuously measured pressure-time data,
m, = 162.6 NB (2)
kh ............ large sensitivity to the value of initial pressure
used for superposition2'3 and lack of stability of
rate and pressure data. ihese limitations are dis-
cussed in the later sections of this paper. It will
and
also be shown that with the excellent rate control
and pressure measurement devices now available, mul-
tirate analysis is a powerful analysis tool for det-
b, = 162.6 iB (log k 2- 3.23 ermining the FPP from SRT data.
kh oNctrw
FACTORS AFFECTING SRT ANALYSIS

+ 0.87s] .............(3) As with any other pressure transient test, SRTs


are also affected by wellbore storage, skin damage,
reservoir fracturing, etc. In addition, other con-
Formation flow capacity, kh, and wellbore skin, s, siderations such as time step size, rate increment
before fracturing can therefore be determined from and changing wellbore storage are important factors
this line using Eqs. 2 and 3, respectively. The to consider for SRT design and/or analysis and will
method breaks down above parting pressure. Data be discussed next.
points above the parting pressure should no longer
fall on the previous straight line, indicating that 1. Time Step Size
i acturing has occurred and early time data are no
nger in a radial flow regime. The pressure corre- The linear relationship between p and q below
onding to the point where this occurs should the parting pressure for a SRT can also be explained
relate to the FPP. by the transient radial flow equation:

Fig. 3 is the Odeh and Jones plot for the simu-


As expected, data for the first three
P- P = q 162.6NS (log kAt
lated case.
i wf kh 2
preparting steps fall on a single straight line.
The slope and intercept yield the correct kh and
ONctrw
skin, respectively. A downward shift of data points
is observed for each successive step when the frac- - 3.23 + 0.87s] ............. (4)
ture Length is increased. The plot indicates that
the data from the fourth step onward are above the
parting pressure. The same general trend has been Ignoring the effect of superposition, a linear rela-
consistently observed with field SRT data. Field tionship between injection pressure (p) at the end
data for steps above the FPP, however, usually show of each step and rate (q) should exist below the
a continued flattening trend. This is because the parting pressure if a constant time step size (At)
fracture lengths are probably longer and extend con- is used.
tinuously during each step above the FPP. For the
simulated case, it was possible to calculate the To demonstrate the impact of changing the time
fracture length values from the equivalent wellbore step size within a SRT, a preparting (radial flow)
radius concept.4 This was done using the skin values six-step SRT with equal rate increments and no well-
calculated from the radial flow semilog straight bore storage or skin was simulated. Time steps of
line data obtained for each step above the FPP. 12 hours were used for the first three steps and
Such a calculation may not be possible on actual SRT then reduced to 1 hour for the remaining three
data if the data are not in the pseudo-radial flow steps. Fig. 4 is the p vs. q plot for this case. A
regime. change (reduction) in slope is observed when the
duration of the injection step is reduced within the
Although not shown, AgarwaL's multirate equiva- test. This indicates that a false parting pressure
lent time9 can also be applied for the analysis of may be interpreted by decreasing the time step size.
SRT data. This requires making a plot of: Conversely, an increase in slope should be observed
if the duration of the injection step is increased
during the test.
Pwfn Pwfn-1(tn-1)
vs
The above discussion emphasizes the necessity
qn-1 - qn
of maintaining equal step time lengths during a SRT.
Further, from our earlier discussion, it should be
evident that ideally the duration steps should be

493
SYSTEMATIC DESIGN AND ANALYSIS OF STEP RATE TESTS TO
DETERMINE FORMATION PARTING PRESSURE SPE 16798

long enough to overcome wellbore storage and achieve Fig. 8 is a log 6p vs. log W plot for Case 1.
radial flow. For a radial s stem, equations are Data for the storage dominated first three steps lie
available in the literature1^ 12 to calculate the
time when wellbore storage effects become negli-
gible. If the time step site thus calculated is too
large, smaller time steps may have to be used for
on a unit slope line. The rapid increase in pres-
sure during step 4 is caused by changing (reduced)
wellbore storage. The p vs. q plots for Case 1
and 2 are compared on Fig. 9. The pretest pressure

practical considerations. The analysis of such a (1000 psi) and end points for steps 1, 2 and 3 have
test, however, will be affected by wellbore storage, a concave upward curvature. This feature is further
and if fillup occurs, by changing wellbore enhanced by including the effect of changing storage
storage.4 This is especially true for some pressure during step 4. The true preparting straight line,
depleted reservoirs where reservoir pressure is formed by end points for steps 4, 5 and 6, for
lower than the hydrostatic pressure. Case 1, extrapolates back close to the pretest pres-
sure point. A straight Line through points above
2. WeLlbore Storage the parting pressure (steps 5 and 6, Case 2), how-
ever, intersects the y-axis at a pressure much above
Three SRTs were simulated to investigate the the pretest pressure. It should be emphasized that
applicability of multirate analysis to SRT data it would be incorrect to force fit a straight line
influenced by wellbore storage. For each case, time through storage and/or changing storage dominated
steps were so chosen that steps end in a(i) purely steps and interpret the intersection of this and a
radial, (ii) storage-radial transition, and true preparting straight line (if one exists) as
(iii) entirely storage dominated flow regime, parting pressure.
respectively. For each case, log-log plots of Agar-
wal's multirate equivalent time9 are presented in Felsenthal 2 attributes the concave upward cur-
Figs. 5, 6 and 7, respectively. Multirate superpo- vature for the early steps on the p vs. q plot to
sition is applicable for the radial flow regime non-D'Arcy flow downstream from the pressure meas-
case, as expected (Fig. 5). Surprisingly, superpo- uring device. While this is certainly a possi-
sition method also appears to work for the storage- bility, wellbore storage and, if wellbore fillup
radial transition flow regime case (Fig. 6). The occurs, changing storage have the same effect on the
true semi-log straight line is, however, not devel- p vs. q plot. Generally, considering the lower
oped for an accurate kh analysis. Applicability of injection rates for the early steps of a SRT, well-
multirate analysis breaks down for the storage domi- bore storage seems to be a more likely explanation
nated case (Fig. 7). This suggests that time step for this behavior.
size should be long enough for the data to be at
least in a storage-radial transition period. Figs. 10 and 11 are the Odeh and Jones multi-

To identify if SRT pressure data are completely


dominated by wellbore storage effects, a new plot is
suggested. Applying multiple rate superposition to
rate plots for Case 1 and 2, respectively. Even
though radial flow superposition is not rigorously
applicable during the storage dominated first three
steps, the following comments are in order for the

the storage dominated flow Eq. (5):4 general characteristics of the plots: (i) the rapid
increase in pressure caused by changing wellbore
storage is reflected as a much steeper plot of data
98t ... ..........(5) points for the fourth step, (ii) data for the fifth
Pi pwf 24C
and sixth steps fall together for the no fracture
extension case shown in Fig. 10, when storage has
been reduced and the effect of pressure response
for the case of multiple rate injection, will yield
caused by changing storage has diminished. For the
fracture extension case shown in Fig. 11, a downward
B shift in data is noted for each successive step
.............(6)
pwf pi 24C [W] above the parting pressure (steps 5 and 6).

Note that it is possible to reach the parting


where W = cumulative injection pressure at the same time that the wellbore com-
pletely fills up. This phenomenon was simulated by
Eq. 6 suggests that a plot of log (p f - p.) initiating fracture extension (i) at the beginning
versus log (W) will yield a line of unit slope or of step 4, and (ii) during step 4. Results were
data completely dominated by a constant wellbore very similar to Fig. 11. In this situation, the
storage. This is verified in the next section. exact parting pressure was found to be masked by the
changing wellbore storage pressure response. How-
3. Changing Wellbore Storage ever, it was possible to distinguish whether the
data following this step were above or below the
Six-step SRTs were simulated to investigate the parting pressure.
effect of changing welLbore storage from a rising
fluid level type (high) to a compressive (low) type. 4. Rate Increment
In each case, C was reduced from 105, for the
D
storage dominated first three steps, to 103 for the Felsenthal2 suggested using rates of 5, 10, 20,
last three steps. Two cases are compared here: 40, 60, 80 and 100 percent of the anticipated max-
(i) Case 1 - no fracture for the entire SRT simula-
tion, and (ii) Case 2 - a fracture of 2.5 ft half-
length introduced at the beginning of step 5 and
subsequently increased to 5.0 ft at the beginning of
imum test rate as a possible rule of thumb. All the
simulated SRTs presented in this paper use equal
rate increments. The theoretical basis for the p
vs. q plot or multirate analysis, however, does not

step 6. Case 2 is intended to simulate fracture
extension following changing wellbore storage.

494
SPE 16798 PRAMOD K. SINCH, RAM C. ACARWAL AND LOREN D. KRASE 5

require equal rate increments for a preparting reduced constant injection rate. Moreover, the
straight line. A radial flow SRT with no welLbore shut-in period is seldom long enough to obtain a
k t torage or skin was simulated to see the effect of uniform static pressure, p., throughout the drainage
anging rate increments. Although not shown, a area. The question arises las to what pressure value
raight line was obtained on the p vs. q plot. to use for multirate analysis. Felsenthal2 suggests
using the intercept of the preparting straight line
Rate increments should be planned to provide at on the p vs q plot when q=0 for p.. We find this
least 4-5 steps before an expected parting pressure criterion can give a false indicaLion of parting
is reached to establish a good preparting straight pressure especially if data are influenced by
line on the p vs. q plot. For example, Bennett storage effects.
et al.^'used a radial reservoir model with best
available reservoir properties and initial condi- To determine the proper value of the pre-SRT
tions for their SRT design. pressure and to establish what pre-SRT time periods
are important for the multirate analysis, two cases
Rate controllers should be used, if possible, were simulated: (1) SRT following a shut-in period,
to maintain constant rates during each step. Actual and (2) SRT following a stabilized injection period.
injection rates should be recorded throughout the These cases are somewhat analogous to the field con-
test along with continuous bottomhole pressure meas- ditions and are schematically shown in Fig. 14. The
urements to enable a good analysis of the SRT data. pressure at the beginning of the SRT is labelled

pref'
5. Skin Damage
1. SRT Following a Shut-In Period
Skin damage causes an additional pressure drop
in the near the wellbore region and reduces the This consists of a constant long-term injection
pressure which is actually applied to the formation. rate, q ( Fig. 14) for 14 days starting with an ini-
Although not shown here, if the skin damage is tial static reservoir pressure of 1000 psi. The
removed due to high injection rates during a SRT, a well was then shut in and followed by a four-step
shift in data will be observed on the p vs. q plot. SRT of 1 hour time steps. To study the impact of
Data for the remaining steps, however, will fall on the duration of the shut-in times, two cases with
another straight line with a reduced slope. How- shut-in periods of 3 and 48 hours were simulated.
ever, this line should extrapolate back towards the
pretest pressure-rate point for a no fracture exten- Odeh and Jones multirate analysis was performed
sion case. Also, a downward shift in data will be (for 48 hour shut-in) using Pre for p, and ignoring
observed on the Odeh and Jones multirate analysis the rate-time history prior tothe SRT: This
lot corresponding to the step where the skin damage involves performing superposition starting with time
removed, but the data for the remaining steps at the beginning of the SRT (t , Fig. 14). Although
ill not show a continued downward shift. It is not shown, data for all the steps fell on a single
therefore possible to distinguish between the semilog straight line with the correct slope. This
removal of skin damage and exceeding the FPP if they suggests that if the shut-in period is much longer
occur as separate incidents during a SRT. than the SRT step length, p can be used as p. for
multirate analysis, and therpce-SRT rate-time his-
ADDITIONAL CONSIDERATIONS FOR MULTIRATE ANALYSIS tory can be ignored. A single straight line is,
however, not obtained for a similar analysis for the
The use of multirate analysis requires a value case of a 3 hour shut-in as can be seen in Fig. 15.
of pressure and rate at the beginning of the test.
It will be shown that if a proper pressure value is The data for the SRT following a 3 hour shut-in
not used, misleading conclusions may be reached was reanalyzed assuming a pseudo-steady state or
regarding the FPP. The Odeh and Jones superposition steady-state (pss/ss) condition to have been
method8 described earlier inherently assumes that achieved during the initial 14 day long-term injec-
the entire reservoir ( drainage area) is shutin and tion period. Here, the 3 hour shut-in period is
stabilized at p. before the start of the test. A included as a transient step, i.e., the superposi-
single preparting straight line is obtained if tion time function is calculated starting with time,
superposition is applied with the correct value of to, at the beginning of the shut-in period. How-
pi (=1000 psi) for the simulated SRT case shown in ever, Pre is used for p. in calculating the pres-
Fig. 3. sure fuEion for the SR^ data. The data now falls
on a single straight line with the correct semilog
The effects of using lower and higher pressures slope as shown in Fig. 16.
than the actual are shown in Figs. 12 and 13,
respectively. Fig. 12 is an Odeh and Jones plot of 2. SRT Following a Stabilized Injection Period
the same data using a p of 900 psi. A downward
shift in data for all tne steps is observed, which A three-step prefrac SRT was simulated
can be misinterpreted as if all the steps are above replacing the shut-in period by a 7 day reduced
parting pressure. Fig. 13 is a multirate plot of injection rate (q ) period (Fig. 14). Here, the
the same data using a pi of 1100 psi. Here, an pressure at the end of the 7 day injection is pref'
upward shift in data is observed for each successive
preparting step. Data for steps above parting pres- Results of the Odeh and Jones multirate anal-
JWre, however, indicate the downward shift of data. ysis using p for p. and ignoring the pre-SRT rate
history is s own in Fig. 17. Note that data for
OF In actual practice, SRTs are usually run after each step plots with a different slope and none of
either shutting in the well or stabilizing it at a them represent the correct semilog slope. This is

495
SYSTEMATIC DES[GN AND ANALYSIS OF STEP RATE TESTS TO
DETERMINE FORMATION PARTING PRESSURE SPE 16798

not surprising since the basic assumption of a


stabilized shut-in (zero rate) condition prior to


^^1/2
the SRT is not correct for this case. Reworking the _ 4.064 qB
............ (7)
superposition for the case of a stabilized low pi - Pwf x h c k
injection rate (q ) prior to the SRT will result in
the pressure funclion to be (Pref
pwf)/(qn qr)
instead of just q in the denominator. The time
function is also adjusted to account for a finite which, when superposed, yields
rate (q ) prior to the SRT. Data for alL the steps
now fa1f on a single straight line with the correct
semilog slope as shown in Fig. 18. If the reduced pi q Pwf 4.064 B r^ 11/2
injection rate period is not long enough for pss/ss
n xfh IL ctkJ
to be achieved, it should be accounted for as a
transient step in calculating the multirate superpo-
sition time similar to the 3 hour shut-in case dis-
cussed earlier.
(q^ qj-1) (t-t )1/2 ........ .....(8)
j_1
nE qn j-1
The above cases indicate that for the multirate
analysis: (i) a knowledge of p. is not required,
and (ii) multirate test data can be analyzed using
The resulting plot is presented in Fig. 21. Data
any pressure point as p at the beginning of a
for the first three preparting steps fall together.
rate change, and by inc^uing the appropriate rate-
Fracture extension can be noted by a definite shift
time history starting from the last pss/ss period to
in data for each subsequent step. This technique
the time corresponding to Pref'
provides a powerful tool to determine the propaga-
tion pressure for fractured wells where linear flow
SRT ON A FRACTURED WELL
is occurring. This also suggests that the SRT for
fractured well can be designed with shorter time
The analysis techniques discussed in the
steps corresponding to the duration of the linear
preceding sections assume transient radial flow
flow period. Although not shown here, Agarwal's
during the preparting steps of a SRT. For a frac-
multirate equivalent time type superposition can
tured well, if the time steps are not tong enough,
also be applied to the linear flow equation.
the radial flow assumption is obviously violated.
Similar requirements and limitations for the appli-
To investigate the proper analysis method for such
cability of linear flow multirate analysis are
test on a fractured well, a six-step SRT with no
expected to apply as was discussed earlier for the
wellbore storage was simulated. A fracture half

q
radial flow multirate analysis.
length, xf, of 130 ft was kept constant for the
first three steps representing steps below the fra c-
For wells with low finite conductivity frac-
ture propagation pressure. The xf was then
tures, a plot of the linear flow superposition
increased to 140 ft, 160 ft and 175 ft at the begi n-
method applied to SRT data, may also exhibit a shift
ning of the fourth, fifth and sixth steps, respec-
in data corresponding to an increase in fracture
tively, to approximately simulate fracture
width (fracture opening). This is due to the
extension. The fracture was of infinite conduc-
increased fracture conductivity. In this situation,
tivity (FCD>500) throughout the simulation run. T he
it may not be possible to distinguish the fracture
time step size was just long enough for data to be
width effects (fracture opening above closure pres-
in a linear-pseudo radial transition flow regime.
sure) from the fracture propagation effects (frac-
ture propagation above FPP). It appears that a
Fig. 19 is the p vs. q plot of the simulated
bilinear flow13 superposition method may be prefer-
SRT. A reduction in slope, corresponding to frac-
able to use for wells with lc:; finite conductivity
ture extension from the fourth step onward, is not
fractures. However, this aspect has not been
clearly evident on this plot. This suggests that
included in this paper.
the conventional p vs. q analysis may not be ade-
quate for determining the true parting (propagation)
FIELD EXAMPLES
pressure for wells with long preexisting fractures.

1. Field Example A
Fig. 20 is the conventional Odeh and Jones plot
for the simulated case. Data for each step have the
This illustrates the analysis of a SRT influ-
characteristic concave upward semilog shape
enced by wellbore storage and changing storage. The
resulting from early time linear flow. Late time
well was shut-in prior to the SRT and the bottomhole
data for the first four steps falls together. There
pressure was allowed to stabilize. Wellbore fillup
is some evidence of fracture extension for steps
occurred during the third step. A log-log plot of
five and six, reflected by a slight shift in data
Ap vs cumulative water injected (Fig. 22) indicates
points for these steps. However, it appears that
that all data points until the middle of the third
the multirate analysis using radial flow superposi-
step are dominated by wellbore storage. The rapid
tion also may not be adequate for determining the
rise in pressure during the third step is simulta-
parting pressure in such cases.
neous with wellbore fillup and is caused by a

The data was reanalyzed by performing an Odeh


and Jones type multiple rate superposition using the
linear flow equation:4
changing wellbore storage.

Data for the initial storage and changing


storage dominated steps 1, 2 and 3 show a concave

upward curvature on the p vs. q plot (Fig. 23).

496
SPE 16798 PRAMOD K. SINGH, RAM C. AGARWAL AND LOREN D. KRASE

straight line drawn through steps after wellbore 4. Data influenced by wellbore storage and
fiLlup has an intercept-much above the pretest pres- changing wellbore storage will plot with a con-
re point (pref - 1060 psi at zero rate) suggesting cave upward curvature on the p vs. q plot. It
at these steps are above the parting pressure. is incorrect to force a straight Line through
is is confirmed by the Odeh and Jones multirate these steps.
analysis plot shown in Fig. 24. FPP is therefore
interpreted to be 2169 psi, the pressure at the end 5. Multirate analysis can be applied to determine
of the third step. the parting pressure provided the time step
size is at least long enough to be in a
2. Field Example B storage-radial transition flow regime. Accu-
rate kh analysis, however, requires the data to
This illustrates the use of linear flow multi- be in a purely radial flow regime.
rate analysis to determine parting pressure for a
fractured well. The well was shut-in for a falloff 6. The rate-time history before a SRT and the
test just before the SRT. Analysis of the falloff pressure at the start of the SRT should be
data indicated a uniform flux fracture of 110 ft recorded for valid multirate analysis. The
half-length. The SRT consisted of 1-hour time steps proper application of multirate analysis has
of approximately 500 bpd rate increments. been demonstrated.

The p (surface pressures, corrected for fric- 7. Linear flow superposition is a powerful tool
tional pressure loss) vs. q plot is shown in for determining parting pressure from SRT data
Fig. 25. Data for the early steps form a concave on a fractured well. Shorter time steps, lim-
upward curvature. This behavior may be observed ited to the duration of linear flow, can be
simply due to superposition effects even for steps used in this case.
with linear flow. Based on the interpretation (two
straight lines) shown in Fig. 25, parting pressure 8. The techniques developed in this paper have
would be interpreted to be between steps 13 and 14, been successfully applied to field SRT data.
at 975 psi.
NOMENCLATURE
The radial flow Odeh and Jones plot for this
case did not give a clear indication of parting b' = intercept of tine from Odeh and Jones anal-
pressure and is not shown. Fig. 26 is the linear ysis (see Eq. 3)
flow Odeh and Jones plot. The falloff before the
SRT is accounted for as a transient step and pres- B = formation volume factor, RB/STB (res m3/stock
ure at the end of the falloff (p ) is used for tank m3)
^perposition. Clearly, data fore first ten
teps fall on a single curve. A shift in data for ct = total system compressibility, psi-' (kPa-')
steps beyond step 10 indicates that these steps are
above the parting pressure. The true FPP is there- C = wellbore storage factor, RB/psi (res m3/kPa)
fore interpreted to be 744 psi, the pressure at the
end of the tenth step. FCD = dimensionless fracture conductivity

Note that for this example the p vs. q plot h = formation thickness, ft (m)
(Fig. 25) does not show a definite change in slope
corresponding to the FPP. In fact, data points for k = formation permeability, md
all the steps form a smooth S-shaped curve.
Interpretation for parting pressure from this plot m' = slope of line from Odeh and Jones analysis
is, therefore, very subjective. Clearly, linear (see Eq. 2)
flow multirate analysis is a superior analysis tool
for this case. p = pressure, psi (kPa)

CONCLUDING REMARKS pref = pressure at the beginning of a rate change


(beginning of the SRT, see Fig. 14), psi
A systematic investigation of a number of sig- (kPa)
nificant factors affecting SRT design and analysis
has been made. Based on the results of this inves- ISp = pressure change, psi (kPa)
tigation, the following conclusions can be made:
q = flow rate, negative for injection, STB/D or
1. Previously available guidelines for SRT proce- Mscf/D ("standard" m3/D)
dure and analysis are not adequate.
qL = long term stabilized rate prior to the SRT
2. Equal size time steps of length long enough to (see Fig. 14), STB/D or Mscf/D ("standard"
be out of wellbore storage should be used for m3/D)
an ideal SRT.
q = reduced or changed rate prior to the SRT (see
3. Data completely dominated by welLbore storage r Fig. 14), STB/D or Mscf/D ("standard" m3/D)
will plot with a unit slope on a log-log plot
of Ap vs. cumulative injection (production) for r = radius, ft (m)
a constant wellbore storage factor.
s = skin factor

497
SYSTEMATIC DESICN AND ANALYSIS OF STEP RATE TESTS TO
8 DETERMINE FORMATION PARTING PRESSURE SPE 16798

t = injection time, hours 4. Earlougher, R. C., Jr.: Advances in Well Test


Analysis, Monograph Series Volume 5, Socie=v of
At = SRT time step size or incremental time during Petroleum Engineers, Dallas (1977).
the injection period, hours
5. Hagoort, J., Weatherhill, B. D., and Settari,
W = cumulative injection, STB ("standard" m3) A.,: "Modeling the Propagation of Waterflood-
Induced Hydraulic Fractures," Soc. Pet. Eng. J.
X f = fracture half-length, ft (m) (Aug. 1980) 293-303.

N = viscosity, cp (Pa•s) 6. Staggs, H. M., Jr.: "Injection Well Skin


Effects: Plus or Minus?" Paper No. 851-41-B
0 = formation porosity, f raction presented at the API Mid-Continent District
Meeting, Oklahoma City, Oklahoma, March 29-31,
Subscripts 1967.

D = dimensionless 7. Aronofsky, J. S. and Jenkins, R.: "A Simpli-


fied Analysis of Unsteady Radial Gas Flow,"
d= drainage Trans., AIME, Vol. 201.

e= external boundary 8. Odeh, A. S. and Jones, L. C.: "Pressure Draw-


down Analysis, Variable-Rate Case," J. Pet.
i = initial Tech. (Aug. 1965) 960-964.

j = index 9. Agarwal, R. G.: "A New Method to Account for


Producing Time Effects When Drawdown Type
n = index Curves are Used to Analyze Pressure Buildup and
Other Test Data," paper SPE 9289 presented at
w = wellbore the 55th Annual Technical Conference and Exhi-
bition, Dallas, Texas, Sept. 21-24, 1980.
wf = flowing conditions
10. Agarwal, Ram G., Al-Hussainy, Rafi, and Ramey,
ACKNOWLEDGEMENTS H. J., Jr.: "An Investigation of Wellbore
Storage and Skin Effect in Unsteady Liquid
The authors thank Amoco Production Company for Flow: I. Analytical Treatment," Soc. Pet. Eng.
permission to prepare and publish this paper. The J. (Sept. 1970) 279-290; Trans., AIME, 249.
helpful comments provided by Ian Palmer and Hussien
Khattab during the preparation of this paper are 11. Ramey, H. J., Jr., Kumar A., and Culati, M. S.:
also acknowledged. "Gas Well Test Analysis Under Water-Drive Con-
ditions," AGA, Arlington, VA ( 1973 ) .
REFERENCES
12. Vongvuthipornchai, S. and Raghavan, R.: "A
1. Nolte, K. G.: "Fracture Design Considerations Note on the Duration of the Transitional Period
Based on Pressure Analysis," paper SPE 10911 of Responses Influenced by Wellbore Storage and
presented at the 1982 SPE Cotton Valley Sympo- Skin," Paper SPE 15273, 1986.
sium, Tyler, TX, May 20, 1982.
13. Cinco-Ley, H. and Samaniego, V. F.: "Transient
2. Felsenthal, M.: "Step-Rate Tests Determine Pressure Analysis for Fractured Wells," J. Pet.
Safe Injection Pressures in Floods," Oil and Tech. (Sept. 1981) 1749-1766.
Gas J. (Oct. 1974) 49-54.

3. Bennett, C. 0. and Clark, T. J.: "Design and


Analysis of a Nitrogen Step-Rate Test, Anschutz
Ranch East Nitrogen Injection Project," paper
SPE 14456 presented at the 60th Annual Tech-
nical Conference and Exhibition, Las Vegas, NV,
Sept. 22-25, 1985.

is
498
.M. • •
^
i^
q3
`+4

O
U q2
^
Q1

t1 t2 t3 t4 t5 3000
0.20
t6
Steps 1-3: Radial Flow
Time Steps 1-3: Radial Flow Steps 4-6: Frac Extension
0 Step 4: xf = 2.5 ft
2500 U
to Step 5: xf = 5.0 ft
{ ;t 0.15
On Step 6: xf = 7.5 ft
a
C
=1 ^ + - - FPP -
2000 Q I c
^ I Cr
--*"
^ I 1 I ^ °
a I { I Steps 4-6: Frac Extension 0.10

^ '/2
1 i I ^ Step 4: xf = 2.5 ft
C Time f 1500
[Step Sizel Step 5: xt = 5.0 ft
1 Step 6: xt = 7.5 ft pi = 1000 psi

I I I I 1000 V.uJ
et et ^ et At at At I 0 2 4 6 8 10 12 14 2'10-2 10-1 100 101 4010'
Time
Injection Rate, MMscf/D Odeh and Jones Time (Radial), hours
Fig. 1-Schamafic of a typical step-te test procedure

Fig. 2-Prauure w. rate for slmulated data ( Ca =0, 9=0). Fig. 3-Odeh and Jonea multlnte analysis of simulated data (Co•0. s=0).

3250 5x10-1 . . . . . , ., . . . . . . .., . . . . . . ..I . . .


-

Steps 1-6: Radial Flow Storage-^-Transition --►f Radial


Steps 1-3: At = 12 hrs 0 ocw° °'i° x2 3 4
Steps 4-6: At = 1 hr N CEW
10-1
rn 2500
n CL Unit Slope---//°
0)
m Yc c
aN)
^
CL 1
1750 Q ° Step 1
CL 10-2 * Step 2 _
* Step 3 =
v Step 4 =

1000 "
0 2 4 6 8 10 12 14 10-2 10-1 10o 101 5x101

Agarwal's Multirate Equivalent Time (Radial), hours


Injection Rate, MMscf1D

Fig. 4-E/lecl of changing Ilmestep size. Fig. 5-Agarvral's multirale equWelent lima anelysis steps ending In radial Ilow.
5x10 - 3x10-1
Storage --H-- Transition u Step 1
^
3 4 ^ 10-1 q Step 2 i v
^ to 4
^ Step 3
i ¢ - a - v Step 4
n 10-
Unit Slope--- 1
q .
d+' a°o
C vN q
Cr Cr X0

a i 10-2 ° qo q
C' ka
C
Q Cr
a o Step 1 CL
K° q
10- 2 q Step 2
K ^Unit Slope
k Step 3
7, Step 4
10-3

1 0-2 10-1 10o 10 10-2 10-1 100


10-3
Agarwal's Multirate Equivalent Time (Radial), hours Agarwal's Multirate Equivalent Time (Radial), hours

Fig. 6-Agarwal's mulluata equivalent lime analysis steps ending in slorffge-radial transition, Fig. 7-Agarwa1's muttlrala aquivalanl 11me analysis steps ending In storage-dominated flow.

3x103 4000
103 Steps 1-6: Radial Flow Steps 1-3: CID = 1x105
o Step 1
Steps 4-6: CD = 1x103
nStep2 C0=1x105 6
.s lep 3 Case 1 - No Fracture
102 o Step 4
N Q 3000 o Steps 1-6: Radial Flow
a * Step 5 Cp=1x103 1
a
N Step 6
101 d)
^
^--
Slope 2000 ^
10o Case 2 - Frac Extension
Z
/ q Step 5: xt = 2.5 ft
o3 v Step 6: xt = 5.0 It
^^2
10 - 1 r, . ... .. , . , . - , . .....^r . ^ . .....^ . 1

1000 0 5
2x100 101 102 103 104 3.ex104 10 15

Cumulative Injection, Mscf Injection Rate, MMscfD

Fig, 6-1p vs. cumulative Injection for changing weltpora storage: Case 1-na fracture. Fig. g-praNura no. rata for changing wellbore storage: 11) no fracture and 12) fracture extension,

0. , , . ^ ^ ^,.I ^ , , . , , ^iI i -r -i^r-rr^ ^ --r---• ^ 0.20


case 2- rac Extension 4
Case 1 - No Fracture
5 Steps 1-4: Radial Flow
o Step 1 ^4
Step 5: xt = 2.5 ft °
5 q Step2 Cp=1x105
y 0. 0.15 Step 6: xt = 5.0 ft ° o, ^
NStep 3
xx
V Step 4 ° 0 Step 1
a xxxxxXxx
oStepS CD=1x10 a q Step2 Cp=1x105 °
a c 0. 0 X Step 6 ° x Step 3
i ^ 0.10
a / v v a o Step 4 °
Cr
a o Step 5 CO = 1 x103
x Step 6 °
0.1 5 0.05 vv° k3
. '3
kO
M q G N
k^`2
q
X N N K N k k CO N N k N k k
n O 1 n C
^ o 0 u C q q o q q 11 O O E3 q O
n _n,l o : 9 , , ,. 0 . ^
OL 0.00
2x10-2 10-1 100 101 4x101 10-1 100 101 4x10
20 0-2
Odeh and Jones Time (Radial), hours Odeh and Jones Time (Radial), hours

Fig, 10-Otleh and Jones analysis lar changing welloora slorage Case 1-no Iraclure. Fig. 1 1-Odeh and Jenes analysis fer changing waIlhore Wage: Case 2-trectura exlensian.

500
0.25 0.20 - 111111! , , , ,,,,,, , . ..,.. , , .
Steps 1-3: Radial Flow
Steps 1-3: Radial Flow
Steps 4-6: Frac Extension ".3
Steps 4-6: Frac Extension
Step 4: xf = 2.5 ft o 2
0 Step 4:xf=2.5ft qq 2
100.20 Step 5: xf = 5.0 ft "3 0 0.15
U
Step 5: xf = 5.0 ft v4
=x
Step 6: xt = 7.5 ft o N
Step 6: xf = 7.5 ft q "q q vvov°vo x o.5 6
o q q" x 4 Q xx
o v°
o ° " vvv ° q 0 pOO^ooo °
xxxxxx
° o
CL 0.15 - o q vvvv 5_
0.10 O
o vv V oo°o Jo
xyY CL I C 0
C. C xx" 6 I ItS
0
1 Q xxxxx
a 0
0
0.10 0.05 0
pi (actual) = 1000 psi pi (actual) = 1000 psi
0
pi (for superposition) = 900 psi pi (for superposition) = 1100 psi

0.05 t- 0 00
2'10-2 10-1 100 101 4'101 2`10-2 10-t 100 101 4'101
Odeh and Jones Time (Radial), hours Odeh and Jones Time (Radial), hours
Fig. 12-Ettect of using lower than actual p, on Odeh and Jones analysis.
Fig. 17-Effect of using higher than actual pi on Odeh and Jones analysts.

q4
^ 0.150
m qL q3

C
q2 I
0 I
q , I i
U I ^
^
c qr # 0 i ° 0.125
rn
Cr
qr=0

Reduced Q 0.100
I- Long Term I ^
+ Step Rate Test ^
^ I injection '' Rate or
Shut-in a`

0,075
t
v
z
fNA 0.050
^ t
L
2'10-3 10-2 10-1 100 3'100
pref f Odeh and Jones Time (Radial), hours
----t----- , , t
At At i At At
Fig. 15-Odeh and Jones analysis of SAT following a short shut-In ignoring the shut-In period.

t0 `1 `2 f3 i4
Time

Fig. lc-Schematic of SAT following shut-in or reduced-rate injection.

0.150 IIII 1 r 0.12 ,^IIIIi , I , ,I ,,,i , . I rrr„


Steps 1-4: Radial Flow Steps 1-3: Radial Flow x
o Step 1 1 3
o Step 1 <^O2
° 0.125 q Step 2 2 0 o Step 2 ^ q
U
" Step 3 1 * Step 3 ^ G° q
a v Step 4 ^ ^
Q 0.08 ^ O -
q

/ / q OO1
0.100 s I, 0
CL a
C .' o
I Q v o
a` i2
0.04 C)
0.075

^
0 050 0.00'
2'10-3 10-2 10-1 0 2'10--3 10-2 10-1 100 3'100
Odeh and Jones Time (Radial), hours
Odeh and Jones Time (Radial), hours
Fig. 18-Otleh and Jones analysis of SAT follow,ng e short shut-in accounting for the shut-in period.
50, Fig. 17-Odeh and Jones analysis of SAT following reduced-rate Injection Ignoring the reduced-rate infection period.
0,175 ....^

Steps 1-3: Radial Flow


. , 1750

Steps 1-3: xf = 130 ft
o Step 1 2 Steps 4-6: Frac Extension
0.150
q Step 2 ^
Step 4: xt = 140 ft
q
U * Step 3
1500 Step 5: xt = 160 ft
>t^p
_
0.125 Step 6: xt= 175ft
CL ^ .
p0
^
CL
0.100
y c
1250
a
2
0.075

0050 1000
0 100 200 300 400 500 600
2'10-3 10-2 10-1 100 2'100
Odeh and Jones Time (Radial), hours Injection Rate, STBJD

Fig. 16-Odeh and Jones analysis of SRT lollowing reduced-rate injection sccounting for the nduced+al. Fig. 19-Presaure as. ral. lor sunufated data (Iracturnd wNl1.
Inject ion period.

1.5
Steps 1-3: xt = 130 ft Steps 1-3: xt = 130 ft
Steps 4-6: Frac Extension Steps 4-6: Frac Extension 3 °4
3 2 x Ya° 040% 5x 6
Step 4: xf = 140 ft q step 4:xt= 140 ft
q
M
F-
^
a
10
Step 5: xt = 160 ft
Step 6: xt = 175 ft

CL
^
CL
1.0 Step 5: xt = 160 It
Step 6: xt = 175 ft

o° 9b
Q^
&51^
1^,°°O° oou^%xx'ex
41° •
o o Step 1 I Cr °° o Step 1
Ia Q o 00
a ° 0 Li Step 2 a- oo° o Step 2
0.5 0.5 » Step 3
0 » Step 3 0 °°
0 v Step 4 o Step 4
O 0 Step 5 o Step 5
0
0 x Step 6 0 X Step 6
0
0.0 00
X1 0_2 10-1 100 101 2x 01 0 1 2 3 4 5

Odeh and Jones Time (Radial), hours Odeh and Jones Time (Linear), (hours) 112

Fig 20-Odeh and Jonas mullirate (radial /low) analysis of cimul,f.d dots practured w.11). Fig. 21-Odeh and Jones multlnl. plnear now) analysis of simulated data (fractured welQ.

3x103

103

Fill-up

N
C. Unit
d 102 ° Step 1 =
° Step 2 =
M Step 3
o Step 4
o Step 5
x Step 6
101 ;_

101 100 101


+ Step 7
I-]
Cumulative Injection, STB
Fig. 22-p vs. cumulative mlKnon for F.eld E.emple A.

507
^

2500 b

Wellbore Fill-up 5 4
3 ^
CL 2000 ^
^
^ a 3

in
m CL
a` ^ 2
02 w
1500 a`

01

pref
1000
0 100 200 300 400 0
10-2 10-1 10o 3x10')
Injection Rate, STBlD
Odeh and Jones Time (Radial), hours
Fg. 23-Pressure vs rate for Field ExamVe A. Fig. 2e-0deh arq Jones radl•I Ilcw ansrysra ror FieM Exampte A.

i 0 0.15
$ 9^ 11 12 13 ^1s
w o>n faxrIm 17
0 s
0 ; 0.10
4
^ CL „ _A • Step 10
^,'d is Step 11
a
L 2 oe m Step 12
Q
Step 13
0.05
Step 14
Step 15
Step 16
^ q Step 17
0 n nn
0 2500 5000 7500 10000 0 0.5 1 1.5 2 2.5
Injection Rate, STB/D
Odeh and Jones Time (Linear), (hours) 1!2
Fig, 25-Pressure vs. rau for Field Example B. Fig. 28-OdaA and Jones Ilnur flow enelyxls for Field Example B.


503
Chapter 9
0

INTERFERENCE AND PULSE TESTS

I. INTRODUCTION

The majority of pressure transient tests conducted by


the petroleum industry, i.e., drawdown, buildup, injectivity,
falloff and multirate, involve only one well. These tests
are relatively simple to run and analyze because the rate
change required to create a pressure disturbance, and measure-
ment of the pressure response caused by this rate change,
occur in the same well. Single-well tests yield useful
information about flow capacity near injection or production
wells, wellbore conditions, the existence of fractures, faults
and boundaries, fracture length, and distances to faults and
boundaries. However, single-well tests tend to give average
reservoir properties and, when run in anisotropic reservoirs,
yield no information about the directional nature of
reservoir properties. Further, single-well tests generally
do not indicate if the test well is in communication with
adjacent wells.
Interference and pulse tests are multi-well tests which
require a minimum of two wells, but may involve several wells
when the directional nature of reservoir properties is
required. Figure 9.1 illustrates two wells being used to
conduct an interference or pulse test. The general procedure
used to conduct a multi-well test is to change the rate at
^ the active well (injector or producer) while measuring the
resulting pressure response at an observation well. The
observation well is generally shut in for the pressure
measurement. It is possible from the pressure response of
the observation well to compute the flow capacity and porosity-
compressibility product between the wells and, when multiple
observation wells are used, to determine the directional
nature of flow capacity.

Fig. 9.1: Active and observation wells used to


conduct an interference or pulse test.

Interference and pulse tests differ primarily in the


rate behavior of the active well. When conducting an
interference test, the rate is generally changed only one
time. A pulse test, however, involves several rate changes
of short duration. The advantages and disadvantages of
both tests will be discussed in subsequent sections. An
excellent paper by Kamali summarizes various types of
interference and pulse testing and dilineates the advantages
and disadvantages of techniques which can be used to analyze
these tests.
Multi-well tests are more difficult to run and analyze
than single-well tests. However, they offer the following

9-?
advantages:

0 tests can determine if two or


•Multi-well
more wells are in pressure communication.
This information is of particular importance
in the selection of injection and production
wells for enhanced oil recovery projects.

• The directional nature of permeability, or


fractures, can be determined in anisotropic
reservoirs. Again, this information is
critical in the design of enhanced oil
recovery projects.

• Multi-well tests generally investigate a larger


portion of the reservoir than single-well
tests. It is commonly believed that multi-
well tests provide information only about
that part of the reservoir between the active and
observation wells. Vela and McKinley2 have
shown, however, that the area of investigation
is approximated by a rectangle of width Zri
and length (r + 2ri), where ri is the radius
of investigation of the active well and r is
the distance between the wells. The
approximate area of investigation is depicted
by Fig. 9.2.

The primary objectives of this chapter are to introduce


the reader to the basic concepts of multi-well testing, and
to illustrate how interference and pulse tests can be analyzed.
Many variations of multi-well tests exist which will not be
covered; this material will only consider applications to
simple systems.

Ll
9-3
ri

Fig. 9.2: Approximate area of investigation


of multi-well test (After Ref. 2).

II. INTERFERENCE TEST

An interference test is conducted by changing the flow


rate of one or more injection or production wells and measuring
the resulting change in pressure at an adjacent observation
well. In its simplest form, this test will involve only two
wells as depicted by Fig. 9.1; however, multiple observation
wells may be employed if information about directional
characteristics of the reservoir are required. Figure 9.3
illustrates the rate history and pressure response of an
interference test in which the active well is placed on
production at constant rate; this causes an immediate pressure
drawdown at the active well and, after some lag time required
for the pressure disturbance to reach the observation well,
a decrease in pressure at the observation well. The obser-
vation well is generally shut in during this test.
The rate change required to conduct an interference
test can also be achieved by shutting in the active well.
The rate history and pressure response for this procedure
is depicted by Fig. 9.4 for a two well test. Again, it is
assumed that the observation well is shut in.
F-1
L_J

9-4

q

¢-^

T I ME

LAG
T IME
• OBSERVATION
IVELL

z \
O :!;

ACTIVE WELL

T I,^iE

Fig. 9.3: Rate history and pressure response


of a two-well interference test
conducted by placing the active
well on production at constant
rate.

9-5

q

^
a

E -r
^

0
I----^ A t

0
t TIME

ACTIVE
WELL

OBSERVAI'I0.^
WELL

/
/ apobs
cz

^j
°
l:STaBLI51-IL'D
T I mL-'
I'RENll
LAG

0 t
TIME

Fig. 9.4: Illustration of rate history and


pressure response for an interference
test where the active well is shut in
for the test.

0
9-6
A. Homogeneous Isotropic Reservoir
^ The simplest testing situation occurs when the reservoir
can be assumed homogeneous and isotropic, i.e., when porosity,
thickness and permeability are the same everywhere, and are
equal in all directions. We will consider first how to analyze
"'a test which involves only one active well and one observation
'well, and then how to modify the calculations for multiple
active wells.

1. Single active well


Consider the two-well system depicted by Fig. 9.1 in
which the active and observation wells are separated by a
distance r. If the active well is placed on production at
constant rate, as illustrated by Fig. 9.3, the pressure change
at the observation well, Apobs' can be predicted by the
exponential integral equation. According to Eq. 2.31,

p(r,t) (9.1)
• Lpobs pi pi pobs

aBU !uctr2
-70.6 kh Ei 948 (9 . 2)
^pobs kt .

The purpose of the interference test is to measure Apobs


and, from Eq. 9.2, to determine the permeability, porosity,
or porosity-compressibility product of the reservoir. This
can be accomplished using (a) type curves, (b) trial-and-
error, or (c) least square analysis.

a. Type Curve Solution


The type curve used to analyze interference tests is
the exponential-integral solution. This curve, a plot of
PD versus tD/rD, was previously presented as Fig. 2.24 and
is reproduced here in Fig. 9.5. This curve, although it
^ represents a zero-skin factor, can be used because the skin
effect does not influence the behavior of the observation

9-7
104 IOS 106 107 108 109

G^+

l0

00

10-I I 10 102 103 104

t D/ / I' 1')

Fig. 9.5: Dimensionless pressure for a single well in an ideal infinite system, i.e.,
the exponential-integral solution.

0 0 0
well; an exception to this occurs when a large negative skin
^ is present because of fracturing3. Wellbore storage effects
are also minimized, but not eliminated, by interference
testing`:-6 Consequently, since only one curve must be
considered for infinite acting systems, type curve matching
is simpler for interference testing than for single-well
testing'.
The procedure for using type curves to analyze inter-
ference data was described in Chapter 2 (pp. 2-75 to 2-84).
This method requires that pressures measured at the obser-
vation well, Pobs' be plotted on tracing paper as Apobs versus
t where Apobs pobs(t=0) and t is test time. It is
pobs
necessary that the log-log scale used to plot these data be
identical to the scale on the type curve. The data curve is
placed on the type curve and moved horizontally and vertically
until the best match is obtained. This match is illustrated
by Fig. 9.6. With the curves in a matched position, a
convenient match point is chosen. Permeability can be
estimated as

k = 141 . 2qBu (pD) M


h (9.3)
(Ap) N1 '

and the porosity-compressibility product as

= 0.0002637k (At) M
(9.4)
^ct ur2 (tD/rD)N1

Examples of using this method to analyze field data were


previously presented in Example 2.7 and Problem 2.4.
The analysis of interference data from bounded
systems can be more difficult than for infinite-acting
reservoirs. Whereas only one solution exists for an
infinite-acting system, the solution for bounded systems
. is a function of drainage geometry and well location.
Earlougher and Ramey8 have studied this problem and present

9-9
DATA
CURVE

tI)/rD
is
Fig. 9.6: Illustration of type curve match for an
interference test.

type curves for more than 150 drainage geometries and well
locations. The analysis of data from a bounded system
where the drainage configuration is not known can be very
difficult because of the large number of solutions from which
one must choose. An example which illustrates the use of
type curves to analyze data from a bounded system is
presented by Earlougher'. #i

b. Trial-and-Error Solution
Equation 9.2 can be solved by trial-and-error using
the following procedure:


9-10
1) Assume a value of ^uct/k.

• 2) Calculate Apobs versus t by substituting the


assumed value of ¢uct/k into Eq. 9.2.

3) Compare values of Opobs calculated using


Eq. 9.2 and those measured during the test.

4) Continue assuming values of ^uct/k until the


best match is obtained between calculated and
measured values of Apobs' The value of ^uct/k
giving the best match is assumed to be the true
reservoir value. Depending on the information
available, k, or ^ct can be computed from the
correct value of ¢uct/k.

This trial-and-error procedure is graphically illustrated by


Fig. 9 . 7.

is c. Least Square Method


The procedure for solving Eq. 9.2 using the least square
method is:

1) Assume a value of ^uct/k.

2) Compute Apobs versus t for the assumed


value of ^uct/k using Eq. 9.2.

3) Compute

n
[(APb)tt (Apobs) calcl 2
i=1

where n = number of data points.


9-11
- MEASURED
^ • .
^ (^uct/k)1 A A A
A

n (^uct/k) 2 AA nn

n (^uct/k)3 ^ n
A n
A
0 A n
A n

A n
n

^n
CORRECT
SOLUTION

TIME

Fig. 9.7: Graphical illustration of trial-and-
error procedure to determine ^uct/k
from an interference test.

4) Repeat Steps 1-3 for other values of


^uct/k.

5) The correct value of ^uct/k is the one


which gives a minimum value of the
summation.

This procedure is illustrated graphically by Fig. 9.8.


If the observation well is producing before the
interference test and is shut in for the test, i.e., as
depicted by Fig. 9.9, the equation used to predict APobs
must account for this. In this case, there will be an

9-12
• N

^
r^l

J
^.f

^
Q

--'^

VALUE
I I^

^uct

Fig. 9.8: Graphical procedure to determine
correct least square value of
Wt/k.

initial pressure buildup at the observation well caused by


shutting it in; this will be followed, however, by a
decrease in pressure caused by depletion (or interference)
from the active well which continues to produce. The
pressure at the observation well can be predicted by
superposition to be

162.6 k qBP log tGOt


pws = p * -

au
aB ct
^ur2
+
• 70.6 kh Ei -948
kt A
(9.5)

9-13
- OBSERVATION "iiELL
-- A C"I I%- 1: ^',ILL

q

Lz^
aA
-------------

i --^ta
77

At
0 ---^

t
T IME

OBSERVATION WELL

•r,
--ACTIVE WELL

pext

z pobs
c

T I ME

Fig. 9.9: Rate and pressure history of a


two-well interference test where
the observation well was flowing
prior to the test. •
9-14
where:

^
d= production rate at the observation well
before it was closed in, STB/D
qA = production rate at the active well, STB/D
t = length of time observation well flowed
before shut-in, hrs
tA = length of time active well has been
producing when pws is calculated, hrs.

Therefore, the pressure change caused by interference at the


observation well will be

(9.6)
Apobs Pext Pobs

qABU !uctrz
kt (9.7)
Apobs = 70.6 ^ Ei -9 48
I A

The pressure, Pext, represents the pressure which would


have occurred at the observation well had the interference
not occurred. This pressure can be determined by ex-
trapolating the Horner straight line obtained before the
interference effect reached the observation well. The
pressure, pobs' is the pressure measured at the observation
well during the test. The relationship between Pext'
pobs and APobs is depicted by Fig. 9.10. The results of
this test can be analyzed by any of the three methods
previously described.
An interference test can also be conducted by shutting
in the active well. This procedure was illustrated by
• Fig. 9.4. The equations and procedures for analyzing this
type of test are similar to those previously described.
An example of this type of test is presented by Earlougher^


9-15
/
// E
/
pext
:r.
^
/
/ l.

"pobs
/• • ^
^

z
0

pobs
J

10 3 102 10 1

t+At
At

r-^
Fig. 9.10: Horner plot showing the pressure
change caused by interference.

2. Multiple Active Wells


When more than one active well is involved in the test,
the effect of each well on pressures at the observation well
must be considered. According to the principle of super-
position, the total pressure change at the observation well
is the summation of pressure changes caused by each of the
active wells. In this situation, the pressure at the
observation well is
y I

B log (tQA t
p* - 162.6
pobs - F t

N
^uctai
qaiEi (-948 kt
+ 70.6 ^ Ai
) (9• 0
i

9-16
where:
N = number of active wells
0 q = rate of production at active well i, STB/D
tAi= length of time active well i has been
producing, hrs
ai = distance from observation well to well i, ft.

Therefore, the pressure change at the observation well


caused by interference from all active wells is

(9.9)
Apobs Pext pobs

N ^uctai
Bu (9.10)
Apobs = 70.6 kh qt^i Ei(-948
ktAl
i=1

If the observation well has been shut in prior to the


^ interference test, Eq. 9.8 can be modified by setting
q=0.
This type of test can be analyzed using the same
methods previously discussed for two-well tests. The
following example illustrates the use of the least square
method to analyze an interference test with two active
wells.

Example 9.1: Analysis of a three-well interference test


using the least square method.

Problem. The following interference data are for a new


reservoir containing three wells as depicted by Fig. 9.11.
The rate history of the three wells is illustrated by Fig.
9.12. The observation well, Well 1, was produced at a rate
of 120 STB/D for 70 hours before being shut in for the
interference test. Pressures recorded in the observation

9-17
well following shut-in are given. Well 2 was on production
at a rate of 190 STB/D for 100 hours prior to the time Well
1 was shut in; Well 2 continued to produce at this rate
during the entire test. At the time Well 1 was shut in,

Well 3 had been producing at 80 STB/D for 50 hours; it also
continued to produce at this rate during the test.
Calculate the average porosity of the reservoir. Other
data are

Po = 0.8 cp h = 30 ft
Bo = 1.15 RB/STB co = 8 x 10 5 psi-1
Sw = 0.2 cw = 3 x 10-6 psi
So = 0.8 C f = 4 x 10-6 psi -1
pi = 4,485 psia rw = 0.276 ft

Fig. 9.11: Well arrangement for


interference test, Example 9.1.


9-18
240

i• 200
WELL 2

^
160

WELL 1
120

lti l: LL 3
^ 80
.=.i
O
r^ 40

0 100 ?on ;nn ;^nn

T IME , H RS

Fig. 9.12: Rate history, Example 9.1.

i•
At, hrs t^^t pobs' psia
0 - 4213
5 15 4380
10 8 4413
20 4.5 4433
30 3. 3 4443
40 2.8 4450
50 2.4 4455
100 1.70 4466
150 1.47 4472
200 1.35 4473
250 1.28 4474
300 1.23 4478
400 1.18 4480

• 500
800
1.14
1.09
4470
4461
1200 1.06 4448
1500 1.05 4439

9-19
Solution. A Horner plot of pressures at the observation
well is presented in Fig. 9.13. It is observed that the
data form a transient flow straight line at early times
before the observation well is affected by production at
Wells 2 and 3. At late times, however, pressures at the
well begin to decrease because of interference effects.
The early time data can be analyzed as a normal buildup
test to obtain permeability. The slope of the Horner
straight line is

m = -79 psi/cycle.

Therefore, from Eq. 5.8,

k = 162.6 m

k = (162.6) ( 120) (1.15) (0.8)


- 79) 30

k = 7.6 md.

The pressure change, opobs' caused by interference


can be computed using 9.1:

Bu 2 ^uctai
qEi(-948 kt
[APobs]caic = 70.6 Al

(70.6) (1.15) (0.8) X


^Apobs1calc ( 7.6 )( 30 )

^u^
{190 Ei - (948) (800) 2 k 100+At

+ 80 Ei 1- (948(600) 2 k SO+At

9-20
i• 4500

4450

4400 -----^ __ .
-_--^- - -

.r.,

4350
n
0

4300

4250

4200
1 10 40

t+At
At

Fig. 9.13: Horner plot, Ex. 9.1.


9-21
= 0.285 {190 Ei -6.07 x 108 k 100+At)
[P0bS]
calc

^u^
0
+ 80 Ei -3.41 x 108 k 50+Ot

The pressure change actually observed during the well


test can be computed using Eq. 9.9, i.e.,

LPobs]test pext pobs

The extrapolated pressure, pext, is obtained from an


extrapolation of the Horner straight line, i.e., Fig. 9.13,
at the time of interest. The pressure, pobs' is the
pressure measured at the observation well.
It is observed from Fig. 9.13 that only the last four
points are affected by interference from the active wells.
Using these four points, Table 9.1 presents the least
square calculations for several assumed values of ^uct/k.
It is obvious from these calculations that the minimum

summation occurs for

^uc t _^
^ = 15 x 10 .

Accordingly, this is considered to be the correct value.


The total compressibility is

ct = coSo + c w S w + cf
.

ct = (8 x 10-5) (0.8) +(3 x 10-6) (0.2) + 4 X 10-6

ct = 68.6 x 10-6 psi-1

Finally, porosity can be calculated as



9-22
• ! !
Table 9.1: Least square caleulat.ions, Example 9.1
(Puct At
^Ap obs1 test ^APobslcalc ^(Ap obs) test (dp obs) calcl 2
k hours)

2.Sx10 500 11 80 4761 38994


800 21 115 8836
1200 34 140 11236
1500 44 163 14161

3x]' 500 1.1 76 4225 28086


800 21 102 6561
1200 34 126 8464
1500 44 138 8836
'7
10x10 500 11 20 81 943
800 21 36 225
N
W
1200 34 48 196
1500 44 65 440
15x10 ' S00 11 10.8 0.04 0.22
800 21. 21.3 0.09
1200 34 34.3 0.09
1500 44 44 0.0

20x10 ' 500 11 5.6 29 343


800 21 13.6 55
1200 34 24.5 90
1500 44 31. 169
l^ x lU ^
uCt

(15 x 10-') (7.6) 6
(0.8) (68.6 x 10 )

= 0.208.

B. Homogeneous -Anisotropic Reservoir


A homogeneous anisotropic reservoir is one in which
porosity, permeability and thickness are the same everwhere,
but permeability varies with direction. If a rate change is
initiated at an active well and the pressure response is
observed at multiple observation wells, it is possible to
determine the maximum and minimum permeabilities, and the
direction of the permeability trend relative to well locations.
The solution presented here was developed by Papadopulos9
and presented in the ground water literature. Rameylo
adopted the Papadopulos solution to the analysis of well
tests in petroleum reservoirs and presented a case history
illustrating application of the technique.
Consider the well configuration shown in Fig. 9.14 with
one active well and multiple observation wells. Well pattern
coordinates can be established by constructing x-y axes
with the active well at the origin. Relative to this
coordinate system are major and minor permeability axes
which represent the directions of maximum and minimum perme-
ability, respectively. The location of the permeability
axes, which are unknown, is a major objective of the test.
Suppose we have a test situation where all wells are
shut-in prior to the test. If the active well is placed
on production (or injection), this will cause a change
in pressure, (pi px v t), at all observation wells
^ ^

9-24
k Y
min

• \
px,y,t
0
(xlI Yl)
• \
(X3,Y3) ^ kmax
^ ^
\^ ^ •

\
.-1 '1 0 x
^
^
\
\
i •
\ •
\ (X2,^'2)
ACTIVE \
WE L L
\

Fig. 9.14: Coordinate system for interference test in


an anisotropic reservoir.

• following a lag period. The change in pressure at each


well can be described by the exponential integral solution
(Eq. 2.70); however, the equation has a different form than
for homogeneous isotropic formations. In this caselo,

PD = - 2 Ei - 4D D
r r2
(9.11)

where
2 z
xx -
PD q B ku h(pi - px ,Y, t (9.12)

tD 0.0002637t r kxxkYY kxY (9.13)


^uct k xx y2 + k y - 2k xy
rD y
Y xy

• Further, it can be shown that

9-25
_ 1
kmax 2 {(kxx + kyy)

+ [(k
xx
- k
yy
1
) 2 + 4k2 ] z} 0
(9.1
xy

_ 1 '1
kmin 2 {(kxx + kyy'

L(kxx - kyy) 2+ 4k^^,] z } (9.15)

k - k
max xx (9.16)
S = arctan

Reference 9 presents a derivation of these equations.


If ^uct is not known, solution of these equations will
require that a minimum of three observation wells be used
in the test. If ^uct is known, the required 'information
can be obtained with only two observation wells.
After a test has been conducted, the pressure change
at each well, (pi can be plotted on log-log
px ,,
paper and matched with the exponential-integral type curve
presented in Fig. 9.4. From the pressure match, we can
compute from Eq. 9.12

k2 ] z = 141.2qBu pD^t (9.17)


[k xx k yy - h
xy (pi-px,v,t^

Notice that the permeability function computed in Eq. 9.17


does not depend upon well position (i.e., x or y).
Accordingly, a match of the data from either of the obser-
vation wells should result in the same value of this function.
Although the type curve match for all observation wells
give the same pressure match, the time match, which depends
upon well position, will be different for each well. With
the time match known for each well, we can compute from

9-26
Eq. 9.13.

^ k - k' D/r')
k
xx yy xy _ ^uc t ^" t DM (9.18)
0.0002637
tM
kxxy2 + kyyx2 - 2kxvxy

With data from three observation wells, Eq. 9.18 yields


three independent relationships which can be solved
simultaneously with Eq. 9.17 to obtain the four unknowns,
i.e., kxx, kyy, kxy, and ^uct. This information can be
substituted into Eqs. 9.14 - 9.16 to solve for kmax' kmin'
and 0.

Example 9.2: Interference test in a homogeneous anisotropic


formation.

Problem. The following data, presented by Ramey10, are


for an interference test which involved one active well
and eight observation wells. The well configuration and
coordinate system for the test is depicted by Fig. 9.15.
The active well was an injector with an injection rate of
115 STB/D. For simplicity, we will only consider data at
three observation wells, i.e., Well 5-E, Well 1-D, and Well
1-E; pressure data for these wells are presented in Table 9.2.
Calculate the maximum and minimum permeabilities, and the
direction of the maximum permeability relative to the x-
axis.

i w = 11S STB/D c o = 7.5 x 10 _ 6 psi


h 25 ft cf = 3.7 x 10 psi-1
B = 1.0 RB/STB T = 72 ° F
w
uw = 1.0 cp pi = 240 psia
20% r ja = 0.563 ft
c `ti = 3.3 x 10-6 psi 1 Well depth = 1,011 ft

9-27
Y

1-C(-470,490) 1-D(0,475) 1 -E1475,5 14 1


• •
0

695 7

5-C(-455,0) 5-E (475, 0)


• ^ 0 x
N
5-D (!NJ)
00


®
\,
9-C(-470, -460) 9 - D 10,-4551 9-E(470, -415)

Fig. 9.15 Well locations and coordinate system for Example 9.2. All
coordinates and distances given in feet.

• • •
• • •
Fable 9.2: Interference test data, Example 9.2 (Ap
represents the pressure increase above the
initial pressure at the observation well).
WELL 5-E WELL 1-D WELL 1-E
t, hrs Ap, psi t, hrs 4p, psi t, hrs Ap, psi
21 4 23.5 6.7 27.5 3
47 11 28.5 7.2 47 5
72 16.3 51 15 72 11
94 21.2 77 20 95 13
115 22 95 25 115 16
122 2S 119 24 12S 16
If 140 22.3 125 23.2 142 13
N

188 19.2 141 19 192 10


210 18 163 18 215 10
285 15 188 14 240 6
215 12 295 5.8
265 10
290 10
Solution. A log-log plot of (pi - px,y,t) versus t was
prepared for each of the three observation wells and is
presented in Fig. 9.16. These three curves were individu-
ally matched with the exponential-integral type curve

presented in Fig. 9.4; the matched data are shown in Fig.
9.17. All three wells have the same match point on the
pressure scale, i.e.,

pDM = 0.26

ApM = 10 psi.

However, the match point on the time scale is different for


all wells. At a match value of tM = 100 hours, the values
of (tD/rD)M for the wells are:

WELL (tD/rD) M rD, ft

1-E 0.70 700


S-E 1.10 475
1-D 1.40 475

Using the match value from the pressure scale, Eq. 9.17
yields the following result:

k2 ]'"z = 141.2qBu pDM


[k k
xx yy xy h pi-px,y M

_ (141.2) (115) (1.0) (1.0) 0.26


25 10

[kxYkvy - kXV] Iz = 16 . 89 md

kxxkyy - kXy = 285.3 (9.1y

9-30

100

. n .
Ln
o- 10
Ci

• n
• lE
• 1D
n SE

10 100 1000
t, HOUR S

Fig. 9.16: Log-log plot of interference data,


Example 9.2.


9-31
10
^

100

• 1-E
• 1-D
10 ® 5-E
a


1
iu

2
10 -
10-1 12 10
tolrp

Fig. 9.17: Type curve match of interference data,


Example 9.2.


9-32
The match points on the time scale can separately be

• substituted into Eq. 9.18 to obtain

z
_ kxxk yy k xy _ uct
(t_ /t?1

k 2 + k x2 - 2k x 0.0002637
1I,
xxy yy xy y

For Well 5-E (x = 475 ft, y = 0),

285.3 _ Wt 1.10
0.0002637 100
kyy(475)2

k = 3.03 x 10 s (9.20)
yy Uct

For Well 1-E (x = 475', y = 514'),

• 285.3
kxx(514) 2 + kyV(475) 2 - 2kxy(475) (514)

_ ^'uct 0.70
0.0002637 100 (9 •-'1)

For Well 1-D (x = 0 , y = 475'),

285.3 _ Cuct 1.40


0.0002637 100
kxx(475)2

k = 2.38 x 10 s
(9.22)
xx ^uct

Equations 9.19 - 9.22 can be solved simultaneously to


obtain

0
9-33
kxX = 15.2 md

k = 19.4 md
YY

kxy = 3.12 md

-6
^uct = 1.57 x 10 cp/psi.

Maximum permeability is determined using Eq. 9.14:

kmax 2 {(kxx + kvy)

+ [(k kYy ) 2 + 4 k^ ] z }
xx , Y

kmax = 2 {(15.2 + 19.4)

+ [(15. 2 - 19 .4) 2 + ( 4) (3. 12) Z] 2}

k = 21.1 md.
max

Minimum permeability is, according to Eq. 9.15,

kmin 2 {(kxx + kyy)

2
[(kxx kyy.) 2+ 4 kxy Z}

kmin = -;T {(15.2 + 19.4) •

- [(15.2 - 19 . 4) Z + 4(3. 12) 2] 2}

kmin = 13.5 md.



9-34
Finally, the direction of the maximum permeability trend
relative to the x-y coordinate system can be expressed in
terms of the angle, 0, where from Eq. 9.16,

kxx
0 = arctan kmax
k
xy

0 = arctan 21.1 - 15.2,


3.12 J

0 = arctan (1.89)

0 = 62.1°

It is often difficult to decide whether a homogeneous


anisotropic model or a heterogeneous anisotropic model (to
be discussed in the next section) should be used to analyze
test data. As Ramey10 clearly pointed out, inspection of
Eqs. 9.11 - 9.16 can assist in making this decision. Equation
9.12 shows in particular that the permeability function which
controls the pressure match on a type curve is independent
of the direction of the line connecting the wells; however,
Eq. 9.13 indicates that the time match does depend on this
direction. Analysis of more than one interference test in
the same area should, therefore, provide information on the
feasibility of using homogeneous anisotropic models. If the
match of pressure is the same in tests run in different
locations, the model is applicable. If not, data should be
analyzed using a heterogeneous model. Unequal thickness will
also cause the pressure matches from different wells to be
unequal. Another test that can be applied is to compare the
permeability calculated from single-well tests to the perme-
ability function calculated from the pressure match using Eq.
9.17; for homogeneous isotropic reservoirs, these should be
equal.

9-35
C. Heterogeneous Anisotropic Reservoirs
Heterogeneous reservoirs are those where porosity,
permeability and thickness vary with position within the
reservoir. Further, the heterogeneities may be areal,
vertical, or both. Determining the nature of heterogeneities
from interference data is a very difficult problem which
requires the use of numerical solutions and computer models
to match reservoir history. These techniques are beyond the
scope of this text and the reader is referred to the
literature"-'8 for additional information.

III. PULSE TESTING

A. Introduction
Pulse testing is a special type of interference testing
which was introduced to the petroleum industry in 1966 by
Johnson, Greenkorn, and 6Voods19. The tests are conducted
by generating a series of pulses at an active well (injector
or producer); the pulses are created by alternately producing
(or injecting) and shutting-in the active well on a regular
pattern as depicted by Fig. 9.18. The production (or
injection) rate should be the same during each period. The
length of all production periods, and all shut-in periods,
should be equal; however, production periods do not have to
equal shut-in periods. These pulses create a very distinctive
pressure response at the observation well which can easily be
distinguished from any pre-existing trend in reservoir pressure,
or random pressure pertubations ("noise"), which could other-
wise be misinterpreted. Because the pressure responses are
very small, commonly less than 0.1 psi, very sensitive
pressure-measuring equipment is generally required.
The objectives of pulse testing are the same as for inter-
ference testing, i.e., to establish if communication exists
between wells, and to determine permeability, porosity, or the
porosity-compressibility product. Pulse testing, however,

9-36

N N N N
C^ CD (D tD

^ N W .P

Z
0

r N

^
.-,-i 3^

TIIME

Fig. 9.18: Illustration of rate history and


pressure response for a pulse
test (After Ref. 7).

offers several advantages over conventional interference


tests:

1) Because the pulse length used in a pulse test


is short, i.e., a few hours to a few days,
boundaries seldom affect the test data.
Therefore, simple infinite-system
equations normally apply. During the

9-37
long test times normally required in
interference testing, boundaries often
affect the pressure data; accordingly,
more complex analysis techniques which
account for boundary effects must be
used.
2) Because of the distinctive pressure
response, there are fewer interpretation
problems caused by random "noise" and by
trends in reservoir pressure at the
observation well. Because of the small
pressure changes which must normally be
recorded, this can be a major problem
in conventional interference tests.

3) Because of shorter test times, pulse


tests cause less disruption of normal
field operations than interference tests.

B. Theoretical Background
Several methods2'19 2 3 of analysis have been developed
which can be used to analyze pulse test data. The Kamal-
Brigham M ethod23 has the advantage of being the most flexible
and convenient to use for hand-calculations'; accordingly,
it is the method which will be presented.
Figure 9.19; which depicts the rate history and pressure
response of a pulse test, i llustrates several parameters
which are required for the analysis of a pulse test. The
pulse period, At p , represents the length of time the well is
shut in. The cycle period, Atcyc, represents the total
length of a cycle, i.e., the shut-in period plus the flow
period.
The response amplitude, Ap, is the pressure increment
between the tangent to two consecutive valleys and the
parallel tangent at the peak between them, or it is the
pressure increment between the tangent to two consecutive
peaks and the parallel tangent at the valley between them.
Analysis of simulated pulse tests show that pulse 1 (the
"first odd pulse") and pulse 2 (the "first even pulse")
have characteristics that differ from all subsequent pulses.
Beyond those initial pulses, all odd pulses have s imilar

9-38

Ap l AP QpJ

a.

H tLIH tL2
F tLJ

PULSE 1 I PULSE 2 1 PULSE 3

Atcvc
A

T I ME

• Fig. 9.19: Schematic showing important pulse


test parameters.

characteristics, and all even pulses exhibit similar behavior.


Another important parameter is the time Zag, tL, which
represents the elapsed time between the end of a pulse and
the pressure peak caused by the pulse. The radius of
investigation concept prepares us to expect a time lag; a
finite period of time is required for a pulse created when
the rate is changed to move from the active well to the
observation well.
The test parameters and reservoir properties can be
used to define the following dimensionless terms:

Pulse Ratio, R':

R, = pulse period
• cycle period

9-39
At
R' (9.23)
= At p
cyc
^
where Atp = pulse period, minutes
at = cycle period, minutes.
cyc

Dimensionless Cycle Period, AtcycD'

k At
(9.24)
Atc cD = cyc
y 56,900^ctur2

where r = distance from active well to observation


well, ft.

Dimensionless Time Lag, tLD'

tL
tLD At
cyc

where tL = time lag, minutes.

Dimensionless Response Amplitude, ApD:

(9.26)
APD i0h6qBu

where Ap = response amplitude, psi.

The pulse ratio, dimensionless time lag, dimensionless


cycle period, and dimensionless response amplitude can be
used to design and analyze pulse tests. The mathematical
relationship between these parameters was developed by
using the exponential-integral solution to the diffusivity
equation for an infinite homogeneous reservoir containing
a single-phase, slightly compressible fluid23.

9-40
Figures 9.20 - 9.27 present the curves relating the
^ dimensionless time lag and pulse ratio to the dimensionless
cycle period and the dimensionless response amplitude.
Notice that selection of the appropriate curve depends on
the pulse being analyzed. Different curves exist for the
following situations: (1) first odd pulse, (2) first even
pulse, (3) all odd pulses except the first, and (4) all
even pulses except the first.
Kamal and Brigham23 also found that the relations among
the dimensionless time lag, dimensionless cycle period, and
the dimensionless response amplitude can be presented in the
form of two simple equations. The equation relating the
dimensionless cycle period to the dimensionless time lag is

AtcycD = CtLD + D (9.27)

where D = -0.325 for odd pulses, and D = -0.675 for even


pulses. A and C are functions of pulse ratio, R', and are
^ presented by Figs. 9.28 and 9.29. The equation relating
dimensionless time lag, dimensionless cycle period, and
dimensionless response amplitude is

ApD = H[F exp (EtLD) + 0.01 I (9.28)


At
cycD

where H = -1 for odd pulses, and H = 1 for even pulses.


E and F are presented as functions of pulse ratio in Figs.
9.30 and 9.31.

C. Homogeneous Isotropic Reservoirs


The permeability, porosity, and thickness are the same
throughout the reservoir in this case. Procedures for both
design and analysis will be presented.


9-41
0.8

0.7

0.6

0. 5

U
>.

4J 0 . 4
1 d
4

0.3

0.2

0.1

t LD

Fig. 9.20: Relationship between dimensionless time lag,


dimensionless cycle period, and pulse ratio
for the first odd pulse.

0 0 0
• ^ •
0.8

,
0. 7 l lr ^' I;i ,i t I I. ` { I. ^ II
{' r R' = 0.9 + ^

0.6
^ 'I ^!t ^ ^I !^1i 1^ ^ ^1^ ^ ; ^ ,:, ( , ;, _^ _
I! '^^ ^' ^ I r' jj ! ^ ! •{-- - I -
{^
I, ^I III '
; t , , ► t , : ;F^ ^ ^ ^i, ,f
" ? ^ ffl i^ fi' ;
^I' 0 .8 I I It ^
t I { I ( }, I' i
I li t G i Iti il l!
ili! li l; .^ ! ^1i ` ! 1
T { • {^ ^{ li ',
0. 1 i ,-
^ T ' , ^ , , ^ f ,, ; ; , ; „
0.5
+J
a
I i i[ '!^^ ' ;
i_ _ i
; ^ I, ,► ^ t ; , ^ it^ - - - ; ;
0 .6 t1 r -
^ , i;
U
^ ^ ^i I ^' '^ ^ '': ^ ^t ^ ' ^ ^I , • i; ^ ,
v 0.4 0.5
^
4-) if ^ I

04 ,^
0 3
0.3

0.2
t J i •1! j^ I f ' l ^ I^ I i
;l 1 if li tj
711
0.2 + i•I ^ ! ^ ^

I ^ ' ^ ^ !,;I ^,j^ ^ ^, ,; I ' 'rf =- ^ '- i^ } ►il CE^ r


.) , .i
o.i0.2 I

tLD

Fig. 9.21: Relationship between dimensionless time lag,


dimensionless cycle period, and pulse ratio for
all odd pulses except the first.
0

; ! ^ i^, ;, t^ ! ^ ^^^
0 .1

7^1 i L!i
=^-; i 't ' 11I' ^` ►
-.001
0 .2

-.002 -
+
'lil li^^ r^' ^^^I J I I II! ^ `I ' I I{ '! ^^il
I j I ^i l l,l I^II !' I^I
' I !i ^f' L I,Tj
N .-.^
! 1T
1 , ,iI ^,1 ,,; ^ j, ' I ^i ^ i , ^ ^, I , ^ ^ •^' ^i ; , ' , I I ^I ' !^
-.003 .4
ar
t , ; ,^ 1: ^ 1; ^ ► f ;,^ ,^ hil
;? , ; , , ;i .^^ ^ i ^
.004
I ; ► i ' lilj I I iil^ !i ! 0:9 '
'i'^ i i! =
I -,I 4 ^
-.005
( ^! ! ^ ,^;. ^
^;I ; ^^^
r-
,
^;!i ^ ^ - _
^^;; ;1 ^ ; ^ ' ;1 - ^^^ - -
0 . 7

► , ^

7

^,I
'; I^ ^
^ I, ^ ', ^i ► :
0 . 6i - ^i^ !^

j ii ► ; f , ! ^ i ^ ;^ ^^
I I i ► ^j, I^ ^ , ^ i ;^ 1^1^ , ^ ,^ j^ ^^ I ^
006
0.02 0.1

tLD

Fig. 9.22: Relationship between dimensionless time lag,


dimensionless response amplitude, and pulse
ratio for the first odd pulse.

0 0 0
• • •
,' •i ^,? `' "; ''' ' ?;^? ^_ ^- ^-^ ,_.i ^^ ^^ - ^ ,^ ;.^ -^ , -; a ^-a ^
0 ,i ^ ! L ^-^
R 0. 1
-^^ f^•

-.001
I '^ ' ' ^ , ;
^
IIIf
^211-
lii iil
il
1
^I . .ll
l.
^ i'
1?
^ II
1 i I
► ' i ^^ ' I^ ' ^'' '^ '^
l ! li' i?' ^ I i ry I!,,
li.
0.2 i ^i I li i^ il I
^ -
^- ^- i ^ l ^ t i ^.{; l ,,, i ^ E • .l ^ ^ i I ( ' `l; f^ ! T ' ' ^^If , ;j ' I_ I{ 1I

i I i I I I li ^i '^^^ i^ l I l I I l I j I I I i I i^:

002 tII I I I^ _ i I ' I


- I 1 1 J

I iI ' I I j

-.003 T
I i I^
4-J

I i I . I 'i ,r , i f
J ^ 1 l ^ ^ III
!1^( il j 1 l i ' I i^
( ^ i ' I - I lli I{ lil I
I I ^( ?. ' ^ i ^^ ^ i 'I i • t
-.004
cn ^ 1 I
0. 5

I ilI ji I I I i i ili

005 Tj
' ► I (^ ^ ;li, , I, ^;`i ^ ^^ 8 , I ^ ; ;;,^ i ^
; ,
^ ^ !` , ;

0. 6

-.006 0. 7

^ ^ ^ i ii^•^ ^ .1^ ^ ^ ^ ,?,; ^ 1 1^ ^^^ ^l; ^^


-.007
0.02 0.1

tLl)

Fig. 9.23: Relationship between dimensionless time lag,


dimensionless response amplitude, and pulse
ratio for all odd pulses except the first.
0.8 °, Wy I I
4 i ^ i,ii y^

R I 0,1 y l
0.7
I
-
--rT

0.6
11 I ! I o.z I
tl ;I 11 ^^

I` II' ^ !t ^ ( I 0.3
11
I II
0.5

t^ I . j i
1
U 4
t' t . I. ^
0.4 ef: 0.5

j I 0 6 yt

0, 7
0.3 I • i ' 'j l n l

0 .8
Ei,
yl^ TT 'ilj !I i I 0.9 I
y t I .
0.2 I I
^ y'r
^;

0., 0.02 U. I

tLD

Fig. 9.24: Relationship between dimensionless time lag,


dimensionless cycle period, and pulse ratio
for the first even pulse.

• • •
. • •
0.8

d`' ^ I
I;I, ^; I^ I,^ ' ► '` {^ ^+ ^ II I!^ ^ j ! ^ !^^ ^ 1I ^i^
0.1
0.7 f t r `l, ft
rl

I^
0.6 1 ^
I
'I

^I
I II'

i; i!
f

0. 2
I

ii
^ I I

, ^ ^j , ^ i , f, E ,
I ^ II j ^! t
f
^
I^I
f ^ ^ ^ '^^ 1 ^, ' I ^, ^^,; , ^ r ,^ ,^, _ -
J ♦ I _ - 1-► , , ^
- I I^ f 1 t ; II
r
i
! li ,til I i^ 0 .3
0.5
+-)
0.4 ^I1 I i
U
i ► f ^! ► ' t^ ^^^^ ^ ^,f , ^ ^ ,. - - _ ^ ,, ` ^ ^^i ^ ^ ^ i ^' ; ^
i ^
4J ^ 0.4 0.5

0 .6

0.7
0.3
Y 77
.s
li

^ ► ' l jr T '
0.9
0.2
^ ^^ j IT-

; } ^ ^,^^ ^i^ ^ ^ i ^ ^
till

0.1
! , ^ ^ :^ ^ ^., ^I : I ^ ^ ^ ' ' ^" _ _ , I
- -- i ^ ^ '! ^ 1^► ^ ^ ^
^ , ^, ► ^ , ^^^ I^^,. + ;
0.02 0.1

t LD

Fig. 9.25 Relationship between dimensionless time lag,


dimensionless cycle period, and pulse ratio
for all even pulses except the first.
009

008

007

.006

^--^

^
.005

.^.

004

003

002 k-i '1


0.02 0.1

tLll

Fig. 9.26: Relationship between dimensionless time lag,


dimensionless response amplitude, and pulse
ratio for the first even pulse.

0 0 0

009

' = 0. 3

008

007

.006
N ^

d .005

I • 004

003

002

tLD

Fig. 9.27: Relationship between dimensionless


time lag, dimensionless response
amplitude, and pulse ratio for all
even pulses except the first.


9-49

-.6
=^ = r - - -.

- - -^ -- -
- _ - T _.. _^ - -
_.---

.. • ti ,
- ^s - - -

-.7

^ .. . _
A -.8
O ALL ---,-
0 THER
- - - -_ _-^ _
ODD 'St PULSE --

-- ''
GGG
G
S

t 04 S
F

, T .. - r ----
-.9 .`. ^ . -^-- Fp ,.
'_ .
T7^ ..__;
I.': -..------
. . . . ..i-.

---j--

1::

_ . • ... _.,_ .. ._:.. _.-.__.._. . - - i.


_ . ._. _.

_^..
...^._ : :.....
__
.-_
.__.
._
_
. _
. r._-.
.-.-
. .........._._ .t7.. ...
.....
.... .
^l.- .
.
^.
.
.._.. .I.. . .
-
.^
-
. ._-
_
..

-1 .0
9 3 _4 .5 .6 7 3

R'

Fig. 9.28: Parameter "A" as a function


of pulse ratio.


9-50
r^
0
00 +,
U

^ CTS

U)
a3 •
0
•rl
`O U +J

N O
Ln

^4
cd (4-+
O Ln

Q)

CY)
I ^ `^'•
CY)

q0
N w

rn co Ln d •
O

• • •
^._ -- , •I

-I ^ -
, _ -

9 ; 5 6
4 7 .8
2 3

H'

Fig. 9.30: Parameter "E" as a


function of pulse ratio.

9-52
ca
it
+

f: t `} i j;(t t p^^E R :r } .}
I ^1: r' ( E;' w
}` r ,l i:i it:; E (F E ,^. St. F v^ pQO P^'\ t :; F t 0
r_1
`'{ ,I ; ^ i7^ {( 0
`+ ... i?E, Fk} !{( { ^- fI , F ;Iii t .^,
{4 it ; t; ^ ,c; ; :i; r c,^ ^ ^ t - , t' ' t, t^ • ^
U
F+ ^ t ^
t t E 1 •I, ' t t ij i I
Ilk F t ft ^ .Ir it

'j ^ }I i E { ,C { i't ,11 '; it ^1! :(E f f i E i


i^it t} at l; C t I,! I t.I
01
tn

( 7^
; tt
i( t i,t( i j'
Ft t

r,
il

Ei^, 4l:t
I

t^^,
4 E
^jl •
^` (^^E
i
{ iI^ I '^
1^ f
.I
4 1
ill I T 11
}

f f t ^ ii!
^
i
'

' i!^!

7t 4
it i i
I} ( f {}^, E{j E^ ^
T !!` t ^( '^I+ tt] ^ r; .}
4 .t;

4
I ,t
! 1
t
t :,, , t4 ;j^
!L.
LO

^•
w •

4-)
N

C^
O
.^
+-)

V)
t^)
tn
O1

1 E^ ^? ^I" i( ii , ^i 'f ^ji ^ :f it^i ;+^j ^^ i, ^, i i ! i'1( if 'it: ;iti j^C ^C'{ {; ^l jf l.
.P { t r ^ ' t rr ;iE
i i} { Ei a a.
l it
t1 , r') -A
^
(I ,;,
.} l
11 i I ^ i i, °
Ei( F 1 E I E ill, f ^; ^j ^ 1^i i I ► ^1' ;i! 14
it ^; i it f; l^ ^ I ^ ► ► I rf fE r^ ^ C ^(^t I1 it N

M 00 ^
r r r ^ C:) C:)
r

• • ^
1. Test Design - Graphical Method
In order to insure good test analysis, it is first
necessary to properly design the test. The most important
factors required for test design are pulse time and the
expected pressure response. According to Brigham22, the
correct pulse time is one that falls around the midpoint
of the range of effectiveness. Use of the following
empirical equations to determine dimensionless time lag
insures that the pulse time will be in the correct range:

tLD = 0.09 + 0.3R' (odd pulses) (9.29)

tLD = 0.09 + 0.3(1 -R') (even pulses) (9.30)

Knowledge of the expected pressure response is needed so


that the range and sensitivity of the pressure gauge and
the length of time needed for the test can be predetermined.
The following procedure23 can be used to design a pulse
test using the correlations presented in Figs. 9.20 - 9.27:

1) Select a pulse ratio, R'. If a particular


ratio is more convenient to accomodate oilfield
operations, then use this ratio. Otherwise,
a ratio near 0.7 is recommended if odd pulses
are to be analyzed, and a ratio near 0.3 is
recommended if even pulses are to be analyzed.
The ratio selected should never be below 0.2
or above 0.8.
2) Compute the dimensionless time lag using
Eqs. 9.29 or 9.30.
3) Determine the dimensionless cycle period using
the dimensionless time lag and the appropriate
curve in Figs. 9.20, 9.21, 9.24, or 9.25.
4) Determine the dimensionless response amplitude
using the dimensionless time lag and the
appropriate curves in Figs. 9.22, 9.23, 9.26,
or 9.27.
5) Approximate values of formation permeability,
porosity, thickness, fluid viscosity, and
total compre.ssibility. Using these values
along with the dimensionless cycle period,

9-54
dimensionless response amplitude, and Eqs. 9.24
and 9.26, calculate the cycle period and the
^ response amplitude.
6) Using the pulse ratio and the cycle period,
calculate the pulsing period and the shut-in
period.

Example 9.3: Graphical design of a pulse test23

Problem. A two-well pulse test is to be conducted in an


oil reservoir which has the following approximate properties:

= 20%
k = 200 md
h = 30 ft
ct = 10 x 10-6 psi-1
uo = 2.0 cp
^ Bo = 1.0 RB/STB.

The active and observation wells are shut in prior to the


test. The distance between active and observation wells is
600 feet. Using a pulse ratio of 0.6, based on convenience,
design a pulse test for this reservoir.

Solution. Since the pulse ratio, R' = 0.6, is near 0.7,


odd pulses should be used rather than even pulses for the
analysis.
The dimensionless time lag is computed using Eq.
9.29:

tLD = 0.09 + 0.3R'

tLD = 0.09 + (0.3) (0.6)

tLD = 0.27.
I*

9-55
Assuming that the first odd pulse will be used for the
analysis, we find from Fig. 9.20 that

AtcycDtLD = 0.423

= 0.423 = 0.423
At
cycD tLD 0.27

AtcycD = 1.567.

The dimensionless response amplitude for the first


odd pulse is determined from Fig. 9.22:

ApDtLD = -0.0055

_ -0.0055 _ -0.0055
(0 7) 2
ApD tLII

^^
^_ 1 ) = 0 0, , 4 ^

The cycle period is computed using Eq. 9.24:

56,900^ctur2AtcycD
^tcyc k

_ (56,900) ( 0 .2) (10 x 10- 6)( 2.0)(600) 2(1-.S-6-/)


^tcyc 200

Atcyc = 642 minutes.

The pulse period, therefore, is

Atp = R' At cyc


9-56
otp = (0.6) (642)

Atp = 385 minutes.

It follows that the shut-in period is (642-385) = 257


minutes.
Finally, the response amplitude is predicted using
Eq. 9.26:

70.6auBOpD
^P = I- h

pp = (7/0.6)(2.0)(l.0)(-0.0754)q
(200) (30)

Ap = -0.00177d psi.

If a production rate of. 100 STB/D is used during the test,


the pressure response during the first odd pulse will
0.177 psi. Accordingly, a very sensitive pressure gauge
will be needed for the test.

2. Test Design - Analytical Method


Test design can also be accomplished using Eqs. 9.27
and 9.28 together with Figs. 9.28 - 9.31 to compute the
dimensionless cycle period and dimensionless pressure
response. The required procedure is:

1) Select a pulse ratio as in the graphical


method an d compute the dimensionless time
lag using Eqs. 9.29 or 9.30.
2) Determine A and C using Figs. 9.28 or 9.29.
3) Determine E and F using Figs. 9.30 or 9.31.
4) Calculate the dimension less cycle period
using Eq . 9 . 27 .

9-57
5) Calculate the dimensionless response amplitude
using Eq. 9.28.
6) Using approximate known values of permeability,
Is
porosity, thickness, fluid viscosity, and total
compressibility, calculate the cycle period and
response amplitude using Eqs. 9.24 and 9.26.

Example 9.4: Design of a pulse test using the analytical


method23.

Problem. Using the analytical method, design a pulse test


for the reservoir described in Ex. 9.3.

Solution. From Ex. 9.3, we are using a pulse ratio of 0.6


which indicates that it will be best to analyze odd pulses.
This design will be based on the assumption that the first
odd pulse will be analyzed.
The dimensionless time lag is calculated in the same
manner as for the graphical method; therefore, from Ex. 9.3,

tLD = 0.27.

The dimensionless cycle period is computed using


Eq. 9.27:

A
AtcvcD = Ct LD + D.

Since we are using odd pulses, D=-0.325. From Fig. 9.28,


A = -0.864, and from Fig. 9.29, C 0.6015. Therefore,

.^tcycD = 0.6015(0.27)-0.864 _ 0.325

1.539.
AtcycD =

The dimensionless response amplitude is determined


using Eq. 9.28: •
ApD = AtcycDH [F exp (EtLD) + 0.01] .

9-58
Since we are using odd pulses, H =-1. From Fig. 9.30,
E = - 4.0, and from Fig. 9.31, F 0.113. Therefore,

APD =(1.539) (-1) [0.113 exp(-4.0 x 0.27) + 0.01]

APD = -0.0744.

The cycle period is computed using Eq. 9.24:

56,900^ctur2AtcvcD
^tcyc =

= (56, 900) (0.2) ( 10 x 106 ) ( 2.0) (600) 2 (1.539)


At
cyc 200

At = 630 minutes.
cyc

Thus, Atp = K' ctcyc

,^tp = 378 minutes,

^
and the shut-in period is (630 _ .^,
^^ 8) = 2^5.,
2 5 2 minutes.
The response amplitude is, from Eq. 9.26,

70.6qBUApD
Ap kh

Qp = (70.6)(1.0)(2.0)(-0.0744)
^ (200) ( 3 0)

Ap = -0.00175q psi.

Again, if a rate of 100 STB/D is used, the pressure response


will be -0.175 psi.
It can be concluded from this example that the
graphical and analytical methods give essentially the
same result.


9-59
3. Test Analysis - Graphical Method
After running a test, pressure at the observation well
is plotted versus time as depicted by Fig. 9.18. The follow-
ing procedure can then be used to analyze the data.

1) Select the pulse to be analyzed and


measure the time lag and response amplitude.
2) Compute the dimensionless time la g using
Eq. 9 . 25 .
3) Determine the dimensionless cycle period
using the dimensionless time lag and the
appropriate curve in Figs. 9.20, 9.21, 9.24,
or 9.25.
4) Determine the dimensionless respo nse
amplitude using the dimensionless time
lag and the appropriate curves in Figs.
9.22, 9.23, 9.26, or 9.27.
S) Compute the value of (kh/u) from Eq. 9.26.
6) Compute the value of (^c t ) using Eq. 9.24.

Example 9.5: Pulse test analysis using the graphical


method23.

Problem. Examples 9.3 and 9.4 described the design of a


pulse test for an oil reservoir. Resulting from this
design procedure, a pulse test was conducted using the
following design parameters:

R' = 0.6
Atcyc = 650 minutes
Flow Period = 390 minutes
Shut-in Period = 260 minutes
q = 100 STB/D.

After running the test, the first odd pulse was analyzed and
found to have the following pressure response and time lag:


9-60
Op = -0.25 psi

t = 195 minutes.
L

Using this information, compute ( kh/u) and (^cth).

Solution. The dimensionless time lag is computed using


Eq. 9.25:

_ tL _ 195
tLD At 650
cyc

tLD = 0.3.

From Fig. 9.20

AtcycDtLD = 0.42;

therefore,
is
_ 0.42 =
1.40.
^tcvcD 0.3

From Fig. 9.22,

ApDtLD = -0.00549;

thus ,
-0.00549 = -0.061.
A PD
pD (0.3 )2

The value of (kh/u) can be determined using Eq.


9.26:

70.6qBApD
kh
u ap
^

9-61
P
kh
-0.2J
(70.6) (100) (1 .0) (-0.U6I)

0
kh = 1,723 md-ft/cp.
u

Finally, the value of (^cth) can be computed from


Eq. 9.24:

khbt
_ cyc
^cth 56,9001ir2AtcycD

(1723) (650)
^c h =
( 5 6 , 9 0 0 ) (600) 2 (1. 4)

^cth = 3.91 x 10-5 ft/psi.


4. Test Analysis - Analytical Method
A pulse test can also be analyzed using the following
analytical procedure:

1) Compute the dimensionless time lag using Eq,


9.25.
2) Compute the dimensionless cycle time using
Eq. 9.27.
3) Compute the dimensionless pressure response
using Eq. 9 .28.
4) Compute the value of (kh/ u) using Eq. 9.26.
5) Compute the value of (^ct ) using Eq. 9.24.

Example 9.6: Pulse test analysis using the analytical


method".

Problem. Using the analytical method, analyze the test


described in Ex. 9.5.
9-62
Solution. From Ex. 9.5, the dimensionless time lag is

0 tLD = 0.30.

The dimensionless cycle period is computed using Eq. 9.27:

AtcycD = CtLD + D.

From Fig. 9.28, A = -0.864, and from Fig. 9.29, C = 0.6015.


Therefore,

0.6015)(0.30-0.864) _ 0.325
AtcycD = (

OtcycD = 1.377.

The dimensionless response amplitude is,-from Eq.


9.28,

^ = _ H [F exp (Et LD ) + 0.011.


Ap D At c ycD

From Fig. 9.30, E = -4.0, and from Fig. 9.31, F = 0.113.


Therefore,

A pD = -1.3"17[0.113 exp (-4.0 x 0.3) + 0.01]

ApD = -0.0606.

The value of (kh/p) is computed using Eq. 9.26, i.e.,

kh 70.6c{BApD
P Ap

kh (70.6)(100)(1.0)(-0.0606)
u -0.25


9-63
kh = 1,711 md-ft/cp.

Finally, the value of (^cth) is determined from Eq.


9.24:

khAt
cyc
^c h =
t 56,900ur20tcvcD

cpc h = (1711) (650)


(S6, 900) (600) 2 (1. 377)

-s
^cth = 3.94 x 10 ft/psi.

D. Homogeneous Anisotropic Reservoirs


The equations used to describe homogeneous anisotropic
reservoirs were developed by Papadopulos9 and modified for
petroleum reservoirs by Ramey10. The same techniques used
to analyze homogeneous isotropic reservoirs can be used
here; however, the equations for calculating reservoir
properties are different. These equations are:

[k k- - k2 z _ 70.6qBp APD (9.31)


xx yy xy h Op

z
kxxkvv - kxv h Atcyc
(9.32 )
^cth kxxy + kvvx2 - 2kxvxy 56,900u AtcycD

, I
= 1
kmax 2 { (kxx + kyy)

+ [kxx - kyy) 2 + 4kXy] z } (9.33)


9-64
kmin 1 {(kxx + kyy)

^
- [(k - k ) Z + 4k2 ^ z} (9.34)
xx yy xy

k - k
max xx (9.35)
0 = arctan
k xy

It is noted from these equations that three perme-


ability related variables must be determined from the
test, i.e., k max, k min, and 0; this compares to only one
variable which had to be determined from the test in a
homogeneous isotropic reservoir. This means that three
tests in different directions, i.e., one test with three
observation wells, must be used to obtain the required
information.

E. Heterogeneous Anisotropic Reservoirs


• Heterogeneous reservoirs require a computer solution
which employs history matching techniques". These
techniques are beyond the scope of this text and will not
be discussed.

F. Vertical Pulse Testing


It is possible to determine vertical permeability by
conducting a pulse test within a single well. The test
is conducted by flowing or injecting in one set of perfo-
rations while measuring the pressure response at another
set. The reader is referred to the literature2" 29 for
the.,details of this test. The major problem associated with
the vertical pulse test is the communication that often
exists between the casing and formation due to a poor
cement job or fractures in the cement.


9-65
IV. SUMMARY

The objective of this chapter has been to introduce



the basic concepts of interference and pulse testing, and
to show how simple tests can be designed and analyzed.
Because of increased activity in secondary and enhanced
oil recovery projects, there is a need for better reservoir
description than can be provided by single-well tests.
Whereas single-well tests only give a measure of average
reservoir properties, multi-well tests have the potential
to detect and determine the nature of reservoir hetero-
geneities, and to establish if communication exists between
wells.


9-66
REFERENCES

1. Kamal, M. M: "Interference and Pulse Testing - A


Review" J. Pet. Tech. (Dec., 1983) 2257-2270.

2. Vela, Saul and McKinley, R. M.: "How Areal


Heterogeneities Affect Pulse Test Results," Soc.
Pet. Eng. J. (June, 1970) 181-191; Trans., AIME, 249.

3. Gringarten, A. C. and Witherspoon, P. A.: "A Method


of Analyzing Pump Test Data From Fractured Aquifers,"
Proc., Symposium on Percolation Through Fissured Rock,
International Society for Rock Mechanics, Stuttgart
(Sept. 18-19, 1972).

4. Prats, M. and Scott, J. B.: "Effect of Wellbore


Storage on Pulse Test Pressure Response," J. Pet.
Tech. (June, 1975) 707-709.

5. Jargon, J. R.: "Effect of Wellbore Storage and


Wellbore Damage at the Active Well on Interference
Test Analysis, J. Pet. Tech. (Aug., 1976) 851-857.

6. Chu, W. C., Garcia-R., J. and Raghavan, R.: "Analysis


of Interference Test Data Influenced by Wellbore
Storage and Skin at the Flowing Well," J. Pet. Tech.
(Jan., 1980) 171-178.

7. Earlougher, R. C., Jr.: Advances in Well Test Analysis,


Monograph Series, SPE, Dallas 1977T 5.

8. Earlougher, R. C., Jr., and Ramey, H. J., Jr.:


"Interference Analysis in Bounded Systems," J. Cdn.
Pet. Tech. (Oct.-Dec., 1973)'33-45.

9. Papadopulos, I. S.: "Nonsteady Flow to a Well in an


Infinite Anisotropic Aquifer," presented at Symposium
of the International Assn. Sci. Hydrology, Dubrovnik,
Yugoslavia (1965).

10. Ramey, H. J., Jr.: "Interference Analysis for Anisotropic


Formations - A Case History," J. Pet. Tech. (Oct. 1975)
1290-1298; Trans., AIME, 259.

11. Kamal, M. M.: "The Use of Pressure Transients to


Describe Reservoir Heterogeneity," J. Pet. Tech.
(Aug., 1979) 1060-1070; 'Trans., AIME, 267.

12. Kruger, W. D.: "Determining Areal Permeability


Distribution by Calculations," J. Pet. Tech. (July,
1961.) 691-696; 'Frans., AIME, 227
-.

9-67
13. Jacquard, P. and Jain, C.: "Permeability Distribution
from Field Pressure Data," Soc. Pet. Eng. J. ( Dec.,
1965) 281-294; Trans., AIME, 234.

14. Jahns, H. 0.: "A Rapid Method for Obtaining a Two-


Dimensional Reservoir Description from Well Pressure
Response Data," Soc. Pet. Eng. J. (Dec., 1966) 315-327.

15. Coats, K. H., Dempsey, J. R., and Henderson, J. H.:


"A New Technique for Determining Reservoir Description
From Field Performance Data," Soc. Pet. Eng. J.
(March, 1970) 66-74; Trans., AIME, 249.

16. Carter, R. D., Kemp, L. F., Jr., Pierce, A. C., and


Williams , D. L.: "Performance Matching with Constraints,"
Soc. Pet. Eng. J. (April, 1974) 187-196; Trans., AIME,
257.

17. Chavent, C., Dupuy, M., and Lemonier, P.: "History


Matching by Use of Optimal Control Theory," Soc. Pet.
Eng. J. (Feb., 1975) 74-86; Trans., AIME, 259.

18. Wasserman, M. L. and Emanuel, A. S.: "History


Matching Three-Dimensional Models Using Optimal
Control Theory," J. Cdn. Pet. Tech. (Oct./Dec., 1976)
70-77.

19. Johnson, C. R., Greenkorn, R. A., and Woods, E. G.:


"Pulse-Testing: A New Method for Describing Reservoir
Flow Properties Between Wells," J. Pet. Tech. (Dec.,
1966) 1599-1604; Trans., AIME, 237.

20. Culham, W. E.: "Amplification of Pulse-Testing Theory,"


J. Pet. Tech. (Oct., 1969) 1245-1247.

21. Startzman, R. A.: "A Further Note on Pulse-Test


Interpretation," J. Pet. Tech. (Sept., 1971) 1143-1144.

22. Brigham , W. E.: "Planning and Analysis of Pulse-Tests,"


J. Pet. Tech. (May, 1970) 618-624; Trans., AIME, 249.

23. Kamal, M. M. and Brigham, W. E.: "Pulse-Testing


Response for Unequal Pulse and Shut-In Periods," Soc.
Pet. Eng. J. (Oct., 1975) 399-410; Trans., AIME, 259.

24. Burns, W. A., Jr.: "New Single-well Test for Determining


Vertical Permeability," J. Pet. Tech. (June, 1969)
743-752; Trans., AIME, 246.

25. Prats, M.: "A Method for Determining the Net Vertical
Permeability Near a Well from In-Situ Measurements,"
J. Pet. Tech. (May, 1970) 637-643; Trans., AIME, 249.

9-68
26. Falade, G. K., and Brigham, W. E.: "The Dynamics of
Vertical Pulse Testing in a Slab Reservoir," SPE 50S5-A
^ presented at the 49th Annual Fall Meeting, Houston,
Texas, 1974.

27. Falade, G. K., and Brigham, W. E.: "The Analysis of


Single-Well Pulse Tests in a Finite-Acting Slab
Reservoir," SPE 5055-B presented at the 49th Annual
Fall Meeting, Houston, Texas, 1974.

28. Hirasaki, G. J.: "Pulse Test and Other Early Transient


Pressure Analysis for In-Situ Estimation of Vertical
Permeability," Soc., Pet. Eng. J. (Feb., 1974) 75-91;
Trans., AIME, 257.

29. Earlougher, R. C., Jr.: "Analysis and Design Methods


for Vertical Well Testing," J. Pet. Tech. (Mar., 1980)
505-514; Trans., AIME, 269.


9-69
NOMENCLATURE - CHAPTER 9

A = constant in Eq. 9.27


a. = distance between active well and Well i when
1 multiple observation wells are used, ft
B = formation volume factor, RB/STB
C = constant in Eq. 9.27
ct = total compressibility, psi-1
D = constant in Eq. 9.27
E = constant in Eq. 9.28
Ei = exponential integral
F = constant in Eq. 9.28
H = constant in Eq. 9.28
h = formation thickness, ft
k = volumetric average effective permeability, md
kmax = maximum principal permeability, md
minimum principal permeability, md
^ kmin =
kxx, kyy, kxy = components of permeability tensor, md
m = slope of semilog straight line, psi/cycle
p* = false pressure from Horner plot, psia
= dimensionless pressure
PD
pDM = match value of PD from type curve
pext = extrapolated pressure, psia
pobs = pressure at observation well, psia
Ap = pressure change, psi
t1pD = dimensionless pressure change
ppM = match value of Ap from type curve, psi
Apobs = pressure change at observation well, psi
q = production or injection rate, STB/D
R' = pulse ratio, dimensionless
r = distance between active well and observation well, ft
rD = dimensionless value of r
ta = length of time active well has been producing during
^ an interference test, hrs.
tD = dimensionless time

9-70
tL = lag time, minutes
tLD = dimensionless lag time
= match value of t from type curve, hrs
tM
Atcyc = cycle time, minutes
AtcycD = dimensionless cycle time
atp = pulse time, minutes
= porosity, fraction
0 = angle of maximum permeability vector relative to
x-y axis, degrees


9-71
SUMMARY OF MAJOR EQUATIONS - CHAPTER 9
0

Equation Number Equation


in Text

k = 141 . 2qBu (PD) M


9. 3
h ^
^p7, 11

0.0002637k ('^t)M
9.4 ^c =
t urz (tD/rD)M

9.6 Apobs Dext pobs

^uctrz
70.6 qABu Ei -948
9'7 Apobs E kt
a
^
N 2
9.10 70.6 Bu ^uctai
AFobs kh LL^^ aa iEi (-948
i=1 ktAi

[k k k2 2
xtir ^
9.12 = -xx yy
pD 141.2qBp h(Pi - Px,y,t)

2
tD = 0.0002637t kxxkvy - kxy
9.13
^uct kxxvZ + kvyx2 - 2kxyxy
rD

9.14 {(kxx + kyy)


kmax T

+ L(kxx kyv) z + 4k^v

9-72
9.15 ^ {(kxx + kyy)
kmin

[(k xx k vy Z + 4k Xy]
-

[kmax - k
9.16 xx
G= arctan
-
x

pDM
9. 17 [k - k2 ] z = 141 . 2qBu
xx k yy xy h pi px,Y,t M

kxxkYV - kXY ^uct (tD/rD) M


9.18
2 + k x2 - 2k xy 0.0002637 t,^^
k
xxy yy xy

At
9.23 R' = Ap
cyc

k At
•i
9.24 = cyc
At
cycD
56,9000ctur2

tL
9.25
tLD At
cyc

9.26
APD = rOhA
^ 6qBu

9.27
AtcvcD = ctLD + D

0
9.28 ^tpD = H [F exp(EtLD) + 0.01]
cycD

• i
9-73
9.29 tLD = 0.09 + 0.3R' (odd pulses)

0
9.30 tLD = 0.09 + 0.3 (1-R')(even pulses)

''2 = 70.6qBu ApD


9.31 [k xx k yy - k2 h Ap
xy^

k xx k yy k xy
z h A t cyc
9.32 ^cth = 2 z 56,900p At cycD
kxxy + kyyx - 2kxyxy

^
.


9-74
0 Chapter 10

DRILLSTEM TESTING

1. INTRODUCTION

A drillstem test (DST) is normally run in a well at


the time it is drilled to determine the productive potential
of a formation. The decision to complete or abandon a well
is often made on the basis of this test. A good DST yields
samples of the fluids present in the test formation, a
measurement of static formation pressure, an estimate of
^ potential flow rate, and a short-time pressure transient
test. Analysis of the pressure transient data can yield
approximate formation properties and an indication of
formation damage. The purpose of this chapter is to intro-
duce the basics of drillstem testing and to discuss the
analysis of pressure transient data obtained from this test.
The reader is referred to the literature1-5 for details of
test equipment and variations in test procedures.
A DST permits a temporary completion of the test
formation. The test is conducted by placing a DST tool on
the end of the drill pipe and lowering it to the test
interval. When the tool is activated, it isolates the
formation from the mud column in the annulus and permits
formation fluids to flow into the pipe. A recorder in the
DST tool provides a record of pressure during the test.
Most drillstem tests run today include two cycles of flow
0 and shut-in.
The performance of a typical drillstem test can best
be described by considering the schematic of a DST pressure
chart presented in Fig. 10.1. Notice that pressure increases

downward on this chart. At point A, the tool is lowered
into the well. Between A and B, the increasing hydrostatic
head of the mud column is recorded as the tool increases in
depth; at point B, the tool reaches the test interval. The
decrease in pressure from B to C indicates the release of
mud column pressure from the test zone as the tool is acti-
vated, and C-D represents the first flow period. Notice that
pressure increases during the flow period because produced
fluids accumulate in the drill pipe above the pressure Qauge.

BASE LINE

A
`l:
J •
D
G

•r.
.f

T I ^1E No

Fig. 10.1: Schematic of a drillstem pressure chart.


10 -'
The well is shut in at D for the first buildup and pressure
increases to E. The tool is opened at E for the final flow
period and pressure drops immediately to F; as fluids enter
the drill pipe, pressure increases from F to G. The tool
is shut in from G to H for the final buildup. The tool is
released at H and the pressure increases to I as the full
hydrostatic head of the mud column is again imposed on the
test formation and the pressure gauge. From I to J, the
DST tool is removed from the hole.
We will discuss first the equipment and procedures
used to conduct a typical test, and then the analysis of
pressure transient data from a test.

II. EQUIPMENT AND PROCEDURES

Selection of appropriate equipment is a prerequisite


^ to a successful drillstem test. There are many equipment
options available but most drillstem test tools include a
set of flow valves, one or two packers, and two or more
clock-driven bourdon-tube pressure gauges. The DST tool
is attached to the end of the drill pipe and lowered to the
formation of interest. When in the correct position, the
packer is set and the flow valves are opened and closed by
rotation of the drill pipe. The purpose of this section is
not to describe all available DST equipment; the reader is
referred to the literature for this purposel 5. Instead,
a typical DST tool will be presented and described.
Three basic drillstem test assemblies used by the
Halliburton Company5 are illustrated by Fig. 10.2. These
tools are used for three different types of tests: the
hook wall packer test, the open hole single packer test,
and the straddle packer test. Figure 10.3 shows the flow
path of fluids in a Halliburton DST tool during various
stages of a drillstem test6.

10-3
Open Hole Open Hole
Hook Wall Packer Test Single Packer Test Straddle Packer Test

Tubing

Pipe

Impact Reverse
Sub (Optional) Impact Reverse
Sub (Optional)
Tubing
Drill Pipe
Dual Closed In Impact Reverse
Pressure Valve Dual Closed In
Sub (Optional) Pressure Valve
Reverse Circulation
Reverse Circulation
Ports
Drill Pipe Ports
Handling Sub 8 Handlino Sub &
Choke Assembly
Dual Closed in
Hydrospring Pressure Valve
Tester Tester
By-Pass Ports Reverse Circulation By-Pass Ports
Ports
B.T. Pressure B.T. Pressure
Recorder
Recorder Handling Sub 8
Choke Assembly Hydraulic Jar
Hydraulic Jar
Hydrospring
Tester V R Safety Joint
V R Safety Joint
By-Pass Ports
By-Pass Ports Upper Body-
By-Pass Ports
Hook Wall Pressure Equalizer
Packer B.T. Pressure Pressure Equalizer
Recorder Ports

Hydraulic Jar

V R Safety Joint
Expanding Shoe
Packer Assembly
Flush Joint
Anchor
is
By-Pass Ports B.T Pressure
Expand Shoe Recorder
Packer Assembly ( Blanked Off)
Expanding Shoe
Packer Assembly

Tailpipe Anchor Pipe


Safety Joint

Flush Joint
Anchor

.....t-:u.
B.T. Pressure B.T. Pressure
Recorder Recorder
(Blanked Off) (Blanked Off)

Thread Protector

Fig. 10.2: Typical drilistem test tools used for three


types of tests (After Ref. 6).


10-4

Reverse
Circulating
Sub

Dual CIP
Circulating
Ports

Dual
Closed In
Pressure Valve
Tester
Valve

By-Pass
Ports

Pressure
Recorder

VR Safety

• Joint^
By-Pass
Ports
Packer

Perforated
Anchor

Blanked Off
Pressure
Recorder

RUNNING FLOWING FORMATION EQUALIZING REVERSE PULLING


IN FORMATION CLOSED IN PRESSURE CIRCULATING OUT

(a) (b) ( C) (d) (e) (f)

Fig. 10.3: Flow path of fluids in a typical DST tool


during various stages of a drillstem test
(After Ref. 6).

10-5
It is noted in Fig. 10.2 that the upper portions of all
test assemblies are similar in that each contains an impact
reversing sub, a dual closed-in valve (DCIP), and a hydro-
spring tester valve. The impact reversing sub is a valve
which can be opened by dropping a bar through the drill pipe
either during or at the end of the final shut-in period. This
permits reverse circulation above the closed tester valve
which removes formation fluids from the drill pipe before
the test assembly is removed from the hole. This process
is illustrated by Figs. 10.3(e) and 10.3(f).
The primary valve in the DST tool is the dual closed-
in pressure valve. When used in conjunction with the hydro-
spring tester valve, this valve permits the two flow and shut-
in periods, and reverse circulation. Flow is controlled by
a sliding valve mandrel which provides two flow positions,
two shut-in pressure periods, and a reverse circulation
period" When the DST tool is going in the hole, the DCIP
valve is open and the hydrospring tester valve is closed;
this is illustrated by Fig. 10.3(a). The first flow period
begins when the packer is set and the hydrospring tester
valve opens; Fig. 10.3(b) illustrates this operation. The
formation is shut-in for the first time by rotating the drill
pipe clockwise so that the DCIP valve moves to the first
shut-in position. Clockwise rotation of the drill pipe
at the appropriate time also results in subsequent flow,
shut-in and reverse circulation.
The hydrospring tester valve serves as a downhole
master valve. This hydraulically controlled valve has a
time delay which causes the valve to open slowly and to close
quickly. When the DST tool is in position at the test
interval and weight is applied by the drill pipe to set
the packer, this weight is also transmitted through the
hydrospring to activate the hydraulic time delay. A few
minutes after the weight is applied, the time delay releases
and opens the hydrospring tester valve. When the weight or
the drill pipe is picked up at the end of the test, the

10-6
hydrospring tester valve closes quickly.
^ A DST tool typically contains two bourdon-type pressure
recorders. One of these pressure recorders is placed in the
upper portion of the tool so that it can measure the pressure
response during flow and shut-in periods. A second recorder,
located near the bottom of the tool, is shut off from the
flow string. This recorder measures annulus pressure below
the bottom packer rather than pressure inside the drill pipe.
Pressures indicated by the upper and lower pressure gauges
should only differ by the hydrostatic head between them; any
deviation from this indicates a probable malfuntion in the
test.
A DST tool may contain a variety of auxilliary equipment.
Two items included in many DST tools are hydraulic jars and a
safety joint. If the drill string becomes stuck, hydraulic
jars provide an impact near the bottom of the string which
may release the stuck tools. When the tool cannot be unstuck,
the drill string can be backed off at the safety joint per-
mitting recovery of the drill pipe and that part of the DST
tool above the safety joint.
The bottom portion of a DST tool, i.e., below the safety
joint, will vary depending upon the type of test run. We will
briefly describe the open hole single packer test, the open
hole straddle packer test, and the hook wall packer test.
Typical equipment for these tests is depicted by Fig. 10.2.
An open hole single packer test utilizes a non-rotating,
expanding packer. Expansion of the packer is achieved by
setting weight from the drill pipe on the packer. Below
the packer is an anchor pipe and a bourdon-tube pressure
recorder. The anchor pipe is perforated to permit flow into
the drill string. When a test is completed, an upward pull
releases the packer and opens bypasses to equalize pressure
across the packer.
An open hole straddle packer test isolates the test
^ formation between two non-rotating packers. An anchor pipe
and pressure recorder will also be located between the

10 -7
packers. An equalizing tube is used to connect the annulus
above the top packer to the area below the bottom packer.
This tube assists in bypassing wellbore fluids around the
packers while going in and out of the hole, and balances the
compressive load created by the annulus hydrostatic pressure.
It is recommended that a third pressure recorder be used
below the bottom packer to indicate whether the packer
remains sealed throughout the test:
A hook wall packer is used when the test is run inside
the casing. Slips on this packer grip the inside of the
casing and support the weight required to expand packer
elements.
Figure 10.3 illustrates the operational sequence of
the major components in a DST tool during a test. When the
DST tool is being run into the hole, the DCIP valve is open
and the hydrospring tester valve is closed; this is depicted
by Fig. 10.3(a). Bypass ports in the tool permit mud to
flow around the outside of the tool and through the packer
while the tool is moving. Also, both pressure recorders are
in communication with the mud column and should read hydro-
static pressure as the tool is lowered into position. The
packer is set when the tool is in position; this closes the
bypass ports and opens the hydrospring tester valve as
indicated by Fig. 10.3(b). At the end of the first flow
period, the tool is shut in for the first buildup by closing
the DCIP valve; this is illustrated by Fig. 10.3(c). The
second flow period is initiated by again opening the DCIP
valve, and the final buildup occurs when the valve is closed.
Following the final buildup, the DCIP valve remains closed
and the hydrospring tester valve is opened. Next, the bypass
ports are opened, as shown by Fig. 10.3(d), and pressure
across the packer is allowed to equalize. As the packer is
unseated, the reverse circulating valve opens and mud is
pumped down the annulus to displace fluids from the drill
pipe. The volume of these displaced fluids is measured at
the surface5 Finally, the tool is pulled out of the hole.

10-8
The initial flow period in a typical drillstem test
should be short, i.e., 5-10 minutes. The purpose of this
flow period is to relieve the hydrostatic head of the mud
column from the formation and permit pressure near the
wellbore to draw down. This short drawdown period will
relieve pressure in the mud-invaded zone, commonly called
"supercharge". The initial shut-in period, generally 30-60
minutes, is designed to allow pressure to build back up to
the initial reservoir pressure. In order to achieve this
objective, it is important that the initial flow period be
sufficiently short. The initial shut-in pressure from a
DST may be the best measurement of initial reservoir pressure
available to an engineer.
The final flow period is designed to recover a sample
of reservoir fluids, and to draw down the pressure in the
reservoir to the largest distance possible. It is desirable
to achieve a stabilized flow rate during this period but this
is generally not possible unless formation fluids flow to the
surface. The length of the final flow period will depend
upon test conditions but generally varies from one hour to
several hours.
The final shut-in period is very important because it
is designed to obtain good pressure buildup data from which
formation properties can be estimated. Also, comparison of
the final shut-in pressure to the initial reservoir pressure
can provide an indication if significant pressure depletion
occurred during the test; if so, this would suggest that the
test well is in a small, non-commercial reservoir. The
length of this shut-in period is generally one to two times
the length of the second flow period.
Table 10.1 presents recommended flow and shut-in times
for a drillstem test conducted in an area where no experience
is available3's
Other variations in drillstem testing include the use of
^ the multiflow evaluator, a tool that allows unlimited sequences
of production and shut-in; and the use of a Closed Chamber DST7
where the test is run while shut-in at the surface.

10-9
Table 10.1: Recommended times for drillstem testing when
experience in an area is not available (From
References 3 and S)
Test Situation During Recommended Minimum
Period Test Time Time(minutes)

Initial All Tests Short-release 3 to 5


Flow hydrostatic mud
pressure

Initial All Tests 60 minutes 30


Shut-in unless total
test time is
too short- 45
minutes then

Final Strong Continu- 60 minutes 60


Flow ing blow

Blow dies Shut in when


blow dies

Reservoir fluid 60 minutes - 60


produced at longer to gauge
surface flow rates if
time is available

Final Strong Continu- Shut-in time 45


Shut-in ous blow during equal to flow
flow period time

Blow dies during Minimum shut-in Two times


flow period time of twice flow time
flow time

Reservoir fluid Shut-in time 30


produced at sur- equal to one-
face during flow half flow time
period


10-10
III. ANALYSIS OF PRESSURE DATA

• Pressure data recorded during a drillstem test can be


analyzed using methods presented in previous chapters. The
first flow period offers no useful data because it is too
short, and because production consists primarily of drilling
mud. Generally, however, data from the first and second shut-
in periods and the second flow period can be analyzed.

A. Pressure Buildup Data


The methods presented in Chapter 5 to analyze pressure
buildup data can also be applied to DST data. However,
because of the short flow periods in drillstem tests and the
inability to control flow rate in most tests, special pre-
cautions must be used. The flow period and shut-in period
in a DST are about the same length; accordingly, only the
Horner plot or Agarwal plot should be used. When analyzing
^ the first buildup, tp is the length of the first flow period.
For the second buildup, tp is usually equal to the length of
the second flow period; an exception to this is when the
initial flow period is very long, in which case tp is the sum
of the lengths of the first and second flow periodsi'S
The techniques used to analyze buildup tests assume that
flow rate is constant prior to shut-in. This is seldom true
in a DST. When testing liquid wells, produced fluids flow
into the drill pipe and cause an increasing hydrostatic
pressure against the formation face as the fluid level moves
upward. This increasing backpressure causes the rate to
decrease and, in some cases, may cause flow to cease. The
only time rate may be constant is when fluids flow to the
surface or when critical flow is present. Average flow rate
is determined by measuring the volume of produced fluids and
dividing this volume by the length of the flow period.
Variations in rate are generally negelected in DST analysis
and this average rate is used. Rate changes can generally be
40

10-11
neglected if the bottomhole pressure increase is small
compared to the preceding pressure drawdown; otherwise,
negelecting rate changes can result in significant errorsy
When rate changes can be measured, the multiple rate analysis
techniques presented in Chapter 4 should be applied. In
general, DST analysis will be subject to greater error than
a conventional buildup test because of the problem of rate
control.
The duration of wellbore storage can be determined
using a log-log plot of the test data. Since the well is
generally shut in close to the formation face, storage
typically does not last long. However, since the shut-in
period does not last long, a short storage period can still
cause problems.
After the slope m of the transient flow straight line
is determined from a Horner plot, permeability can be
estimated as

k = - lb2.0 IL

It is probable that B and u will have to be estimated from


available correlations since lab studies will not be
available at the time of the DST.
The total skin factor can be computed using
Eq. 5.15:

[PTft_O) - pws ktpnt


s = 1.151 log
L m ^uctrw(tp + At)
+ 3.231. (10.2)

If p«s is read from the Horner straight line at At = 1 hour,


i.e., pws = p1hr,

10-1?
pwf(at=o) - plhr ktp
s = 1.151 log
m ^uctr^ (tp + 1)

+ 3.23^. (10.3)

The flow efficiency can be computed as

pw f(Gt=O) - Aps
PR (10.4)
E=
pwf(At=0)
PR

where Ops = -0.87ms. (10.5)

Alternatively, the effect of skin can be expressed in


terms of the damage ratio (DR) as

PR pwf(Qt=O)
DR = - . (10.6)
pwf(At=0) - ^'ps
PR

The average pressure, PR' in Eqs. 10.4 and 10.6 is equal


to the initial reservoir pressure. Since there has been
insignificant production at the time of a DST, the initial
pressure can be estimated by extrapolating the Horner
straight line to (tp + ^t)/©t = 1. Since we have two
buildup periods, the extrapolated pressure should be
determined using each of these tests. If the extrapolated
pressure from the final shut-in period is significantly
lower than the pressure obtained from the initial shut-in,
this is an indication that the reservoir is very small or
that the test is bad. If the validity of the test can be
confirmed, this result suggests that the test formation
may be non-commercial.
The effects of flow barriers, layers, fractures,
changes in lithology, etc., will generally affect DST
buildup data in the same manner described in Chapter 5

10-13
for conventional buildup tests. Because of the short flow
periods in a DST, it is not likely that barriers or
Is
boundaries will affect DST data. The radius of investl'-
gation of a DST can be estimated using Eq. 3.14:

rd = 0.029 Fu^ z. (10.7)


t

The Ramey type curve can also be used to analyze DST


buildup data. Conventionally, the buildup data are
prepared on a log-log plot of [pjvs - pwf(At=0)] versus At
for this purpose. However, it must be recalled from Chapter
5 that the Ramey type curve is a pressure drawdown solution
and should not be used to analyze pressure buildup data
unless t >> At. Since this condition is not satisfied in
p
a DST, it is required that the log-log plot be prepared
using equivalent time, Ate, where

t r'ti t 0
(10.8)
^te = t p+ At
P

The use of Ate in place of At on the log-log plot permits


the Ramey curve to be used for DST buildup analysis.
Pressure data from both DST shut-in periods can be
analyzed using the methods discussed. However, data from
the second buildup period are considered more reliable
because of the longer flow time and better definition of
produced fluids associated with the preceding flow period.
The flow period which precedes the first buildup is very
short and production during this period consists primarily
of drilling fluids. Accordingly, both flow rate and fluid
properties can be difficult to define for the first buildup.

B. Flow-Period Data
Pressure data from the flow periods are not
analyzed because of variations in flow rate which are typically

10-14
present. When a flow period begins, produced fluids enter
the drill pipe and, as they flow toward the surface, cause
a continually increasing backpressure against the formation
face. This backpressure causes the flow rate to decrease
until the fluid level reaches the surface. If the fluid
level reaches the surface during the test, it may be possible
to establish a constant rate at this time. Sometimes, the
rising fluid level causes a sufficient backpressure to kill
the well. In this situation, there will be no buildup of
pressure during the following shut-in period; accordingly,
the only pressure data which can be analyzed will be from
the flow period.
If the variation in rate during the flow period can be
measured, the multi-rate equations introduced in Chapter 4
can be used to evaluate the data. Also, type curves
developed by Ramey, Agarwal and Martine can be used to
analyze the data provided flow does not reach the surface
and there is no significant change in wellbore storage.
Earloughers provides details of using these curves and
presents an example illustrating their application.

IV. WIRELINE FORMATION TESTS

A wireline formation tester9 12 which can be run on a


wireline from a logging truck offers an alternative to
drillstem testing. This method offers the advantage of
being quick and inexpensive; however, data from this method
are considered inferior to that available from a DST.
The wireline formation tester contains a pad which,
when the tool is in proper position, expands against the
formation. When flow is established, formation fluids
flow into a sample chamber and the pressure response is
measured with a pressure transducer and recorded at the
surface. This method can be expected to give a reasonable
fluid sample and an acceptable value of pi. Permeability

10-15
can be computed from the pressure data but the degree of
uncertainity is high. :\ skin Cactor cannot be obtained
from the datas

V. INTERPRETATION OF DST PRESSURE CHARTS.

Evaluation of the DST chart at the wellsite is extremely


important. The decision of whether to set pipe or abandon a
well is often made on the basis of a DST; accordingly, when
a test is completed, the DST chart should be evaluated
for any mechanical or operational problems so that the test
can be rerun if necessary. According to Earloughers, a aood
DST chart should have the following characteristics:

1) The pressure base line should be straight


and clear.
?) Recorded initial and final hydrostatic mud
pressures should be the same, and should
be consistent with depth and mud weight.
3) Pressure curves during flow and buildup
periods should be smooth curves.

An engineer familiar with the characteristics of DST


charts can identify a variety of problems by visual analysis.
Chart shapes associated with a variety of common problems
have been studied and summarized in the literature"' 14 .
DST charts for many problem situations were presented by
Earlougher5 and are reproduced in Figs. 10.4 - 10.17.


10-16
^ W
W
a:
W
cra V)(n
W l \^
Cr
d

TIME --+ TIME

Fig. 10A: Tight hole condition. Fig. 10.5: Leaking drillpipe or


This may cause pressure surging mud loss to some formation, or
or tool sticking (Ref. 5). both, are indicated by the hydro-
static mud-pressure decrease
shown here (Ref. 5).

W W
^ ^ .\ f N
(n ^
W W
M
a

TIME ^
TIME- ►
0
Fig. 1.O.b: Delav while going into Fig. 10.7: The stair-stepping pattern
the hole without mud loss (Ref. in the buildup curves indicates a
S). malfunctioning pressure gauge or
recorder. Such test data cannot be
analyzed (Ref. 5).

W W
X X
(n tn
^
W
In W ----- -----
CE
M ^ a

1 - ------ ------- i

TIME ----p TIME - ►

Fig. 10.8: Clock stopped (Ref. 5) Fig. 10.9: Clock ran away (Ref. 5).


10-17

TIME -^ TI ME ----w

Fig. 10.10: The S shape of the Fig. 10.11: An S shape occurring


latter part of the flow curve only in the buildup portion of
and the early part of the the curve indicates gas is going
buildup curve indicates fluid into solution in the wellbore.
communication around the This mechanism is characterized
packer. This may be caused by by a sharp transition between
a fracture or a poorly seated the flow and buildup curves
packer (Ref. 5). (Ref. 5).

N
^ \\i
W
x
a

TIME -+ TIME ---



Fig. 10.12: An S-shaped curve Fig. 10.13: This behavior indicates
occurs only in the first flow a plugged bottomhom-hole choke or
period when the volume below perforated anchor. The up-and-
the closed-in pressure valve down nature of the pressure curve
is large compared with the is caused by momentary breakthrough
volume of the fluid flowed and release of the pressure
during the flow period. A (Ref. S).
wellbore storage effect
caused by the relatively
large volume between the
hydrospring tester and DCIP
valves (Ref. 5).


10-18
W \
^ WCr M
^
N !A
W W

1
TIME-.w TIME-----o

Fi 10.14: The flat portion Fig. 10.15: A decrease in slope


in the second flow period in either flow period indicates
indicates the well is flow- fillup of the drill collar and
ing at the surface (Ref. 5) transition to a drillpipe of a
larger internal diameter (Ref. 5)

N -A
N
W

. . t

TIME -+ TIME ^

Fig. 10.16: This behavior Fig. 10.17: The rippled appearance


typically occurs in gas. in the flow curve indicates that
reservoirs when flow occurs gas has broken through the liquid
at the surface. The pressure in the drillstring and the well
decrease at Point G is caused is flowing by heads (Ref. S.).
by the water cushion flowing
at the surface, which
decreases average density
of the flowing column
(Ref. S).


10-19
REFERENCES

IS 1. McAlister, J. A., Nutter, B. P., and Lebourg, M.:


"A New System of Tools for Better Control and
Interpretation of Drillstem Tests," J. Pet. Tech.
(Feb., 1965) 207-214; Trans., AIME, ?34.

2. Edwards, A. G. and Winn, R. H.: "A Summary of Modern


Tools and Techniques Used in Drill Stem Testing,"
Publication T-4069, Halliburton Co., Duncan, Oklahoma
(Sept., 1973).

3. "Review of Basic Formation Evaluation," Form J-328,


Johnston Schlumberger, Houston (1974).

4. Edwards, A. G. and Shryock, S. H.: "New Generation


Drill Stem Testing Tools/Technology," Pet. Eng.
(July, 1974) 46-61.

5. Earlougher, R. C., Jr.: Advances in Well Test Analysis,


Monograph Series, Society of Petroleum Engineers,
Dallas (1977), S.

6. "Halliburton Formation Testing Worldwide" Form T-4062,


Halliburton Services.

7. Alexander, L. G.: "Theory and Practice of the Closed-


Chamber Drill-Stem Test Method," J. Pet. Tech. (Dec.,
1977) 1539-1544.

8. Ramey, H. J., Jr., Agarwal, R. G. and Martin, Ian:


"Analysis of 'Slug Test' or DST Flow Period Data,"
J. Cdn. Pet. Tech. (July - Sept., 1975) 37-42.

9. Moran, J. H. and Finklea, E. E.: "Theoretical Analysis


of Pressure Phenomena Associated With the Wireline
Formation Tester", J. Pet. Tech. (Aug., 1962) 899-908;
Trans., AIME, 225.

10. Burnett, 0. W. and Mixa, E.: "Application of the


Formation Interval Tester in the Rocky Mountain Area,''
Drill. and Prod. Prac., API (1964) 131-140.

11. Banks, K. M.: "Recent Achievements With the Formation


Tester in Canada," J. Cdn. Pet. Tech. (July - Sept.,
1963) 84-94.

12. Schultz, A. L., Bell, W. T., and Urbanosky, H. J.:


"Advances in Uncased-Hole, Wireline Formation Tester
Techniques," J. Pet. Tech. (Nov., 1975) 1331-1336.


10-20
13. Murphy, W. C.: "The Interpretation and Calculation of
Formation Characteristics From Formation Test Data,"
Pamphlet T-101, Halliburton Co., Duncan, Ok. (1970).

14. Timmerman, E. H. and van Poollen, H. K.: "Practical


Use of Drill-Stem Tests," J. Cdn. Pet. Tech. (April-June
1972) 31-41.


10-21
0 NOMENCLATURE - CHAPTER 10

B = formation volume factor, RB/STB


ct = total compressibility factor, psi-
DR = damage ratio, dimensionless
h = net formation thickness, ft
k = effective permeability, md
m = slope of semilog straight line, psi/cycle
= volumetric average reservoir pressure, psia
PR
pw_f(At=0) = flowing formation face pressure at time of
shut-in, psia
= shut-in formation face pressure, psia
pw.s
plhr = pressure from semilog straight line at At = 1 hour, psia
q = flow rate, STB/D
rd = drainage radius, ft
r = wellbore radius, ft
1V
s = total skin factor, dimensionless
tp = producing time prior to shut-in, hrs
Ap s = pressure loss due to skin, psi
At = shut-in time, hrs
At e = equivalent time, hrs
u = viscosity, cp
= porosity, fraction


10-"
0 SUMMARY OF MAJOR EQUATIONS - CHAPTER 10

Equation Number Equation
in Text

10.1 k = -162.6 mh

10.3 s= 1.151 pwf(Gt=O) - plhr


m

kt
log p + 3.231
^ijoti'I^(tp + 1) 1

PR Pwt(dt=0) - APs
10.4 E
^ plZ - I'Itil f(^t=0)

lU . ^ ^^ps = -0 . 8 7ms

10.6 DR = PR Pwf (At=0)
PR - Pwf(At=0) - pps

z
10.; rd = 0.029 kt

t At
10.8
Ate = t p+ At
p

• • i
10-23

APPENDIX A

FLUID AND ROCK

PROPERTY

CORRELATIONS

0
O I 2 3 a 3 6 7 0

ir

ti

Z
0

^
W
a

^ N

PSEUDO-REDUCED PRESSURE, pp.

^ Fig. A.1: Real gas deviation factor for natural gases


as a function of pseudoreduced pressure and
temperature. After Standing and Katzi.

A-2
4au -- ,^ Affd^
W

109
N 9
LL^l

^ a
C /DATED L /MESTONES
Q Fa-

Qa
3

ag

0 U4j VAN DER KNAAPS


Ojr CORRELATION
0]
ON T •
WM • ^
NW
air
a •w
o 10
K s

K F- . • ^
0
J • •
4 HALL'S
`t n CORRELATION •
U^...
• • • •
F- ^ W •
^W • :
Cr
a
^ 2 • • •
OQ
0 $
:D Ir W
WW
00.
2 J
W
H 9
W
W 1 0 5 10 15 20 25 30 35
0
A INITIAL POROSITY AT ZERO NET PRESSURE, jSj
GAS GRAVITY, yo, (AIR n 1)

Fig. A.2: Correlation of pseudo- Fig. A.3: Pore-volume compressibility


critical properties of at 75-percent lithostatic
condensate well fluids pressure vs initial sample
and miscellaneous natural porosity for limestones.
gas with fluid gravity. After Newman3.
After Brown et al?

9 • •
• • •
W
1O
^ 10t
^ 9 df
N ^ W
W Q 7
a
a , c^ FRIABLE SANDSTONES
U CONSOL IDATED SANDSTONES
H
0
F
y 0
, O Op p O

^
J O O C O
J
0 O O
aR
h 1^ 00

0
00
Q
T O
OO° O ° 0
a
0
o to
_ 0
K
-0
HALL'S CORRELATION u
u
Y
90 d
. -
0- 0
O
H . J OO 00
O
HALLS
H fN
!) , CORRELATION
U) 0 00 0
W m p
lb- o gio^ ^ o 0 O
a
a 0
2 0 0 0 0
8 0 8 0
W 0o ° 0
0
0 0

0
O 5 10 IS 20 25 30 35 0 5 10 15 20 25 30 35
a 0
INITIAL POROSITY AT ZERO NET PRESSURE, 4tj INITIAL POROSITY AT ZERO NET PRESSURE, 4ol

Fig. A.4: Pore-volume compress- Fig. A.5: Pore-volume compressibility


ibility at 75-percent at 75-percent lithostatic
lithostatic pressure vs pressure vs initial sample
initial sample porosity porosity for friable sand-
for consolidated sand- stones. After Newman3.
stones. After Newman?

SIO=
9 p
N -o
W
oc 7
a s
V 41AFCONSOLIDArED SANDSTONES
TED SANDSTONES
F-
s ^ o
^
H ♦
O
2
1- 0
!
J
O
a`
In 00
2 O

o $^ o
po
0 0
0
p lo 00
rf 0
v 7 0
r
_J
m
N
H
W
!

4
HALL'S
CORRELATION
-^

O

d'
a
O
^
W Y
a
D
J
9
W
cc
^ 0 s 10 t3 20 25 30 35
INITIAL POROSITY AT ZERO NET PRESSURE, 01

Fig. A.6: Pore-volume compressibility


at 75-percent lithostatic
pressure vs initial sample
porosity for unconsolidated
sandstones. After Newman!


A-5
• • •
GAS GRAVITY, rQ, (AIR • I.OOOY

u
u
a

W
C
W

N
0

W
2
Oh 0

N
0
V

MOLECULAR WEIGHT. M

Fig. A.7: Viscosity of natural gases at 1 atm After Carr,


Kobayashi, and Burrows4.
1010

9.0

8.0

7.0

p 6.0
^

O
H
4

0 5.0
U
N_
^
^
PSEUDOREDUCED
PRESSURE, ppr
4.0
`=o

3.0 a0

FkkJ

2.0

I.OLII
1.0 1.4 1.0 2.2 2.6 3.0

PSEUDOREDUCED TEMPERATURE, Tpr

Fig. A.8: Effect of temperature and pressure on gas


viscosity;
After Carr, Kobayashi,
and Burrows'.

A-7
• 6.0

5.0

4.0

}
^
^
^^V! 3.0
1

2.0

1.0

1.0 10 70

PSEUDOREDUCED PRESSURE, ppr

Fig. A.9: Effect of pressure and temperature on gas viscosity.


After Carr, Kobayashi, and Burrows:'


A-8
i
0
{.1
}
^
J
U m_
a
c^ N
tn
PSEUDOREDUCED W
H TEMPERATURE, r, K
J a.

T 0
U) O
N
W
M O
a w
7! 0
0 7
U O
W
O M
W 0
U 0
D
O W
W \
U)
ir IL
0
0
W
U)
a

107
3 tO is
PSEUDOREDUCED PRESSURE, Ppr PSEUDOREDUCED PRESSURE, Ppr

Fig. A.10: Correlation of pseudo- Fig. A.11: Correlation of pseudo-


reduced compressibility reduced compressibility
for natural gases. for natural gases. After
After Trubes Trubes

0 0 9
REFERENCES

• 1. Standing, Marshall B. and Katz, Donald L.: "Density


of Natural Gases," Trans., AIME (1942) 146, 140-149.
is
2. Brown, George G., Katz, Donald L., Oberfell, George G.,
and Alden, Richard C.: Natural Gasoline and the
Volatile Hydrocarbons, Natural Gasoline Assn. of
America, Tulsa (1948).

3. Newman, G. H.: "Pore-Volume Compressibility of Consoli-


dated, Friable, and Unconsolidated Reservoir Rocks
Under Hydrostatic Loading," J. Pet. Tech. (Feb. 1973)
129-134.

4. Carr, Norman L., Kobayashi, Riki, and Burrows, David


B.: "Viscosity of Hydrocarbon Gases Under Pressure,"
Trans., AIME (1954) 201, 264-272.

5. Trube, Albert S.: "Compressibility of Natural Gases,"


Trans., AIME (1957) 210, 355-357.

11 •

is •
A-10
WELL WITH WELLE
INFINITE ACTING R )SITY BEHAVIOR - interporosity f low
ROaETAOL JOHNSTON

The use of this type-cum is described : INTERPRETING WELL SERVOIRS by D. BOURDET , J. A. AYOUB , T. M. WHITTLE, Y. M. PI IAZEFF. ^- ^•
Pp+4P
arw2km C_ 0.8936 C FOR OIL -PD kh _ kh Tsc 2 M-090134
AP FOR GAS _ pD p dp
kf D (pcthrW2 141.2 q By 5.030104q T Psc µ(P)7(P) © Copyright 1983
((DVCt)f Po Flopetrol Johnston,
tD = 0.000295 kh At kh all rights reserved.
^St.ap.
((AVCt)t+(Q1VCt)m CD µ C CDPD= 141.2q 8µ ^DPD= 5.030 104q T sc µ(P)pZ(p) at.^p, Subject to change
without notice
102
CDe2S At
^ 10 60 FP

Q
O
U
0

CL
D
0
Cr
010
w
>
>
cc
w
0
w
Cr
W
U)
w
Cr
a-
0
z

0
CL
w
¢
D
^
W1
Cr
a.
U)
U)
w
-J
z
0
rn
z
w
a

10 - 1
10-1 104

DIMENSIONLESS TIME GROUP tn/Cn


00 WELL WITH WELLBORE STORAGE AND SKIN FLOPETAOLJONNSTON

INFINITE ACTING RESERVOIR WITH DOUBLE POROSITY BEHAVIOR - transient i :y flow l-l911111u-IaNlaM
The use of this type-curve is described in World Oil - April 1984: NEW TYPE CURVES AID ANALYSIS OF )NE WELLTESTS b D.BOURDET, A.ALAGOA, J.A. AYOUB , Y. M. PIRARD
Po+dpp FLOPETBOLJOHNSTON ATA
226 me ET51em
kh kh Tsc 2
a rW2 km =1.8914 (C De2S) f+m for slab matrix blocks CD = 0.8936 C FOR OIL_ PD = 4p FOR GAS _ PD = dp 17530 VauxleP2na- FRANCF
Postal address B P 557
kf Ae-2S rpcthrW2 141.2qBµ 5.030 104q T Psc po µ(P)z (P) 77006 Melm, Codex fRANCF
FLOPETflOLJOHNSTON NAM
^ ((PVct)f 1.0508 (CDe25)f+m tD p - kh At vp, _ tD p kh Tsc 2 p dt op 100 Macca B^ouleva^c
forspherical matrix blocks tD = 0 . 000295 kh At Sugariand, Texas 77478, u5 A
Fq)VCt)f+((hVCt)m ,,e-2S CD µ C CD D 141.2qBµ CD D- 5.030 104q T psc It(P)Z(P) Posts '. address P0 Box 36369
HouSron Texas 77036.. USA
102
CDe2S At

1060 tp
1050
10100
0

o 1

0
Q
a 6-
0
U
4-
^

a- 3-
:D
0
10
w

w 10-1
Cr
8-
w
Cr
a
0
z

0
CL
w 3-

Cl)
w
cc: 1
CL
Cl)
tn
U-1
J
z
0
0
z
w
2
0

10 - 1
10-1 1 10 102 103 104
DIMFNSIONI FSS TIMF GRC)I IP t../f'.,
WELL WITH UNIFORM FLUX VERTICAL FRACTURE . INFINITE ACTING RESERVOIR WITH HOMOGENEOUS BEHAVIOR rtor^aa,}o►^srori
The use of this type curve is described in World Oil October 1985 : HOW TO SIMPLIFY THE ANALYSIS OF FRACTURED WELL TESTS by A.ALAGOA, D. BOURDET ar t. AYOU B ^. ^.
SPE 12778 : INFINITE CONDUCTIVITY FRACTURE IN A DOUBLE POROSITY MEDIUM by O.PHOUZE et al.
FLOPETROLJONNSTON ATA
Pc + ^p 22E ruc Err
CDf : 0.8936 C FOR OIL - kh kh Tsc 2 77531Vadxlc Pend FRANCE
=
PD :1 p FOR GAS - _ Postal atltlrers : 6.P 557
Q)cth xf2 141.2qBu PD 5.030x104q T Psc ^^ (p)Z(P)
7700,6 Mel^n Cedex FRANCE
FLOPETROL JOHNSTON NAM
,OE kfarno BoWevard
0.000264 kAt kh ,At,AP kh Tsc 2 Pc P d t4p' SugatlandTexs 77478, U.S.A
tDi = tDf PD' tDf PD'= Postal address P 0 Box 36369
141.2qB!,t 5.030x104q T Psc (P)Z(P) liousloq Texas 77036, US A

10 ^^l rfxf2
-77

r: ,. . ... -.. ` . . .. . . ,,_ f '..


_ _ .1_...^. - ..
.. .-
. ^
. , . , . - -
._.. .-. . ...:.
._ -._ ^-_^- .. :
7 ._... .: .. .- ,
- . : 77. . ' ^ .^' .' . . , . ; .
6 l.
!
^ l;
5 - Y- - --- - -- -^^ .! . ^ .. .._ _^ ..^
_: , ^ '.. «...
4 . . +. .^ .. ,. . . . , _, .. ... « . _ , _ . . I .. . . _ ^
-_;. 7 r
:.. .., ^ __ . _ .^..
_.,..._.. .r^ _.^. . . .:. ^^ .: .. .. _ .,
. _._ : , - ._
, .. . . 'r._
T:
. ,... -. . ... . :. ... ..
,
_.. . ,. . . ._. - . -. ., ; . . . . _ ,, .
.
..
_14 f
77:- 7
3 y^ :.... : ..::...i.- ..: -t_ --f.':
'O a ^.:4
Q . .,
,
^
l -^-[ -.--- ,
t ...
O ^ r
2 f f I ... j 4 :
I I
a I
. l ..'
1 . _. .. ^1.. ., .. . . . I . _ ..
0 L_ `__ T ^. ^
- -
CC T .._ -- -.-^
(3 l;
, . . . . .^.. . . . . . ^ ^ . . ;. ..., , _ _. . . . . __ 1^ ...
w
>
^ . _ ..,
Q 1 ODf t.:_.
- } !
Ir 0. 0
w 7
0 - -
w 6 0.003 .^
fr
D
U)
C/)
w
Ir
a
0
z

0
Q.
w
^
D
Cl)
Cl)
w
tr
a
^
U)
w
I
z
0
U)
z
w
:^F
0
PTA6:1: Chapter 6, Problem 1- Frac'd Oi l Well Pressure Buildup
Storage: unit slope; linear flow: half slope; type curve m atching
2*1 02
o °
Legend
o TEST DATA
102 d(P)/d(In(T)) DERSPT
-^ i -- o -
---
^
^
a
_
Q
4

Y,
o
o i

10' -- ^ ^ ^
---- ^^ , , ----r-- -;
10 10v 10 10` 3*102

tp* Ot/(tp+At) (hours)


.p
Q

a
0
cr
0
w
^

cc
w
0
w
cc
^
^
w
cr
a-
0
z

0
CL
w
w
u^
U)
w
w
d
U)
(n
w
^
z
0
cn
z
w
2 I G^^1--
0

10 - 2 10-4
10-3 10-2 t0-^ 1 10
DIMENSIONLESS TIME tD1

Вам также может понравиться